You are on page 1of 910

CURSO DE SEGURIDAD INFORMTICA Y CRIPTOGRAFA

PRLOGO DE LA TERCERA EDICIN v 3.1 EN INTERNET


Madrid, marzo de 2003

Estimado amigo o amiga:

Transcurridos ya doce meses desde la publicacin en Internet de la 2 versin de este curso
gratuito en diapositivas, en los cuales he tenido el grato placer de observar cmo da a da el
nmero de descargas del archivo desde el servidor Web de la Red Temtica Iberoamericana
de Criptografa y Seguridad de la Informacin CriptoRed, iba logrando unas cifras para m
verdaderamente astronmicas (ms de 40.000 slo desde ese servidor, y me imagino que unas
cuantas ms por medio de copia de discos y descargas desde otros sitios Web), ha llegado el
momento de hacer una revisin y actualizacin de dicho curso.

La razn de escribir un libro en formato diapositivas naci prcticamente despus de la
publicacin del libro Aplicaciones Criptogrficas editado por el Departamento de Publicaciones
de la EUIUPM en junio de 1999, al ver que en una materia tan cambiante como es la seguridad
informtica y la criptografa era casi una utopa tener una publicacin relativamente al da
sobre tcnicas, algoritmos, esquemas, etc. Por tanto, comenc a elaborar unos apuntes de
clases para mis alumnos con un formato algo ms explcito que unas simples diapositivas de
apoyo a una clase magistral pero que, a la vez, pudiera ser til como material de aprendizaje
para cualquiera que quisiera adentrarse en este fascinante mundo de la seguridad informtica.
Naci as una primera versin del curso en el mes de septiembre de 2001, una segunda versin
en febrero de 2002 y, por ltimo, sta que adems de su publicacin en Internet como
material de libre distribucin, se publica en formato libro con ISBN y depsito legal.

Por lo tanto, adems de su publicacin en Internet para su descarga gratuita slo para fines
personales y docentes, existe una edicin del mismo en papel orientada bsicamente para los
alumnos de la asignatura de Seguridad Informtica que imparto y para quien desee tener ese
documento en formato libro- por intermedio del Departamento de Publicaciones de la Escuela
Universitaria de Informtica de la Universidad Politcnica de Madrid, bajo el patrocinio de la
Fundacin General de la UPM, con ISBN y depsito legal. Por ello, queda prohibida la venta del
mismo en los trminos habituales de cualquier propiedad intelectual protegida por copyright
como el indicado, excepto a travs de dicho departamento de publicaciones.

He corregido algunas erratas, modificado prrafos para su mejor comprensin, incluido ms
ejemplos y actualizado todos los captulos. En el captulo de presentacin del curso se incluye
el contenido por temas para una bsqueda ms rpida y efectiva. Se ha incluido un archivo
dedicado a la bibliografa, enlaces, software y tablas de inters criptogrfico. Adems, al final
de cada captulo o leccin se presenta un conjunto de cuestiones y ejercicios (en total 268)
para el lector. que le servirn a modo de evaluacin personal del avance y comprensin de cada
tema. Espero que las soluciones a los mismos estn disponible en los prximos meses en la
pgina Web de la asignatura.

Esta tercera versin del libro electrnico cuenta con 905 diapositivas, 218 ms que en la
versin v2 del ao 2002 y ms del doble comparado con la versin v1 del ao 2001. Siempre
van quedando -y me temo que por mucho tiempo- temas pendientes que por uno u otro motivo
voy dejando para ms adelante; entre ellos el de tcnicas criptogrficas avanzadas como por
ejemplo la esteganografa, la criptografa visual y la criptografa cuntica, el tema de la
seguridad fsica, de los modelos, normativas y planes de seguridad, la legislacin vigente, las
Autoridades de Certificacin, sistemas de cifra con curvas elpticas, etc.
Pg. i

Pido entonces mis disculpas si precisamente aquel tema sobre seguridad informtica que usted
estaba buscando no lo encuentra en estos apuntes. Adems, cada vez se vuelve ms difcil
lograr un documento ms o menos completo con una cantidad adecuada de pginas; en este
caso estas 900 pginas estn ya en el tamao lmite para poder publicarlo en formato libro. En
este orden de cosas, le informo que el curso que tiene en sus manos est fuertemente
orientado hacia los algoritmos, sistemas de cifra, de firma digital y protocolos criptogrficos,
enfocado eso s hacia el mundo de los ordenadores personales e Internet.

La seguridad informtica como una materia integral que engloba a diversas disciplinas como las
matemticas, algoritmos, lenguajes, sistemas operativos, sistemas de gestin de bases de
datos, protocolos y administracin de redes, modelos y polticas de seguridad, la planificacin
estratgica, hardware, software, auditora, forensia, etc., es cada vez ms amplia e imposible
de abordar en un solo texto. Ms de alguno entre los que me incluyo, hemos propuesto en algn
congreso o foro la necesidad de una nueva carrera: la Ingeniera en Seguridad Informtica.

Para asociar un nombre de archivo con un tema en particular y al mismo tiempo mantener los
nombres genricos de archivos ya utilizados en la versin anterior, excepto el primer archivo
que corresponde a la presentacin del curso, todos los archivos siguen denominndose
SItema??.ppt, si bien de forma interna cada uno de ellos tiene un ttulo diferente en las
propiedades de archivo, por lo que puede saber el nombre de la leccin correspondiente con
slo seleccionarlo o pasar el ratn por sobre el icono, sin tener que abrirlo a priori.

Para un mejor seguimiento de su aprendizaje, le recomiendo que descargue desde el servidor
de CriptoRed o la pgina Web de la asignatura los exmenes de la asignatura de Seguridad
Informtica que imparto desde 1994 (en estos momentos 17 exmenes con 122 pginas) con
sus correspondientes soluciones y, cmo no, el software de prcticas de dicha asignatura que
encontrar en el mismo servidor y el cuaderno de prcticas de la misma.

Jorge Rami Aguirre


PRLOGO DE LA SEGUNDA EDICIN
Madrid, febrero de 2002

Estimado amigo o amiga:

En estas primeras pginas del Libro Electrnico de Seguridad Informtica en Diapositivas y
de Libre Distribucin por Internet que ha descargado del servidor de la Red Temtica
CriptoRed o de mi pgina personal en la Escuela Universitaria de Informtica de la
Universidad Politcnica de Madrid, Espaa, he incluido una versin actualizada y breve del
prlogo del libro de la asignatura Aplicaciones Criptogrficas que dio paso a este curso.

Como posiblemente ya lo sepa, el curso de Seguridad Informtica en diapositivas con formato
Power Point es un proyecto en constante renovacin. En esta fase de comienzos del ao 2002
se ha ampliado contenidos y adaptado la versin que exista desde 2000, para hacerla
compatible con su impresin en papel. Ms adelante, en una siguiente fase, se incluir
informacin de inters, ejercicios, enlaces, etc.

Pg. ii
CURSO DE SEGURIDAD INFORMTICA Y CRIPTOGRAFA
Por lo tanto, le recomiendo que para una correcta visualizacin de su contenido, imprima ahora
los captulos ppt en formato documento, en lo posible con dos diapositivas por pgina hoja y
con la opcin escala de grises. Recuerde que aunque el fondo de las diapositivas sea azul, el
fondo de impresin deber ser blanco y las letras negras. Debe tener en cuenta adems que el
formato de impresin est adecuado para papel tamao DIN A4 que es el usado en Europa y
particularmente en Espaa, si bien en Amrica es ms estndar el tamao carta.

Sin ms, y con la esperanza de que el libro electrnico de este curso que en su primera versin
super las dos mil descargas desde el servidor pueda serle de provecho, reciba un cordial
saludo

Jorge Rami Aguirre


APLICACIONES CRIPTOGRFICAS
PRLOGO DE LA 2 EDICIN 1999 DEL LIBRO DE LA ASIGNATURA
(reducida y actualizada)

Uno de los retos ms fascinantes de la informtica del futuro, inmersa en sistemas de
interconexin de redes y autopistas de la informacin, con un espacio virtual para el
intercambio de mensajes y datos a travs de canales por definicin vulnerables ser, sin lugar
a dudas, la proteccin de la informacin. Representada por archivos confidenciales o mensajes
que se intercambian dos o ms interlocutores autenticados y cuyo contenido en muchos casos
debe mantenerse en secreto por razones personales, empresariales, polticas o de otra ndole,
la informacin es el bien ms preciado en estos das. Por poner slo un ejemplo sencillo y
comn, un problema de gran actualidad es el asociado con el correo electrnico que se
transmite a travs de redes y cuyo nivel seguridad deja mucho que desear. Internet es un
claro ejemplo de estas amenazas en tanto es un entorno abierto en su sentido ms amplio. Por
lo visto en estos pocos aos de existencia de la llamada red de redes, sobran los comentarios
acerca de prdida de privacidad, accesos no autorizados, ataques y otros delitos informticos
a nivel nacional e internacional.

Ante tales amenazas, la nica solucin consiste en proteger nuestros datos mediante el uso de
tcnicas criptogrficas. Esto nos permitir asegurar al menos dos elementos bsicos de la
Seguridad Informtica, a saber la confidencialidad o secre o de la informacin y la in egridad
del mensaje, adems de la autenticidad del emisor.
t t

A la luz de lo anterior, a mediados de esta dcada hacen su aparicin en los nuevos planes de
estudios de Ingeniera Informtica que comienzan a implantarse en las universidades
espaolas, diversas asignaturas sobre seguridad informtica y proteccin de la informacin,
con el objeto de formar a los futuros ingenieros en estos temas. Lo que a comienzos de los 90
apareca como una tmida apuesta por esta enseanza en carreras relacionadas principalmente
con la Informtica y las Telecomunicaciones, hoy despus de 10 aos permite ver un mapa
universitario en Espaa en el que ninguna universidad tecnolgica se queda fuera en cuanto a la
oferta de este tipo de asignaturas, superndose las 40. Es ms, comienzan ya a plantearse
perfiles en los nuevos planes de estudio con una clara orientacin hacia la seguridad
informtica de forma integral. Personalmente creo que esto es slo el comienzo; incluso me he
atrevido a proponer ya no slo un perfil, sino una carrera de ingeniera en la que cerca de un
30% de los crditos estn relacionados con esta materia, en una ponencia presentada en un
Pg. iii
congreso el ao 2001 y que podr encontrar en el mismo servidor del que ha descargado este
curso. Puede parecer una utopa, para otros tal vez una locura, pero por lo que he visto en
estos ltimos aos, tengo el presentimiento de que quizs el tiempo me d la razn y veamos
en algunos aos ms como Responsable de Seguridad Informtica precisamente a un Ingeniero
en Seguridad Informtica.

Estoy plenamente consciente que quedan muchos temas interesantes fuera del contexto del
libro, como son las nuevas tcnicas de cifra con curvas elpticas, estudio y profundizacin en
sistemas actuales de clave secreta que tendrn un importante protagonismo en el futuro como
son Skipjack y Rijndael; tcnicas y protocolos de autenticacin; la teora de cifra y
factorizacin de polinomios; tcnicas, algoritmos y esquemas de cifra en flujo; gestin de
claves, proteccin en entornos de red; esquemas de firma; certificados digitales y
Autoridades de Certificacin; criptografa visual, criptografa cuntica, esteganografa,
autenticacin biomtrica, etc. Resulta imposible abordar tantos temas y plasmarlos en un libro
que tenga un tamao relativamente normal. Por ello, he decido que las actualizaciones del
mismo desde el ao 2000 sean en formato electrnico como el que acaba de instalar en su
computador; ello permite una ms rpida y gil actualizacin y, por otra parte, como es de libre
distribucin, el alcance es mucho mayor. Recuerde que estos apuntes se actualizan de forma
peridica; como consejo le recomiendo que visite la pgina Web indicada dos o tres veces al
ao, seguro encontrar una versin nueva.

Tras estas palabras, slo me queda animarle a adentrarse al fascinante mundo de la
criptografa y escritura secreta que, por muy lejano y misterioso que pueda hoy parecerle, en
este ao 2002 la encontrar en muchos sistemas de acceso, consulta, navegacin, intercambio
de datos, etc., la mayora de ellos relacionados con Internet. El cifrado de la informacin, la
firma digital y toda la teora relacionada con este tema ser en esta primera dcada del siglo
XXI algo tan comn como lo fue en la dcada anterior escribir con un procesador de textos,
establecer una llamada telefnica mediante un telfono mvil o incluso ver la televisin va
satlite. En una sociedad informatizada al mximo, nos guste o no la proteccin de esta
informacin y ms an la autenticidad de la misma, nos llevar de forma irremediable a un
nico punto: el uso de tcnicas criptogrficas. No s qu opina usted, en mi caso estoy seguro
de ello.
Madrid, junio de 1999 (y febrero de 2002)
El autor


Dr. Ingeniero de Telecomunicacin por la Universidad Politcnica de Madrid. Profesor titular del
Departamento de Lenguajes, Proyectos y Sistemas Informticos de la Escuela Universitaria de
Informtica de la Universidad Politcnica de Madrid. Es profesor y coordinador de la asignatura de
Seguridad Informtica que se imparte desde el ao 1994 como asignatura optativa de tercer curso
en la titulacin de Ingeniero Tcnico en Informtica de Gestin del Plan de Estudios 1992. Es
Coordinador General de CriptoRed, la Red Temtica Iberoamericana de Criptografa y Seguridad
de la Informacin, en la que participan destacados profesionales e investigadores de ms de un
centenar de universidades y centros de investigacin, as como empresas de sector de las NTIs.
Es partner de la Red ECET, European Computer Education and Training, teniendo entre otros
objetivos la introduccin de las asignaturas de seguridad informtica en los planes de estudios
superiores de las universidades europeas, y miembro del Subcomit de Seguridad de T.I. (SC 27)
del Comit Tcnico de Normalizacin de Tecnologa de la Informacin (CTN 71) de AENOR.
jramio@eui.upm.es
Web personal: http://www.lpsi.eui.upm.es/~jramio
Web asignatura: http://www.lpsi.eui.upm.es/SInformatica/SInformatica.htm
Web CriptoRed: http://www.criptored.upm.es/
Pg. iv

- Documento de libre distribucin en Internet -
Prohibida su venta, excepto a travs del Departamento de Publicaciones de
la Escuela Universitaria de Informtica de la UPM - Espaa


CURSO DE
SEGURIDAD INFORMTICA
Y CRIPTOGRAFA



LIBRO ELECTRNICO DE LIBRE DISTRIBUCIN EN
INTERNET PARA LA ENSEANZA Y EL APRENDIZAJE DE
LA SEGURIDAD INFORMTICA Y LA PROTECCIN DE LA
INFORMACIN MEDIANTE TCNICAS CRIPTOGRFICAS


DIAPOSITIVAS ANIMADAS QUE PERMITEN SU IMPRESIN EN PAPEL




TERCERA EDICIN
Versin v 3.1









Autor: Jorge Rami Aguirre

UNIVERSIDAD POLITCNICA DE MADRID - ESPAA
Marzo de 2003
JRA, Madrid 2003




ISBN: 84-86451-69-8
Depsito Legal: M-10039-2003
Presentacin del Curso
Seguridad Informtica y Criptografa
Tercera edicin v 3.1 3 de marzo de 2003
Ultima actualizacin: 03/03/03
Archivo con 24 diapositivas
Curso completo: 905 diapositivas
Jorge Rami Aguirre
Universidad Politcnica de Madrid
Material Docente de
Libre Distribucin
v 3.1
Este archivo forma parte de un curso sobre Seguridad Informtica y Criptografa. Se autoriza la
reproduccin en computador e impresin en papel slo con fines docentes o personales, respetando
en todo caso los crditos del autor. Queda prohibida su venta, excepto a travs del Departamento de
Publicaciones de la Escuela Universitaria de Informtica, Universidad Politcnica de Madrid, Espaa.
Curso de Seguridad Informtica y Criptografa JRA
Seguridad Informtica y Criptografa. Presentacin del Curso
Diapositiva 2
Jorge Rami Aguirre Madrid (Espaa) 2003
Los derechos de autor
ISBN: 84-86451-69-8
Depsito Legal: M-10039-2003
Este documento electrnico puede ser descargado
libre y gratuitamente desde Internet para su
ejecucin e impresin, slo para fines educativos y/o
personales, respetando su integridad y manteniendo
los crditos del autor en el pie de pgina.
Queda por tanto prohibida su venta, excepto a
travs del Departamento de Publicaciones de la
Escuela Universitaria de Informtica de la
Universidad Politcnica de Madrid, Espaa.
Recuerde que este curso en
diapositivas es el resultado
de miles de horas de trabajo.
Seguridad Informtica y Criptografa. Presentacin del Curso
Diapositiva 3
Jorge Rami Aguirre Madrid (Espaa) 2003
Temas del curso (1)
Nmero de diapositivas
Presentacin del curso ........................................................................... 24
Tema 00: Introduccin a la Criptografa ................................................ 14
Tema 01: Introduccin a la Seguridad Informtica ................................ 54
Tema 02: Calidad de la Informacin y Virus ......................................... 25
Tema 03: Introduccin a la Seguridad Fsica ........................................ 37
Tema 04: Teora de la Informacin ........................................................ 57
Tema 05: Teora de los Nmeros ........................................................... 67
Tema 06: Teora de la Complejidad Algortmica ................................... 28
Tema 07: Sistemas de Cifra Clsicos ..................................................... 40
Tema 08: Sistemas de Cifra Modernos ................................................... 33
Tema 09: Sistemas de Cifra en Flujo ...................................................... 49
Seguridad Informtica y Criptografa. Presentacin del Curso
Diapositiva 4
Jorge Rami Aguirre Madrid (Espaa) 2003
Temas del curso (2)
Nmero de diapositivas
Tema 10: Cifrado Simtrico en Bloque ................................................. 87
Tema 11: Cifrado Asimtrico con Mochilas ......................................... 28
Tema 12: Cifrado Asimtrico Exponencial ........................................... 56
Tema 13: Funciones Hash en Criptografa ............................................ 30
Tema 14: Autenticacin y Firma Digital ............................................... 50
Tema 15: Certificados Digitales ............................................................ 11
Tema 16: Aplicaciones Criptogrficas .................................................. 93
Tema 17: Protocolos y Esquemas Criptogrficos .................................. 72
Tema 18: Bibliografa, Enlaces, SW y Tablas ....................................... 50
Total diapositivas del curso en la versin v3.1 ..................................... 905
Seguridad Informtica y Criptografa. Presentacin del Curso
Diapositiva 5
Jorge Rami Aguirre Madrid (Espaa) 2003
Resumen del Contenido (1)
Pgina diapositiva
Presentacin del curso .............................................................................. 1
Tema 00: Introduccin a la Criptografa ................................................. 25
Definicin de criptografa ..................................................................... 27
Confidencialidad e integridad .............................................................. 30
Clasificacin de los criptosistemas ....................................................... 31
Criptosistemas simtricos ..................................................................... 33
Criptosistemas asimtricos ................................................................... 34
Sistema de cifra hbrido ........................................................................ 38
Tema 01: Introduccin a la Seguridad Informtica ................................ 39
Delito informtico ................................................................................. 48
Principios de la seguridad informtica ................................................ 50
Debilidades del sistema informtico ..................................................... 54
Amenazas del sistema ........................................................................... 56
Elementos de la seguridad informtica ................................................ 63
Seguridad Informtica y Criptografa. Presentacin del Curso
Diapositiva 6
Jorge Rami Aguirre Madrid (Espaa) 2003
Resumen del Contenido (2)
Pgina diapositiva
Esquema de un criptosistema ................................................................ 66
Recomendaciones de Bacon y Kerkchoffs ............................................ 75
Fortaleza y tipos de ataques ................................................................. 78
Cifrado en bloque v/s cifrado en flujo .................................................. 80
Confidencialidad v/s integridad ........................................................... 82
Tema 02: Calidad de la Informacin y Virus .......................................... 93
Concepto e importancia de la informacin .......................................... 95
Vulnerabilidades de la informacin ..................................................... 99
Ejemplos de delitos informticos ....................................................... 106
Introduccin a los virus informticos ................................................. 111
Tema 03: Introduccin a la Seguridad Fsica ....................................... 119
Seguridad fsica en entornos de PCs .................................................. 122
Anlisis de riesgo ............................................................................... 123
Polticas de seguridad ........................................................................ 133
Seguridad Informtica y Criptografa. Presentacin del Curso
Diapositiva 7
Jorge Rami Aguirre Madrid (Espaa) 2003
Resumen del Contenido (3)
Pgina diapositiva
Modelos de seguridad ......................................................................... 137
Criterios y normativas de seguridad .................................................. 141
Planes de contingencia ....................................................................... 144
Tema 04: Teora de la Informacin ...................................................... 157
Cantidad de informacin .................................................................... 161
Definicin logartmica ........................................................................ 169
Grado de indeterminacin .................................................................. 170
Entropa de los mensajes .................................................................... 175
Codificador ptimo ............................................................................. 178
Entropa condicional ........................................................................... 181
Ratio r del lenguaje ............................................................................. 184
Redundancia del lenguaje ................................................................... 188
Secreto en un sistema criptogrfico .................................................... 194
Distancia de unicidad ......................................................................... 202
Seguridad Informtica y Criptografa. Presentacin del Curso
Diapositiva 8
Jorge Rami Aguirre Madrid (Espaa) 2003
Resumen del Contenido (4)
Pgina diapositiva
Esquema de cifrador aleatorio de Hellmann ..................................... 203
Cantidad de trabajo ............................................................................ 209
Tema 05: Teora de los Nmeros .......................................................... 215
Operaciones y propiedades de la congruencia ................................... 217
Conjunto completo de restos .............................................................. 221
Homomorfismo de los enteros ............................................................ 222
Inversos en un cuerpo ......................................................................... 227
Conjunto reducido de restos ............................................................... 232
Funcin de Euler ................................................................................ 234
Teorema de Euler ............................................................................... 240
Algoritmo extendido de Euclides ........................................................ 246
Teorema del resto chino ..................................................................... 252
Raz primitiva de un cuerpo ............................................................... 260
Algoritmo de exponenciacin rpida ................................................. 270
Seguridad Informtica y Criptografa. Presentacin del Curso
Diapositiva 9
Jorge Rami Aguirre Madrid (Espaa) 2003
Resumen del Contenido (5)
Pgina diapositiva
Clculos en campos de Galois ............................................................ 274
Tema 06: Teora de la Complejidad Algortmica ................................. 283
Nmero de operaciones bit ................................................................ 286
La funcin O(n) ................................................................................... 288
Algoritmos de complejidad lineal ....................................................... 291
Algoritmos de complejidad exponencial ............................................. 293
El problema de la mochila .................................................................. 298
El problema de la factorizacin ......................................................... 301
El problema del logaritmo discreto .................................................... 304
Tema 07: Sistemas de Cifra Clsicos ................................................... 311
Herramientas y clasificacin de la criptografa clsica .................... 316
Cifradores de esctala y Polybios ....................................................... 319
Cifrador por sustitucin y cifrado del Csar ...................................... 322
Cifrador monoalfabeto afn ................................................................ 325
Seguridad Informtica y Criptografa. Presentacin del Curso
Diapositiva 10
Jorge Rami Aguirre Madrid (Espaa) 2003
Resumen del Contenido (6)
Pgina diapositiva
Cifrador polialfabtico de Vigenre .................................................. 327
Ataque por mtodo de Kasiski ............................................................ 329
Indice de Coincidencia IC .................................................................. 333
Cifrador poligrmico de Playfair ....................................................... 334
Cifrador poligrmico de Hill .............................................................. 336
Ataque por el mtodo de Gausss Jordan ............................................ 339
El cifrador de Vernam ....................................................................... 343
Tema 08: Sistemas de Cifra Modernos ................................................. 351
Clasificacin de los criptosistemas modernos ................................... 353
Introduccin al cifrado en flujo .......................................................... 354
Introduccin al cifrado en bloque ...................................................... 358
Funciones unidireccionales con trampa ............................................ 362
Cifrado con clave pblica de destino ................................................. 365
Cifrado con clave privada de origen .................................................. 371
Seguridad Informtica y Criptografa. Presentacin del Curso
Diapositiva 11
Jorge Rami Aguirre Madrid (Espaa) 2003
Resumen del Contenido (8)
Pgina diapositiva
Comparativas entre sistemas simtricos y asimtricos ...................... 374
Tema 09: Sistemas de Cifra en Flujo .................................................... 385
Rachas de dgitos ............................................................................... 388
Autocorrelacin fuera de fase ............................................................ 390
Postulados de Golomb ........................................................................ 393
Generador de congruencia lineal ....................................................... 400
Registros de desplazamiento .............................................................. 404
Generadores no lineales: NLFSR ....................................................... 405
Generadores lineales: LFSR .............................................................. 407
Ataque de Berlekamp-Massey ............................................................ 418
Complejidad lineal ............................................................................. 421
Algoritmos A5/1 y A5/2 ...................................................................... 425
Tema 10: Cifrado Simtrico en Bloque ................................................ 435
Cifradores tipo Feistel ....................................................................... 437
Seguridad Informtica y Criptografa. Presentacin del Curso
Diapositiva 12
Jorge Rami Aguirre Madrid (Espaa) 2003
Resumen del Contenido (9)
Pgina diapositiva
Data Encryption Standard DES ......................................................... 447
Modos de cifra .................................................................................... 470
Cifrado DES mltiple. Triple DES ..................................................... 478
International Data Encryption Standard IDEA .................................. 484
Algoritmos RC2, RC5, SAFER, Blowfish, CAST, Skipjack ................ 501
DES Challenge, introduccin al AES y Rijndael ................................ 507
Tema 11: Cifrado Asimtrico con Mochilas ........................................ 523
El problema de la mochila ................................................................. 524
Mochilas simples ................................................................................ 529
Mochila de Merkle y Hellman ............................................................ 532
Criptoanlisis de Shamir y Zippel ...................................................... 542
Tema 12: Cifrado Asimtrico Exponencial .......................................... 551
Cifrado genrico tipo RSA ................................................................. 554
Intercambio de clave de Diffie y Hellman .......................................... 557
Seguridad Informtica y Criptografa. Presentacin del Curso
Diapositiva 13
Jorge Rami Aguirre Madrid (Espaa) 2003
Resumen del Contenido (10)
Pgina diapositiva
Algoritmo de cifra RSA ...................................................................... 565
Uso del Teorema del Resto Chino en RSA .......................................... 567
Nmeros primos seguros .................................................................... 572
Claves privadas parejas ..................................................................... 574
Mensajes no cifrables ......................................................................... 577
Ataque por factorizacin de n ............................................................ 583
Ataque al secreto de M por cifrado cclico ........................................ 585
Ataque por la paradoja del cumpleaos ............................................ 588
Algoritmo de cifra de Pohlig y Hellman ............................................ 591
Algoritmo de cifra de ElGamal ........................................................... 595
Tema 13: Funciones Hash en Criptografa ........................................... 607
Propiedades de las funciones hash ..................................................... 611
Algoritmos de resumen ....................................................................... 614
Message Digest 5 MD5 ....................................................................... 615
Seguridad Informtica y Criptografa. Presentacin del Curso
Diapositiva 14
Jorge Rami Aguirre Madrid (Espaa) 2003
Resumen del Contenido (11)
Pgina diapositiva
Secure Hash Algorithm SHA-1............................................................ 625
Tema 14: Autenticacin y Firma Digital .............................................. 637
Los problemas de la integridad .......................................................... 639
Autenticacin con sistemas simtricos ............................................... 644
Autenticacin con MAC o Checksum ................................................. 646
Autenticacin con HMAC ................................................................... 649
Autenticacin de Needham y Schroeder ............................................. 653
Autenticacin con Kerberos .............................................................. 658
Autenticacin con sistemas asimtricos ............................................. 668
Caractersticas de la firma digital ..................................................... 669
Firma digital RSA ............................................................................... 670
Firma digital ElGamal ....................................................................... 674
Firma digital DSS Digital Signature Standard .................................. 680
Seguridad Informtica y Criptografa. Presentacin del Curso
Diapositiva 15
Jorge Rami Aguirre Madrid (Espaa) 2003
Resumen del Contenido (12)
Pgina diapositiva
Tema 15: Certificados Digitales ........................................................... 687
Certificado digital X.509 .................................................................... 690
Introduccin a las Autoridades de Certificacin ................................ 693
Tema 16: Aplicaciones Criptogrficas ................................................. 699
Private Enhanced Mail PEM .............................................................. 703
Pretty Good Privacy PGP................................................................... 707
Cifrado local PGP con IDEA ............................................................. 712
Generacin de claves asimtricas y anillos de claves ....................... 715
Gestin del anillo de claves pblicas ................................................. 722
Cifrado RSA con clave pblica de destino ......................................... 725
Descifrado RSA con clave privada de destino ................................... 728
Firma digital RSA ............................................................................... 729
Formato de un mensaje PGP ............................................................. 731
Versiones de PGP en entorno Windows ............................................. 732
Seguridad Informtica y Criptografa. Presentacin del Curso
Diapositiva 16
Jorge Rami Aguirre Madrid (Espaa) 2003
Resumen del Contenido (13)
Pgina diapositiva
Instalacin y generacin de claves de la versin 6.5.1 ...................... 735
Operaciones con el portapapeles en la versin 6.5.1 ......................... 761
Versin 7.0.3: caractersticas y operaciones con ficheros ................. 768
Versin 8.0: caractersticas y operaciones de cifra ........................... 777
Estndar PKCS .................................................................................. 784
Caractersticas del estndar PKCS #1 de RSA .................................. 786
Tema 17: Protocolos y Esquemas Criptogrficos ................................ 793
Definicin y ejemplos de protocolos criptogrficos .......................... 794
Transferencia inconsciente o trascordada de Rabin .......................... 800
El problema del lanzamiento de la moneda ....................................... 808
Solucin segn el esquema de Blum ................................................... 810
Restos cuadrticos y enteros de Blum ................................................ 812
La firma de contratos ......................................................................... 820
Firma de contratos segn algoritmo de Even .................................... 824
Seguridad Informtica y Criptografa. Presentacin del Curso
Diapositiva 17
Jorge Rami Aguirre Madrid (Espaa) 2003
Resumen del Contenido (14)
Pgina diapositiva
Firma digital ciega ............................................................................. 827
Correo electrnico certificado ........................................................... 830
Pker mental con cifra simtrica y asimtrica ................................... 835
Canal subliminal: transferencia con conocimiento nulo ................... 840
Voto electrnico y esquema electoral ................................................. 847
Tema 18: Bibliografa, Enlaces, SW y Tablas ..................................... 865
Bibliografa en castellano ................................................................... 866
Bibliografa en ingls ......................................................................... 871
Enlaces en Internet ............................................................................. 885
Software de prcticas de la asignatura .............................................. 888
Tablas de frecuencia de caracteres y codificacin cifra clsica ........ 897
Tablas ASCII y ANSI .......................................................................... 903
Tabla cdigo base 64 .......................................................................... 907
Tablas de primos y polinomios primitivos .......................................... 909
Seguridad Informtica y Criptografa. Presentacin del Curso
Diapositiva 18
Jorge Rami Aguirre Madrid (Espaa) 2003
El curso, la asignatura y este libro
Este libro electrnico en su calidad de curso en diapositivas de
libre distribucin en Internet, es parte del material docente que
se usa en la asignatura de Seguridad Informtica y que el autor
imparte desde el ao 1994 en la Escuela Universitaria de
Informtica de la Universidad Politcnica de Madrid, Espaa.
Existe tambin una versin libro editada por el Departamento de
Publicaciones de la Escuela Universitaria de Informtica con
ISBN: 84-86451-69-8 y Depsito Legal: M-10039-2003.
Es posible que le resulte ms econmico adquirir el libro en vez
imprimirlo o sacar fotocopias. Para cualquier consulta en este
sentido sobre condiciones de compra y envo del mismo, dentro
o fuera de Espaa, pngase en contacto con el Departamento de
Publicaciones, EUI-UPM, Carretera de Valencia Km 7, 28031,
Madrid, Espaa. O bien por email a publicaciones@eui.upm.es
o telefnicamente al nmero +34 91 3367905. Precio 16.
Seguridad Informtica y Criptografa. Presentacin del Curso
Diapositiva 19
Jorge Rami Aguirre Madrid (Espaa) 2003
Sobre el autor
Jorge Rami Aguirre es profesor titular en el Departamento de Lenguajes,
Proyectos y Sistemas Informticos de la Universidad Politcnica de
Madrid.
http://www.lpsi.eui.upm.es
Desde el curso 1994/1995 imparte la asignatura de Seguridad Informtica
en la titulacin de Ingeniero Tcnico en Informtica de Gestin en la
Escuela Universitaria de Informtica de dicha universidad.
http://www.lpsi.eui.upm.es/SInformatica/SInformatica.htm
Es el creador y Coordinador General de CriptoRed, la Red Temtica
Iberoamericana de Criptografa y Seguridad de la Informacin, desde
diciembre de 1999 y que en febrero de 2003 cuenta con ms de 110
universidades representadas y una centena de empresas.
http://www.criptored.upm.es
Ha impartido diversas conferencias y cursos sobre criptografa y seguridad
informtica en Argentina, Bolivia, Chile, Colombia, Cuba, Espaa,
Mxico, Uruguay y Venezuela.
http://www.lpsi.eui.upm.es/~jramio
Seguridad Informtica y Criptografa. Presentacin del Curso
Diapositiva 20
Jorge Rami Aguirre Madrid (Espaa) 2003
El primer libro de la asignatura
Aplicaciones Criptogrficas, 2 edicin, Junio de 1999
Departamento de Publicaciones - Escuela Universitaria de
Informtica - Universidad Politcnica de Madrid
Carretera de Valencia km. 7 - 28031 Madrid (Espaa)
I.S.B.N.: 84-87238-57-2. Depsito Legal: M-24709-1999
Si dotamos a Internet de las medidas de proteccin y seguridad que nos
ofrecen las tcnicas criptogrficas, dejar de ser ese peligroso y catico
tabln de anuncios para convertirse en el supermercado electrnico del
futuro y la herramienta de trabajo de la generacin del prximo siglo.
A Anita y Jordi
Seguridad Informtica y Criptografa. Presentacin del Curso
Diapositiva 21
Jorge Rami Aguirre Madrid (Espaa) 2003
Descargas de la versin v2 del libro
Febrero 2002 16.020
Marzo 2002 5.892
Abril 2002 3.655
Mayo 2002 3.077
Junio 2002 2.717
Julio 2002 2.940
Agosto 2002 1.602
Septiembre 2002 719
Octubre 2002 674
Noviembre 2002 765
Diciembre 2002 462
Enero 2003 690
Febrero 2003 852
Nmero total de descargas de la versin v2
desde el 11/02/02 al 28/02/03: 40.065.
La versin v1 del libro electrnico alcanz unas descargas ligeramente
superiores a 2.000 durante el ao 2001 desde el mismo servidor.
Seguridad Informtica y Criptografa. Presentacin del Curso
Diapositiva 22
Jorge Rami Aguirre Madrid (Espaa) 2003
Nuevo formato imprimible
Esta edicin ha sido adaptada para que, adems de su animacin
como material de apoyo docente y de autoaprendizaje, pueda
tambin imprimirse en papel como un documento de estudio y
consulta. Esto en caso de que opte por no adquirir el libro en el
Departamento de Publicaciones segn ya se ha comentado.
Estos archivos forman parte de un Proyecto Docente del autor y
cuyo objetivo es la elaboracin de una documentacin seria sobre
seguridad informtica y criptografa, en formato electrnico de
libre distribucin en Internet.
IMPORTANTE: Para una lectura ms cmoda, se recomienda
imprimir dos diapositivas por pgina en formato documentos DIN
A4 con impresin en segundo plano, el fondo debe ser blanco. Si
imprime ms de dos diapositivas por pgina, algunas de ellas se
vern con un tamao demasiado pequeo.
Seguridad Informtica y Criptografa. Presentacin del Curso
Diapositiva 23
Jorge Rami Aguirre Madrid (Espaa) 2003
Diferencias con la versin v2
El libro en su versin v3.1 cuenta con 19 temas, uno ms que en la versin
v2 de febrero de 2002, al incluir uno dedicado a la bibliografa, enlaces,
software y tablas. Se han revisado y actualizado todos los captulos (en
especial los temas de cifra simtrica y cifra asimtrica, de autenticacin,
protocolos y correo seguro con PGP). Adems, al final de cada captulo se
ha incluido un conjunto de cuestiones y ejercicios, en total 268 preguntas.
En la medida de lo posible, las soluciones a esta cuestiones y ejercicios se
irn incluyendo de forma gradual en la pgina Web de la asignatura.
En esta versin se siguen usando los mismos nombres para los archivos,
es decir SItema??.ppt. No obstante, puede saber el contenido de cada
captulo y su nombre sencillamente viendo las propiedades de ste, sin
necesidad de abrirlo. En algunas versiones de Windows, esto se obtiene
slo con pasar el ratn sobre el icono del archivo correspondiente. Si
desea ir a una diapositiva especfica dentro de un archivo, use la Vista
Normal de Power Point, preferiblemente con la opcin contraer activada.
Seguridad Informtica y Criptografa. Presentacin del Curso
Diapositiva 24
Jorge Rami Aguirre Madrid (Espaa) 2003
Actualizacin de los archivos
Tenga en cuenta que los temas y contenidos de estos archivos sern
actualizados de forma continua. Luego, cuando observe en las pginas
Web indicadas la existencia de una versin ms actualizada del archivo
SItemas.zip que la que tiene en su ordenador, reemplace el archivo zip
anterior y, al descomprimirlo, reemplace todos los archivos ppt.
NOTA: No obstante, la documentacin ser en esencia la misma que la
del libro editado mientras se mantenga la versin 3.1. En la pgina Web
del autor y de la asignatura se irn poniendo en un archivo aparte las
nuevas diapositivas que se vayan generando de cara a una nueva edicin.
El curso comienza con un primer archivo llamado SItema00 cuyo objetivo
es entregar una introduccin a la criptografa. Si est comenzando estos
estudios, le recomiendo su visualizacin previa al curso en tanto es un
resumen claro y preciso sobre muchos de los temas y algoritmos que sern
tratados en ms detalle a lo largo de los captulos.
Fin presentacin
Tema 0
Una Introduccin a la Criptografa
Seguridad Informtica y Criptografa
Ultima actualizacin: 03/03/03
Archivo con 14 diapositivas
Jorge Rami Aguirre
Universidad Politcnica de Madrid
Material Docente de
Libre Distribucin
v 3.1
Este archivo forma parte de un curso sobre Seguridad Informtica y Criptografa. Se autoriza la
reproduccin en computador e impresin en papel slo con fines docentes o personales, respetando
en todo caso los crditos del autor. Queda prohibida su venta, excepto a travs del Departamento de
Publicaciones de la Escuela Universitaria de Informtica, Universidad Politcnica de Madrid, Espaa.
Curso de Seguridad Informtica y Criptografa JRA
Jorge Rami Aguirre Madrid (Espaa) 2003
Seguridad Informtica y Criptografa. Tema 0: Una Introduccin a la Criptografa
Diapositiva 26
Nota del autor
El contenido de este tema de
introduccin est orientado
a una primera visin de tipo
generalista al tema sobre en el
que se profundizar a lo largo
del libro: la seguridad y proteccin
de la informacin mediante el uso de tcnicas y
algoritmos criptogrficos. En particular, se trata de
visin rpida del concepto de cifra y firma digital
asociado a los criptosistemas y que sern analizados
en detalle en los prximos captulos. Si lo desea, puede
utilizar estas diapositivas para una charla introductoria al
tema de la seguridad y criptografa de unos 30 minutos.
Jorge Rami Aguirre Madrid (Espaa) 2003
Seguridad Informtica y Criptografa. Tema 0: Una Introduccin a la Criptografa
Diapositiva 27
Criptografa segn la RAE
Criptografa
una definicin ...
La Real Academia Espaola define criptografa
(oculto + escritura) como:
"el arte de escribir mensajes con una
clave secreta o de modo enigmtico".
Resulta difcil dar una definicin tan poco
ajustada a la realidad actual. Vase la siguiente diapositiva
Jorge Rami Aguirre Madrid (Espaa) 2003
Seguridad Informtica y Criptografa. Tema 0: Una Introduccin a la Criptografa
Diapositiva 28
Imprecisiones de esta definicin
Arte: la criptografa ha dejado de ser un arte: es una ciencia.
Escritura de mensajes: ya no slo se escriben mensajes; se
enva o se guarda en un ordenador todo tipo de documentos e
informacin de distintos formatos (txt, doc, exe, gif, jpg, ...).
Una clave: los sistemas actuales usan ms de una clave.
Clave secreta: existirn sistemas de clave secreta que usan
una sola clave y sistemas de clave pblica (muy importantes)
que usan dos: una clave privada (secreta) y la otra pblica.
Representacin enigmtica: la representacin binaria de la
informacin podra ser enigmtica para nosotros los humanos
pero jams para los ordenadores ... es su lenguaje natural.
Jorge Rami Aguirre Madrid (Espaa) 2003
Seguridad Informtica y Criptografa. Tema 0: Una Introduccin a la Criptografa
Diapositiva 29
Una definicin ms tcnica de criptografa
Criptografa
Lo que realmente es
Rama inicial de las Matemticas y en la actualidad de la
Informtica y la Telemtica, que hace uso de mtodos y
tcnicas con el objeto principal de cifrar un mensaje o
archivo por medio de un algoritmo, usando una o ms
claves. Esto da lugar a diferentes tipos de criptosistemas
que permiten asegurar cuatro aspectos fundamentales de
la seguridad informtica: confidencialidad, integridad,
disponibilidad y no repudio de emisor y receptor.
Jorge Rami Aguirre Madrid (Espaa) 2003
Seguridad Informtica y Criptografa. Tema 0: Una Introduccin a la Criptografa
Diapositiva 30
Confidencialidad e integridad
Criptosistema
Medio de
Transmisor Transmisin Receptor
M C
Cifrador
Mensaje cifrado Descifrador
T R
MT
C M
Usurpacin de identidad
por un intruso (I)
Interceptacin del mensaje
por un intruso (I)
Cualquier medio de transmisin es inseguro
Integridad
Confidencialidad
Estos dos principios de la seguridad informtica, el de la
confidencialidad y la integridad, (adems de la disponibilidad y el
no repudio) sern muy importantes en un sistema de intercambio
de informacin segura a travs de Internet.
Jorge Rami Aguirre Madrid (Espaa) 2003
Seguridad Informtica y Criptografa. Tema 0: Una Introduccin a la Criptografa
Diapositiva 31
Tipos de criptosistemas
Clasificacin de los criptosistemas
Segn el tratamiento del mensaje se dividen en:
Cifrado en bloque (DES, IDEA, RSA) 64-128 bits
Cifrado en flujo (A5, RC4, SEAL) cifrado bit a bit
Segn el tipo de claves se dividen en:
Cifrado con clave secreta
Cifrado con clave pblica
Sistemas simtricos
Sistemas asimtricos
Jorge Rami Aguirre Madrid (Espaa) 2003
Seguridad Informtica y Criptografa. Tema 0: Una Introduccin a la Criptografa
Diapositiva 32
Criptosistemas simtricos y asimtricos
Criptosistemas simtricos:
Existir una nica clave (secreta) que deben compartir emisor
y receptor. Con la misma clave se cifra y se descifra por lo que
la seguridad reside slo en mantener dicha clave en secreto.
Criptosistemas asimtricos:
Cada usuario crea un par de claves, una privada y otra pblica,
inversas dentro de un cuerpo finito. Lo que se cifra en emisin
con una clave, se descifra en recepcin con la clave inversa. La
seguridad del sistema reside en la dificultad computacional de
descubrir la clave privada a partir de la pblica. Para ello usan
funciones matemticas de un solo sentido con trampa.
Jorge Rami Aguirre Madrid (Espaa) 2003
Seguridad Informtica y Criptografa. Tema 0: Una Introduccin a la Criptografa
Diapositiva 33
Criptosistemas simtricos
Medio de
k
Transmisin
k
M
C
Texto Texto
Base Criptograma Base
E
K
MT
D
K
M
C
protegida
Intruso
E
K
protegida
D
K
La confidencialidad y la integridad se
lograrn si se protegen las claves en el cifrado
y en el descifrado. Es decir, se obtienen
simultneamente si se protege la clave secreta.
Cifrado con criptosistemas de clave secreta
M no permitido C no permitido
Confidencialidad
Integridad
DES, TDES,
IDEA, CAST,
RIJNDAEL
DES, TDES,
IDEA, CAST,
RIJNDAEL
Jorge Rami Aguirre Madrid (Espaa) 2003
Seguridad Informtica y Criptografa. Tema 0: Una Introduccin a la Criptografa
Diapositiva 34
Criptosistemas asimtricos (1)
Intruso
Medio de
Clave pblica
del usuario B
Transmisin
M
C Usuario A
Criptograma
E
B
MT
D
B
M
C
Clave privada
del usuario B
Usuario B
protegida
D
B
Cifrado con clave pblica del receptor
Intercambio de claves RSA
M no permitido
Observe que se
cifra con la
clave pblica
del destinatario.
Confidencialidad
Un sistema similar es el
intercambio de clave de
Diffie y Hellman (DH)
Las cifras E
B
y D
B
(claves) son
inversas dentro de un cuerpo
Jorge Rami Aguirre Madrid (Espaa) 2003
Seguridad Informtica y Criptografa. Tema 0: Una Introduccin a la Criptografa
Diapositiva 35
Criptosistemas asimtricos (2)
Intruso
Criptograma
C
Medio de
Clave privada
del usuario A
Transmisin
M
Usuario A
D
A
MT
E
A
M
C
Clave pblica
del usuario A
Usuario B
D
A
protegida
Cifrado con clave privada del emisor
Firma digital RSA Firmas: RSA y DSS
Observe que se cifra
con la clave privada
del emisor.
C no permitido
Integridad
Se firma sobre un hash
H(M) del mensaje, por
ejemplo MD5 o SHA-1
La firma DSS est
basada en el algoritmo
de cifra de El Gamal
Las cifras D
A
y E
A
(claves) son
inversas dentro de un cuerpo
Jorge Rami Aguirre Madrid (Espaa) 2003
Seguridad Informtica y Criptografa. Tema 0: Una Introduccin a la Criptografa
Diapositiva 36
Tipos de cifra con sistemas asimtricos
Criptosistemas de clave pblica
Integridad
Confidencialidad
La confidencialidad y la integridad se obtienen por separado
Firma
digital
Informacin cifrada
Autenticacin
del usuario A;
integridad de M
B B
D
A
M
E
A
M
Usuario B
k privada
de A
k pblica
de A
k pblica
de B
k privada
de B
D
A B
C
Usuario A
E DD
Jorge Rami Aguirre Madrid (Espaa) 2003
Seguridad Informtica y Criptografa. Tema 0: Una Introduccin a la Criptografa
Diapositiva 37
Qu usar, simtricos o asimtricos?
Los sistemas de clave pblica son muy
lentos pero tienen firma digital.
Los sistemas de clave secreta son muy
rpidos pero no tienen firma digital.
Qu hacer?
Cifrado de la informacin:
Sistemas de clave secreta
Firma e intercambio de clave de sesin:
Sistemas de clave pblica
Jorge Rami Aguirre Madrid (Espaa) 2003
Seguridad Informtica y Criptografa. Tema 0: Una Introduccin a la Criptografa
Diapositiva 38
Sistema hbrido de cifra y firma
Firma
digital
sobre
H(M)
Cifrado simtrico
Intercambio clave de sesin k usando
E
B
(k) en emisin y D
B
(k) en recepcin
Autenticacin
del usuario e
integridad
k k
M
E
A
M
Usuario A
k privada
de A
Usuario B
k pblica
de A
Cifrado asimtrico
k secreta
k
D
A
k
C
k secreta
Fin del Tema 0
Tema 1
Introduccin a la Seguridad Informtica
Seguridad Informtica y Criptografa
Ultima actualizacin: 03/03/03
Archivo con 54 diapositivas
Jorge Rami Aguirre
Universidad Politcnica de Madrid
Material Docente de
Libre Distribucin
v 3.1
Este archivo forma parte de un curso sobre Seguridad Informtica y Criptografa. Se autoriza la
reproduccin en computador e impresin en papel slo con fines docentes o personales, respetando
en todo caso los crditos del autor. Queda prohibida su venta, excepto a travs del Departamento de
Publicaciones de la Escuela Universitaria de Informtica, Universidad Politcnica de Madrid, Espaa.
Curso de Seguridad Informtica y Criptografa JRA
Jorge Rami Aguirre Madrid (Espaa) 2003
Seguridad Informtica y Criptografa. Tema 1: Introduccin a la Seguridad Informtica
Diapositiva 40
Conectado o desconectado?
No podemos aceptar esa
afirmacin popular que
dice que el computador
ms seguro ...
... es aquel que est
apagado y, por tanto,
desconectado de la red.
A pesar de todas las
amenazas del entorno
que, como veremos,
sern muchas y variadas.
Jorge Rami Aguirre Madrid (Espaa) 2003
Seguridad Informtica y Criptografa. Tema 1: Introduccin a la Seguridad Informtica
Diapositiva 41
Hay conciencia de las debilidades?
Amenazas
La seguridad informtica
ser un motivo de
preocupacin.
A finales del siglo XX las empresas, organismos y particulares
comienzan a tener verdadera conciencia de su importancia.
internas o externas
Jorge Rami Aguirre Madrid (Espaa) 2003
Seguridad Informtica y Criptografa. Tema 1: Introduccin a la Seguridad Informtica
Diapositiva 42
Acontecimientos en dos ltimas dcadas
A partir de los aos 80 el uso del ordenador
personal comienza a ser comn. Asoma ya la
preocupacin por la integridad de los datos.
En la dcada de los aos 90 proliferan los ataques
a sistemas informticos, aparecen los virus y se
toma conciencia del peligro que nos acecha como
usuarios de PCs y equipos conectados a Internet.
Las amenazas se generalizan a finales de los 90.
En los 00s los acontecimientos fuerzan a que se
tome en serio la seguridad informtica.
Jorge Rami Aguirre Madrid (Espaa) 2003
Seguridad Informtica y Criptografa. Tema 1: Introduccin a la Seguridad Informtica
Diapositiva 43
Qu hay de nuevo en los 00s?
Principalmente por el uso de Internet, el tema de la
proteccin de la informacin se transforma en una
necesidad y con ello se populariza la terminologa
tcnica asociada a la criptologa:
Cifrado, descifrado, criptoanlisis, firma digital.
Autoridades de Certificacin, comercio electrnico.
Ya no slo se transmiten estas enseanzas en las
universidades. El usuario final desea saber, por
ejemplo, qu significa firmar un e-mail.
Productos futuros: Seguridad aadida
Jorge Rami Aguirre Madrid (Espaa) 2003
Seguridad Informtica y Criptografa. Tema 1: Introduccin a la Seguridad Informtica
Diapositiva 44
Una definicin de criptografa
Criptografa:
Rama de las Matemticas y en la actualidad de la
Informtica que hace uso de mtodos y herramientas
con el objeto principal de cifrar un mensaje o archivo
por medio de un algoritmo y una o ms claves, dando
lugar a distintos criptosistemas que permiten asegurar,
al menos, dos aspectos bsicos de la seguridad como
son la confidencialidad y la integridad de la informacin.
Aqu tenemos una definicin algo menos
afortunada de criptografa que vemos en el
diccionario de la Real Academia Espaola ...
Jorge Rami Aguirre Madrid (Espaa) 2003
Seguridad Informtica y Criptografa. Tema 1: Introduccin a la Seguridad Informtica
Diapositiva 45
Una definicin menos afortunada...

Criptografa:
Arte de escribir mensajes con una
clave secreta o de modo enigmtico.
Desde el punto de vista de la ingeniera y la informtica,
es difcil encontrar una definicin menos apropiada .
Hoy ya no es un arte sino una ciencia.
No slo se protegen mensajes que se escriben, sino
archivos y documentos en general que se generan.
Muchos sistemas usan dos claves: secreta y pblica.
No hay nada de enigmtico en una cadena de bits.
Jorge Rami Aguirre Madrid (Espaa) 2003
Seguridad Informtica y Criptografa. Tema 1: Introduccin a la Seguridad Informtica
Diapositiva 46
Cifrar o encriptar?
Cifra o cifrado:
Tcnica que, en general, protege o autentica a un documento o
usuario al aplicar un algoritmo criptogrfico. Sin conocer una clave
especfica, no ser posible descifrarlo o recuperarlo.
En algunos pases por influencia del ingls se usar la palabra
encriptar. Si bien esta palabra todava no existe, bien podra ser el
acto de meter a alguien dentro de una cripta, ... algo
bastante distinto a lo que deseamos expresar .
Situaciones parecidas a sta encontraremos muchas. Por ejemplo,
podemos ver en algunos documentos las palabras autentificacin,
securizar y otras parecidas, que a la fecha no estn recogidas en el
diccionario de la Real Academia Espaola. Peor an, no podemos
encontrar en ese diccionario palabras tan comunes como factorizar,
factorizacin, primalidad, criptlogo, criptgrafo, criptoanalista, ...
Jorge Rami Aguirre Madrid (Espaa) 2003
Seguridad Informtica y Criptografa. Tema 1: Introduccin a la Seguridad Informtica
Diapositiva 47
Algunas definiciones previas
Criptologa: ciencia que estudia e investiga todo aquello
relacionado con la criptografa: incluye cifra y criptoanlisis.
Criptgrafo: mquina o artilugio para cifrar.
Criptlogo: persona que trabaja de forma legtima para
proteger la informacin creando algoritmos criptogrficos.
Criptoanalista: persona cuya funcin es romper algoritmos de
cifra en busca de debilidades, la clave o del texto en claro.
Texto en claro: documento original. Se denotar como M.
Criptograma: documento/texto cifrado. Se denotar como C.
Claves: datos (llaves) privados/pblicos que permitirn cifrar.
Jorge Rami Aguirre Madrid (Espaa) 2003
Seguridad Informtica y Criptografa. Tema 1: Introduccin a la Seguridad Informtica
Diapositiva 48
Es el delito informtico interesante?
El delito informtico parece ser un buen negocio:
Objeto Pequeo: la informacin est almacenada en
contenedores pequeos: no es necesario un camin para
robar un banco, llevarse las joyas, el dinero, ...
Contacto Fsico: no existe contacto fsico en la mayora
de los casos. Se asegura el anonimato y la integridad
fsica del propio delincuente.
Alto Valor: el objeto codiciado tiene un alto valor. El
contenido (los datos) puede valer mucho ms que el
soporte que los almacena: computador, disquete, CD, ...
Solucin? uso de tcnicas criptogrficas.
Jorge Rami Aguirre Madrid (Espaa) 2003
Seguridad Informtica y Criptografa. Tema 1: Introduccin a la Seguridad Informtica
Diapositiva 49
Seguridad Fsica v/s Seguridad Lgica
El estudio de la seguridad informtica podemos
plantearlo desde dos enfoques:
Seguridad Fsica: proteccin del sistema ante las
amenazas fsicas, planes de contingencia, control de
acceso fsico, polticas de seguridad, normativas, etc.
Este tema ser tratado brevemente en el captulo 3.
Seguridad Lgica: proteccin de la informacin en su
propio medio mediante el enmascaramiento de la
misma usando tcnicas de criptografa. Este enfoque
propio de las Aplicaciones Criptogrficas es el que ser
tratado a lo largo de todo el curso.
No obstante, tenga en cuenta que esta clasificacin en la
prctica no es tan rigurosa.
Jorge Rami Aguirre Madrid (Espaa) 2003
Seguridad Informtica y Criptografa. Tema 1: Introduccin a la Seguridad Informtica
Diapositiva 50
Principios de la seguridad informtica
Veremos a continuacin los tres
principios bsicos de la seguridad
informtica: el del acceso ms fcil,
el de la caducidad del secreto, y el de
la eficiencia de las medidas tomadas.
Tras los acontecimientos del 11 de
septiembre de 2001, que de alguna
forma ha hecho a la gente pensar en
las debilidades de los sistemas, vale
la pena tenerlos muy en cuenta.
Deberamos
aprender la
leccin
Jorge Rami Aguirre Madrid (Espaa) 2003
Seguridad Informtica y Criptografa. Tema 1: Introduccin a la Seguridad Informtica
Diapositiva 51
1
er
principio de la seguridad informtica
PREGUNTA:
Cules son los puntos dbiles
de un sistema informtico?
El intruso al sistema utilizar cualquier artilugio
que haga ms fcil su acceso y posterior ataque.
Existir una diversidad de frentes desde los que
puede producirse un ataque. Esto dificulta el
anlisis de riesgos porque el delincuente aplica la
filosofa de ataque hacia el punto ms dbil.
Jorge Rami Aguirre Madrid (Espaa) 2003
Seguridad Informtica y Criptografa. Tema 1: Introduccin a la Seguridad Informtica
Diapositiva 52
2 principio de la seguridad informtica
PREGUNTA:
Cunto tiempo deber
protegerse un dato?
Los datos confidenciales deben protegerse slo
hasta ese secreto pierda su valor.
Se habla, por tanto, de la caducidad del sistema de
proteccin: tiempo en el que debe mantenerse la
confidencialidad o secreto del dato.
Esto nos llevar a la fortaleza del sistema de cifra.
Jorge Rami Aguirre Madrid (Espaa) 2003
Seguridad Informtica y Criptografa. Tema 1: Introduccin a la Seguridad Informtica
Diapositiva 53
3
er
principio de la seguridad informtica
Las medidas de control se implementan para ser
utilizadas de forma efectiva. Deben ser eficientes,
fciles de usar y apropiadas al medio.
Que funcionen en el momento oportuno.
Que lo hagan optimizando los recursos del sistema.
Que pasen desapercibidas para el usuario.
Y lo ms importante: ningn sistema de
control resulta efectivo hasta que es
utilizado al surgir la necesidad de
aplicarlo. Este es uno de los grandes
problemas de la Seguridad Informtica.
Medidas de control
S.I.
Jorge Rami Aguirre Madrid (Espaa) 2003
Seguridad Informtica y Criptografa. Tema 1: Introduccin a la Seguridad Informtica
Diapositiva 54
Debilidades del sistema informtico (1)
HARDWARE - SOFTWARE - DATOS
MEMORIA - USUARIOS
Los tres primeros puntos conforman el llamado Tringulo de
Debilidades del Sistema:
Hardware: pueden producirse errores intermitentes, conexiones
suelta, desconexin de tarjetas, etc.
Software: puede producirse la sustraccin de programas, ejecucin
errnea, modificacin, defectos en llamadas al sistema, etc.
Datos: puede producirse la alteracin de contenidos, introduccin
de datos falsos, manipulacin fraudulenta de datos, etc.
Jorge Rami Aguirre Madrid (Espaa) 2003
Seguridad Informtica y Criptografa. Tema 1: Introduccin a la Seguridad Informtica
Diapositiva 55
Debilidades del sistema informtico (2)
Memoria: puede producirse la introduccin de un virus, mal uso de
la gestin de memoria, bloqueo del sistema, etc.
Usuarios: puede producirse la suplantacin de identidad, el acceso
no autorizado, visualizacin de datos confidenciales, etc.
Es muy difcil disear un plan que contemple minimizar de
forma eficiente todos estos aspectos negativos.
Debido al Principio de Acceso ms Fcil, no se deber
descuidar ninguno de los cinco elementos susceptibles de
ataque del sistema informtico.
Jorge Rami Aguirre Madrid (Espaa) 2003
Seguridad Informtica y Criptografa. Tema 1: Introduccin a la Seguridad Informtica
Diapositiva 56
Amenazas del sistema
Las amenazas afectan
principalmente al
Hardware, al Software
y a los Datos. stas se
deben a fenmenos de:
Interrupcin
Interceptacin
Modificacin
Generacin
Interrupcin Interceptacin
Flujo Normal
Modificacin Generacin
Jorge Rami Aguirre Madrid (Espaa) 2003
Seguridad Informtica y Criptografa. Tema 1: Introduccin a la Seguridad Informtica
Diapositiva 57
Amenazas de interrupcin
Se daa, pierde o deja de funcionar un punto del
sistema.
Su deteccin es inmediata.
Interrupcin
Intruso
Ejemplos: Destruccin del hardware.
Borrado de programas, datos.
Fallos en el sistema operativo.
Jorge Rami Aguirre Madrid (Espaa) 2003
Seguridad Informtica y Criptografa. Tema 1: Introduccin a la Seguridad Informtica
Diapositiva 58
Amenazas de interceptacin
Interceptacin
Intruso
Acceso a la informacin por parte de personas no
autorizadas. Uso de privilegios no adquiridos.
Su deteccin es difcil, a veces no deja huellas.
Ejemplos: Copias ilcitas de programas.
Escucha en lnea de datos.
Jorge Rami Aguirre Madrid (Espaa) 2003
Seguridad Informtica y Criptografa. Tema 1: Introduccin a la Seguridad Informtica
Diapositiva 59
Amenazas de modificacin
Modificacin
Intruso
Acceso no autorizado que cambia el entorno para
su beneficio.
Su deteccin es difcil segn las circunstancias.
Ejemplos: Modificacin de bases de datos.
Modificacin de elementos del HW.
Jorge Rami Aguirre Madrid (Espaa) 2003
Seguridad Informtica y Criptografa. Tema 1: Introduccin a la Seguridad Informtica
Diapositiva 60
Amenazas de generacin
Generacin
Intruso
Creacin de nuevos objetos dentro del sistema.
Su deteccin es difcil: delitos de falsificacin.
Ejemplos: Aadir transacciones en red.
Aadir registros en base de datos.
Jorge Rami Aguirre Madrid (Espaa) 2003
Seguridad Informtica y Criptografa. Tema 1: Introduccin a la Seguridad Informtica
Diapositiva 61
El tringulo de debilidades
Interrupcin Interceptacin Modificacin Generacin
(prdida) (acceso) (cambio) (alteracin)
DATOS
HW SW
Interrupcin (denegar servicio) Modificacin (falsificacin)
Interceptacin (robo) Interrupcin (borrado)
Interceptacin (copia)
Ejemplos de ataques
Los datos sern la
parte ms vulnerable
del sistema.
Jorge Rami Aguirre Madrid (Espaa) 2003
Seguridad Informtica y Criptografa. Tema 1: Introduccin a la Seguridad Informtica
Diapositiva 62
Ataques caractersticos
Hardware:
Agua, fuego, electricidad, polvo, cigarrillos, comida.
Software:
Adems de algunos de hardware, borrados accidentales
o intencionados, esttica, fallos de lneas de programa,
bombas lgicas, robo, copias ilegales.
Datos:
Tiene los mismos puntos dbiles que el software. Pero
hay dos problemas aadidos: no tienen valor intrnseco
pero s su interpretacin y, por otra parte, algunos datos
pueden ser de carcter pblico.
Jorge Rami Aguirre Madrid (Espaa) 2003
Seguridad Informtica y Criptografa. Tema 1: Introduccin a la Seguridad Informtica
Diapositiva 63
Confidencialidad, integridad, disponibilidad
Confidencialidad
Los componentes del sistema sern accesibles slo por
aquellos usuarios autorizados.
Integridad
Los componentes del sistema slo pueden ser creados
y modificados por los usuarios autorizados.
Disponibilidad
Los usuarios deben tener disponibles todos los
componentes del sistema cuando as lo deseen.
Jorge Rami Aguirre Madrid (Espaa) 2003
Seguridad Informtica y Criptografa. Tema 1: Introduccin a la Seguridad Informtica
Diapositiva 64
No repudio de origen y destino
No Repudio
Este trmino se ha introducido en los ltimos aos
como una caracterstica ms de los elementos que
conforman la seguridad en un sistema informtico.
Est asociado a la aceptacin de un protocolo de
comunicacin entre emisor y receptor (cliente y
servidor) normalmente a travs del intercambio de
sendos certificados digitales de autenticacin.
Se habla entonces de No Repudio de Origen y No
Repudio de Destino, forzando a que se cumplan todas
las operaciones por ambas partes en una comunicacin.
Jorge Rami Aguirre Madrid (Espaa) 2003
Seguridad Informtica y Criptografa. Tema 1: Introduccin a la Seguridad Informtica
Diapositiva 65
Datos seguros
Si se cumplen estos principios, diremos en general
que los datos estn protegidos y seguros.
Confidencialidad Integridad Disponibilidad
Datos Seguros
DATOS DATOS
DATOS
DATOS
Jorge Rami Aguirre Madrid (Espaa) 2003
Seguridad Informtica y Criptografa. Tema 1: Introduccin a la Seguridad Informtica
Diapositiva 66
Sistema de cifra
Medio de
Transmisin Transmisor Receptor
M C
T R
MT
C M
Mensaje cifrado
Interceptacin del mensaje
por un intruso
Cifrador
Descifrador
Usurpacin de identidad
por un intruso
Sea cual sea el medio de transmisin (enlace, red telefnica,
red de datos, disco magntico, disco ptico, etc.) ste ser
siempre y por definicin inseguro.
Jorge Rami Aguirre Madrid (Espaa) 2003
Seguridad Informtica y Criptografa. Tema 1: Introduccin a la Seguridad Informtica
Diapositiva 67
Esquema de un criptosistema
Texto
Base Base
Canal inseguro
Clave
Cifrador Descifrador
Texto
C
K Clave K
Transmisor Receptor
Texto cifrado
M M
Espacio de Mensajes M
Espacio de Textos Cifrados C
Espacio de Claves K
Transformaciones de Cifrado y de Descifrado
Se habla
entonces de:
Jorge Rami Aguirre Madrid (Espaa) 2003
Seguridad Informtica y Criptografa. Tema 1: Introduccin a la Seguridad Informtica
Diapositiva 68
Funciones y operaciones de cifra
C = E(M)
M = D(C)
M = D(E(M))
Si se usa una clave k:
C = E(k,M) o E
k
(M)
M = D(k, E(k,M))
M = D(k
D
, E(k
E
,M))
Las operaciones D y E son
inversas o bien lo son las
claves que intervienen. Esto
ltimo es lo ms normal, con
los inversos dentro de un
cuerpo finito. Por lo tanto, se
recupera el mensaje en claro.
E: Cifrado del mensaje M
D: Descifrado del criptograma C
En este ltimo caso los algoritmos E y D son iguales
Jorge Rami Aguirre Madrid (Espaa) 2003
Seguridad Informtica y Criptografa. Tema 1: Introduccin a la Seguridad Informtica
Diapositiva 69
Espacio de mensajes M
Componentes de un mensaje inteligible (bits, bytes,
pixels, signos, caracteres, etc.) que provienen de un
alfabeto previamente establecido como en el ejemplo.
El lenguaje tiene unas reglas sintcticas y semnticas.
En algunos casos y para los sistemas de cifra clsicos
la longitud del alfabeto indicar el mdulo en el cual
se trabaja. En los modernos, no guarda relacin.
Habr mensajes con sentido y mensajes sin sentido.
M = {m
1
, m
2
, ..., m
n
}
Est muy claro que esto
es un texto en claro...
Jorge Rami Aguirre Madrid (Espaa) 2003
Seguridad Informtica y Criptografa. Tema 1: Introduccin a la Seguridad Informtica
Diapositiva 70
Espacio de textos cifrados C
Normalmente el alfabeto es el mismo que el utilizado
para crear el mensaje en claro.
Supondremos que el espacio de los textos cifrados C y
el espacio de los mensaje M (con y sin sentido) tienen
igual magnitud.
En este caso, a diferencia del espacio de mensajes M,
sern vlidos todo tipo de criptogramas.
C = {c
1
, c
2
, ..., c
n
}
VjbmljYSB3kZSBNYWRy+
WQgQ0ExLTAr8BgN...
Jorge Rami Aguirre Madrid (Espaa) 2003
Seguridad Informtica y Criptografa. Tema 1: Introduccin a la Seguridad Informtica
Diapositiva 71
Espacio de claves K
K = {k
1
, k
2
, ..., k
n
}
Si el espacio de claves K es tan grande como el de los
mensajes M, se obtendr un criptosistema con secreto
perfecto.
Se supone que es un conjunto altamente aleatorio de
caracteres, palabras, bits, bytes, etc., en funcin del
sistema de cifra. Al menos una de las claves en un
criptosistema se guardar en secreto.
Jorge Rami Aguirre Madrid (Espaa) 2003
Seguridad Informtica y Criptografa. Tema 1: Introduccin a la Seguridad Informtica
Diapositiva 72
Transformaciones de cifrado E
k
E
k
: M C k K
E
k
es una aplicacin con una clave k, que est en el
espacio de claves K, sobre el mensaje M y que lo
transforma en el criptograma C.
Es el algoritmo de cifra. Slo en algunos sistemas
clsicos el algoritmo es secreto. Por lo general el
algoritmo de cifra ser de dominio pblico y su cdigo
fuente debera estar disponible en Internet.
Jorge Rami Aguirre Madrid (Espaa) 2003
Seguridad Informtica y Criptografa. Tema 1: Introduccin a la Seguridad Informtica
Diapositiva 73
Transformaciones de descifrado D
k
D
k
es una aplicacin con una clave k, que est en el
espacio de claves K, sobre el criptograma C y que lo
transforma en el texto en claro M.
Se usa el concepto de inverso. D
k
ser la operacin
inversa de E
k
o bien -que es lo ms comn- se usa la
misma transformacin E
k
para descifrar pero con una
clave k que es la inversa de k dentro de un cuerpo.
D
k
: C M k K
Jorge Rami Aguirre Madrid (Espaa) 2003
Seguridad Informtica y Criptografa. Tema 1: Introduccin a la Seguridad Informtica
Diapositiva 74
Criptosistemas de clave secreta
Mensaje cifrado
Medio de
Transmisin Clave
M
C
Texto
Base
Base
Cifrado Descifrado
MT
M C
Cifrado: E
k
Descifrado: D
k
E
k
D
k
k k
Clave
nica
Clave
Texto
Jorge Rami Aguirre Madrid (Espaa) 2003
Seguridad Informtica y Criptografa. Tema 1: Introduccin a la Seguridad Informtica
Diapositiva 75
Requisitos de un criptosistema
Algoritmo de cifrado/descifrado rpido y fiable.
Posibilidad de transmitir ficheros por una lnea de
datos, almacenarlos o transferirlos.
No debe existir retardo debido al cifrado o descifrado.
La seguridad del sistema deber residir solamente en
el secreto de una clave y no de las funciones de cifra.
La fortaleza del sistema se entender como la
imposibilidad computacional (tiempo de clculo en
aos que excede cualquier valor razonable) de romper
la cifra o encontrar la clave secreta a partir de otros
datos de carcter pblico.
Jorge Rami Aguirre Madrid (Espaa) 2003
Seguridad Informtica y Criptografa. Tema 1: Introduccin a la Seguridad Informtica
Diapositiva 76
Recomendaciones de Bacon
Filsofo y estadista ingls del siglo XVI
Dado un texto en claro M y un algoritmo de cifra E
k
,
el clculo de E
k
(M) y su inversa debe ser sencillo.
Ser imposible encontrar el texto en claro M a partir
del criptograma C si se desconoce la funcin de
descifrado D
k
.
El criptograma deber contener caracteres distribuidos
para que su apariencia sea inocente y no d pistas a un
intruso.
Teniendo en cuenta los siglos trascurridos desde estas
afirmaciones, stas siguen siendo vlidas hoy en da.
Jorge Rami Aguirre Madrid (Espaa) 2003
Seguridad Informtica y Criptografa. Tema 1: Introduccin a la Seguridad Informtica
Diapositiva 77
Recomendaciones de Kerckhoffs
Profesor holands en Pars del siglo XIX
K
1
: El sistema debe ser en la prctica imposible de
criptoanalizar.
K
2
: Las limitaciones del sistema no deben plantear
dificultades a sus usuarios.
K
3
: Mtodo de eleccin de claves fcil de recordar.
K
4
: Transmisin del texto cifrado por telgrafo.
K
5
: El criptgrafo debe ser portable.
K
6
: No debe existir una larga lista de reglas de uso.
Al igual que en el caso anterior, siguen siendo vlidas.
Jorge Rami Aguirre Madrid (Espaa) 2003
Seguridad Informtica y Criptografa. Tema 1: Introduccin a la Seguridad Informtica
Diapositiva 78
Fortaleza: tipos de ataques
Conociendo el algoritmo de cifra, el criptoanalista
intentar romper la cifra:
1. Contando nicamente con el criptograma.
2. Contando con texto en claro conocido.
3. Eligiendo un texto en claro.
4. A partir de texto cifrado elegido.
ATAQUE POR FUERZA BRUTA
5. Buscando combinaciones de claves.
Mayor
trabajo
Jorge Rami Aguirre Madrid (Espaa) 2003
Seguridad Informtica y Criptografa. Tema 1: Introduccin a la Seguridad Informtica
Diapositiva 79
Clasificacin de los criptosistemas
Sistemas de cifra: clsicos
v/s modernos
Clasificacin histrica y
cultural (no cientfica).
Sistemas de cifra: en bloque
v/s en flujo
Clasificacin de acuerdo a
cmo se produce la cifra.
Sistemas de clave: secreta v/s pblica
Clasificacin de acuerdo a la cifra usando una nica
clave secreta o bien sistemas con dos claves, una de
ellas pblica y la otra privada.
Jorge Rami Aguirre Madrid (Espaa) 2003
Seguridad Informtica y Criptografa. Tema 1: Introduccin a la Seguridad Informtica
Diapositiva 80
Cifrado en bloque y en flujo
CIFRADO EN BLOQUE:
El mismo algoritmo de cifra se aplica a un bloque de
informacin (grupo de caracteres, nmero de bytes,
etc.) repetidas veces, usando la misma clave. El
bloque ser normalmente de 64 128 bits.
CIFRADO EN FLUJO:
El algoritmo de cifra se aplica a un elemento de
informacin (carcter, bit) mediante un flujo de clave
en teora aleatoria y mayor que el mensaje. La cifra se
hace bit a bit.
Jorge Rami Aguirre Madrid (Espaa) 2003
Seguridad Informtica y Criptografa. Tema 1: Introduccin a la Seguridad Informtica
Diapositiva 81
Comparativa cifrado en bloque v/s flujo
CIFRADO EN BLOQUE
Ventajas: Desventajas:
* Alta difusin de los elementos * Baja velocidad de cifrado al tener
en el criptograma. que leer antes el bloque completo.
* Inmune: imposible introducir * Propenso a errores de cifra. Un
bloques extraos sin detectarlo. error se propagar a todo el bloque.
CIFRADO EN FLUJO
Ventajas: Desventajas:
* Alta velocidad de cifra al no * Baja difusin de elementos en el
tener en cuenta otros elementos. criptograma.
* Resistente a errores. La cifra es * Vulnerable. Pueden alterarse los
independiente en cada elemento. elementos por separado.
Jorge Rami Aguirre Madrid (Espaa) 2003
Seguridad Informtica y Criptografa. Tema 1: Introduccin a la Seguridad Informtica
Diapositiva 82
Confidencialidad v/s integridad
Vamos a ver cmo se obtienen en cada uno de
estos sistemas de cifra (cifrado con clave secreta
y cifrado con clave pblica) los dos aspectos ms
relevantes de la seguridad informtica:
La confidencialidad
y la integridad
Llegaremos a un concepto de mucha utilidad en
criptografa al analizar el sistema con clave pblica...
Jorge Rami Aguirre Madrid (Espaa) 2003
Seguridad Informtica y Criptografa. Tema 1: Introduccin a la Seguridad Informtica
Diapositiva 83
Confidencialidad con clave secreta
protegida
Buscamos la
confidencialidad
Medio de
k
Transmisin
k
M
C
Texto Texto
Base Base
M no permitido
E
K
MT
D
K
M
C
D
K
intruso
Criptograma
Protegemos el
extremo receptor
El criptoanalista no podr descifrar el criptograma C o
cualquier otro texto cifrado bajo la transformacin E
K
.
Jorge Rami Aguirre Madrid (Espaa) 2003
Seguridad Informtica y Criptografa. Tema 1: Introduccin a la Seguridad Informtica
Diapositiva 84
Integridad con clave secreta
protegida
Buscamos la
integridad
Medio de
k
Transmisin
k
M
C
Texto Texto
Base
Criptograma
Base
C no permitido
E
K
MT
D
K
M
C
E
K
intruso
Hola... soy Paquito!
Protegemos ahora el extremo emisor
El criptoanalista no podr cifrar un texto en claro M y
enviarlo al destinatario como C = E
K
(M).
Jorge Rami Aguirre Madrid (Espaa) 2003
Seguridad Informtica y Criptografa. Tema 1: Introduccin a la Seguridad Informtica
Diapositiva 85
Resumen para sistemas de clave secreta
protegida
Medio de
k
Transmisin
k
M
C
Texto Texto
Base
Criptograma
Base
C no permitido
E
K
MT
D
K
M
C
E
K
Hola... soy Paquito!
M no permitido
protegida
D
K
Confidencialidad Integridad
La confidencialidad y la integridad se lograrn
simultneamente si se protege la clave secreta.
Era algo
obvio
Jorge Rami Aguirre Madrid (Espaa) 2003
Seguridad Informtica y Criptografa. Tema 1: Introduccin a la Seguridad Informtica
Diapositiva 86
Confidencialidad con clave pblica
protegida
Buscamos la
confidencialidad
Medio de
clave pblica de B
Transmisin
M
C
Texto Texto
Base Base
M no permitido
E
B
MT
D
B
M
C
D
B
intruso
Criptograma
clave privada de B
Observe que se cifra
con la clave pblica
del usuario receptor
Usuario A Usuario B
Cada usuario U
usa dos funciones:
una pblica E
U
y
otra privada D
U
Cada usuario U
usa dos funciones:
una pblica E
U
y
otra privada D
U
C = E
B
(M)
M = D
B
(C) = D
B
(E
B
(M))
D
B
y E
B
son operaciones
inversas dentro de un cuerpo
Jorge Rami Aguirre Madrid (Espaa) 2003
Seguridad Informtica y Criptografa. Tema 1: Introduccin a la Seguridad Informtica
Diapositiva 87
Integridad con clave pblica
Medio de
clave privada de A
Transmisin
M
C
Texto Texto
Base Base
D
A
MT
E
A
M
C
D
A
Criptograma
clave pblica de A
Observe que se cifra
con la clave privada
del usuario emisor
Usuario A Usuario B
protegida
Buscamos la
integridad
C no permitido
intruso
Hola... soy Paquito
otra vez!
C = D
A
(M)
M = E
A
(C) = E
A
(D
A
(M))
D
A
y E
A
son operaciones
inversas dentro de un cuerpo
Jorge Rami Aguirre Madrid (Espaa) 2003
Seguridad Informtica y Criptografa. Tema 1: Introduccin a la Seguridad Informtica
Diapositiva 88
Resumen para sistemas con clave pblica
D E D
K privada
de A
M
C
Confidencialidad
Usuario A Usuario B
Integridad
B
E
A B
D
A
K privada
de B
K pblica
de B
K pblica
de A
D
A
La integridad y la confidencialidad se
obtendrn ahora por separado ...
M
Esta caracterstica
ser muy importante
B
C = E
B
(D
A
(M)) Cifrado del mensaje con firma digital
M = E
A
(D
B
(C)) Descifrado y comprobacin de firma
Jorge Rami Aguirre Madrid (Espaa) 2003
Seguridad Informtica y Criptografa. Tema 1: Introduccin a la Seguridad Informtica
Diapositiva 89
La gestin de las claves secretas
D
k
AB
k
AC
k
BC
k
AD
k
BD
k
CD
k
AE
k
BE
k
CE
k
DE
X Y
k
XY
k
YX
Nmero Claves:
n (n-1)
2
2 usuarios: N = 1 3 usuarios: N = 3 4 usuarios: N = 6 5 usuarios: N = 10
Definicin previa:
N = n de claves
Clave
secreta
E
A
C
B
Muy mala gestin de claves n
2
Jorge Rami Aguirre Madrid (Espaa) 2003
Seguridad Informtica y Criptografa. Tema 1: Introduccin a la Seguridad Informtica
Diapositiva 90
La solucin hbrida
Es entonces la clave pblica la solucin a
todos nuestros problemas?
NO !
Tendr como inconveniente principal (debido a las
funciones de cifra empleadas) una tasa o velocidad de
cifra mucho ms baja que la de los criptosistemas de
clave secreta.
Solucin?
Sistemas de cifra hbridos:
los esquemas actuales
de protocolos seguros en
Internet funcionan as.
Fin del Tema 1
Jorge Rami Aguirre Madrid (Espaa) 2003
Seguridad Informtica y Criptografa. Tema 1: Introduccin a la Seguridad Informtica
Diapositiva 91
Cuestiones y ejercicios (1 de 2)
1. Un empleado poco satisfecho ha robado varios discos duros de muy
alta calidad con datos de la empresa. Qu importa ms, el costo de
esos discos o el valor de los datos? Justifique su respuesta.
2. En una empresa se comienza a planificar estrategias de acceso a las
dependencias, polticas de backup, de proteccin de los equipos ante
fuego, agua, etc. Eso es seguridad fsica o lgica? Por qu?
3. En nuestra empresa alguien usa software pirata. Es una amenaza de
interrupcin, interceptacin, modificacin o de generacin?
4. Una clave de sesin en Internet para proteger una operacin de cifra
dura 45 segundos. Si alguien intercepta el criptograma, debemos
preocuparnos si sabemos que la prxima vez la clave ser otra?
5. Si se prueban todas las combinaciones posibles de una clave para
romper un criptograma, qu tipo de ataque estamos realizando?
Jorge Rami Aguirre Madrid (Espaa) 2003
Seguridad Informtica y Criptografa. Tema 1: Introduccin a la Seguridad Informtica
Diapositiva 92
Cuestiones y ejercicios (2 de 2)
6. Si protegemos una clave en el extremo emisor, qu buscamos, la
confidencialidad o la integridad? Y si es en el extremo receptor?
7. Por qu en un sistema simtrico se obtiene la confidencialidad y la
integridad al mismo tiempo protegiendo la clave?
8. Explique qu significa que en un sistema de cifra asimtrica se
obtengan la confidencialidad y la integridad por separado.
9. Si se cifra un mensaje con la clave privada del emisor, qu se
obtiene? Y si el emisor cifra con la clave pblica del receptor?
10. Tiene sentido que el emisor cifre de forma asimtrica con su clave
pblica? Qu logramos con ello? Para qu servira?
11. Queremos comunicarnos 10 usuarios con un sistema de cifra de
clave secreta nica entre cada dos miembros. Cuntas claves sern
necesarias? Es eficiente el sistema? Y si hay un usuario ms?
Tema 2
Calidad de la Informacin y Virus
Seguridad Informtica y Criptografa
Ultima actualizacin: 03/03/03
Archivo con 25 diapositivas
Jorge Rami Aguirre
Universidad Politcnica de Madrid
Material Docente de
Libre Distribucin
v 3.1
Este archivo forma parte de un curso sobre Seguridad Informtica y Criptografa. Se autoriza la
reproduccin en computador e impresin en papel slo con fines docentes o personales, respetando
en todo caso los crditos del autor. Queda prohibida su venta, excepto a travs del Departamento de
Publicaciones de la Escuela Universitaria de Informtica, Universidad Politcnica de Madrid, Espaa.
Curso de Seguridad Informtica y Criptografa JRA
Jorge Rami Aguirre Madrid (Espaa) 2003
Seguridad Informtica y Criptografa. Tema 2: Calidad de la Informacin y Virus
Diapositiva 94
Nota del autor
El contenido de este tema
corresponde a los primeras
apuntes del autor.
Tenga en cuenta entonces
que se trata de un borrador, simplemente unas notas
sueltas que tratan genricamente este tema y la nica
intencin de mantener este captulo en el libro es
porque algn da ser ampliado, actualizado y
tratado como se merece.
Jorge Rami Aguirre Madrid (Espaa) 2003
Seguridad Informtica y Criptografa. Tema 2: Calidad de la Informacin y Virus
Diapositiva 95
Qu es la informacin?
El concepto en ingeniera:
Estudio de las estadsticas y caractersticas del
lenguaje que nos permitir su anlisis desde un
punto de vista matemtico, cientfico y tcnico.
El concepto en la empresa:
Conjunto de datos propios que se gestionan y
mensajes que se intercambian personas y/o
mquinas dentro de una organizacin.
Jorge Rami Aguirre Madrid (Espaa) 2003
Seguridad Informtica y Criptografa. Tema 2: Calidad de la Informacin y Virus
Diapositiva 96
Teora de la Informacin
El estudio hecho por Claude Shannon en aos
posteriores a la 2 Guerra Mundial ha permitido:
Cuantificar la cantidad de informacin.
Medir la entropa de la informacin.
Definir un sistema con secreto perfecto.
Calcular la redundancia y ratio del lenguaje.
Encontrar la distancia de unicidad.
Aunque todo el estudio est orientado hacia los
criptosistemas clsicos que cifran letras, se ver
en detalle en un captulo posterior pues permite
definir y analizar sistemas con secreto perfecto.
Jorge Rami Aguirre Madrid (Espaa) 2003
Seguridad Informtica y Criptografa. Tema 2: Calidad de la Informacin y Virus
Diapositiva 97
La informacin en la empresa
Se entender como:
Todo el conjunto de datos y ficheros de la empresa.
Todos los mensajes intercambiados.
Todo el historial de clientes y proveedores.
Todo el historial de productos, ... etc.
En definitiva, el know-how de la organizacin.
Si esta informacin se pierde o deteriora, le ser
muy difcil a la empresa recuperarse y seguir
siendo competitiva polticas de seguridad.
Jorge Rami Aguirre Madrid (Espaa) 2003
Seguridad Informtica y Criptografa. Tema 2: Calidad de la Informacin y Virus
Diapositiva 98
Importancia de la informacin
La implantacin de una poltica y medidas de seguridad
informtica en la empresa comienza a tenerse en cuenta slo a
finales de la dcada pasada. En este nuevo siglo es un factor
estratgico en el desarrollo y vida de la misma. Tras los
acontecimientos en las torres gemelas del ao 2001, varias
empresas han desaparecido por haber perdido toda su
informacin. Es una seal de peligro y un aviso.
El xito de una empresa depender de la
calidad de la informacin que genera y
gestiona. As, una empresa tendr una
informacin de calidad si sta permite, entre
otras caractersticas, la confidencialidad, la
integridad y disponibilidad.
Jorge Rami Aguirre Madrid (Espaa) 2003
Seguridad Informtica y Criptografa. Tema 2: Calidad de la Informacin y Virus
Diapositiva 99
Vulnerabilidad de la informacin
La informacin (datos) se ver afectada
por muchos factores, incidiendo
bsicamente en los aspectos de
confidencialidad, integridad y
disponibilidad de la misma.
Desde el punto de vista de la empresa,
uno de los problema ms importantes
puede ser el que est relacionado con el
delito o crimen informtico, por
factores externos e internos.
descontento
Jorge Rami Aguirre Madrid (Espaa) 2003
Seguridad Informtica y Criptografa. Tema 2: Calidad de la Informacin y Virus
Diapositiva 100
Implantar polticas de seguridad
Esto se ver agravado por otros temas,
entre ellos los aspectos legales y las
caractersticas de los nuevos entornos de
trabajo de la empresa del siglo XXI.
... slo ahora el
tema comienza a
tomarse en serio.
Poltica 1
Poltica 2
Poltica 3
Poltica 4
Solucin
La solucin parece
muy sencilla: aplicar
tcnicas y polticas
de seguridad...
Jorge Rami Aguirre Madrid (Espaa) 2003
Seguridad Informtica y Criptografa. Tema 2: Calidad de la Informacin y Virus
Diapositiva 101
Acciones contra los datos
Una persona no autorizada podra:
Clasificar y desclasificar los datos.
Filtrar informacin.
Alterar la informacin.
Borrar la informacin.
Usurpar datos.
Hojear informacin clasificada.
Deducir datos confidenciales.
Por lo tanto,
deberemos
proteger
nuestros datos
Jorge Rami Aguirre Madrid (Espaa) 2003
Seguridad Informtica y Criptografa. Tema 2: Calidad de la Informacin y Virus
Diapositiva 102
Proteccin de los datos
La medida ms eficiente para la proteccin de los datos es
determinar una buena poltica de copias de seguridad o
backups:
Copia de seguridad completa
Todos los datos (la primera vez).
Copias de seguridad incrementales
Slo se copian los ficheros creados o modificados desde el
ltimo backup.
Elaboracin de un plan de backup en funcin del
volumen de informacin generada
Tipo de copias, ciclo de esta operacin, etiquetado correcto.
Diarias, semanales, mensuales: creacin de tablas.
Establecer quin cmo y dnde se guardan esos datos.
Jorge Rami Aguirre Madrid (Espaa) 2003
Seguridad Informtica y Criptografa. Tema 2: Calidad de la Informacin y Virus
Diapositiva 103
Hackers y crakers
Hacker:
Definicin inicial de los ingenieros del MIT que
hacan alardes de sus conocimientos en informtica.
Pirata Informtico.
Cracker:
Persona que intenta de forma ilegal romper la
seguridad de un sistema por diversin o inters.
No existe uniformidad de criterios en su clasificacin;
no obstante, su accin cada da se vuelve ms tcnica,
sofisticada y debemos implementar medidas para
proteger nuestra informacin ante tales ataques.
Jorge Rami Aguirre Madrid (Espaa) 2003
Seguridad Informtica y Criptografa. Tema 2: Calidad de la Informacin y Virus
Diapositiva 104
Puntos vulnerables en la red
Las empresas relacionadas con las Nuevas Tecnologas
de la Informacin NTIs hacen uso de varias tcnicas y
herramientas de redes para el intercambio de datos:
Transferencia de ficheros (ftp)
Transferencia de datos e informacin a travs de
Internet (http)
Conexiones remotas a mquinas y servidores
(telnet)
Todo esto presentar graves riesgos de ataques de
hackers y otros delincuentes informticos, pero ...
Jorge Rami Aguirre Madrid (Espaa) 2003
Seguridad Informtica y Criptografa. Tema 2: Calidad de la Informacin y Virus
Diapositiva 105
Dnde est el enemigo?
Por muy organizados que puedan estar
estos grupos de vndalos o delincuentes,
primero que nada hay que ponerse en el
lugar que nos corresponde y no caer en la
paranoia. Adems, debemos pensar que el
peor enemigo puede estar dentro de casa...
La solucin sigue siendo la misma: la
puesta en marcha de una adecuada
poltica de seguridad en la empresa.
Jorge Rami Aguirre Madrid (Espaa) 2003
Seguridad Informtica y Criptografa. Tema 2: Calidad de la Informacin y Virus
Diapositiva 106
Delitos informticos
Son acciones que vulneran la confidencialidad,
integridad y disponibilidad de la informacin.
Ataques a un sistema informtico:
Fraude Malversacin Robo
Sabotaje Espionaje Chantaje
Revelacin Mascarada Virus
Gusanos Caballos de Troya ... etc.
Veamos algunos
Jorge Rami Aguirre Madrid (Espaa) 2003
Seguridad Informtica y Criptografa. Tema 2: Calidad de la Informacin y Virus
Diapositiva 107
Fraude y sabotaje
Fraude
Acto deliberado de manipulacin de datos perjudicando
a una persona fsica o jurdica que sufre de esta forma
una prdida econmica. El autor del delito logra de esta
forma un beneficio normalmente econmico.
Sabotaje
Accin con la que se desea perjudicar a una empresa
entorpeciendo deliberadamente su marcha, averiando
sus equipos, herramientas, programas, etc. El autor no
logra normalmente con ello beneficios econmicos pero
pone en jaque mate a la organizacin.
Jorge Rami Aguirre Madrid (Espaa) 2003
Seguridad Informtica y Criptografa. Tema 2: Calidad de la Informacin y Virus
Diapositiva 108
Chantaje y mascarada
Chantaje
Accin que consiste en exigir una cantidad de dinero a
cambio de no dar a conocer informacin privilegiada o
confidencial y que puede afectar gravemente a la
empresa, por lo general a su imagen corporativa.
Mascarada
Utilizacin de una clave por una persona no autorizada
y que accede al sistema suplantando una identidad. De
esta forma el intruso se hace dueo de la informacin,
documentacin y datos de otros usuarios con los que
puede, por ejemplo, chantajear a la organizacin.
Jorge Rami Aguirre Madrid (Espaa) 2003
Seguridad Informtica y Criptografa. Tema 2: Calidad de la Informacin y Virus
Diapositiva 109
Virus y gusanos
Virus
Cdigo diseado para introducirse en un programa,
modificar o destruir datos. Se copia automticamente a
otros programas para seguir su ciclo de vida. Es comn
que se expanda a travs de plantillas, las macros de
aplicaciones y archivos ejecutables.
Gusanos
Virus que se activa y transmite a travs de la red. Tiene
como finalidad su multiplicacin hasta agotar el espacio
en disco o RAM. Suele ser uno de los ataques ms
dainos porque normalmente produce un colapso en la
red (p.e. el gusano de Internet de Robert Morris Jr.).
Jorge Rami Aguirre Madrid (Espaa) 2003
Seguridad Informtica y Criptografa. Tema 2: Calidad de la Informacin y Virus
Diapositiva 110
Caballos de Troya
Caballos de Troya
Virus que entra al ordenador y posteriormente acta de
forma similar a este hecho de la mitologa griega. As,
parece ser una cosa o programa inofensivo cuando en
realidad est haciendo otra y expandindose. Ejemplo:
el huevo de Pascua de Windows 95.
Y hay muchos ms delitos. Incluso aparecern nuevos delitos
y ataques a los sistemas informticos y redes que a fecha de
hoy no sabemos cmo sern ni qu vulnerabilidad atacarn...
Este enfrentamiento entre el bien y el mal es inevitable en
un sistema abierto ... y las comunicaciones hoy son as.
Jorge Rami Aguirre Madrid (Espaa) 2003
Seguridad Informtica y Criptografa. Tema 2: Calidad de la Informacin y Virus
Diapositiva 111
Virus informticos
Nota del autor
Las prximas diapositivas son una breve y elemental
introduccin al tema de los virus informticos, orientado adems
slo al mundo de los PCs y del llamado entorno Windows. No
pretende ser ni mucho menos un documento que trate este tema
con la profundidad que debera hacerse y se merece.
Mucha gente cataloga a ste como un tema menor, sin embargo
dentro de las empresas es uno de los mayores problemas con los
que se enfrentan los responsables de seguridad informtica.
Se incluye aqu este apartado como un factor ms a tener en
cuenta en cuanto a la calidad de la informacin que manejamos,
dado que sta puede verse afectada por este tipo de ataques tan
comunes hoy en da por ejemplo va e-mail.
Jorge Rami Aguirre Madrid (Espaa) 2003
Seguridad Informtica y Criptografa. Tema 2: Calidad de la Informacin y Virus
Diapositiva 112
Historia y tipos de virus
Primer ejemplo: John von Neuman (1949)
Primer virus: M. Gouglas de Bell Laboratories
crea el Core War en 1960.
Primeros ataques a PCs entre 1985 y 1987:
Virus Jerusalem y Brain.
Inofensivos (pelota, letras, etc.)
Slo molestan y entorpecen el trabajo pero no
destruyen informacin. Podran residir en el PC.
Malignos (Viernes 13, Melissa, Nimbda, etc.)
Destruyen los datos y afectan a la integridad y la
disponibilidad del sistema. Hay que eliminarnos.
Jorge Rami Aguirre Madrid (Espaa) 2003
Seguridad Informtica y Criptografa. Tema 2: Calidad de la Informacin y Virus
Diapositiva 113
Transmisin de un virus
Se transmiten slo mediante la ejecucin de un
programa. Esto es muy importante recordarlo.
El correo electrnico por definicin no puede
contener virus al ser slo texto. No obstante,
muchas veces contienen archivos aadidos o los
visualizadores ejecutan cdigo en el cliente de
correo del usuario y stos pueden tener incluido
un virus. Ah est el peligro.
El entorno web es mucho ms peligroso. Un
enlace puede lanzar un programa en Java u otro
lenguaje que se ejecute y afecte el disco duro.
Jorge Rami Aguirre Madrid (Espaa) 2003
Seguridad Informtica y Criptografa. Tema 2: Calidad de la Informacin y Virus
Diapositiva 114
Tipos de ataque de un virus
Estn aquellos que infectan a programas con
extensin exe, com y sys por ejemplo.
Residen en memoria al ejecutarse el husped y de
ah se propagan a otros archivos.
Y tambin aquellos que infectan el sistema y el
sector de arranque y tablas de entrada (reas
determinadas del disco).
Se instalan directamente all y por lo tanto residen
en memoria.
Jorge Rami Aguirre Madrid (Espaa) 2003
Seguridad Informtica y Criptografa. Tema 2: Calidad de la Informacin y Virus
Diapositiva 115
Algunas medidas bsicas de prevencin
Proteger los discos extrables con la pestaa, una
proteccin de tipo hardware muy elemental.
Instalar un antivirus y actualizarlo al menos una
vez al mes; recomendable cada 15 das.
Ejecutar el antivirus de vez en cuando al disco
duro (por ejemplo una vez al mes) y siempre a los
disquetes y archivos que se descargan de Internet.
Usar siempre software legal, con licencia.
Controlar el acceso de extraos al computador.
Jorge Rami Aguirre Madrid (Espaa) 2003
Seguridad Informtica y Criptografa. Tema 2: Calidad de la Informacin y Virus
Diapositiva 116
Qu hacer en caso de estar infectado?
Detener las conexiones remotas.
No mover el ratn ni activar el teclado.
Apagar el sistema y desconectarlo.
Arrancar con un disquete de arranque o emergencia
protegido y ejecutar luego un programa antivirus.
Hacer copia de seguridad de ficheros.
Formatear el disco duro a bajo nivel si no queda otra
solucin .
Instalar nuevamente el sistema operativo y restaurar
las copias de seguridad.
Fin del Tema 2
Jorge Rami Aguirre Madrid (Espaa) 2003
Seguridad Informtica y Criptografa. Tema 2: Calidad de la Informacin y Virus
Diapositiva 117
Cuestiones y ejercicios
1. Qu diferencia hay entre el concepto de informacin y su calidad
segn lo entienda una empresa o los estudios de ingeniera?
2. Por qu se dice que la informacin de una empresa es su activo
ms valioso? Compare este activo con el personal de la misma y
pngase en situaciones en las que ambos se pierden, qu situacin
podra ser es ms perjudicial para la continuidad de dicha empresa?
3. Como supervisores de seguridad hemos detectado que alguien est
realizando acciones no lcitas, por ejemplo copias no autorizadas de
informacin. Qu actitud debemos tomar?
4. Qu medidas seran la ms adecuadas de cara a minimizar el ataque
por virus en nuestra empresa?
5. Si deseamos que nuestra empresa est debidamente protegida tanto
fsica como lgicamente, qu debemos hacer?
Tema 3
Introduccin a la Seguridad Fsica
Seguridad Informtica y Criptografa
Ultima actualizacin: 03/03/03
Archivo con 37 diapositivas
Jorge Rami Aguirre
Universidad Politcnica de Madrid
Material Docente de
Libre Distribucin
v 3.1
Este archivo forma parte de un curso sobre Seguridad Informtica y Criptografa. Se autoriza la
reproduccin en computador e impresin en papel slo con fines docentes o personales, respetando
en todo caso los crditos del autor. Queda prohibida su venta, excepto a travs del Departamento de
Publicaciones de la Escuela Universitaria de Informtica, Universidad Politcnica de Madrid, Espaa.
Curso de Seguridad Informtica y Criptografa JRA
Jorge Rami Aguirre Madrid (Espaa) 2003
Seguridad Informtica y Criptografa. Tema 3: Introduccin a la Seguridad Fsica
Diapositiva 120
Nota del autor
El contenido de este archivo
corresponde slo a un primer
borrador en relacin con el
tema de la seguridad fsica.
Actualmente sta tiene una
importancia igual y en algunos casos superior a la
seguridad lgica. Los acontecimientos acaecidos a
comienzos de este siglo han puesto al descubierto
cun vulnerable puede ser un sistema ante un ataque;
por ello se han actualizado algunos temas pero este
documento sigue siendo un borrador de trabajo.
Jorge Rami Aguirre Madrid (Espaa) 2003
Seguridad Informtica y Criptografa. Tema 3: Introduccin a la Seguridad Fsica
Diapositiva 121
Seguridad Fsica
Los datos deben protegerse aplicando:
Seguridad Lgica
Uso de herramientas de proteccin de la informacin
en el mismo medio en el que se genera o transmite.
Protocolos de autenticacin entre cliente y servidor.
Aplicacin de normativas.
Seguridad Fsica
Procedimientos de proteccin fsica del sistema:
acceso personas, incendio, agua, terremotos, etc.
Medidas de prevencin de riesgos tanto fsicos como
lgicos a travs de una poltica de seguridad, planes de
contingencia, aplicacin de normativas, etc.
Jorge Rami Aguirre Madrid (Espaa) 2003
Seguridad Informtica y Criptografa. Tema 3: Introduccin a la Seguridad Fsica
Diapositiva 122
La seguridad Fsica en entornos de PCs
Anclajes a mesas de trabajo.
Cerraduras.
Tarjetas con alarma.
Etiquetas con adhesivos especiales.
Bloqueo de disquetera.
Protectores de teclado.
Tarjeta de control de acceso al hardware.
Suministro ininterrumpido de corriente.
Toma de tierra.
Eliminacin de la esttica... etc.
Temas a tener
en cuenta en un
entorno PC
Jorge Rami Aguirre Madrid (Espaa) 2003
Seguridad Informtica y Criptografa. Tema 3: Introduccin a la Seguridad Fsica
Diapositiva 123
Anlisis de riesgo: plan estratgico
Es el proceso de identificacin y evaluacin del
riesgo a sufrir un ataque y perder datos, tiempo y
horas de trabajo, comparndolo con el costo que
significara la prevencin de este suceso.
Su anlisis no slo nos lleva a establecer un nivel
adecuado de seguridad, sino que permite conocer
mejor el sistema que vamos a proteger.
Jorge Rami Aguirre Madrid (Espaa) 2003
Seguridad Informtica y Criptografa. Tema 3: Introduccin a la Seguridad Fsica
Diapositiva 124
Informacin del anlisis de riesgo
Informacin que se obtiene en un anlisis de riesgo:
Determinacin precisa de los recursos sensibles de la
organizacin.
Identificacin de las amenazas del sistema.
Identificacin de las vulnerabilidades especficas del
sistema.
Identificacin de posibles prdidas.
Identificacin de la probabilidad de ocurrencia de una
prdida.
Derivacin de contramedidas efectivas.
Identificacin de herramientas de seguridad.
Implementacin de un sistema de seguridad eficiente en
costes y tiempo.
Jorge Rami Aguirre Madrid (Espaa) 2003
Seguridad Informtica y Criptografa. Tema 3: Introduccin a la Seguridad Fsica
Diapositiva 125
Ecuacin bsica del anlisis de riesgo
B > P L ?
B: Carga o gasto que significa la prevencin
de una prdida especfica por vulnerabilidad.
P: Probabilidad de ocurrencia de esa prdida
especfica.
L: Impacto total de dicha prdida especfica.
Jorge Rami Aguirre Madrid (Espaa) 2003
Seguridad Informtica y Criptografa. Tema 3: Introduccin a la Seguridad Fsica
Diapositiva 126
Cundo y cunto invertir en seguridad?
Si B P L
Hay que implementar una medida de
prevencin.
Si B > P L
No es necesaria una medida de prevencin.
... al menos matemticamente. No obstante, siempre
puede ocurrir una desgracia que est fuera de todo
clculo. Por ejemplo, el ataque a las torres gemelas y
sus posteriores consecuencias informticas no estaba
contemplado en ningn plan contingencia...
Jorge Rami Aguirre Madrid (Espaa) 2003
Seguridad Informtica y Criptografa. Tema 3: Introduccin a la Seguridad Fsica
Diapositiva 127
Efectividad del coste de la medida
Las medidas y herramientas de control han de
tener menos coste que el valor de las posibles
prdidas y el impacto de stas si se produce el
riesgo temido.
Ley bsica: el costo del control ha de ser menor
que el activo que protege. Algo totalmente lgico
y que tanto los directivos como los responsables
de seguridad de la empresa debern estimar de
forma adecuada a su realidad.
Jorge Rami Aguirre Madrid (Espaa) 2003
Seguridad Informtica y Criptografa. Tema 3: Introduccin a la Seguridad Fsica
Diapositiva 128
El factor L en la ecuacin de riesgo
Factor L (en B P L)
El factor de impacto total L es difcil de evaluar.
Incluye daos a la informacin, a los equipos,
prdidas por reparacin, por volver a levantar el
sistema, prdidas por horas de trabajo, etc.
Siempre habr una parte subjetiva.
La prdida de datos puede llevar a una prdida de
oportunidades por el llamado efecto cascada.
En la organizacin debe existir una comisin
especializada interna o externa que sea capaz de
evaluar todas las posibles prdidas y cuantificarlas.
Jorge Rami Aguirre Madrid (Espaa) 2003
Seguridad Informtica y Criptografa. Tema 3: Introduccin a la Seguridad Fsica
Diapositiva 129
El factor P en la ecuacin de riesgo
Factor P (en B P L)
El factor P est relacionado con la determinacin
del impacto total L y depende del entorno en el
que est la posible prdida. Como este valor es
difcil de cuantificar, dicha probabilidad puede
asociarse a una tendencia o frecuencia conocida.
Conocido P para un L dado, se obtiene la
probabilidad de prdida relativa de la ocurrencia PL
que se comparar con B, el peso que supone
implantar la medida de prevencin respectiva.
Jorge Rami Aguirre Madrid (Espaa) 2003
Seguridad Informtica y Criptografa. Tema 3: Introduccin a la Seguridad Fsica
Diapositiva 130
El factor B en la ecuacin de riesgo
Factor B (en B P L)
Indica qu se requiere para prevenir una prdida.
Es la cantidad de dinero que vamos a disponer
para mitigar la posible prdida.
Ejemplo: la carga de prevencin para que un sistema
informtico minimice el riesgo de que sus servidores
sean atacados desde fuera incluye la instalacin de
software y hardware adecuado, un cortafuegos, un
sistema de deteccin de intrusos, una configuracin
de red segura, una poltica de seguimiento de accesos
y de passwords, personal tcnico cualificado, etc.
Todo ello importa una cantidad de dinero especfica.
Jorge Rami Aguirre Madrid (Espaa) 2003
Seguridad Informtica y Criptografa. Tema 3: Introduccin a la Seguridad Fsica
Diapositiva 131
Cuantificacin de la proteccin
B P L ?
Cunta proteccin es necesaria?
En nuestro ejemplo: qu configuracin de red usar,
en qu entorno trabajar, qu tipo de cortafuegos, etc.
Eso depender del novel de seguridad que nuestra
empresa desee o crea oportuno.
De qu forma nos protegeremos?
Una casa puede protegerse con puertas, cerraduras,
barras en ventanas, sistemas de alarmas, etc.
En un sistema informtico podemos aplicar medidas
fsicas, polticas de seguridad de accesos, planes de
contingencia y recuperacin, cortafuegos, cifrado de
la informacin, firmas, pasarelas seguras, etc.
Jorge Rami Aguirre Madrid (Espaa) 2003
Seguridad Informtica y Criptografa. Tema 3: Introduccin a la Seguridad Fsica
Diapositiva 132
Pasos en un anlisis de riesgos
1. Identificacin costo
posibles prdidas (L)
Identificar amenazas
3. Identificar posibles
acciones (gasto) y sus
implicaciones. (B)
Seleccionar acciones a
implementar.
Se
cierra
el
ciclo
B PL ?
2. Determinar susceptibilidad.
La probabilidad de prdida (P)
Jorge Rami Aguirre Madrid (Espaa) 2003
Seguridad Informtica y Criptografa. Tema 3: Introduccin a la Seguridad Fsica
Diapositiva 133
Algunas polticas de seguridad
Polticas administrativas
Procedimientos administrativos.
Polticas de control de acceso
Privilegios de acceso del usuario o programa.
Polticas de flujo de informacin
Normas bajo la cuales se comunican los
sujetos dentro del sistema.
Jorge Rami Aguirre Madrid (Espaa) 2003
Seguridad Informtica y Criptografa. Tema 3: Introduccin a la Seguridad Fsica
Diapositiva 134
Aspectos administrativos
Polticas administrativas
Se establecen aquellos procedimientos de
carcter administrativo en la organizacin
como por ejemplo en el desarrollo de
programas: modularidad en aplicaciones,
revisin sistemtica, etc.
Se establecen responsabilidades compartidas
por todos los usuarios, cada uno en su nivel.
Jorge Rami Aguirre Madrid (Espaa) 2003
Seguridad Informtica y Criptografa. Tema 3: Introduccin a la Seguridad Fsica
Diapositiva 135
Control de accesos
Polticas de control de acceso
Poltica de menor privilegio
Acceso estricto a objetos determinados, con
mnimos privilegios para los usuarios.
Poltica de comparticin
Acceso de mximo privilegio en el que cada
usuario puede acceder a todos los objetos.
Granularidad
Nmero de objetos accesibles. Se habla entonces
de granularidad gruesa y fina.
Jorge Rami Aguirre Madrid (Espaa) 2003
Seguridad Informtica y Criptografa. Tema 3: Introduccin a la Seguridad Fsica
Diapositiva 136
Control de flujo
Polticas de control de flujo
La informacin a la que se accede, se enva y
recibe por:
Canales claros o canales ocultos? Seguros o no?
Qu es lo que hay que potenciar?
La confidencialidad o la integridad?
La disponibilidad? ... El no repudio?
Segn cada organizacin y su entorno de trabajo y
servicios ofrecidos, habr diferencias. En algunos
sistemas primarn unos ms que otros, en funcin
de cun secreta es la informacin que procesan.
Jorge Rami Aguirre Madrid (Espaa) 2003
Seguridad Informtica y Criptografa. Tema 3: Introduccin a la Seguridad Fsica
Diapositiva 137
Modelos de seguridad
Modelo de Bell LaPadula (BLP)
Rgido. Confidencialidad y con autoridad.
Modelo de Take-Grant (TG)
Derechos especiales: tomar y otorgar.
Modelo de Clark-Wilson (CW)
Orientacin comercial: integridad.
Modelo de Goguen-Meseguer (GM)
No interferencia entre usuarios.
Modelo de Matriz de Accesos (MA)
Estados y transiciones entre estados
Tipo Graham-Dennig (GD)
Tipo Harrison-Ruzzo-Ullman (HRU)
Se definirn
brevemente
en prxima
diapositiva
Jorge Rami Aguirre Madrid (Espaa) 2003
Seguridad Informtica y Criptografa. Tema 3: Introduccin a la Seguridad Fsica
Diapositiva 138
Modelo de Bell LaPadula BLP
La escritura hacia abajo est prohibida.
La lectura hacia arriba est prohibida.
Es el llamado principio de tranquilidad.
No lectura hacia arriba Secreto mximo
usuario dado de alta
con un nivel secreto Secreto
No escritura hacia abajo No clasificado
Jorge Rami Aguirre Madrid (Espaa) 2003
Seguridad Informtica y Criptografa. Tema 3: Introduccin a la Seguridad Fsica
Diapositiva 139
Modelo de Take Grant TG
Se describe mediante grafos orientados:
el vrtice es un objeto o sujeto.
un arco es un derecho.
Se ocupa slo de aquellos derechos que
pueden ser transferidos.
Jorge Rami Aguirre Madrid (Espaa) 2003
Seguridad Informtica y Criptografa. Tema 3: Introduccin a la Seguridad Fsica
Diapositiva 140
Modelo de Clark Wilson CW
Basado en polticas de integridad
Elementos de datos restringidos.
sobre stos debe hacerse un chequeo de
consistencia.
Elementos de datos no restringidos.
Procedimientos de transformacin.
trata los dos elementos.
Procedimientos de verificacin de integridad.
Jorge Rami Aguirre Madrid (Espaa) 2003
Seguridad Informtica y Criptografa. Tema 3: Introduccin a la Seguridad Fsica
Diapositiva 141
Criterios y normativas de seguridad
Criterio de evaluacin TSEC
Trusted Computer System Evaluation Criteria, tambin
conocido como Orange Book.
Criterio de evaluacin ITSEC
Information Technology Security Evaluation Criteria.
Criterio de evaluacin CC
Common Criteria: incluye los dos anteriores.
Ley Orgnica de Proteccin de Datos LOPD (1999, Espaa)
Establece un conjunto de medidas de seguridad de debido
cumplimiento por parte de empresas y organismos.
Normativa internacional 17799
Desarrolla un protocolo de condiciones mnimas de seguridad
informtica de amplio espectro.
Jorge Rami Aguirre Madrid (Espaa) 2003
Seguridad Informtica y Criptografa. Tema 3: Introduccin a la Seguridad Fsica
Diapositiva 142
La ley de seguridad informtica en Espaa
Ley Orgnica de Proteccin de Datos LOPD se desarrolla en Espaa
en el ao 1999 y comienza a aplicarse ya en el siglo XXI.
Se crea una Agencia de Proteccin de datos APD que debe velar por
el cumplimiento de esta ley mediante la realizacin de auditoras, al
menos cada dos aos. La APD la forman 9 personas.
Se definen las funciones del Responsable de Fichero y del Encargado
de Tratamiento.
Las faltas se clasifican como leves, graves y muy graves con multas
de 60.000, 300.000 y 600.000 .
En el Real Decreto 994/1999 sobre Medidas de seguridad de los
ficheros automatizados que contengan datos de carcter personal se
definen las funciones del Responsable de Seguridad.
Establece un conjunto de procedimientos de obligado cumplimiento
de forma que adems de proteger la privacidad de las personas, se
cumplan los principios de la seguridad informtica fsica y lgica.
Jorge Rami Aguirre Madrid (Espaa) 2003
Seguridad Informtica y Criptografa. Tema 3: Introduccin a la Seguridad Fsica
Diapositiva 143
La normativa 17799
Cdigo de buenas prcticas para la Gestin de la Seguridad de la
Informacin: PNE-ISO/IEC 17799 (Proyecto de Norma Espaola)
Antecedentes
Introduccin
Objeto y campo de la aplicacin
Trminos y definiciones
Poltica de seguridad
Aspectos organizativos para la seguridad
Clasificacin y control de los archivos
Seguridad ligada al personal
Seguridad fsica y del entorno
Gestin de comunicaciones y operaciones
Control de accesos
Desarrollo y mantenimiento de sistemas
Gestin de continuidad del negocio
Conformidad
En 70 pginas y diez
apartados, presenta unas
normas, recomendaciones y
criterios bsicos para
establecer unas polticas de
seguridad. stas van desde
los conceptos de seguridad
fsica hasta los de seguridad
lgica. Parte de la norma
elaborada por la British
Standards Institution BSI,
adoptada por International
Standards Organization ISO
y la International Electronic
Commission IEC.
Jorge Rami Aguirre Madrid (Espaa) 2003
Seguridad Informtica y Criptografa. Tema 3: Introduccin a la Seguridad Fsica
Diapositiva 144
Planes de contingencia
Un Plan de Contingencia consiste en un anlisis
pormenorizado de las reas que componen nuestra
organizacin que nos servir para establecer una poltica de
recuperacin ante un desastre.
Es un conjunto de datos estratgicos de la empresa y que
se plasma en un documento con el fin de protegerse ante
eventualidades.
Adems de aumentar su seguridad, con un plan estratgico la
empresa tambin gana en el conocimiento de fortalezas y
debilidades.
Pero si no lo hace, se expone a sufrir una prdida irreparable
mucho ms costosa que la implantacin de este plan.
Jorge Rami Aguirre Madrid (Espaa) 2003
Seguridad Informtica y Criptografa. Tema 3: Introduccin a la Seguridad Fsica
Diapositiva 145
Desastres naturales y su prevencin
Desastres naturales
Huracn
Tormenta
Inundacin
Tornado
Vendaval
Incendio
Otros
Medidas prevencin
Emplazamientos
adecuados
Proteccin fachadas,
ventanas, puertas
Jorge Rami Aguirre Madrid (Espaa) 2003
Seguridad Informtica y Criptografa. Tema 3: Introduccin a la Seguridad Fsica
Diapositiva 146
Vandalismo informtico y su prevencin
Terrorismo
Sabotaje
Robo
Virus
Programas malignos
Medidas de prevencin
Fortificacin entradas
Guardia Jurado
Patrullas
Circuito cerrado TV
Control de accesos
Proteccin de software y
hardware con antivirus,
cortafuegos, etc.
Jorge Rami Aguirre Madrid (Espaa) 2003
Seguridad Informtica y Criptografa. Tema 3: Introduccin a la Seguridad Fsica
Diapositiva 147
Amenazas del agua y su prevencin
Amenazas
Inundaciones por
causas propias de la
empresa
Inundaciones por
causas ajenas
Pequeos incidentes
personales (botella de
agua, taza con caf)
Medidas prevencin
Revisar conductos de
agua.
Localizar la sala con los
equipos ms caros en un
sitio libre de estos
problemas.
Instalar sistemas de
drenaje de emergencia.
Concienciar empleados.
Jorge Rami Aguirre Madrid (Espaa) 2003
Seguridad Informtica y Criptografa. Tema 3: Introduccin a la Seguridad Fsica
Diapositiva 148
Amenazas del fuego y su prevencin
Amenazas
Una mala instalacin
elctrica.
descuidos personales
como fumar en la sala
de ordenadores.
Papeleras mal ubicadas
en la que se tira un
cigarrillo no apagado.
Vulnerabilidades del
sistema por humo.
Medidas prevencin
Detector humo y calor.
Materiales ignfugos.
Almacn de papel
separado de mquinas.
Estado del falso suelo.
Extintores revisados.
Es la amenaza ms
temida por su rpido
poder destructor.
Jorge Rami Aguirre Madrid (Espaa) 2003
Seguridad Informtica y Criptografa. Tema 3: Introduccin a la Seguridad Fsica
Diapositiva 149
Qu sucede si se produce una catstrofe?
Las empresas dependen hoy en da de los equipos
informticos y de todos los datos que hay all
almacenados (nminas, clientes, facturas, ...).
Dependen tambin cada vez ms de las
comunicaciones a travs de redes de datos.
Si falla el sistema informtico y ste no puede
recuperarse, la empresa puede desaparecer porque
no tiene tiempo de salir nuevamente al mercado
con ciertas expectativas de xito, aunque conserve
a todo su personal.
Jorge Rami Aguirre Madrid (Espaa) 2003
Seguridad Informtica y Criptografa. Tema 3: Introduccin a la Seguridad Fsica
Diapositiva 150
Tiempos de recuperacin ante desastres
Perodo mximo de paro de una empresa
sin poner en peligro su supervivencia:
Sector Seguros: 5,6 das
Sector Fabricacin: 4,9 das
Sector Industrial: 4,8 das
Sector Distribucin: 3,3 das
Sector Financiero: 2,0 das
Ref. Estudio de la Universidad de Minnesota (1996)
Jorge Rami Aguirre Madrid (Espaa) 2003
Seguridad Informtica y Criptografa. Tema 3: Introduccin a la Seguridad Fsica
Diapositiva 151
Prdidas por no contar con un plan
Prdida de clientes.
Prdida de imagen.
Prdida de ingresos por beneficios.
Prdida de ingresos por ventas y cobros.
Prdida de ingresos por produccin.
Prdida de competitividad.
Prdida de credibilidad en el sector.
Jorge Rami Aguirre Madrid (Espaa) 2003
Seguridad Informtica y Criptografa. Tema 3: Introduccin a la Seguridad Fsica
Diapositiva 152
Implantacin de medidas bsicas
Plan de emergencia
Vidas, heridos, activos, evacuacin personal.
Inventariar recursos siniestrados.
Evaluar el coste de la inactividad.
Plan de recuperacin
Acciones tendentes a volver a la situacin que
exista antes del desastre.
Jorge Rami Aguirre Madrid (Espaa) 2003
Seguridad Informtica y Criptografa. Tema 3: Introduccin a la Seguridad Fsica
Diapositiva 153
Plan de continuidad
Instalaciones alternativas
Oficina de servicios propia.
Acuerdo con empresa vendedora de HW y SW.
Acuerdo recproco entre dos o ms empresas.
Arranque en fro; sala vaca propia.
Arranque en caliente: centro equipado.
Sistema Up Start: caravana, unidad mvil.
Sistema Hot Start: centro gemelo.
Fin del Tema 3
Jorge Rami Aguirre Madrid (Espaa) 2003
Seguridad Informtica y Criptografa. Tema 3: Introduccin a la Seguridad Fsica
Diapositiva 154
Cuestiones y ejercicios (1 de 2)
1. Qu es y qu significa hacer un anlisis de riesgos?
2. Explique el sentido de las ecuaciones B > PL y B PL.
3. Tras un estudio, obtenemos B > PL, podemos estar totalmente
tranquilos al no utilizar medida alguna de prevencin?
4. Explique qu significan los factores L y P en la ecuacin B > PL.
5. Cules son los pasos a seguir en un anlisis de riesgos de acuerdo a
los factores de la ecuacin de B > PL?
6. En algunos sistemas de gestin de informacin a veces prima ms el
elemento confidencialidad, en cambio en otros ms el de integridad.
D algunos ejemplos en que pueda cumplirse al menos en parte este
escenario. Qu opina respecto a una transaccin electrnica?
7. Comente el modelo de seguridad de Bell Lapadula. Por qu se le
llama el modelo de la tranquilidad?
Jorge Rami Aguirre Madrid (Espaa) 2003
Seguridad Informtica y Criptografa. Tema 3: Introduccin a la Seguridad Fsica
Diapositiva 155
Cuestiones y ejercicios (2 de 2)
8. Ud. es el responsable de seguridad y detecta que un empleado est
robando informacin confidencial, cmo reaccionara?
9. Cules pueden ser las prdidas en una empresa si no se cuenta con
un adecuado Plan de Contingencia y sucede un desastre?
10. Qu es un Plan de Contingencia y por qu es importante?
11. Nuestra empresa est a medias entre el rubro distribucin y el de las
finanzas. Resulta estratgico tener aqu un Plan de Contingencia?
12. Qu soluciones tenemos para que un banco no se vea afectado por
un desastre y pueda seguir trabajando con sus clientes con un tiempo
de recuperacin bajo o mnimo? Cmo sera su coste?
13. Se pueden prever situaciones extremas como lo acontecido con las
torres gemelas? En que tipo de empresas o instituciones no deben
descartarse estos extremos? En mi empresa que vende pasteles?
Tema 4
Teora de la Informacin
Seguridad Informtica y Criptografa
Ultima actualizacin: 03/03/03
Archivo con 57 diapositivas
Jorge Rami Aguirre
Universidad Politcnica de Madrid
Material Docente de
Libre Distribucin
v 3.1
Este archivo forma parte de un curso sobre Seguridad Informtica y Criptografa. Se autoriza la
reproduccin en computador e impresin en papel slo con fines docentes o personales, respetando
en todo caso los crditos del autor. Queda prohibida su venta, excepto a travs del Departamento de
Publicaciones de la Escuela Universitaria de Informtica, Universidad Politcnica de Madrid, Espaa.
Curso de Seguridad Informtica y Criptografa JRA
Jorge Rami Aguirre Madrid (Espaa) 2003
Seguridad Informtica y Criptografa. Tema 4: Teora de la Informacin
Diapositiva 158
Fundamentos de la Seguridad Informtica
Los pilares sobre los que descansa toda la teora
asociada a los criptosistemas son tres:
La teora de la informacin
Estudio de la cantidad de informacin y entropa.
La teora de los nmeros
Estudio de las matemticas discretas y cuerpos finitos.
La teora de la complejidad de los algoritmos
Clasificacin de los problemas.
Lo veremos en ...
este archivo
archivo SItema05
archivo SItema06
Jorge Rami Aguirre Madrid (Espaa) 2003
Seguridad Informtica y Criptografa. Tema 4: Teora de la Informacin
Diapositiva 159
Teora de la informacin
Informacin:
Conjunto de datos o mensajes inteligibles creados
con un lenguaje de representacin y que debemos
proteger ante las amenazas del entorno, durante
su transmisin o almacenamiento, usando entre
otras cosas, las tcnicas criptogrficas.
La teora de la informacin mide la
cantidad de informacin que
contiene un mensaje a travs del
nmero medio de bits necesario para
codificar todos los posibles mensajes
con un codificador ptimo.
Veremos estas
dos definiciones
ms adelante.
Jorge Rami Aguirre Madrid (Espaa) 2003
Seguridad Informtica y Criptografa. Tema 4: Teora de la Informacin
Diapositiva 160
Representacin de la informacin
Puede ser numrica, alfabtica, simblica, por lenguaje.
Ejemplo: 24/01/03 24-01-03 24-1-03 24/01/2003
01/24/03 01-24-03 1-24-03 01-24-2003 ...
- Todos son el da 24 de enero del ao 2003.
Vitaminas: B
12
, C, ...
Grupo sanguneo: A2 Rh+ ...
Elementos: Fe, Si, Hg ...
Compuestos qumicos: H
2
O, CO
2
...
Ms comn Lenguaje con cdigo: Hoy hace calor
Qu informacin nos
entrega el mensaje
Hoy hace calor?
Qu informacin nos
entrega el mensaje
Hoy hace calor?
Jorge Rami Aguirre Madrid (Espaa) 2003
Seguridad Informtica y Criptografa. Tema 4: Teora de la Informacin
Diapositiva 161
Pero qu es la informacin?
Veremos qu informacin nos entrega un mensaje
dependiendo del contexto en que nos encontremos.
Esto puede ser:
a) En funcin de la extensin del mensaje recibido.
b) En funcin de la utilidad del mensaje recibido.
c) En funcin de la sorpresa del mensaje recibido.
d) Dependiendo del entorno de esa sorpresa.
e) En funcin de la probabilidad de recibir un mensaje.
Este es el entorno del estudio realizado por Claude
Shannon orientado a la ingeniera y que aqu nos interesa.
Jorge Rami Aguirre Madrid (Espaa) 2003
Seguridad Informtica y Criptografa. Tema 4: Teora de la Informacin
Diapositiva 162
Cantidad de informacin (caso 1)
En funcin de la extensin del mensaje
Ante una pregunta cualquiera, una respuesta concreta y
extensa nos entregar mayor informacin sobre el tema
en particular, y diremos que estamos ante una mayor
cantidad de informacin.
Pregunta: Hace calor all? (una playa en particular)
Respuesta 1: S, hace mucho calor.
Respuesta 2: Cuando no sopla el viento, el calor all es
inaguantable pues supera los 42 grados a la sombra.
Dnde hay una mayor cantidad de informacin?
Respuesta 2: Cuando no sopla el viento, el calor all es
inaguantable pues supera los 42 grados a la sombra.
Jorge Rami Aguirre Madrid (Espaa) 2003
Seguridad Informtica y Criptografa. Tema 4: Teora de la Informacin
Diapositiva 163
Cantidad de informacin (caso 2)
En funcin de la utilidad del mensaje
Ante una pregunta cualquiera, una respuesta ms til y
clara nos dejar con la sensacin de haber recibido una
mayor cantidad de informacin.
Pregunta: Hace calor all? (una playa en particular)
Respuesta 1: S, sobre 30 grados.
Respuesta 2: Si no hay viento del sur y el mar est en
calma, es normal que la temperatura suba bastante.
Respuesta 1: S, sobre 30 grados.
Dnde hay una mayor cantidad de informacin?
Jorge Rami Aguirre Madrid (Espaa) 2003
Seguridad Informtica y Criptografa. Tema 4: Teora de la Informacin
Diapositiva 164
Cantidad de informacin (caso 3)
En funcin de la sorpresa del mensaje
Ante una pregunta cualquiera, una respuesta ms
inesperada y sorprendente, nos dar la sensacin de
contener una mayor cantidad de informacin.
Pregunta: Hace calor all? (ahora Finlandia en otoo)
Respuesta 1: S, muchsimo. Es insoportable.
Respuesta 2: En esta poca del ao, la temperatura es
ms suave y el tiempo muy agradable.
Respuesta 1: S, muchsimo. Es insoportable.
Dnde hay una mayor cantidad de informacin?
Jorge Rami Aguirre Madrid (Espaa) 2003
Seguridad Informtica y Criptografa. Tema 4: Teora de la Informacin
Diapositiva 165
Cantidad de informacin (caso 4)
Dependencia del entorno (sorpresa)
Ante una pregunta cualquiera, una respuesta inesperada
y sorprendente en el entorno, nos dar la sensacin de
contener una mayor cantidad de informacin.
Pregunta: Hace calor all?
(ahora las mismas respuestas hablan de la temperatura en un horno)
Respuesta 1: S, muchsimo. Es insoportable.
Respuesta 2: En esta poca del ao, la temperatura es
ms suave y el tiempo muy agradable.
Respuesta 2: En esta poca del ao, la temperatura es
ms suave y el tiempo muy agradable. ?
Dnde hay una mayor cantidad de informacin?
Jorge Rami Aguirre Madrid (Espaa) 2003
Seguridad Informtica y Criptografa. Tema 4: Teora de la Informacin
Diapositiva 166
Cantidad de informacin (caso 5)
En funcin de la probabilidad de recibir un mensaje
Este enfoque probabilstico es el que nos interesar en
cuanto a la definicin de Cantidad de Informacin.
Dnde le da alegra a su cuerpo Macarena?
Respuesta 1: En un pas de Europa.
Respuesta 2: En una ciudad de Espaa.
Respuesta 3: En el nmero 7 de la calle de la Sierpes en
Sevilla, Espaa.
Respuesta 3: En el nmero 7 de la calle de la Sierpes en
Sevilla, Espaa.
Por qu?
Dnde hay una mayor cantidad de informacin?
Jorge Rami Aguirre Madrid (Espaa) 2003
Seguridad Informtica y Criptografa. Tema 4: Teora de la Informacin
Diapositiva 167
Incertidumbre e informacin
Ante varios mensajes posibles, en principio
todos equiprobables, aquel que tenga una
menor probabilidad ser el que contenga
una mayor cantidad de informacin.
En el ejemplo anterior:
Al ser ms extenso el nmero de calles y sus nmeros
en una ciudad que el nmero de ciudades en Espaa
y esto ltimo mayor que los pases en Europa, la
ltima respuesta tendr una mayor incertidumbre. Si
suponemos todos los estados equiprobables, entonces
la cantidad de informacin de la respuesta tercera
ser mayor que las dems.
Jorge Rami Aguirre Madrid (Espaa) 2003
Seguridad Informtica y Criptografa. Tema 4: Teora de la Informacin
Diapositiva 168
Concepto de variable aleatoria
Sea X una variable aleatoria con n estados posibles con
X = x
i
una ocurrencia isima:
X = {x
1
, x
2
, x
3
, ..., x
n-1
, x
n
}
p
1
= p(x
1
), p
2
= p(x
2
), ..., p
n
= p(x
n
)
Como:
0 p
i
1 para i = 1, 2, ..., n
Entonces:
n
p
i
= 1
i = 1
La probabilidad de que ocurra p
1
o
p
2
o p
3
, etc. ser siempre la unidad
porque seguro ser uno de ellos.
Jorge Rami Aguirre Madrid (Espaa) 2003
Seguridad Informtica y Criptografa. Tema 4: Teora de la Informacin
Diapositiva 169
Definicin logartmica
Definiremos c
i
a la cantidad de informacin del
estado i, como el logaritmo en base dos de la
probabilidad de que ocurra el estado isimo.
c
i
= - log
2
(p
i
)
- Logaritmo: p(x
i
) = 1 no hay incertidumbre: c
i
= 0
p(x
i
) = 0 mxima incertidumbre: c
i

- Signo: p(x
i
) < 1 log p(x
i
) ser negativo
- Base: Fenmeno binario dos estados (bit)
1
c
i
p
i
0
0
Jorge Rami Aguirre Madrid (Espaa) 2003
Seguridad Informtica y Criptografa. Tema 4: Teora de la Informacin
Diapositiva 170
Grado de indeterminacin
Combinacin n 1 Combinacin n 5
Combinacin n 2 Combinacin n 6
Combinacin n 3 Combinacin n 7
Combinacin n 4 Combinacin n 8
Qu cantidad de informacin tiene cada uno de los estados?
Esta ser la combinacin elegida
Grado de indeterminacin previo
Grado de indeterminacin posterior
c
i
=
Si hay equiprobabilidad
entonces p(x
i
) = 1/8
Ejemplo del mago: En una bolsa hay un crculo, un
cuadrado y un tringulo: negros o blancos.
Jorge Rami Aguirre Madrid (Espaa) 2003
Seguridad Informtica y Criptografa. Tema 4: Teora de la Informacin
Diapositiva 171
Solucin al ejemplo del mago
Incertidumbre inicial I
i
= 8
Daremos algunas pistas :
Las figuras no son del mismo color: I
i
baja de 8 a 6 al
descartarse las combinaciones 1 y 8.
El crculo es blanco: I
i
baja de 6 a 3 (descarte 5, 6 y 7).
Hay dos figuras blancas: I
i
baja de 3 a 2 (descarte 4).
El cuadrado es negro: I
i
baja de 2 a 1 (descarte 2.)
Veamos esto ahora
matemticamente ...
Combinacin n 1 Combinacin n 5
Combinacin n 2 Combinacin n 6
Combinacin n 3 Combinacin n 7
Combinacin n 4 Combinacin n 8
Se acaba la incertidumbre pues la solucin es la combinacin 3.
Los 8 estados sern equiprobables: p(x
i
) = 1/8
Jorge Rami Aguirre Madrid (Espaa) 2003
Seguridad Informtica y Criptografa. Tema 4: Teora de la Informacin
Diapositiva 172
Solucin matemtica al ejemplo del mago
Las figuras no son del mismo color. I
i
baja de 8 a 6:
c
i1
= log (8/6) = log 8 - log 6
El crculo es blanco. I
i
baja de 6 a 3:
c
i2
= log (6/3) = log 6 - log 3
Hay dos figuras blancas. I
i
baja de 3 a 2:
c
i3
= log (3/2) = log 3 - log 2
El cuadrado es negro. I
i
baja de 2 a 1:
c
i4
= log (2/1) = log 2 - log 1
Todas las magnitudes se pueden sumar como escalares:
c
i
= c
i1
+ c
i2
+ c
i3
+ c
i4
= log 8 - log 1 = log 8
Jorge Rami Aguirre Madrid (Espaa) 2003
Seguridad Informtica y Criptografa. Tema 4: Teora de la Informacin
Diapositiva 173
Base del logaritmo
Sean: I
i
la indeterminacin inicial
I
f
la indeterminacin final
c
i
= log (I
i
/ I
f
) = log I
i
- log I
f
La cantidad de informacin tiene como unidad de medida la
de un fenmeno de slo dos estados, un fenmeno binario.
Luego:
c
i
= log
b
(2/1) = log
b
2 - log
b
1
Si log
b
2 debe ser igual a 1 entonces la base b = 2.
Precisamente a esta unidad se le llama bit (binary digit)
Ejemplo anterior: c
i
= log
2
8 = 3 Slo 3 preguntas!
Jorge Rami Aguirre Madrid (Espaa) 2003
Seguridad Informtica y Criptografa. Tema 4: Teora de la Informacin
Diapositiva 174
Con slo tres preguntas...
Con slo tres preguntas ms o menos inteligentes
podemos pasar de la incertidumbre total a la certeza:
Pregunta 1: Est entre la opcin 1 y la 4? S
Pregunta 2: Est entre la opcin 1 y la 2? No
Pregunta 3: Es la opcin 4? No Se acaba la indeterminacin
Combinacin n 1 Combinacin n 5
Combinacin n 2 Combinacin n 6
Combinacin n 3 Combinacin n 7
Combinacin n 4 Combinacin n 8
Jorge Rami Aguirre Madrid (Espaa) 2003
Seguridad Informtica y Criptografa. Tema 4: Teora de la Informacin
Diapositiva 175
Entropa de los mensajes
Si un fenmeno tiene un grado de indeterminacin
k y sus estados son equiprobables, la probabilidad
p de que se d uno de esos estados ser 1/k. Luego:
c
i
= log
2
(k/1) = log
2
[1/(1/k)] = - log
2
p
Si ahora cada uno de estos estados tiene una
probabilidad distinta p
i
, la entropa H ser igual a
la suma ponderada de la cantidad de informacin:
H = - p
1
log
2
p
1
- p
2
log
2
p
2
- ... - p
k
log
2
p
k
k
H = - p
i
log
2
p
i
i = 1
Ecuacin no inmediata
Jorge Rami Aguirre Madrid (Espaa) 2003
Seguridad Informtica y Criptografa. Tema 4: Teora de la Informacin
Diapositiva 176
Definicin de entropa
La entropa de un mensaje X, que se representa por H(X),
es el valor medio ponderado de la cantidad de informacin
de los diversos estados del mensaje.
Es una medida de la incertidumbre media acerca de una
variable aleatoria y el nmero de bits de informacin.
k
H(X) = - p(x
i
) log
2
p(x
i
)
i = 1
El concepto de incertidumbre en H puede aceptarse. Lo que
llama la atencin es lo del nmero de bits de informacin.
Esto lo
veremos ms
adelante
Jorge Rami Aguirre Madrid (Espaa) 2003
Seguridad Informtica y Criptografa. Tema 4: Teora de la Informacin
Diapositiva 177
Propiedades de la entropa
a) La entropa es no negativa y se anula si y slo si un estado de la
variable es igual a 1 y el resto 0 (demostracin sencilla).
b) La entropa es mxima, mayor incertidumbre del mensaje,
cuando todos los valores posibles de la variable X son
equiprobables (empricamente fcil; demostracin no directa).
Si hay n estados equiprobables, entonces p
i
= 1/n.
Luego:
H(X) = - p
i
log
2
p
i
= - n(1/n) log
2
(1/n) = - (log
2
1 - log
2
n)
i
H(X)
mx
= log
2
n
Jorge Rami Aguirre Madrid (Espaa) 2003
Seguridad Informtica y Criptografa. Tema 4: Teora de la Informacin
Diapositiva 178
Codificador ptimo
Nos falta encontrar el segundo trmino pendiente en la
definicin de cantidad de informacin: codificador ptimo.
Introduciendo el signo negativo dentro del logaritmo en la
expresin de la entropa, sta nos quedar como:
Veamos un ejemplo
de codificacin
H(X) = p(x) log
2
[1/p(x)]
i
La expresin log
2
[1/p(x)] representa el nmero necesario de
bits para codificar el mensaje X en un codificador ptimo.
Codificador ptimo es aquel que para codificar un
mensaje X usa el menor nmero posible de bits.
Jorge Rami Aguirre Madrid (Espaa) 2003
Seguridad Informtica y Criptografa. Tema 4: Teora de la Informacin
Diapositiva 179
Codificacin con el mtodo de Huffman
Letra Frecuencia Ocurrencias
E 1 vez 3 6 9 15
I 2 veces
A 3 veces I E A M
3 veces I E A
M 6 veces I E A
I E
Cdigo ptimo:
Mensaje: MI MAMA ME MIMA
Creacin del rbol de
frecuencias observadas
M = 1 = 01 A = 000 I = 0010 E = 0011
Mensaje: 1 0010 01 1 000 1 000 01 1 0011 01 1 0010 1 000 (33 bits)
Pregunta: Con cuntos bits codificara si se usara ASCII? Saque conclusiones.
Jorge Rami Aguirre Madrid (Espaa) 2003
Seguridad Informtica y Criptografa. Tema 4: Teora de la Informacin
Diapositiva 180
Nmero necesario de bits y entropa
Para el mensaje M = M
1
M
2
M
1
M
1
M
3
M
1
M
2
M
3
de 8 caracteres se pide
calcular el nmero de bits ptimo de codificacin:
Solucin:
p(M
1
) = 0.5; p(M
2
) = 0.25; p(M
3
) = 0.25; y obviamente p(M
i
) = 1.0.
Para M
1
: log
2
[1/ P(M
1
)] = log
2
2 = 1 (necesitamos 1 bit)
Para M
2
: log
2
[1/ P(M
2
)] = log
2
4 = 2 (necesitamos 2 bits)
Para M
3
: log
2
[1/ P(M
3
)] = log
2
4 = 2 (necesitamos 2 bits)
Luego si M
1
se codifica como 0, M
2
como 10 y M
3
como 11, el mensaje
M se codificar como: 0 10 0 0 11 0 10 11, es decir se transmiten 12 bits.
Si calcula la entropa de M obtendr H(M) = 1,5 y al mismo valor se llega
con el concepto de nmero medio de bits: se ha usado 12 bits para
codificar un mensaje M de 8 elementos 12/8 = 1,5 bits por elemento.
Jorge Rami Aguirre Madrid (Espaa) 2003
Seguridad Informtica y Criptografa. Tema 4: Teora de la Informacin
Diapositiva 181
Entropa condicional: equivocacin de X
Si existe una segunda
variable Y que influya
sobre X, esto nos
entregar importante
informacin adicional.
H(X/Y) = - p
(x,y)
log
2
p
(x,y)
x,y
Donde p(x,y) = p(y)p(x/y) y la
relacin p(x/y) es la probabilidad
de que se obtenga un estado X
conocido el valor de Y.
El resultado ms
interesante es que...
La entropa se
reduce: hay ms
orden y menos
incertidumbre.
La entropa se
reduce: hay ms
orden y menos
incertidumbre.
Luego:
H(X/Y) = - p
(y)
p
(x/y)
log
2
p
(x/y)
y x
Jorge Rami Aguirre Madrid (Espaa) 2003
Seguridad Informtica y Criptografa. Tema 4: Teora de la Informacin
Diapositiva 182
Ejemplo de entropa condicional
Sea X = {x
1
, x
2
, x
3
, x
4
} con p(x
i
) = 0.25
Sea ahora Y = {y
1
, y
2
, y
3
} con p(y
1
) = 0.5; p(y
2
) = 0.25; p(y
3
) = 0.25
Luego H(X) = 4 log
2
4 = 2.0 y H(Y) = 2 log
2
4 + log
2
2 = 1.5
Adems hay las siguientes dependencias entre X e Y:
Si Y = y
1
X = x
1
o x
2
o x
3
o x
4
(cualquiera con igual probabilidad)
Si Y = y
2
X = x
2
o x
3
(cualquiera con igual probabilidad)
Si Y = y
3
X = x
3
o x
4
(cualquiera con igual probabilidad)
y=3 x=4
Como H(X/Y) = - p
(y)
p
(x/y)
log
2
p
(x/y)
y=1 x=1
H(X/Y) = - p(y
1
)[p(x
1
/y
1
)log
2
p(x
1
/y
1
) + p(x
2
/y
1
)log
2
p(x
2
/y
1
) + p(x
3
/y
1
)log
2
p(x
3
/y
1
) + p(x
4
/y
1
)log
2
p(x
4
/y
1
)]
- p(y
2
)[p(x
1
/y
2
)log
2
p(x
1
/y
2
) + p(x
2
/y
2
)log
2
p(x
2
/y
2
) + p(x
3
/y
2
)log
2
p(x
3
/y
2
) + p(x
4
/y
2
)log
2
p(x
4
/y
2
)]
- p(y
3
)[p(x
1
/y
3
)log
2
p(x
1
/y
3
) + p(x
2
/y
3
)log
2
p(x
2
/y
3
) + p(x
3
/y
3
)log
2
p(x
3
/y
3
) + p(x
4
/y
3
)log
2
p(x
4
/y
3
)]
Calculando, se obtiene H(X/Y) = 1.0 + 0.25 + 0.25 = 1.5. La entropa de
X ha bajado en medio bit con el conocimiento de su relacin con Y.
Jorge Rami Aguirre Madrid (Espaa) 2003
Seguridad Informtica y Criptografa. Tema 4: Teora de la Informacin
Diapositiva 183
Importancia de la entropa condicional
Equivocacin de la clave k
Cul es la probabilidad de
que a un criptograma C le
corresponda una cifra con
una clave k?
H(K/C) = - p
(c)
p
(k/c)
log
2
p
(k/c)
Servir como un parmetro para la evaluacin de la fortaleza
de un criptosistema segn equivocacin de clave y mensaje.
c k
Equivocacin del mensaje M
Cul es la probabilidad de
que a un criptograma C le
corresponda un mensaje en
claro M?
H(M/C) = - p
(c)
p
(m/c)
log
2
p
(m/c)
c m
Jorge Rami Aguirre Madrid (Espaa) 2003
Seguridad Informtica y Criptografa. Tema 4: Teora de la Informacin
Diapositiva 184
Ratio r del lenguaje
Ratio r
Es el nmero de bits de informacin en cada carcter
para mensajes con una longitud igual a N caracteres.
Luego, segn la definicin de entropa, se tiene:
r = H(X)/N (bits/letra)
Si codificramos un mensaje letra a letra suponiendo
adems equiprobabilidad entre las letras, se obtiene la
ratio absoluta del lenguaje, R:
R = H(X)
castellano = 27 letras
R
castellano
= log
2
n = log
2
27 = 4.75 (bits/letra)
Jorge Rami Aguirre Madrid (Espaa) 2003
Seguridad Informtica y Criptografa. Tema 4: Teora de la Informacin
Diapositiva 185
Ratio verdadera del lenguaje
Ratio verdadera
- Como las letras que aparecen en un texto no tienen
igual probabilidad, su frecuencia de aparicin es
distinta, los lenguajes est muy estructurados, hay
bloques de dos palabras (digramas) caractersticos,
trigramas, poligramas, etc., la ratio baja mucho...
1.2 < r < 1.5
- A este valor se llega codificando los mensajes con
monogramas, digramas, trigramas, etc.
Jorge Rami Aguirre Madrid (Espaa) 2003
Seguridad Informtica y Criptografa. Tema 4: Teora de la Informacin
Diapositiva 186
Significado de la ratio del lenguaje
Qu significa esto?
Si un alfabeto consta de L elementos existirn 2
RN
mensajes posibles de longitud N, la entropa mxima
ser H(X)
mx
= log
2
L, y slo habr 2
rN
mensajes que
tengan sentido.
Importante: No significa que podamos codificar
todos los mensajes de 27 caracteres con 2 bits (esto
sera imposible). Significa que la informacin que
contiene cada letra es tan slo de 1.5 bits.
Veamos un ejemplo
Jorge Rami Aguirre Madrid (Espaa) 2003
Seguridad Informtica y Criptografa. Tema 4: Teora de la Informacin
Diapositiva 187
Ejemplo de ratio del lenguaje
Un subalfabeto del castellano mdulo 27 consta de 5 caracteres
A, E, O, S, T todos ellos equiprobables, lo que puede aceptarse
como representativo del lenguaje y ms o menos cierto. Cuntos
mensaje de longitud 4 existen y cuntos con sentido?
Solucin:
R = log
2
5 = 2,3219, luego existirn 2
R4
= 2
2.32194
= 625 = 5
4
Como 1.2 < r < 1.5 entonces cabe esperar x mensajes con sentido
de longitud 4 del orden: 2
1.24
< x < 2
1.54
es decir 27 < x < 64.
Buscando en un diccionario encontramos 45 palabras, que es
precisamente el valor medio (64+27)/2 = 45.
aeta, asas, asea, asee, aseo, ases, asta, atea, atas, ates, ateo, atoa,
atoe, atoo, osas, oses, osos, oste, otea, otee, oteo, easo, esas, eses,
esos, esta, este esto, etas, tasa, tase, taso, teas, tesa, tese, teso,
teta, seas, seso, seta, seto, sosa, sota, sote, soto.
Jorge Rami Aguirre Madrid (Espaa) 2003
Seguridad Informtica y Criptografa. Tema 4: Teora de la Informacin
Diapositiva 188
Redundancia del lenguaje
La redundancia D del lenguaje ser la diferencia
entre la ratio absoluta y la ratio real:
D = R - r
3.25 < D < 3.55
Qu significa esto?
El nmero de bits extras (bits redundantes) necesarios
para codificar un mensaje suponiendo un alfabeto de 27
caracteres (codificacin con 5 bits puesto que 2
5
= 32 y
2
4
= 16) ser aproximadamente igual a 3.5.
D/R ser un factor proporcional, luego:
68.42 < % Red. Lenguaje (D/R) < 74.73
Jorge Rami Aguirre Madrid (Espaa) 2003
Seguridad Informtica y Criptografa. Tema 4: Teora de la Informacin
Diapositiva 189
Es nuestro lenguaje redundante?
El estudio de Shannon demuestra que la estructura del
lenguaje produce esta redundancia:
Diferencia de frecuencias de aparicin de las letras.
Existencia de digramas comunes.
Existencia de trigramas comunes.
Existencia de poligramas comunes.
Estructuracin de frases y oraciones con sentido.
Y nuestra misin es
crear algoritmos que
sean seguros y eviten
estos ataques.
Esto dar pistas al criptoanalista para atacar un sistema.
Jorge Rami Aguirre Madrid (Espaa) 2003
Seguridad Informtica y Criptografa. Tema 4: Teora de la Informacin
Diapositiva 190
Un ejemplo de redundancia (parte 1)
Todos los lenguajes sern redundantes. Esto quiere
decir que la misma cantidad de informacin se puede
entregar con menos smbolos o bits.
Sea el siguiente mensaje M = HBNVZNCRC
1
a
ayuda:
En el mensaje original se han quitado las vocales.
Esto nos permite suponer que entre consonantes habr cero,
una, dos o tres vocales, segn reglas del lenguaje...
M = __H__B__N__V__Z__N__C__R__C__
Jorge Rami Aguirre Madrid (Espaa) 2003
Seguridad Informtica y Criptografa. Tema 4: Teora de la Informacin
Diapositiva 191
Un ejemplo de redundancia (parte 2)
2
a
ayuda:
El mensaje original contiene cinco palabras.
Esto nos permite limitar el nmero de mensajes posibles
que tengan sentido. En estas condiciones podran existir
muchos mensajes de 5 palabras, aunque no cumpliesen de
forma lgica con las reglas del lenguaje. Un ejemplo
podra ser...
M = AH BUENO AVE ZONA CERCA
Tenamos el mensaje M = HBNVZNCRC y adems
M = __H__B__N__V__Z__N__C__R__C__
Jorge Rami Aguirre Madrid (Espaa) 2003
Seguridad Informtica y Criptografa. Tema 4: Teora de la Informacin
Diapositiva 192
Un ejemplo de redundancia (parte 3)
3
a
ayuda y siguientes:
a) El mensaje original tiene que ver con un circo.
b) Corresponde al estribillo de una cancin infantil.
c) Los espacios estn en: M = HB N VZ N CRC.
Seguro que habr adivinado ya el mensaje....
M = AH BUENO AVE ZONA CERCA
M = __H__B__N__V__Z__N__C__R__C__

Tenamos el mensaje M = HBNVZNCRC y adems


M = HABA UNA VEZ UN CIRCO
Jorge Rami Aguirre Madrid (Espaa) 2003
Seguridad Informtica y Criptografa. Tema 4: Teora de la Informacin
Diapositiva 193
Redundancia y entropa condicional
El ejemplo anterior, adems de demostrar que todos los
lenguajes son redundantes, es un claro exponente de lo que se
entiende en la prctica por entropa condicional.
Cada vez que vamos dando nuevas pistas, disminuye la
incertidumbre del mensaje hasta que sta se anula y por lo
tanto la entropa es igual a 0 ya que existe un nico mensaje
posible con probabilidad igual a la unidad.
Algo similar ocurre cuando resolvemos un crucigrama y lo
anteriormente resuelto nos sirve como pistas para descubrir
palabras nuevas. Mientras ms palabras tengamos, ms fcil
se hace avanzar en su resolucin.
Jorge Rami Aguirre Madrid (Espaa) 2003
Seguridad Informtica y Criptografa. Tema 4: Teora de la Informacin
Diapositiva 194
Secreto de un sistema criptogrfico
Shannon midi el secreto de un criptosistema como la
incertidumbre del mensaje en claro conocido el
criptograma recibido:
Mensajes M = {M
1
, M
2
, ..., M
3
} p(M) = 1
M
Criptogramas C = {C
1
, C
2
, ..., C
3
} p(C) = 1
C
Claves K = {K
1
, K
2
, ..., K
3
} p(K) = 1
K
Cuando tendr nuestro sistema un secreto perfecto?
Jorge Rami Aguirre Madrid (Espaa) 2003
Seguridad Informtica y Criptografa. Tema 4: Teora de la Informacin
Diapositiva 195
Definiciones previas secreto criptogrfico
p(M): Probabilidad de enviar un mensaje M. Si
hay n mensajes M
i
equiprobables, p(M
i
) = 1/n.
p(C): Probabilidad de recibir un criptograma C. Si
cada uno de los n criptogramas recibidos C
i
es
equiprobable, p(C
i
) = 1/n.
p
M
(C): Probabilidad de que, a partir de un texto en
claro M
i
, se obtenga un criptograma C
i
.
p
C
(M): Probabilidad de que, una vez recibido un
criptograma C
i
, ste provenga de un texto claro M
i
.
Jorge Rami Aguirre Madrid (Espaa) 2003
Seguridad Informtica y Criptografa. Tema 4: Teora de la Informacin
Diapositiva 196
Secreto criptogrfico perfecto (1)
Un sistema tiene secreto perfecto si el conocimiento del texto
cifrado no proporciona ninguna informacin acerca del
mensaje. Es decir, cuando la probabilidad de acierto al recibir
el elemento i +1 es la misma que en el estado i.
Secreto perfecto p(M) = p
C
(M)
La probabilidad p de enviar un mensaje M con texto en claro p(M) o
probabilidad a priori ser igual a la probabilidad p de que, conocido
un criptograma C, ste se corresponda a un mensaje M cifrado con la
clave K. Esta ltima (probabilidad a posteriori) es p
C
(M).
Jorge Rami Aguirre Madrid (Espaa) 2003
Seguridad Informtica y Criptografa. Tema 4: Teora de la Informacin
Diapositiva 197
Secreto criptogrfico perfecto (2)
La probabilidad p de recibir un texto cifrado C al
cifrar un mensaje M usando una clave K ser p
M
(C).
Luego, M debe haberse cifrado con alguna clave K:
p
M
(C) = p(K) donde E
K
(M) = C
K
k
j
/ E
kj
(M
i
) = C
i
En el fondo esto viene a significar que para lograr un
secreto perfecto, el espacio de claves debe ser al
menos de igual tamao que el espacio de mensajes.
Jorge Rami Aguirre Madrid (Espaa) 2003
Seguridad Informtica y Criptografa. Tema 4: Teora de la Informacin
Diapositiva 198
Secreto criptogrfico perfecto (3)
La condicin necesaria y suficiente del secreto
perfecto es que para cualquier valor de M se
cumpla que la probabilidad de recibir C,
resultado de la cifra de un mensaje M con una
clave K, sea la misma que recibir el criptograma
C, resultado de la cifra de otro mensaje M
distinto, cifrado con otra clave.
p
M
(C) = p(C)
para todo valor de M
Veamos algunos
ejemplos
Jorge Rami Aguirre Madrid (Espaa) 2003
Seguridad Informtica y Criptografa. Tema 4: Teora de la Informacin
Diapositiva 199
Cifrador con secreto perfecto
Sea el siguiente escenario:
Espacio de Mensajes Espacio de Claves Espacio de Cifrados
M
1
k
1
C
1
k
3
k
2
k
2
M
2
k
3
C
2
k
1
k
3
k
1
M
3
C
3
k
2
p(M) = 1/3 para todo M p(C) = 1/3
p
M
(C) =1/3 p
C
(M) = 1/3
Jorge Rami Aguirre Madrid (Espaa) 2003
Seguridad Informtica y Criptografa. Tema 4: Teora de la Informacin
Diapositiva 200
Cifrador sin secreto perfecto (1)
Sea ahora el siguiente escenario:
Probabilidad de que un mensaje M
i
se convierta en un criptograma C
i
: [P
Mi
(C
i
)]
y que un criptograma C
i
sea el resultado de la cifra de un mensaje M
i
: [P
Ci
(M
i
) ]?

Espacio de Mensajes Espacio de Claves Espacio de Cifrados

k
1

M
1
C
1

k
3
k
2

k
2

M
2
k
3
C
2

k
1

k
3
k
1

M
3
k
2
C
3



C
4



p(M
1
) = 1/3 p(C
1
) = 3/9
p(M
2
) = 1/3
p(M
3
) = 1/3
p(C
2
) = 2/9
p(C
3
) = 2/9
p(C
4
) = 2/9
Algo ms
Jorge Rami Aguirre Madrid (Espaa) 2003
Seguridad Informtica y Criptografa. Tema 4: Teora de la Informacin
Diapositiva 201
Cifrador sin secreto perfecto (2)
Esquema anterior:
k
1
M
1
C
1
k
3
k
2
k
2
M
2
k
3
C
2
K
1
K
3
k
1
M
3
k
2
C
3
C
4
p
C1
(M
1
) = 1/3 p
C1
(M
2
) = 1/3 p
C1
(M
3
) = 1/3
p
C2
(M
1
) = 1/2 p
C2
(M
2
) = 1/2 p
C2
(M
3
) = 0
p
C3
(M
1
) = 1/2 p
C3
(M
2
) = 0 p
C3
(M
3
) = 1/2
p
C4
(M
1
) = 0 p
C4
(M
2
) = 1/2 p
C4
(M
3
) = 1/2
p
M1
(C
1
) = 1/3 p
M1
(C
2
) = 1/3 p
M1
(C
3
) = 1/3 p
M1
(C
4
) = 0
p
M2
(C
1
) = 1/3 p
M2
(C
2
) = 1/3 p
M2
(C
3
) = 0 p
M2
(C
4
) = 1/3
p
M3
(C
1
) = 1/3 p
M3
(C
2
) = 0 p
M3
(C
3
) = 1/3 p
M3
(C
4
) = 1/3
Jorge Rami Aguirre Madrid (Espaa) 2003
Seguridad Informtica y Criptografa. Tema 4: Teora de la Informacin
Diapositiva 202
Distancia de unicidad
Se entender por Distancia de Unicidad al bloque N de
texto cifrado o criptograma mnimo necesario para que se
pueda intentar con ciertas expectativas de xito un ataque
en bsqueda de la clave usada para cifrar.
Este valor se obtiene cuando la equivocacin de la clave
H
C
(K) se acerca a cero o tiende a anularse.
A medida que tenga un criptograma ms largo, y por tanto
ms informacin, se supone que la tarea de ataque del
criptoanalista se va facilitando.
Se busca el tamao N de criptograma que permita
esperar que la solucin de K sea nica. Se supondr
un cifrador aleatorio Modelo de Hellmann
Jorge Rami Aguirre Madrid (Espaa) 2003
Seguridad Informtica y Criptografa. Tema 4: Teora de la Informacin
Diapositiva 203
Parmetros del modelo de Hellman (1)
Existirn 2
RN
mensajes posibles de longitud N.
Existirn 2
rN
mensajes de longitud N con sentido.
El espacio de mensajes de longitud N se dividir:
Mensajes con sentido: M
CS
= 2
rN
Mensajes sin sentido: M
SS
= 2
RN
- 2
rN
Los 2
rN
mensajes con sentido sern equiprobables
siendo su valor p(M
CS
) = 1/2
rN
= 2
-rN
El resto de mensajes (2
RN
- 2
rN
) tendr una
probabilidad nula p(M
SS
) = 0.
Jorge Rami Aguirre Madrid (Espaa) 2003
Seguridad Informtica y Criptografa. Tema 4: Teora de la Informacin
Diapositiva 204
Parmetros del modelo de Hellman (2)
Existirn 2
H(K)
claves equiprobables.
En donde H(K) es la entropa de la clave.
Con p(K) = 1/2
H(K)
= 2
-H(K)
Con estas claves se cifrarn todos los mensajes
con sentido dando lugar a 2
RN
textos cifrados
posibles de longitud N.
Los criptogramas obtenidos sern equiprobables.
Por sencillez, veremos el modelo de cifrador
aleatorio de Hellman slo con dos claves k
1
y k
2
.
Jorge Rami Aguirre Madrid (Espaa) 2003
Seguridad Informtica y Criptografa. Tema 4: Teora de la Informacin
Diapositiva 205
Esquema de cifrador aleatorio de Hellmann
Espacio de Mensajes Espacio de Claves Espacio de Cifrados

k1
M
1
C
1

k2
M
2

k1
C
2

k2

k2
M
3
C
3

k1

k1

k2
M
4
C
4
k1
M
5
C
5

k2
M
6 k1
C
6

k2
M
7
C
7
M
8
C
8
M
9
C
9
M
10
C
10
Verdaderas SV
SV: Un criptograma est asociado
slo a un texto en claro con sentido
cifrado con una nica clave k
i
.
SF: Cualquier otra solucin de
cifra distinta a la anterior.
SV: C
3
= E
k1
(M
5
) C
4
= E
k1
(M
2
)
C
6
= E
k2
(M
1
) C
7
= E
k1
(M
3
)
C
9
= E
k1
(M
6
) C
10
= E
k2
(M
6
)
SF: C
2
= E
k1
(M
4
) C
2
= E
k2
(M
4
)
C
5
= E
k2
(M
2
) C
5
= E
k2
(M
5
)
C
1
= E
k1
(M
1
) C
1
= E
k2
(M
3
)
SF C
2
: Condicin obvia
SF C
5
: Condicin dbil
SF C
1
: Condicin fuerte
Falsas SF
Soluciones:
Jorge Rami Aguirre Madrid (Espaa) 2003
Seguridad Informtica y Criptografa. Tema 4: Teora de la Informacin
Diapositiva 206
Clculo de la distancia de unicidad (1)
Para cada solucin correcta de un texto M cifrado con
una clave k del espacio 2
H(K)
, existirn otras (2
H(K)
-1)
claves con la misma probabilidad de entregar una
solucin falta SF.
Sea q la probabilidad de obtener un mensaje con sentido:
q = 2
rN
/ 2
RN
= 2
(r - R)N
= 2
-DN
Luego:
SF = (2
H(K)
-1) q = (2
H(K)
-1) 2
-DN
= 2
H(K) - DN
- 2
-DN
SF 2
H(K) - DN
log
2
SF = H(K) - DN
Jorge Rami Aguirre Madrid (Espaa) 2003
Seguridad Informtica y Criptografa. Tema 4: Teora de la Informacin
Diapositiva 207
Clculo de la distancia de unicidad (2)
La solucin SF = 0 es imposible porque slo se llega a
ella de forma asinttica con un valor de N infinito como
se muestra en la diapositiva siguiente.
Se acepta entonces que haya como mximo una sola
solucin falsa, de ah su nombre de unicidad, luego:
SF = 2
H(K) DN
SF = 1 H(K) - DN = 0
Por lo tanto:
Nmero mnimo de bytes o caracteres
en C para intentar un ataque.
N = H(K) / D
Jorge Rami Aguirre Madrid (Espaa) 2003
Seguridad Informtica y Criptografa. Tema 4: Teora de la Informacin
Diapositiva 208
Ejemplos de distancia de unicidad
Para el cifrador del Csar mdulo 27 en el que la clave es b, todos
los posibles desplazamientos de caracteres, 1 b 26, su entropa
H(X) = log
2
26 = 4.7 bits por lo que N = 4.7/3.4 = 1.4 caracteres.
Para el mismo cifrador del Csar pero con clave, si el alfabeto tiene n
caracteres, existirn n! posibles claves. En este caso la entropa de la
clave puede aproximarse como H(X) = log
2
27! 27log
2
(27/e), por
lo que N = 27log2 (27/2.72)/3.4 = 27.4 caracteres.
En el sistema DES la clave verdadera es de 56 bits por lo que su
entropa H(X) = 56. Si el mensaje slo contiene letras maysculas
(27 elementos) podramos decir que N = 56/3.4 = 16,5 caracteres.
Nota: aunque el valor de N sea ahora ms bajo no quiere decir en
absoluto que el DES sea menos seguro que el cifrador del Csar con
clave. Este ltimo se puede atacar fcilmente con estadsticas del
lenguaje muy elementales y el DES no. Adems, recuerde que debe
contar con un criptograma varias veces mayor que el valor de N si
desea que su criptoanlisis tenga alguna posibilidad de xito.
Jorge Rami Aguirre Madrid (Espaa) 2003
Seguridad Informtica y Criptografa. Tema 4: Teora de la Informacin
Diapositiva 209
Cantidad de trabajo Q
Cantidad de Trabajo
Q
H(M/C)
n
N Cantidad de caracteres
(A)
(B)
(C)
(A) Inicialmente hay que hacer un arduo trabajo para obtener algo
coherente. Habr muchas soluciones falsas.
(D)
Solucin nica
(B) Cuando se tiene una cantidad adecuada de texto cifrado, la
cantidad de trabajo disminuye. Se descartan algunas soluciones.
(C) Cuando se anula la equivocacin de la clave, H(M/C) = 0,
disminuyen las soluciones falsas y la solucin tiende a ser nica.
Jorge Rami Aguirre Madrid (Espaa) 2003
Seguridad Informtica y Criptografa. Tema 4: Teora de la Informacin
Diapositiva 210
El uso de tcnicas de difusin
Para lograr un mayor secreto en las operaciones de cifra,
Shannon propuso usar dos tcnicas: difusin y confusin.
Difusin: es la transformacin sobre el texto en claro con el
objeto de dispersar las propiedades estadsticas del lenguaje
sobre todo el criptograma. Se logra con transposiciones.
TRANSPOSICIONES
La transposicin consiste bsicamente en una permutacin, es
decir, cambiar los caracteres de lugar segn una regla, una
funcin, etc. Por ejemplo el carcter primero se posiciona en
el lugar cuarto, el segundo en el lugar tercero, etc.
Jorge Rami Aguirre Madrid (Espaa) 2003
Seguridad Informtica y Criptografa. Tema 4: Teora de la Informacin
Diapositiva 211
El uso de tcnicas de confusin
Confusin: Transformacin sobre el texto en claro con objeto
de mezclar los elementos de ste, aumentando la complejidad
de la dependencia funcional entre clave y criptograma. Se
obtiene a travs de sustituciones.
SUSTITUCIONES
La sustitucin consiste bsicamente modificar la informacin,
es decir, sustituir un carcter por otro de acuerdo a una regla,
una funcin, etc. Por ejemplo cambiar la letra A por la letra
M, la letra B por la letra X , etc.
Ambas tcnicas sern usadas en los sistemas clsicos y
aunque no parezca, tambin lo usar el cifrador DES.
Fin del Tema 4
Jorge Rami Aguirre Madrid (Espaa) 2003
Seguridad Informtica y Criptografa. Tema 4: Teora de la Informacin
Diapositiva 212
Cuestiones y ejercicios (1 de 2)
1. Al despertar ponemos la radio y escuchamos noticias que no nos
llaman la atencin. Por qu decimos que no haba informacin?
2. Justifique la definicin logartmica de cantidad de informacin, es
decir la razn de que c
i
= - log (p
i
).
3. Por qu usamos la base 2 en el logaritmo que define c
i
?
4. Cul es el nmero mnimo e inteligente- de preguntas que hay que
hacer para pasar de la incertidumbre a la certeza en un sistema de n
estados equiprobables? Y si no son equiprobables?
5. Por qu la entropa es no nula y se anula s y slo si uno de los
estados de la variable es igual a la unidad?
6. Codificamos en binario un sistema con 256 estados equiprobables.
Si no usamos un codificador ptimo, cuntos bits son necesarios?
Mediante un codificador ptimo, usaremos ms o menos bits?
Jorge Rami Aguirre Madrid (Espaa) 2003
Seguridad Informtica y Criptografa. Tema 4: Teora de la Informacin
Diapositiva 213
Cuestiones y ejercicios (2 de 2)
7. Qu representa la expresin log
2
[1/p(x)] en la entropa H(X)? Si
p(x
1
)=0,6; p(x
2
)=0,3; p(x
3
)=0,1 calcule log
2
[1/p(x)] qu opina?
8. Definimos un alfabeto con 71 elementos (maysculas y minsculas,
minsculas acentuadas, nmeros, punto y coma). Si estos elementos
son equiprobables, cul es la ratio absoluta de este alfabeto?
9. La ratio verdadera es mayor o menor que la absoluta? Por qu?
10. Un alfabeto consta de 8 elementos equiprobables. Cuntos posibles
mensajes de tamao 4 existen? De stos, cuntos mensajes podran
tener sentido si esos 8 elementos representan al idioma castellano?
11. Cundo decimos que un sistema tiene secreto perfecto? En un
sistema real, es eso posible? Piense en algn ejemplo y comntelo.
12. Por qu se dice que hay que minimizar las soluciones falsas SF en
el modelo de Hellman para romper la clave? Es la clave k nica?
Tema 5
Teora de los Nmeros
Seguridad Informtica y Criptografa
Ultima actualizacin: 03/03/03
Archivo con 67 diapositivas
Jorge Rami Aguirre
Universidad Politcnica de Madrid
Material Docente de
Libre Distribucin
v 3.1
Este archivo forma parte de un curso sobre Seguridad Informtica y Criptografa. Se autoriza la
reproduccin en computador e impresin en papel slo con fines docentes o personales, respetando
en todo caso los crditos del autor. Queda prohibida su venta, excepto a travs del Departamento de
Publicaciones de la Escuela Universitaria de Informtica, Universidad Politcnica de Madrid, Espaa.
Curso de Seguridad Informtica y Criptografa JRA
Jorge Rami Aguirre Madrid (Espaa) 2003
Seguridad Informtica y Criptografa. Tema 5: Teora de los Nmeros
Diapositiva 216
Conceptos bsicos de congruencia
Es la base matemtica (matemticas discretas) en
la que se sustentan las operaciones de cifra.
Concepto de congruencia:
Sean dos nmeros enteros a y b: se dice que a es
congruente con b en el mdulo o cuerpo n (Z
n
) si y
slo si existe algn entero k que divide de forma
exacta la diferencia (a - b)
a - b = k n
a
n
b
a b mod n
Jorge Rami Aguirre Madrid (Espaa) 2003
Seguridad Informtica y Criptografa. Tema 5: Teora de los Nmeros
Diapositiva 217
Operaciones de congruencia en Z
n
Es 18 congruente con 3 mdulo 5?
18 3 mod 5?
S, porque: 18-3 = 15 = k5 con k = 3
Cmo se usar esto en criptografa?
Esta operacin en Z
n
se expresar as:
18 mod 5 = 3
El valor 3 ser el resto o residuo.
El conjunto de nmeros que forman los restos dentro de
un cuerpo Z
n
sern muy importantes en criptografa.
Jorge Rami Aguirre Madrid (Espaa) 2003
Seguridad Informtica y Criptografa. Tema 5: Teora de los Nmeros
Diapositiva 218
Propiedades de la congruencia en Z
n
Propiedad Reflexiva:
a a mod n a Z
Propiedad Simtrica:
a b mod n b a mod n a,b Z
Propiedad Transitiva:
Si a b mod n y b c mod n
a c mod n a,b,c Z
Jorge Rami Aguirre Madrid (Espaa) 2003
Seguridad Informtica y Criptografa. Tema 5: Teora de los Nmeros
Diapositiva 219
Propiedades de las operaciones en Z
n
(1)
Propiedad Asociativa:
a + (b + c) mod n (a + b) + c mod n
Propiedad Conmutativa:
a + b mod n b + a mod n
a b mod n b a mod n
Propiedad Distributiva:
a (b+c) mod n ((a b) + (a c)) mod n
Se usar el signo =
en vez de
(algo propio de los
Campos de Galois)
a (b+c) mod n = ((a b) + (a c)) mod n
Jorge Rami Aguirre Madrid (Espaa) 2003
Seguridad Informtica y Criptografa. Tema 5: Teora de los Nmeros
Diapositiva 220
Propiedades de las operaciones en Z
n
(2)
Existencia de Identidad:
a + 0 mod n = 0 + a mod n = a mod n = a
a 1 mod n = 1 a mod n = a mod n = a
Existencia de Inversos:
a + (-a) mod n = 0
a (a
-1
) mod n = 1 (si a 0)
Reducibilidad:
(a + b) mod n = [(a mod n) + (b mod n)] mod n
(a b) mod n = [(a mod n) (b mod n)] mod n

Ambos importantes en
criptografa
No siempre existe

Jorge Rami Aguirre Madrid (Espaa) 2003


Seguridad Informtica y Criptografa. Tema 5: Teora de los Nmeros
Diapositiva 221
Conjunto completo de restos CCR
Para cualquier entero positivo n, el conjunto
completo de restos ser CCR = {0, 1, 2, ... n-1},
es decir:
a Z ! r
i
CCR / a r
i
mod n
CCR (11) = {0, 1, 2, 3, 4, 5, 6, 7, 8, 9, 10}
CCR (6) = {0, 1, 2, 3, 4, 5} = {12, 7, 20, 9, 16, 35}
El segundo conjunto es equivalente: 12 0, 7 1...
Jorge Rami Aguirre Madrid (Espaa) 2003
Seguridad Informtica y Criptografa. Tema 5: Teora de los Nmeros
Diapositiva 222
Homomorfismo de los enteros
Enteros Enteros mod n
a
1
, a
2
(a
1
mod n), (a
2
mod n)
op (y posterior reduccin mod n) op
(a
1
op a
2
) mod n (a
1
mod n) op (a
2
mod n) mod n
es lo mismo que
esta otra
Esta operacin ...
Jorge Rami Aguirre Madrid (Espaa) 2003
Seguridad Informtica y Criptografa. Tema 5: Teora de los Nmeros
Diapositiva 223
Un ejemplo de homomorfismo

Ejemplo: una calculadora capaz de


trabajar slo con tres dgitos ...
88 93 mod 13
8.184 mod 13
Resultado: 7
88 93 mod 13
8.184 mod 13
Resultado: 7
Solucin por homomorfismo:
88 93 mod 13
[(88) mod 13 (93) mod 13] mod 13
10 2 mod 13
20 mod 13 Resultado: 7
se llega a lo mismo, pero...
Se desbordara
la memoria de
nuestro sistema

... y hemos usado siempre nmeros de 3
dgitos. En este caso la operacin mxima
sera 1212 = 144, es decir tres dgitos.

Ahora ya no
se desborda
la memoria
Jorge Rami Aguirre Madrid (Espaa) 2003
Seguridad Informtica y Criptografa. Tema 5: Teora de los Nmeros
Diapositiva 224
Divisibilidad de los nmeros
En criptografa muchas veces nos interesar encontrar el
mximo comn denominador mcd entre dos nmeros a y b.
Para la existencia de inversos en un cuerpo n, la base a y el
mdulo n debern ser primos entre s. mcd (a, n) = 1
Algoritmo de Euclides:
a) Si x divide a a y b a = x a y b = x b
b) Por lo tanto: a - k b = x a - k x b
a - k b = x (a - k b)
c) Entonces se concluye que x divide a (a - k b)
Jorge Rami Aguirre Madrid (Espaa) 2003
Seguridad Informtica y Criptografa. Tema 5: Teora de los Nmeros
Diapositiva 225
El mximo comn denominador mcd
Como hemos llegado a que x divide a (a k b) esto nos
permitir encontrar el mcd (a, b):
Si a > b entonces a = d
1
b + r
(con d
i
un entero y r un resto)
Luego mcd (a, b) = mcd (b ,r) (a > b > r 0)
porque:
Si b > r entonces b = d
2
r + r
(con r un entero y r un resto)
Jorge Rami Aguirre Madrid (Espaa) 2003
Seguridad Informtica y Criptografa. Tema 5: Teora de los Nmeros
Diapositiva 226
Divisibilidad con algoritmo de Euclides
mcd (148, 40)
148 = 3 40 + 28
40 = 1 28 + 12
28 = 2 12 + 4
12 = 3 4 + 0
mcd (148, 40) = 4
mcd (385, 78)
385 = 4 78+ 73
78 = 1 73 + 5
73 = 14 5 + 3
5 = 1 3 + 2
3 = 1 2 + 1
2 = 2 1 + 0
mcd (385, 78) = 1
148 = 2
2
37
40 = 2
3
5
Factor comn
2
2
= 4
No hay
factor comn
385 = 5 7 11
78 = 2 3 13
Ser importante
en criptografa

Jorge Rami Aguirre Madrid (Espaa) 2003
Seguridad Informtica y Criptografa. Tema 5: Teora de los Nmeros
Diapositiva 227
Inversin de una operacin de cifra
En criptografa deber estar permitido invertir una
operacin para recuperar un cifrado descifrar.
Si bien la cifra es una funcin, en lenguaje coloquial
la operacin de cifrado sera una multiplicacin y la
operacin de descifrado una divisin.
La analoga anterior slo ser vlida en el cuerpo de
los enteros Z
n
con inverso.
Luego, si en una operacin de cifra la funcin es el
valor a dentro de un cuerpo n, deberemos encontrar el
inverso a
-1
mod n para descifrar; en otras palabras ...
Jorge Rami Aguirre Madrid (Espaa) 2003
Seguridad Informtica y Criptografa. Tema 5: Teora de los Nmeros
Diapositiva 228
Inversos en un cuerpo
Si a x mod n = 1
x ser el inverso multiplicativo (a
-1
) de a en el mdulo n
No siempre existen los inversos multiplicativos. En
realidad lo raro es que existan.
Por ejemplo, en Z = 2 no existirn inversos pues la
nica solucin para ax mod 2 = 1, con a, x = {0, 1},
sera x = 1 para a = 1, una solucin trivial. El cero
nunca ser solucin del inverso multiplicativo.
Jorge Rami Aguirre Madrid (Espaa) 2003
Seguridad Informtica y Criptografa. Tema 5: Teora de los Nmeros
Diapositiva 229
Existencia del inverso por primalidad
inverso a
-1
en mod n ssi mcd (a, n) = 1
Si mcd (a,n) = 1, el resultado de ai mod n (para i todos los
restos de n) sern valores distintos dentro del cuerpo n.
mcd (a, n) = 1 x ! 0<x<n / a x mod n = 1
Sea: a = 4 y n = 9. Valores de i = {1, 2, 3, 4, 5, 6, 7, 8}
41 mod 9 = 4 42 mod 9 = 8 43 mod 9 = 3
44 mod 9 = 7 45 mod 9 = 2 46 mod 9 = 6
47 mod 9 = 1 48 mod 9 = 5
Si mcd (a,n) 1
S
O
L
U
C
I

N
I
C
A
Jorge Rami Aguirre Madrid (Espaa) 2003
Seguridad Informtica y Criptografa. Tema 5: Teora de los Nmeros
Diapositiva 230
Inexistencia de inverso (no primalidad)
Y si no hay primalidad entre a y n?
Si mcd (a, n) 1
No existe ningn x que 0 < x < n / a x mod n = 1
Sea: a = 3 y n = 6 Valores de i = {1, 2, 3, 4, 5}
31 mod 6 = 3 32 mod 6 = 0 33 mod 6 = 3
34 mod 6 = 0 35 mod 6 = 3
No existe el inverso para ningn resto del cuerpo.

Jorge Rami Aguirre Madrid (Espaa) 2003


Seguridad Informtica y Criptografa. Tema 5: Teora de los Nmeros
Diapositiva 231
Inversos aditivos y multiplicativos
(A+B) mod 5 (AB) mod 5
B + 0 1 2 3 4 B 0 1 2 3 4
A 0 0 1 2 3 4 A 0 0 0 0 0 0
1 1 2 3 4 0 1 0 1 2 3 4
2 2 3 4 0 1 2 0 2 4 1 3
3 3 4 0 1 2 3 0 3 1 4 2
4 4 0 1 2 3 4 0 4 3 2 1
o En la operacin suma siempre existir el inverso o valor identidad
de la adicin (0) para cualquier resto del cuerpo.
o En la operacin producto, de existir un inverso o valor de identidad
de la multiplicacin (1) ste es nico. La condicin para ello es que
el nmero y el mdulo sean primos entre s. Por ejemplo para n = 4,
el resto 2 no tendr inverso multiplicativo, en cambio el resto 3 s.
0+0 = 0
11 = 1
Es trivial
Jorge Rami Aguirre Madrid (Espaa) 2003
Seguridad Informtica y Criptografa. Tema 5: Teora de los Nmeros
Diapositiva 232
Conjunto reducido de restos CRR
El conjunto reducido de restos, conocido como
CRR de n, es el subconjunto {0, 1, ... n
i
, ... n-1}
de restos primos con el grupo n.
Si n es primo, todos los restos sern primos con l.
Como el cero no es una solucin, entonces:
CRR = {1, ..., n
i
, ... n-1} / mcd (n
i
, n) = 1
Ejemplo: CRR mod 8 = {1, 3, 5, 7}
CRR mod 5 = {1, 2, 3, 4}
Jorge Rami Aguirre Madrid (Espaa) 2003
Seguridad Informtica y Criptografa. Tema 5: Teora de los Nmeros
Diapositiva 233
Utilidad del CRR
Qu utilidad tiene esto en criptografa?
El conocimiento del CRR permitir aplicar un algoritmo
para el clculo del inverso multiplicativo de un nmero x
dentro de un cuerpo o grupo n a travs de la funcin (n),
denominada Funcin de Euler o Indicador de Euler.
Ser muy importante tanto en los sistemas
simtricos que trabajan en un mdulo (con
excepcin del DES) como en los sistemas
asimtricos. En ambos casos la cifra y las
claves estn relacionadas con el CRR.
Jorge Rami Aguirre Madrid (Espaa) 2003
Seguridad Informtica y Criptografa. Tema 5: Teora de los Nmeros
Diapositiva 234
Funcin de Euler (n)
Funcin (n) de Euler
Entregar el nmero de elementos del CRR.
Podremos representar cualquier nmero n de estas
cuatro formas:
Veamos cada
uno de ellos
a) n es un nmero primo.
b) n se representa como n = p
k
con p primo y k entero.
c) n es el producto n = pq con p y q primos.
d) n es un nmero cualquiera (genrico).
t
n = p
1
e1
p
2
e2
... p
t
et
= p
i
ei
i=1
Jorge Rami Aguirre Madrid (Espaa) 2003
Seguridad Informtica y Criptografa. Tema 5: Teora de los Nmeros
Diapositiva 235
Funcin (n) de Euler (n = p)
Caso 1: n es un nmero primo
Si n es primo, (n) ser igual a CCR menos el 0.
(n) = n - 1
Si n es primo, entonces CRR = CCR - 1 ya que todos
los restos de n, excepto el cero, sern primos entre s.
CRR(7) = {1,2,3,4,5,6} seis elementos
(7) = n - 1 = 7-1 = 6
(11) = 11-1 = 10; (23) = 23-1 = 22
Ejemplo
Se usar en sistemas ElGamal y DSS
Jorge Rami Aguirre Madrid (Espaa) 2003
Seguridad Informtica y Criptografa. Tema 5: Teora de los Nmeros
Diapositiva 236
Funcin (n) de Euler (n = p
k
)
Caso 2: n = p
k
(con p primo y k un entero)
(n) = (p
k
) = p
k
- p
k-1
(p
k
) = p
k-1
(p-1)
De los p
k
elementos del CCR, restaremos todos los
mltiplos 1p, 2p, 3p, ...(p
k-1
-1)p y el cero.
CRR(16) = {1,3,5,7,9,11,13,15} ocho elementos
(16) = (2
4
) = 2
4-1
(2-1) = 2
3
1 = 8
(125) = (5
3
) = 5
3-1
(5-1) = 5
2
4 = 254 = 100
Ejemplo
Jorge Rami Aguirre Madrid (Espaa) 2003
Seguridad Informtica y Criptografa. Tema 5: Teora de los Nmeros
Diapositiva 237
Funcin (n) de Euler (n = pq) (1)
Caso 3: n = pq (con p y q primos)
(n) = (pq) = (p)(q) = (p-1)(q-1)
De los pq elementos del CCR, restaremos todos los
mltiplos de p = 1p, 2p, ... (q - 1)p, todos los
mltiplos de q = 1q, 2q, ... (p - 1)q y el cero.
(pq) = pq - [(q-1) + (p-1) +1] = pq - q - p + 1
(p-1)(q-1)
Jorge Rami Aguirre Madrid (Espaa) 2003
Seguridad Informtica y Criptografa. Tema 5: Teora de los Nmeros
Diapositiva 238
Funcin (n) de Euler (n = pq) (2)
Ejemplo CRR(15) = {1,2,4,7,8,11,13,14} ocho elementos
(15) = (35) = (3-1)(5-1) = 24 = 8
(143) = (1113) = (11-1)(13-1) = 1012 = 120
Ser una de las funciones ms utilizadas ya
que es la base del sistema RSA que durante
muchos aos ha sido un estndar de hecho.
Jorge Rami Aguirre Madrid (Espaa) 2003
Seguridad Informtica y Criptografa. Tema 5: Teora de los Nmeros
Diapositiva 239
Funcin (n) de Euler (n = genrico)
Caso 4: n = p
1
e1
p
2
e2
... p
t
et
(p
i
son primos)
t
(n) = p
i
ei-1
(p
i
- 1)
i=1
(demostracin no inmediata)
CRR(20) = {1, 3, 7, 9, 11, 13, 17, 19} ocho elementos
(20) = (2
2
5) = 2
2-1
(2-1)5
1-1
(5-1) = 2
1
114 = 8
(360) = (2
3
3
2
5) = 2
3-1
(2-1)3
2-1
(3-1)5
1-1
(5-1) = 96
Ejemplo
Jorge Rami Aguirre Madrid (Espaa) 2003
Seguridad Informtica y Criptografa. Tema 5: Teora de los Nmeros
Diapositiva 240
Teorema de Euler
Dice que si mcd (a,n) = 1 a
(n)
mod n = 1
Ahora igualamos ax mod n = 1 y a
(n)
mod n = 1
a
(n)
a
-1
mod n = x mod n
x = a
(n)-1
mod n
El valor x ser el inverso de a en el cuerpo n
Nota: Observe que se ha dividido por a en el clculo anterior.
Esto se puede hacer porque mcd (a, n) = 1 y por lo tanto hay
un nico valor inverso en el cuerpo n que lo permite.
Jorge Rami Aguirre Madrid (Espaa) 2003
Seguridad Informtica y Criptografa. Tema 5: Teora de los Nmeros
Diapositiva 241
Clculo de inversos con Teorema Euler
Ejemplo
Cul es el inverso de 4 en mdulo 9? inv (4, 9)
Pregunta: Existe a x mod n = 4 x mod 9 = 1?
Como mcd (4, 9) = 1 S ... aunque 4 y 9 no son primos.
(9) = 6 x = 4
6-1
mod 9 = 7 74 = 28 mod 9 = 1
Resulta obvio que: inv (4, 9) = 7 e inv (7, 9) = 4
Jorge Rami Aguirre Madrid (Espaa) 2003
Seguridad Informtica y Criptografa. Tema 5: Teora de los Nmeros
Diapositiva 242
Teorema de Euler para n = pq
Si el factor a es primo relativo con n y n es el
producto de 2 primos, seguir cumplindose el
Teorema de Euler tambin en dichos primos.
Por ejemplo:
Si n = pq (n) = (p-1)(q-1)
a / mcd {a, (p,q)} = 1
se cumple que:
a
(n)
mod p = 1
a
(n)
mod q = 1
En el captulo
dedicado a la
cifra con clave
pblica RSA,
relacionaremos
este tema con el
Teorema del
Resto Chino.
Jorge Rami Aguirre Madrid (Espaa) 2003
Seguridad Informtica y Criptografa. Tema 5: Teora de los Nmeros
Diapositiva 243
Ejemplo Teorema de Euler para n = pq
Sea n = pq = 711 = 77
(n) = (p - 1)(q - 1) = (7 - 1)(11 - 1) = 610 = 60
Si k = 1, 2, 3, ...
Para a = k7 a
(n)
mod n = k7
60
mod 77 = 56
Para a = k11 a
(n)
mod n = k11
60
mod 77 = 22
Para a k7,11 a
(n)
mod n = a
60
mod 77 = 1
Y se cumple que:
Para a k7,11 a
(n)
mod p = a
60
mod 7 = 1
a
(n)
mod q = a
60
mod 11 = 1
En caso contrario: a
(n)
mod p = 0
a
(n)
mod q = 0
Jorge Rami Aguirre Madrid (Espaa) 2003
Seguridad Informtica y Criptografa. Tema 5: Teora de los Nmeros
Diapositiva 244
Teorema de Fermat
Si el cuerpo de trabajo n es un primo p
mcd (a, p) = 1 a
(p)
mod p = 1
Entonces a x mod p = 1 y a
(n)
mod p = 1
Adems, en este caso (p) = p-1 por lo que igualando las
dos ecuaciones de arriba tenemos:
a
(p)
a
-1
mod p = x mod p
x = a
p-2
mod p
Luego x ser e inverso de a en el primo p.
Jorge Rami Aguirre Madrid (Espaa) 2003
Seguridad Informtica y Criptografa. Tema 5: Teora de los Nmeros
Diapositiva 245
Qu hacemos si no se conoce (n)?
Calcular a
i
mod n cuando los valores de i y a son
grandes, se hace tedioso pues hay que utilizar la
propiedad de la reducibilidad repetidas veces.
Si no conocemos (n) o no queremos usar el
teorema de Euler/Fermat, siempre podremos
encontrar el inverso de a en el cuerpo n usando el
Algoritmo Extendido de Euclides
Es el mtodo ms rpido y prctico
Jorge Rami Aguirre Madrid (Espaa) 2003
Seguridad Informtica y Criptografa. Tema 5: Teora de los Nmeros
Diapositiva 246
Algoritmo Extendido de Euclides AEE
Si mcd (a, n) = 1 a x mod n = 1 x = inv (a, n)
Luego podemos escribir:
n = C
1
a + r
1
a > r
1
a = C
2
r
1
+ r
2
r
1
> r
2
r
1
= C
3
r
2
+ r
3
r
2
> r
3
... ...
r
n-2
= C
n
r
n-1
+ 1 r
n-1
> 1
r
n-1
= C
n+1
1 + 0
Concluye el algoritmo
Si volvemos hacia
atrs desde este
valor, obtenemos
el inverso de a en
el cuerpo n.
Si volvemos hacia
atrs desde este
valor, obtenemos
el inverso de a en
el cuerpo n.
Jorge Rami Aguirre Madrid (Espaa) 2003
Seguridad Informtica y Criptografa. Tema 5: Teora de los Nmeros
Diapositiva 247
Tabla de restos del AEE
Ordenando por restos desde el valor 1 se llega a una expresin
del tipo (k
1
n + k
2
a) mod n = 1, en donde el inverso de a en
n lo dar el coeficiente k
2
puesto que k
1
n mod n = 0.
C
1
C
2
C
3
C
4
... C
n-1
C
n
C
n+1
n a r
1
r
2
r
3
... r
n-2
r
n-1
1
(k
1
n + k
2
a) mod n = 1
Tabla de restos
Vuelta hacia atrs
Jorge Rami Aguirre Madrid (Espaa) 2003
Seguridad Informtica y Criptografa. Tema 5: Teora de los Nmeros
Diapositiva 248
Clculo de inversos con el AEE
Encontrar el inv (9, 25) por el mtodo de restos de Euclides.
a) 25 = 29 + 7
b) 9 = 17 + 2
c) 7 = 32 + 1
d) 2 = 21 + 0
7 = 25 - 29
2 = 9 - 1(25 - 29) = 39 -125
1 = (25 - 29) - 3(39 -125)
1 = 425 - 119 mod 25
7 = 25 - 29
2 = 9 - 17
1 = 7 - 32
2 1 3 2
25 9 7 2 1 0
El inv (9,25) = -11
-11 + 25 = 14
inv (9, 25) = 14
El inv (9,25) = -11
-11 + 25 = 14
inv (9, 25) = 14
restos
Tabla de Restos
Jorge Rami Aguirre Madrid (Espaa) 2003
Seguridad Informtica y Criptografa. Tema 5: Teora de los Nmeros
Diapositiva 249
Algoritmo para el clculo de inversos
i y
i
g
i
u
i
v
i
0 - 25 1 0
1 - 9 0 1
2 2 7 1 -2
3 1 2 -1 3
4 3 1 4 -11
5 2 0 -9 25
Para encontrar x = inv (A, B)
Hacer (g
0
, g
1
, u
0
, u
1
, v
0
, v
1
, i) = (B, A, 1, 0, 0, 1, 1)
Mientras g
i
0 hacer
Hacer y
i+1
= parte entera (g
i-1
/g
i
)
Hacer g
i+1
= g
i-1
- y
i+1
g
i
Hacer u
i+1
= u
i-1
- y
i+1
u
i
Hacer v
i+1
= v
i-1
- y
i+1
v
i
Hacer i = i+1
Si (v
i-1
< 0)
Hacer v
i-1
= v
i-1
+ B
Hacer x = v
i-1
x = inv (A, B)
x = inv (9, 25)
x = inv (9, 25) = -11+25 = 14
Ejemplo
Jorge Rami Aguirre Madrid (Espaa) 2003
Seguridad Informtica y Criptografa. Tema 5: Teora de los Nmeros
Diapositiva 250
Caractersticas de inversos en n = 27
Para el alfabeto castellano con maysculas (n = 27) tenemos:
x inv (x, 27) x inv (x, 27) x inv (x, 27)
1 1 10 19 19 10
2 14 11 5 20 23
4 7 13 25 22 16
5 11 14 2 23 20
7 4 16 22 25 13
8 17 17 8 26 26

27 = 3
3
luego no existe inverso para a = 3, 6, 9, 12, 15, 18, 21, 24.
Inversos en sistemas de
cifra clsicos orientados a
alfabetos de 27 caracteres.
inv (x, n) = a inv (a, n) = x
inv (1, n) = 1
Jorge Rami Aguirre Madrid (Espaa) 2003
Seguridad Informtica y Criptografa. Tema 5: Teora de los Nmeros
Diapositiva 251
Habr inversos si mcd (a, n) 1?
Pueden existir inversos?
No, pero...
Si a x mod n = b con b 1 y mcd (a, n) = m, siendo
m divisor de b, habr m soluciones vlidas.
Esto no nos sirve en criptografa ...
6x mod 10 = 4 mcd (6, 10) = 2
No existe inv (6, 10) pero ... habr 2 soluciones vlidas
x
1
= 4 64 mod 10 = 24 mod 10 = 4
x
2
= 9 69 mod 10 = 54 mod 10 = 4

?
Jorge Rami Aguirre Madrid (Espaa) 2003
Seguridad Informtica y Criptografa. Tema 5: Teora de los Nmeros
Diapositiva 252
Teorema del Resto Chino TRC
Si n = d
1
d
2
d
3
... d
t
con d
i
= p
i
ei
(p primo)
El sistema de ecuaciones:
x mod d
i
= x
i
(i = 1, 2, 3, ... t)
tiene una solucin comn en [0, n-1]
t
x = (n/d
i
)y
i
x
i
mod n
i=1
con y
i
= inv [(n/d
i
), d
i
]
desarrollo
Jorge Rami Aguirre Madrid (Espaa) 2003
Seguridad Informtica y Criptografa. Tema 5: Teora de los Nmeros
Diapositiva 253
Ejemplo de aplicacin del TRC (1)
Encontrar x de forma que : 12 x mod 3.960 = 36
Tenemos la ecuacin genrica: a x
i
mod d
i
= b
n = 3.960 n = 2
3
3
2
511 = d
1
d
2
d
3
d
4
= 89511
a = 12
b = 36 Como n d
4
, existirn 4 soluciones de x
i
ax
1
mod d
1
= b mod d
1
12x
1
mod 8 = 36 mod 8 = 4
ax
2
mod d
2
= b mod d
2
12x
2
mod 9 = 36 mod 9 = 0
ax
3
mod d
3
= b mod d
3
12x
3
mod 5 = 36 mod 5 = 1
ax
4
mod d
4
= b mod d
4
12x
4
mod 11 = 36 mod 11 = 3
4 ecuaciones en x
Resolviendo para x
i
Jorge Rami Aguirre Madrid (Espaa) 2003
Seguridad Informtica y Criptografa. Tema 5: Teora de los Nmeros
Diapositiva 254
Ejemplo de aplicacin del TRC (2)
x
1
= 1 x
2
= 0
x
3
= 3 x
4
= 3
4 ecuaciones en x
12x
1
mod 8 = 4 4x
1
mod 8 = 4 x
1
= 1
12x
2
mod 9 = 0 3x
2
mod 9 = 0 x
2
= 0
12x
3
mod 5 = 1 2x
3
mod 5 = 1 x
3
= 3
12x
4
mod 11 = 3 1x
4
mod 11 = 3 x
4
= 3
Jorge Rami Aguirre Madrid (Espaa) 2003
Seguridad Informtica y Criptografa. Tema 5: Teora de los Nmeros
Diapositiva 255
Ejemplo de aplicacin del TRC (3)
Resolvemos ahora la
ecuacin auxiliar del
Teorema Resto Chino
y
i
= inv [(n/d
i
), d
i
]
y
1
= 7 y
2
= 8
y
3
= 3 y
4
= 7
y
1
= inv [(n/d
1
), d
1
] y
1
= inv[(3960/8),8] = inv (495,8)
y
2
= inv [(n/d
2
), d
2
] y
2
= inv[(3960/9),9] = inv (440,9)
y
3
= inv [(n/d
3
), d
3
] y
3
= inv[(3960/5),5] = inv (792,5)
y
4
= inv[(n/d
4
), d
4
] y
4
= inv[(3960/11),11] = inv(360,11)
Jorge Rami Aguirre Madrid (Espaa) 2003
Seguridad Informtica y Criptografa. Tema 5: Teora de los Nmeros
Diapositiva 256
Ejemplo de aplicacin del TRC (4)
Aplicando ecuacin del Resto Chino
para el caso 12 x mod 3.960 = 36
con d
1
= 8, d
2
= 9, d
3
= 5, d
4
= 11:
x
1
= 1 x
2
= 0
x
3
= 3 x
4
= 3
t
x = (n/d
i
)y
i
x
i
mod n
i=1
t
x = (n/d
i
)y
i
x
i
mod n
i=1
y
1
= 7 y
2
= 8
y
3
= 3 y
4
= 7
x = [(n/d
1
)y
1
x
1
+ (n/d
2
)y
2
x
2
+ (n/d
3
)y
3
x
3
+ (n/d
4
)y
4
x
4
]
x = [49571 + 44080 + 79233 + 36073] mod 3.960
x = [3.465 + 0 + 7.128 + 7.560] mod 3.960 = 2.313
Jorge Rami Aguirre Madrid (Espaa) 2003
Seguridad Informtica y Criptografa. Tema 5: Teora de los Nmeros
Diapositiva 257
Todo marcha bien en este ejemplo?
Es la solucin de 12x mod 3.960 = 36 nica?
NO
Qu ha sucedido?
Puesto que mcd (a, n) = mcd (12, 3.960) = 12, ya hemos visto
en una diapositiva anterior que habr 12 soluciones vlidas.
x
1
= 3; x
2
= 333; x
3
= 663; x
4
= 993 ..... x
8
= 2.313 ...
x
i
= 3 + (i-1)330 mod 3.960 ... hasta llegar a x
12
= 3.633
Observe que x = 2.313, uno de los valores solucin,
fue el resultado encontrado en el ejercicio anterior.
Jorge Rami Aguirre Madrid (Espaa) 2003
Seguridad Informtica y Criptografa. Tema 5: Teora de los Nmeros
Diapositiva 258
Otros casos de aplicacin del TRC
Qu sucede ahora con:
12x mod 3.960 = 35?
12x mod 3.960 = 35
mcd (a, n) = 12 no es un
divisor de b = 35, luego
aqu no existe solucin.
Tenamos que
3.960 = 2
3
3
2
511
49x mod 3.960 = 1
S existir x, en este caso es el
inverso de 49, y ser nico ya que
49 = 77 no tiene factores en n.
Encuentre x como ejercicio
Qu sucede ahora con:
49x mod 3.960 = 1?
Jorge Rami Aguirre Madrid (Espaa) 2003
Seguridad Informtica y Criptografa. Tema 5: Teora de los Nmeros
Diapositiva 259
Slo sirve para esto el TRC?
Calcular el inverso de 49 en el cuerpo 3.960 por medio
del Teorema del Resto Chino es algo tedioso ..........
ya lo habr comprobado .
No obstante, ya habr comprobado que en este caso el
inverso de 49 en el cuerpo 3.960 es x = 889.
Para qu sirve entonces este algoritmo?
Entre otras cosas, cuando veamos el sistema de cifra
RSA y el tema dedicado a Protocolos Criptogrficos,
encontraremos una interesante aplicacin del Teorema.
Jorge Rami Aguirre Madrid (Espaa) 2003
Seguridad Informtica y Criptografa. Tema 5: Teora de los Nmeros
Diapositiva 260
Raz primitiva o generador g de grupo p
Un generador o raz primitiva de un nmero primo
p n es aquel que, elevado a todos los restos del
cuerpo y reducido mdulo n, genera todo el grupo.
As, g es un generador si: 1 a p-1
g
a
mod p = b (con 1 b p-1, todos los b )
Sea p = 3 CCR = {1,2} (el cero no es solucin)
Resto 1: no generar nada porque 1
k
mod p = 1
Resto 2: 2
1
mod 3 = 2; 2
2
mod 3 = 1
Luego el 2 es un generador del cuerpo n = 3
Jorge Rami Aguirre Madrid (Espaa) 2003
Seguridad Informtica y Criptografa. Tema 5: Teora de los Nmeros
Diapositiva 261
Cuntas races hay en un grupo?
Existen muchos nmeros dentro del CRR que son
generadores del cuerpo ... pero
Su bsqueda no es fcil ... alguna solucin?
Conociendo la factorizacin de p-1 (q
1
, q
2
, ..., q
n
)
con q
i
los factores primos de p-1, diremos que un
nmero g ser generador en p si q
i
:
g
(p-1)/qi
mod p 1
Si algn resultado es igual a 1, g no ser generador.
Ejemplo
En cambio...
Jorge Rami Aguirre Madrid (Espaa) 2003
Seguridad Informtica y Criptografa. Tema 5: Teora de los Nmeros
Diapositiva 262
Bsqueda de races primitivas (1)
BSQUEDA DE RACES EN EL CUERPO Z
13
*
Generadores en Z
13
Como p = 13 p-1 = 12 = 2
2
3
Luego: q
1
= 2 q
2
= 3
Si se cumple g
(p-1)/qi
mod p 1 q
i
entonces g ser un generador de p
g: 2,
2
(13-1)/2
mod 13 = 2
6
mod 13 = 12
2
(13-1)/3
mod 13 = 2
4
mod 13 = 3 El resto 2 es generador
Resto 2
3
(13-1)/2
mod 13 = 3
6
mod 13 = 1
3
(13-1)/3
mod 13 = 3
4
mod 13 = 3 El resto 3 no es generador
Resto 3
Jorge Rami Aguirre Madrid (Espaa) 2003
Seguridad Informtica y Criptografa. Tema 5: Teora de los Nmeros
Diapositiva 263
Bsqueda de races primitivas (2)
6, 7, g: 2, Generadores en Z
13
4
(13-1)/2
mod 13 = 4
6
mod 13 = 1
4
(13-1)/3
mod 13 = 4
4
mod 13 = 9 El resto 4 no es generador
Resto 4
5
(13-1)/2
mod 13 = 5
6
mod 13 = 12
5
(13-1)/3
mod 13 = 5
4
mod 13 = 1 El resto 5 no es generador
Resto 5
6
(13 -1)/2
mod 13 = 6
6
mod 13 = 12
6
(13-1)/3
mod 13 = 6
4
mod 13 = 9 El resto 6 es generador
Resto 6
7
(13-1)/2
mod 13 = 7
6
mod 13 = 12
7
(13-1)/3
mod 13 = 7
4
mod 13 = 9 El resto 7 es generador
Resto 7
Jorge Rami Aguirre Madrid (Espaa) 2003
Seguridad Informtica y Criptografa. Tema 5: Teora de los Nmeros
Diapositiva 264
Bsqueda de races primitivas (3)
Generadores en Z
13
6, 7, 11 g: 2,
8
(13-1)/2
mod 13 = 8
6
mod 13 = 12
8
(13-1)/3
mod 13 = 8
4
mod 13 = 1 El resto 8 no es generador
Resto 8
9
(13-1)/2
mod 13 = 9
6
mod 13 = 1
9
(13-1)/3
mod 13 = 9
4
mod 13 = 9 El resto 9 no es generador
Resto 9
10
(13-1)/2
mod 13 = 10
6
mod 13 = 1
10
(13-1)/3
mod 13 = 10
4
mod 13 = 3 El resto 10 no es generador
Resto 10
11
(13-1)/2
mod 13 = 11
6
mod 13 = 12
11
(13-1)/3
mod 13 = 11
4
mod 13 = 3 El resto 11 es generador
Resto 11
Jorge Rami Aguirre Madrid (Espaa) 2003
Seguridad Informtica y Criptografa. Tema 5: Teora de los Nmeros
Diapositiva 265
Bsqueda de races primitivas (4)
Generadores en Z
13
6, 7, 11 g: 2,
12
(13-1)/2
mod 13 = 12
6
mod 13 = 1
12
(13-1)/3
mod 13 = 12
4
mod 13 = 1 El resto 12 no es generador
Resto 12
La tasa de generadores en el grupo p
ser aproximadamente = (p-1)/(p-1).
Por lo tanto por lo general el 30% de los
elementos del Conjunto Reducido de
Restos de p ser un generador en p.
La tasa de generadores en el grupo p
ser aproximadamente = (p-1)/(p-1).
Por lo tanto por lo general el 30% de los
elementos del Conjunto Reducido de
Restos de p ser un generador en p.
= (12)/12
= 4/12 = 1/3
Jorge Rami Aguirre Madrid (Espaa) 2003
Seguridad Informtica y Criptografa. Tema 5: Teora de los Nmeros
Diapositiva 266
Generadores en cuerpos de primos seguros
Un nmero primo p se dice que es un primo seguro o primo
fuerte si: p = 2p + 1 (con p tambin primo).
Por ejemplo:
Si p = 11, luego p = 211 + 1 = 23 (es primo y es seguro)
En este caso la tasa de nmeros generadores del cuerpo ser
mayor que en el caso anterior (con p = 13 era del 30%).
Probabilidad:
pseguro
= (p-1)/p-1
Casi la mitad de los nmeros del
grupo sern generadores en p.
Comprobacin
Jorge Rami Aguirre Madrid (Espaa) 2003
Seguridad Informtica y Criptografa. Tema 5: Teora de los Nmeros
Diapositiva 267
Comprobacin de generadores en p = 2p+1
p = 11; 2p = 22; p = 2p + 1 = 23 primo seguro
Como 2p = p - 1 existirn:
(p) = [p- 1] elementos de orden (p) en el CRR
(11) = 10 = {1,2,3,4,5,6,7,8,9,10}
(2p) = [p- 1] elementos de orden (p-1) en el CRR
(22) = 10 = {1,3,5,7,9,13,15,17,19,21}
= (p- 1)/(p-1) = (p- 1)/2p
Sigue
Jorge Rami Aguirre Madrid (Espaa) 2003
Seguridad Informtica y Criptografa. Tema 5: Teora de los Nmeros
Diapositiva 268
Comprobacin de generadores en p = 2p+1
Usando la ecuacin g
(p-1)/qi
mod p
En este caso con q
1
= 2 y q
2
= 11
g
(23-1)/2
mod 23 = g
11
mod 23
g
(23-1)/11
mod 23 = g
2
mod 23
Encontramos los siguientes 10 generadores en p = 23
{5, 7, 10, 11, 14, 15, 17, 19, 20, 21}
Prcticamente la mitad de los valores de CRR que en
este caso es igual a 23 1 = 22.
Jorge Rami Aguirre Madrid (Espaa) 2003
Seguridad Informtica y Criptografa. Tema 5: Teora de los Nmeros
Diapositiva 269
Utilidad de la raz primitiva en criptografa
Para qu sirve conocer la raz primitiva de p?
La utilidad de este concepto en
criptografa lo veremos cuando
se estudien los sistemas de clave
pblica y, en particular, el
protocolo de intercambio de
claves de Diffie y Hellman.
Tambin se recurrir a esta
propiedad de los primos cuando
estudiemos la firma digital
segn estndar DSS (ElGamal).
?
Jorge Rami Aguirre Madrid (Espaa) 2003
Seguridad Informtica y Criptografa. Tema 5: Teora de los Nmeros
Diapositiva 270
La exponenciacin en la cifra asimtrica
Una de las ms interesantes aplicaciones de la
matemtica discreta en criptografa es la cifra
asimtrica en la que la operacin bsica es una
exponenciacin A
B
mod n, en donde n es un
primo o un producto de primos grandes.
Esta operacin se realizar en el intercambio de
clave y en la firma digital.
Cmo hacer estos clculos de forma rpida y
eficiente, sin tener que aplicar la reducibilidad?
Tenemos una solucin aplicando un algoritmo
de exponenciacin rpida.
Jorge Rami Aguirre Madrid (Espaa) 2003
Seguridad Informtica y Criptografa. Tema 5: Teora de los Nmeros
Diapositiva 271
Un mtodo de exponenciacin rpida
En x
y
mod n se representa el exponente y en binario.
Se calculan los productos x
2
j
con j = 0 hasta n-1, siendo n
el nmero de bits que representan el valor y en binario.
Slo se toman en cuenta los productos en los que en la
posicin j del valor y en binario aparece un 1.
Ejemplo
Calcular z = 12
37
mod 221 = 207
12
37
es un nmero de 40 dgitos:
8505622499821102144576131684114829934592
Jorge Rami Aguirre Madrid (Espaa) 2003
Seguridad Informtica y Criptografa. Tema 5: Teora de los Nmeros
Diapositiva 272
Ejemplo de exponenciacin rpida
Calcular z = 12
37
mod 221 = 207
37
10
= 100101
2
mod 221
j 0 1 2 3 4 5
12 144 183 118 1 1
interesante
12
2
mod 221
144
2
mod 221
x
2
j
x = 12
Bits 5 4 3 2 1 0
z = 121831 mod 221 = 207
En vez de 36 multiplicaciones y sus reducciones mdulo
221 en cada paso ... 72 operaciones...
Hemos realizado cinco multiplicaciones (para j = 0 el valor
es x) con sus reducciones mdulo 221, ms dos al final y su
correspondiente reduccin. Un ahorro superior al 80% .
Jorge Rami Aguirre Madrid (Espaa) 2003
Seguridad Informtica y Criptografa. Tema 5: Teora de los Nmeros
Diapositiva 273
Algoritmo de exponenciacin rpida
Ejemplo: calcule 19
83
mod 91 = 24
Hallar x = A
B
mod n
Obtener representacin
binaria del exponente B
de k bits:
B
2
b
k-1
b
k-2
...b
i
...b
1
b
0
Hacer x = 1
Para i = k-1, ..., 0 hacer
x = x
2
mod n
Si (b
i
= 1) entonces
x = xA mod n
83
10
= 1010011
2
= b
6
b
5
b
4
b
3
b
2
b
1
b
0
x = 1
i=6 b
6
=1 x = 1
2
19 mod 91 = 19 x = 19
i=5 b
5
=0 x = 19
2
mod 91 = 88 x = 88
i=4 b
4
=1 x = 88
2
19 mod 91 = 80 x = 80
i=3 b
3
=0 x = 80
2
mod 91 = 30 x = 30
i=2 b
2
=0 x = 30
2
mod 91 = 81 x = 81
i=1 b
1
=1 x = 81
2
19 mod 91 = 80 x = 80
i=0 b
0
=1 x = 80
2
19 mod 91 = 24 x = 24
19
83
= 1,369458509879505101557376746718e+106 (calculadora Windows)
Jorge Rami Aguirre Madrid (Espaa) 2003
Seguridad Informtica y Criptografa. Tema 5: Teora de los Nmeros
Diapositiva 274
Clculos en campos de Galois (GF)
Cuando trabajamos en un cuerpo primo p, sabemos que se
asegura la existencia de un nico inverso multiplicativo.
En este caso se dice que estamos trabajando en Campos
de Galois GF(p).
Algunos usos en criptografa:
Sistemas de clave pblica cuando la operacin de
cifra es C = M
e
mod p (cifrador ElGamal) o bien RSA
usando el Teorema del Resto Chino para descifrar.
Aplicaciones en GF(q
n
), polinomios mdulo q y de
grado n: a(x) = a
n-1
x
n-1
+ a
n-2
x
n-2
+ ... + a
1
x + a
0
:
Cifrador de flujo A5, RIJNDAEL, curvas elpticas.
Jorge Rami Aguirre Madrid (Espaa) 2003
Seguridad Informtica y Criptografa. Tema 5: Teora de los Nmeros
Diapositiva 275
Campos de Galois del tipo GF(q
n
)
a(x) = a
n-1
x
n-1
+ a
n-2
x
n-2
+ ... + a
1
x + a
0
Es un polinomio de grado n-1 o menor.
Los elementos a
i
son parte del CCR del mdulo q.
Cada elemento a(x) es un resto mdulo p(x), siendo
p(x) un polinomio irreducible de grado n (que no puede
ser factorizado en polinomios de grado menor que n).
GF(2
n
) es interesante porque CCR(2) = {0, 1} bits.
GF(2
3
) 8 elementos o restos polinmicos que son:
0, 1, x, x+1, x
2
, x
2
+1, x
2
+x, x
2
+x+1, los 8 restos de un
polinomio de grado n-1 (n = 3).
Jorge Rami Aguirre Madrid (Espaa) 2003
Seguridad Informtica y Criptografa. Tema 5: Teora de los Nmeros
Diapositiva 276
Suma en campos de Galois GF(2
n
)
Si el mdulo de trabajo es 2 (con restos = bits 0 y 1), las
operaciones suma y resta sern un OR Exclusivo:
0 1 mod 2 = 1 1 0 mod 2 = 1
0 0 mod 2 = 0 1 1 mod 2 = 0
0 1 x x+1
0 0 1 x x+1
1 1 0 x+1 x
x x x+1 0 1
x+1 x+1 x 1 0
Restos: 0, 1, x, x+1
CG(2
2
)
Como los resultados debern
pertenecer al cuerpo, aplicaremos
Reduccin por Coeficientes:
Por ejemplo:
x + (x +1) = 2x + 1 mod 2 = 1
1 + 1 = 2 mod 2 = 0
Jorge Rami Aguirre Madrid (Espaa) 2003
Seguridad Informtica y Criptografa. Tema 5: Teora de los Nmeros
Diapositiva 277
Producto en campos de Galois GF(2
n
)
La operacin multiplicacin puede entregar elementos
que no pertenezcan al cuerpo, potencias iguales o
mayores que n Reduccin por Exponente.
0 1 x x+1
0 0 0 0 0
1 0 1 x x+1
x 0 x x+1 1
x+1 0 x+1 1 x
Restos: 0, 1, x, x+1
CG(2
2
) Sea el polinomio irreducible de
grado n = 2, p(x) = x
2
+ x + 1
Luego: x
2
= x + 1
Por ejemplo:
(x + 1)(x + 1) = x
2
+ 2x + 1
(x + 1)(x + 1) = (x + 1) + 2x +1
(x + 1)(x + 1) = 3x + 2 mod 2 = x
Jorge Rami Aguirre Madrid (Espaa) 2003
Seguridad Informtica y Criptografa. Tema 5: Teora de los Nmeros
Diapositiva 278
Operaciones con campos de Galois en AES
La suma y multiplicacin de polinomios dentro de un
cuerpo binario descritas en las dispositivas anteriores,
conforman las operaciones bsicas del algoritmo de
cifra Advanced Encryption Algorithm AES, que con
el nombre RIJNDAEL es el estndar mundial desde
finales de 2001, desplazando al ya viejo DES.
En este caso, se trabaja con 8 bits por lo que las
operaciones se realizan en GF(2
8
). En el captulo 10
sobre sistemas de cifra con clave secreta encontrar
ejemplos de suma y multiplicacin polinmica dentro
de este cuerpo binario.
Fin del Tema 5
Jorge Rami Aguirre Madrid (Espaa) 2003
Seguridad Informtica y Criptografa. Tema 5: Teora de los Nmeros
Diapositiva 279
Cuestiones y ejercicios (1 de 3)
1. Qu significa para la criptografa el homomorfismo de los enteros?
2. Si una funcin de cifra multiplica el mensaje por el valor a dentro
del cuerpo n, para qu nos sirve conocer el inverso de a en n?
3. En un cuerpo de cifra n, existen siempre los inversos aditivos y los
inversos multiplicativos? Debe cumplirse alguna condicin?
4. Si en un cuerpo n el inverso de a es a
1
, es ese valor nico?
5. Cifraremos en un cuerpo n = 131. Cul es el conjunto completo de
restos? Cul es el conjunto reducido de restos?
6. Para cifrar un mensaje M = 104 debemos elegir el cuerpo de cifra
entre el valor n = 127 y n = 133, cul de los dos usara y por qu?
7. Qu nos dice la funcin (n) de Euler?
8. Qu papel cumple el algoritmo extendido de Euclides en la
criptografa? Por qu es importante? En qu se basa?
Jorge Rami Aguirre Madrid (Espaa) 2003
Seguridad Informtica y Criptografa. Tema 5: Teora de los Nmeros
Diapositiva 280
Cuestiones y ejercicios (2 de 3)
9. Si en el cuerpo n = 37 el inv (21, 37) = 30, cul es el inv (30, 37)?
10. Usando el algoritmo extendido de Euclides calcule los siguientes
inversos: inv (7, 19); inv (21, 52), inv (11, 33), inv (41, 43).
11. Cuntas soluciones x
i
hay a la expresin 8x mod 20 = 12?
Explique lo que sucede. Tiene esto inters en criptografa?
12. Qu viene a significar el Teorema del Resto Chino? Aunque an
no lo ha estudiado, le ve alguna utilidad en criptografa?
13. Calcule inv (49, 390) usando el Teorema del Resto Chino.
14. Defina lo que es una raz primitiva o generador de un cuerpo. Es
necesario que ese cuerpo sea un primo?
15. Cuntos generadores podemos esperar en el cuerpo n = 17? Y si
ahora n = 7, cuntos generadores habr? Comprubelo calculando
todos los exponentes del conjunto completo de restos de n = 7.
Jorge Rami Aguirre Madrid (Espaa) 2003
Seguridad Informtica y Criptografa. Tema 5: Teora de los Nmeros
Diapositiva 281
Cuestiones y ejercicios (3 de 3)
16. Cmo se define un primo seguro? Cuntos generadores tiene?
17. A partir de los valores p = 13, p = 17, p = 19 y p = 23 queremos
obtener un primo seguro, con cul o cules de ellos lo logramos?
18. Usando el algoritmo de exponenciacin rpida calcule los siguientes
valores: 23
32
mod 51; 100
125
mod 201; 1.000
100.000
mod 2.500.
19. Compruebe los resultados con la calculadora de Windows. Qu
sucede para nmeros muy grandes como el ltimo?
20. Cuntas operaciones bsicas se han hecho en cada caso y en cunto
se ha optimizado el clculo?
21. En GF(2
n
) reduzca por coeficientes 5x
5
+ x
4
+ 2x
3
+ 3x
2
+ 6x + 2.
22. Reduzca (x
3
+ 1)(x
2
+ x +1) por exponente en GF(2
n
) usando como
polinomio primitivo p(x) = x
4
+ x + 1, es decir x
4
= x + 1.
Tema 6
Teora de la Complejidad Algortmica
Seguridad Informtica y Criptografa
Ultima actualizacin: 03/03/03
Archivo con 28 diapositivas
Jorge Rami Aguirre
Universidad Politcnica de Madrid
Material Docente de
Libre Distribucin
v 3.1
Este archivo forma parte de un curso sobre Seguridad Informtica y Criptografa. Se autoriza la
reproduccin en computador e impresin en papel slo con fines docentes o personales, respetando
en todo caso los crditos del autor. Queda prohibida su venta, excepto a travs del Departamento de
Publicaciones de la Escuela Universitaria de Informtica, Universidad Politcnica de Madrid, Espaa.
Curso de Seguridad Informtica y Criptografa JRA
Jorge Rami Aguirre Madrid (Espaa) 2003
Seguridad Informtica y Criptografa. Tema 6: Teora de la Complejidad Algortmica
Diapositiva 284
Nota del autor
El contenido de este tema
corresponde slo a una breve
introduccin a la complejidad
de los algoritmos, con el objeto
de que el lector pueda hacerse
una idea de la importancia de
este tema en el anlisis y diseo de los algoritmos de
cifra y firma digital que se vern en este curso. La
fortaleza de estos algoritmos y de los protocolos que
los incluyen depender de la complejidad asociada al
criptoanlisis o ataque de los mismos.
Jorge Rami Aguirre Madrid (Espaa) 2003
Seguridad Informtica y Criptografa. Tema 6: Teora de la Complejidad Algortmica
Diapositiva 285
Introduccin a la teora de la complejidad
La teora de la complejidad de los algoritmos nos
permitir conocer si un algoritmo tiene fortaleza y tener
as una idea de su vulnerabilidad computacional.
Complejidad Computacional
Los algoritmos se clasifican segn el tiempo de
ejecucin y en funcin del tamao de la entrada.
Complejidad Polinomial
Complejidad Exponencial
Esto dar lugar a tipos de problemas que nos interesarn.
Jorge Rami Aguirre Madrid (Espaa) 2003
Seguridad Informtica y Criptografa. Tema 6: Teora de la Complejidad Algortmica
Diapositiva 286
Operaciones bit en la suma
Si deseamos sumar dos nmeros binarios n y m, ambos de k bits
realizaremos k operaciones bit puesto que cada operacin bsica
con los dgitos de una columna es una operacin bit.
* Recuerde que 0+0 = 0, 0+1=1, 1+0 = 1, 1+1 = 0 con bit 1 de acarreo.
Si un nmero tiene menos bits, se rellena con ceros por la izquierda.
Ejemplo: encontrar el nmero de operaciones bit necesarias en la
suma en binario de 13+7 1101 + 0111 (de k = 4 bits)
1 1 1 1 (acarreo)
1 1 0 1
+ 0 1 1 1
1 0 1 0 0
Cada operacin bsica que hacemos con una columna se conoce
como operacin bit, luego necesitamos k = 4 operaciones bit.
Jorge Rami Aguirre Madrid (Espaa) 2003
Seguridad Informtica y Criptografa. Tema 6: Teora de la Complejidad Algortmica
Diapositiva 287
Operaciones bit en la multiplicacin
Para la multiplicacin de un nmero n de k bits por un nmero
m de h bits, el nmero de operaciones bit ser igual a 2kh.
Suponemos que k h.
* Recuerde que 0x0 = 0, 0x1=0, 1x0 = 0, 1x1 = 1.
Ejemplo: encontrar el nmero de operaciones bit necesarias en la
multiplicacin en binario 10x5 1010 x 101 (4 y 3 bits)
1 0 1 0 x 1 0 1
1 0 1 0
0 0 0 0
+ 1 0 1 0 (procedemos ahora a sumar)
1 1 0 0 1 0
Como cada operacin bsica entre dos bits es una operacin bit,
hemos realizado hk = 34 multiplicaciones y luego kh = 43
sumas, es decir en total 2kh = 24 operaciones bit.
Jorge Rami Aguirre Madrid (Espaa) 2003
Seguridad Informtica y Criptografa. Tema 6: Teora de la Complejidad Algortmica
Diapositiva 288
La funcin O(n)
Las operaciones dependern del tamao de la entrada por lo
que esta complejidad se expresar en trminos del tiempo T
necesario para el clculo del algoritmo y del espacio S que
utiliza en memoria, y se expresar mediante una funcin
f (n), donde n es el tamao de la entrada.
Esta funcin ser una aproximacin pues el resultado exacto
depender de la velocidad del procesador.
f (n) = O(g(n))
f = O(n) ssi c
o
,n
o
/ f(n) c
o
g(n)
Ejemplo
Jorge Rami Aguirre Madrid (Espaa) 2003
Seguridad Informtica y Criptografa. Tema 6: Teora de la Complejidad Algortmica
Diapositiva 289
Complejidad de una funcin f(n)
Si f (n) = 4n
2
+ 2n + 5 f = O(n
2
)?
se cumple que c
o
g(n) = c
o
n
2
f (n)? Sea c
o
= 6
c
o
n
o
c
o
n
o
2
f (n) = 4n
2
+ 2n + 5 c
o
n
2
f (n)?
6 1 6 11 No
6 2 24 25 No
6 3 54 38 S
6 4 96 77 S
Se cumple
siempre
Luego, la complejidad de f (n) es exponencial.
Jorge Rami Aguirre Madrid (Espaa) 2003
Seguridad Informtica y Criptografa. Tema 6: Teora de la Complejidad Algortmica
Diapositiva 290
Tiempos de ejecucin
En la expresin O(n) aparecer el trmino que domina al
crecer el valor de n.
El tiempo de ejecucin de un algoritmo T
1
que realiza
2n+1 operaciones es de tipo O(n); uno T
2
que realiza
3n
2
+n+3 operaciones ser de tipo O(n
2
), etc.
Para realizar la suma de la diapositiva anterior necesitamos
O(n) = O(log n) operaciones bit y para el caso de la
multiplicacin, stas sern O(nm) = O(log n log m)
operaciones bit.
+ Operacin binaria: n+m (de k bits cada uno)
Operacin binaria: nm (de k y h bits respectivamente)
Jorge Rami Aguirre Madrid (Espaa) 2003
Seguridad Informtica y Criptografa. Tema 6: Teora de la Complejidad Algortmica
Diapositiva 291
Algoritmos de complejidad lineal
Un algoritmo se dice que tiene tiempo de ejecucin
polinomial si ste depende polinmicamente del
tamao de la entrada.
Si la entrada es de tamao n y t es un entero, el
nmero de operaciones bit ser O(log
t
n).
Si t = 1, el sistema es lineal Suma
Si t = 2, el sistema es cuadrtico Producto
Si t = 3, el sistema es cbico mcd Euclides
Ejemplos
Jorge Rami Aguirre Madrid (Espaa) 2003
Seguridad Informtica y Criptografa. Tema 6: Teora de la Complejidad Algortmica
Diapositiva 292
Ejemplo de complejidad lineal
Ejemplo: El tiempo de ejecucin de un algoritmo es
O(log
3
n). Si doblamos la entrada, en cunto aumenta
este tiempo?
Solucin: En el primer caso el tiempo es O(log
3
n) y en
el segundo O(log
3
2n). Luego para este sistema lineal
el tiempo se incrementar en log
3
2 operaciones bit.
Estos son los denominados problemas fciles y son
los que involucrarn un proceso de cifra y descifrado
(o firma) por parte del o de los usuarios autorizados.
Jorge Rami Aguirre Madrid (Espaa) 2003
Seguridad Informtica y Criptografa. Tema 6: Teora de la Complejidad Algortmica
Diapositiva 293
Algoritmos de complejidad exponencial
Un algoritmo se dice que tiene tiempo de ejecucin
exponencial si ste depende exponencialmente del
tamao de la entrada.
Si la entrada es de tamao n y t es un entero, el
nmero de operaciones bit ser O(n
t
).
Para t = 2, ser exponencial de orden 2
Para t = 3, ser exponencial de orden 3
n!
Ejemplo
Jorge Rami Aguirre Madrid (Espaa) 2003
Seguridad Informtica y Criptografa. Tema 6: Teora de la Complejidad Algortmica
Diapositiva 294
Ejemplo de complejidad exponencial
Ejemplo: El tiempo de ejecucin de un algoritmo es
O(n
3
). Si doblamos la entrada, en cunto aumenta
este tiempo?
Solucin: En el primer caso el tiempo es O(n
3
) y en el
segundo O(2n
3
) = O(8n
3
). Para este sistema exponencial
el tiempo se incrementar en 8 operaciones bit.
Estos son los denominados problemas difciles y son a
los que deber enfrentarse un criptoanalista o atacante
que desea romper una cifra o la clave de un usuario.
Jorge Rami Aguirre Madrid (Espaa) 2003
Seguridad Informtica y Criptografa. Tema 6: Teora de la Complejidad Algortmica
Diapositiva 295
Comparativas de complejidad
Los algoritmos polinmicos y exponenciales se
comparan por su complejidad O(n
t
).
Polinmico constante O(1)
Polinmico lineal O(n)
Polinmico cuadrtico O(n
2
)
Polinmico cbico O(n
3
) ... etc.
Exponencial O(d
h(n)
)
donde d es una constante y h(n) un polinomio
Si suponemos un ordenador capaz de realizar
10
9
instrucciones por segundo se tiene el cuadro:
Jorge Rami Aguirre Madrid (Espaa) 2003
Seguridad Informtica y Criptografa. Tema 6: Teora de la Complejidad Algortmica
Diapositiva 296
Tabla comparativa de tiempos
Entrada O(n) O(n
2
) O(n
3
) O(2
n
)
n = 10 10
-8
seg 10
-7
seg 10
-6
seg 10
-6
seg
n = 10
2
10
-7
seg 10
-5
seg 10
-3
seg 410
13
aos
n = 10
3
10
-6
seg 10
-3
seg 1 seg Muy grande
Incrementos de un
orden de magnitud
Computacionalmente
imposible
Entrada/10
9
: Para n = 100 O(n
2
) = 100
2
/10
9
= 10
-5
seg
Jorge Rami Aguirre Madrid (Espaa) 2003
Seguridad Informtica y Criptografa. Tema 6: Teora de la Complejidad Algortmica
Diapositiva 297
Problemas de tipo NP
En criptografa nos interesan las funciones f(x) de un solo
sentido, es decir:
Fcil calcular f(x) pero muy difcil calcular f
-1
(x)
salvo que conozcamos un secreto o trampa
Porque dan lugar a problemas tipo NP, polinomiales no
deterministas, computacionalmente difciles de tratar.
Problema de la mochila
Problema de la factorizacin
Problema del logaritmo discreto
Otros ...
Veremos cada
uno de ellos
Jorge Rami Aguirre Madrid (Espaa) 2003
Seguridad Informtica y Criptografa. Tema 6: Teora de la Complejidad Algortmica
Diapositiva 298
El problema de la mochila
Enunciado:
Dada una mochila de determinadas dimensiones de alto,
ancho y fondo, y un conjunto de elementos de distintos
tamaos menores que ella y de cualquier dimensin, ...
es posible llenar la mochila (completa) con distintos
elementos de ese conjunto sin repetir ninguno de ellos?
Es un problema de tipo NP en el
que el algoritmo debe realizar en
cada paso una seleccin iterativa
entre diferentes opciones.
Jorge Rami Aguirre Madrid (Espaa) 2003
Seguridad Informtica y Criptografa. Tema 6: Teora de la Complejidad Algortmica
Diapositiva 299
Ejemplo del problema de la mochila
S
1
= a
1
+a
2
+a
3
S
2
= a
1
+a
2
S
3
= a
1
+a
3
S
4
= a
1
S
5
= a
2
+a
3
S
6
= a
2
S
7
= a
3
S
8
=
S = a
1
+ a
2
+ a
3
A = {a
1
, a
2
, a
3
}
Se incluye a
1
en la suma S?
S No
Se incluye a
2
en la suma?
S No S No
Se incluye a
3
?
S No S No S No S No
S
1
S
2
a
1
a
3
a
3
a
2
S
3
S
4
S
7
S
8
S
5
S
6
a
3
a
3
a
2
Sea una mochila
con 4 elementos
{2, 4, 9, 10}
Cuntas sumas
posibles hay?
Solucin: 2
4
= 16
, 2, 4, 6,
9, 10, 11, 12,
13, 14, 15, 16,
19, 21, 23, 25.
Los resultados son todos
distintos: una casualidad
Repita con
{2, 4, 6, 10}

Hemos tenido que evaluar 2


3
= 8 valores (carcter exponencial)
Jorge Rami Aguirre Madrid (Espaa) 2003
Seguridad Informtica y Criptografa. Tema 6: Teora de la Complejidad Algortmica
Diapositiva 300
Inters de las mochilas en criptografa
Por qu tiene inters este problema en criptografa?
a) Es de tipo NP completo: su resolucin por lo general
implica una complejidad exponencial. Luego, ser
difcil de atacar o criptoanalizar.
b) Existe un caso en el que la resolucin es lineal y, si la
solucin existe, es nica. Se da si A = {a
1
, a
2
, a
3
, .., a
n
}
est ordenado de menor a mayor y en donde cada a
i
es
mayor que la suma de los a
j
que le preceden.
Esto dar lugar a los criptosistemas de mochila
tramposa que veremos en un prximo captulo.
Jorge Rami Aguirre Madrid (Espaa) 2003
Seguridad Informtica y Criptografa. Tema 6: Teora de la Complejidad Algortmica
Diapositiva 301
El problema de la factorizacin PFNG
Dado un nmero n que es el resultado del producto de
dos primos n = pq, se pide encontrar estos factores.
Cuando el valor n es muy grande, el Problema de la
Factorizacin de Nmeros Grandes PFNG se vuelve
computacionalmente intratable.
No obstante, el caso inverso, dado dos nmeros p y q,
encontrar el resultado pq = n, se trata de un problema
de tipo polinomial.
Este problema se usar en la generacin del par de
claves del sistema de cifra con clave pblica RSA.
Jorge Rami Aguirre Madrid (Espaa) 2003
Seguridad Informtica y Criptografa. Tema 6: Teora de la Complejidad Algortmica
Diapositiva 302
Un ejemplo del PFNG
Clculo fcil o polinomial (funcin directa)
Calcule a mano los siguientes productos de dos primos y
tome el tiempo aproximado que tarda en la operacin:
a) 1331 b) 113131 c) 1.0131.031 calcule...
Clculo difcil o no polinomial (funcin inversa)
Usando la criba de Eratstenes, factorice en dos primos los
siguientes nmeros y vuelva a tomar el tiempo empleado:
a) 629 b) 17.399 c) 1.052.627 calcule...
En el caso a) son primos de 2 dgitos, en c) de 3 y en d) de 4.
No vale usar
calculadora...
A qu conclusiones
puede llegar ahora?
Jorge Rami Aguirre Madrid (Espaa) 2003
Seguridad Informtica y Criptografa. Tema 6: Teora de la Complejidad Algortmica
Diapositiva 303
Solucin al ejemplo anterior
Clculo fcil o polinomial
a) 1331 = 403 b) 113131 = 14.803 c) 10131031 = 1.044.403
A medida que aumenta el tamao de la entrada, el tiempo de
clculo aumenta proporcionalmente con los dgitos.
Clculo difcil o no polinomial
a) 629 b) 17.399 c) 1.052.627
Aqu resulta evidente que el tiempo de clculo (da igual que el
algoritmo sea ste u otro ms depurado y eficaz) aumenta mucho al
incrementar en un dgito los nmeros en cuestin.
Solucin: a), b) y c) son el producto de los nmeros primos
inmediatamente superiores a los de arriba (ver tabla en SItema18).
Un computador
experimentar lo
mismo....
Jorge Rami Aguirre Madrid (Espaa) 2003
Seguridad Informtica y Criptografa. Tema 6: Teora de la Complejidad Algortmica
Diapositiva 304
El problema del logaritmo discreto PLD
Dado un par de enteros y que pertenecen al Campo
de Galois GF(p), se pide encontrar un entero x de forma
que x = log

mod p.
Si el valor p es muy grande, el Problema del Logaritmo
Discreto PLD es computacionalmente intratable.
No obstante, el caso inverso, dado dos nmeros y x,
encontrar =
x
mod p es un problema polinomial.
Este problema se usar en la creacin de las claves del
sistema de cifra con clave pblica ElGamal y en el
protocolo de intercambio de clave de Diffie y Hellman.
Jorge Rami Aguirre Madrid (Espaa) 2003
Seguridad Informtica y Criptografa. Tema 6: Teora de la Complejidad Algortmica
Diapositiva 305
Un ejemplo del PLD
Clculo fcil o polinomial (funcin directa)
Calcule a mano las siguientes exponenciaciones mod p y
tome el tiempo aproximado que tarda en la operacin:
a) 5
4
mod 7 b) 8
17
mod 41 c) 92
11
mod 251
5
4
= 625
8
17
= 2.251.799.813.685.248
92
11
= 3.996.373.778.857.415.671.808
Nota: Haciendo uso de la propiedad de reducibilidad del captulo 5,
podr reducir significativamente el tiempo de clculo. No obstante,
este tiempo ser de tipo polinomial segn el tamao de la entrada.
Jorge Rami Aguirre Madrid (Espaa) 2003
Seguridad Informtica y Criptografa. Tema 6: Teora de la Complejidad Algortmica
Diapositiva 306
Solucin al ejemplo anterior
Clculo difcil o no polinomial (funcin inversa)
Aunque existen varios algoritmos para este tipo de clculos
(al igual que para la factorizacin) use la fuerza bruta que se
explica a continuacin para encontrar los siguientes valores y
vuelva a tomar el tiempo empleado:
a) log
5
2 mod 7 b) log
8
39 mod 41 c) log
92
217 mod 251
Aplicando fuerza bruta en el 1
er
caso (la base elevada a todos
los restos de p) al final se obtiene que log
5
2 mod 7 = 4.
5
1
mod 7 = 5 5
2
mod 7 = 4 5
3
mod 7 = 6
5
4
mod 7 = 2 5
5
mod 7 = 3 5
6
mod 7 = 1
En media deber
recorrer la mitad
del espacio...
Jorge Rami Aguirre Madrid (Espaa) 2003
Seguridad Informtica y Criptografa. Tema 6: Teora de la Complejidad Algortmica
Diapositiva 307
Logaritmo discreto con generador
log
2
1 mod 13 = 0 log
2
2 mod 13 = 1 log
2
3 mod 13 = 4
log
2
4 mod 13 = 2 log
2
5 mod 13 = 9 log
2
6 mod 13 = 5
log
2
7 mod 13 = 11 log
2
8 mod 13 = 3 log
2
9 mod 13 = 8
log
2
10 mod 13 = 10 log
2
11 mod 13 = 7 log
2
12 mod 13 = 6
2
0
mod 13 = 1 2
1
mod 13 = 2 2
2
mod 13 = 4
2
3
mod 13 = 8 2
4
mod 13 = 3 2
5
mod 13 = 6
2
6
mod 13 = 12 2
7
mod 13 = 11 2
8
mod 13 = 9
2
9
mod 13 = 5 2
10
mod 13 = 10 2
11
mod 13 = 7
Es
decir
Se cumplir adems que a
p-1
mod p = a
0
mod p = 1.
Jorge Rami Aguirre Madrid (Espaa) 2003
Seguridad Informtica y Criptografa. Tema 6: Teora de la Complejidad Algortmica
Diapositiva 308
Logaritmo discreto con no generador
En p=13, el 2
es generador,
pero no as el
nmero 3...
Luego
3
0
mod 13 = 1 3
1
mod 13 = 3 3
2
mod 13 = 9
3
3
mod 13 = 1 3
4
mod 13 = 3 3
5
mod 13 = 9
3
6
mod 13 = 1 3
7
mod 13 = 3 3
8
mod 13 = 9
3
9
mod 13 = 1 3
10
mod 13 = 3 3
11
mod 13 = 9
log
3
1 mod 13 = 0 log
3
2 mod 13 = NE log
3
3 mod 13 = 1
log
3
4 mod 13 = NE log
3
5 mod 13 = NE log
3
6 mod 13 = NE
log
3
7 mod 13 = NE log
3
8 mod 13 = NE log
3
9 mod 13 = 2
log
3
10 mod 13 = NE log
3
11 mod 13 = NE log
3
12 mod 13 = NE
NE = no existe
Jorge Rami Aguirre Madrid (Espaa) 2003
Seguridad Informtica y Criptografa. Tema 6: Teora de la Complejidad Algortmica
Diapositiva 309
Hay ms funciones NP?
Existen otros problemas matemticos que dan lugar a
problemas del tipo NP basados en estas funciones
unidireccionales (one way functions) pero las dos
ltimas funciones vistas factorizacin de nmeros
grandes y logaritmo discreto- son las que ms uso
tienen, de momento, en la criptografa.
Algunos de ellos se presentarn en el Tema dedicado
a los Protocolos Criptogrficos.
Fin del Tema 6
Jorge Rami Aguirre Madrid (Espaa) 2003
Seguridad Informtica y Criptografa. Tema 6: Teora de la Complejidad Algortmica
Diapositiva 310
Cuestiones y ejercicios
1. Deseamos sumar de forma binaria el nmero 15 (1111) y el nmero
10 (1010), ambos de k = 4 bits. Haga la suma binaria y verifique
que el nmero de operaciones bit desarrolladas es k = 4.
2. Si multiplicamos en binario 101011, donde k = 4 bits y h = 2 bits,
compruebe que el nmero de operaciones bit realizadas es 2kh.
3. Por qu son interesantes los problemas de tipo NP en criptografa?
4. Defina el problema de la mochila y su posible utilizacin en un
sistema de cifra.
5. Factorice mentalmente el valor n = 143. Intente hacer lo mismo
para n = 1.243. Qu opina ahora del problema de la factorizacin?
6. A partir de la ecuacin = x

mod p, defina el problema del


logaritmo discreto. Qu utilidad tiene en criptografa?
Tema 7
Sistemas de Cifra Clsicos
Seguridad Informtica y Criptografa
Ultima actualizacin:17/03/03
Archivo con 40 diapositivas
Jorge Rami Aguirre
Universidad Politcnica de Madrid
Material Docente de
Libre Distribucin
v 3.1
Este archivo forma parte de un curso sobre Seguridad Informtica y Criptografa. Se autoriza la
reproduccin en computador e impresin en papel slo con fines docentes o personales, respetando
en todo caso los crditos del autor. Queda prohibida su venta, excepto a travs del Departamento de
Publicaciones de la Escuela Universitaria de Informtica, Universidad Politcnica de Madrid, Espaa.
Curso de Seguridad Informtica y Criptografa JRA
Jorge Rami Aguirre Madrid (Espaa) 2003
Seguridad Informtica y Criptografa. Tema 7: Sistemas de Cifra Clsicos
Diapositiva 312
Nota del autor
El contenido de este tema
corresponde a unos primeros
apuntes. No tiene la importancia
de los dems temas tratados en
este libro, pero si lo desea puede
encontrar una explicacin ms detallada sobre
la historia de la criptografa y de los sistemas de cifra
clsicos en el libro gua de la asignatura comentado
en la presentacin de estos apuntes o, en su caso,
usando el Libro Electrnico de Criptografa Clsica,
software de libre distribucin que puede descargar
desde Internet como se comenta al final del archivo.
Jorge Rami Aguirre Madrid (Espaa) 2003
Seguridad Informtica y Criptografa. Tema 7: Sistemas de Cifra Clsicos
Diapositiva 313
Clasificacin histrica de criptosistemas
Los criptosistemas pueden clasificarse segn:
a) Su relacin con la Historia en:
- Sistemas Clsicos y Sistemas Modernos
No es sta ni mucho menos la mejor clasificacin desde
el punto de vista de la ingeniera y la informtica ...
No obstante, permitir comprobar el desarrollo de estas
tcnicas de cifra hoy en da rudimentarias y en algunos
casos simples, desde una perspectiva histrica que es
interesante como cultura general para todo ingeniero.
Jorge Rami Aguirre Madrid (Espaa) 2003
Seguridad Informtica y Criptografa. Tema 7: Sistemas de Cifra Clsicos
Diapositiva 314
Clasificacin actual de los criptosistemas
o bien segn:
b) El tratamiento de la informacin a cifrar en:
- Cifrado en Bloque y Cifrado en Flujo
c) El tipo de clave utilizada en la cifra en:
- Sistema con Clave Secreta y Sistema con Clave Pblica
Se ver en Temas 8 y 9 Cifra en flujo
Cifra en bloque Se ver en Temas 8 y 10
Se ver en Temas 10 y 14 Cifra con clave secreta
Se ver en Temas 11, 12 y 14 Cifra con clave pblica
Jorge Rami Aguirre Madrid (Espaa) 2003
Seguridad Informtica y Criptografa. Tema 7: Sistemas de Cifra Clsicos
Diapositiva 315
Primera aproximacin histrica
La criptografa es casi tan antigua como las
primeras civilizaciones de nuestro planeta.
Ya en el siglo V antes de J.C. se usaban tcnicas
de cifra para proteger a la informacin.
Se pretenda garantizar slo la confidencialidad y
la autenticidad de los mensajes.
Los mayores avances se lograron en la Segunda
Guerra Mundial: los pases en conflicto tenan un
gran nmero de tcnicos encargados de romper
los mensajes cifrados de los teletipos.
Jorge Rami Aguirre Madrid (Espaa) 2003
Seguridad Informtica y Criptografa. Tema 7: Sistemas de Cifra Clsicos
Diapositiva 316
Herramientas de la criptografa clsica
Tanto mquinas, artilugios de cifra, como los
algoritmos que trabajaban matemticamente
dentro de un cuerpo finito n, hacen uso de dos
tcnicas bsicas orientadas a caracteres y que,
muchos siglos despus, propone Shannon:
Sustitucin: un carcter o letra se modifica o
sustituye por otro elemento en la cifra.
Transposicin: los caracteres o letras del mensaje
se redistribuyen sin modificarlos y segn unas
reglas, dentro del criptograma. Tambin se le
conoce como permutacin.
Jorge Rami Aguirre Madrid (Espaa) 2003
Seguridad Informtica y Criptografa. Tema 7: Sistemas de Cifra Clsicos
Diapositiva 317
TRANSPOSICIN
SUSTITUCIN
MONOGRMICA POLIGRMICA NO PERIDICA PERIDICA
ALFABETO
ESTNDAR
ALFABETO
MIXTO
DIGRMICA
N-GRMICA
LINEALES PROGRESIVOS
CSAR
PLAYFAIR
HILL
VERNAM
ENIGMA
VIGENRE
AFN
OTROS
ALFABETO
ESTNDAR
ALFABETO
MIXTO
OTROS
COLUMNAS
FILAS
SERIES
GRUPOS
MONOALFABTICA POLIALFABTICA
ESCTALA
y algunos
ejemplos
Clasificacin de los criptosistemas clsicos
Jorge Rami Aguirre Madrid (Espaa) 2003
Seguridad Informtica y Criptografa. Tema 7: Sistemas de Cifra Clsicos
Diapositiva 318
Hitos histricos en la criptografa
La criptografa clsica abarca desde tiempos
inmemoriales hasta la mitad del siglo XX.
El punto de inflexin en esta clasificacin la
marcan tres hechos relevantes:
En el ao 1948 se publica el estudio de Claude
Shannon sobre la Teora de la Informacin.
En 1974 aparece el estndar de cifra DES.
En el ao 1976 se publica el estudio realizado por W.
Diffie y M. Hellman sobre la aplicacin de funciones
matemticas de un solo sentido a un modelo de cifra,
denominado cifrado con clave pblica.
C
I
F
R
A
D
O
D
I
G
I
T
A
L
Jorge Rami Aguirre Madrid (Espaa) 2003
Seguridad Informtica y Criptografa. Tema 7: Sistemas de Cifra Clsicos
Diapositiva 319
Primer cifrador por transposicin: esctala
La esctala era usada en el siglo V a.d.C. por el pueblo
griego de los lacedemonios. Consista en un bastn en el
que se enrollaba una cinta de cuero y luego se escriba en
ella el mensaje de forma longitudinal.
Al desenrollar la cinta, las letras aparecen desordenadas.
La nica posibilidad de recuperar el texto en claro pasaba
por enrollar dicha cinta en un bastn con el mismo
dimetro que el usado en el extremo emisor y leer el
mensaje de forma longitudinal. La clave del sistema est
en el dimetro del bastn. Se trata de una cifra por
transposicin pues los caracteres del criptograma son los
mismos que en el texto en claro distribuidos de otra forma.
Jorge Rami Aguirre Madrid (Espaa) 2003
Seguridad Informtica y Criptografa. Tema 7: Sistemas de Cifra Clsicos
Diapositiva 320
Mtodo de cifra de la esctala
A S I C I F R A B
A N C O N L A E S
C I T A L A
En ese bastn resida la
fortaleza de un pueblo.
Hoy en da el popular
bastn de mando que se
le entrega al Alcalde de
una ciudad proviene de
esos tiempos remotos.
El texto en claro es:
M = ASI CIFRABAN CON LA ESCITALA
El texto cifrado o criptograma ser:
C = AAC SNI ICT COA INL FLA RA AE BS
Jorge Rami Aguirre Madrid (Espaa) 2003
Seguridad Informtica y Criptografa. Tema 7: Sistemas de Cifra Clsicos
Diapositiva 321
Primer cifrador por sustitucin: Polybios
Es el cifrador por sustitucin de caracteres ms antiguo que se
conoce (siglo II a.d.C.) pero como duplica el tamao del texto
en claro, con letras o nmeros, resulta poco interesante.
A B C D E 1 2 3 4 5
A A B C D E 1 A B C D E
B F G H IJ K 2 F G H IJ K
C L M N O P 3 L M N O P
D Q R S T U 4 Q R S T U
E V W X Y Z 5 V W X Y Z
M
1
= QU BUENA IDEA
C
1
= DA DE AE AB DE AE
CC AA BD AD AE EA
M
2
= LA DEL GRIEGO
C
2
= 31 11 14 15 31 22
42 24 15 22 34
Jorge Rami Aguirre Madrid (Espaa) 2003
Seguridad Informtica y Criptografa. Tema 7: Sistemas de Cifra Clsicos
Diapositiva 322
El cifrador del Csar
En el siglo I a.d.C., Julio Csar usa este cifrador, cuyo
algoritmo consiste en el desplazamiento de tres espacios
hacia la derecha de los caracteres del texto en claro. Es
un cifrador por sustitucin monoalfabtico en el que las
operaciones se realizan mdulo n, siendo n el nmero de
elementos del alfabeto (en aquel entonces latn).
M
i
A B C D E F G H I J K L M N O P Q R S T U V W X Y Z
C
i
D E F G H I J K L M N O P Q R S T U V W X Y Z A B C
0 1 2 3 4 5 6 7 8 9 10 11 12 13 14 15 16 17 18 19 20 21 22 23 24 25 26
Alfabeto de cifrado del Csar para castellano mod 27
Jorge Rami Aguirre Madrid (Espaa) 2003
Seguridad Informtica y Criptografa. Tema 7: Sistemas de Cifra Clsicos
Diapositiva 323
Ejemplo de cifra del Csar en mod 27
M
i
A B C D E F G H I J K L M N O P Q R S T U V W X Y Z
C
i
D E F G H I J K L M N O P Q R S T U V W X Y Z A B C
C
i
= M
i
+ 3 mod 27
M = EL PATIO DE MI CASA ES PARTICULAR
C = H SDWLR GH OL FDVD HV SDUWLFXDU
Cada letra se cifrar siempre igual. Es una gran debilidad y
hace que este sistema sea muy vulnerable y fcil de atacar
simplemente usando las estadsticas del lenguaje.
Jorge Rami Aguirre Madrid (Espaa) 2003
Seguridad Informtica y Criptografa. Tema 7: Sistemas de Cifra Clsicos
Diapositiva 324
Criptoanlisis del cifrador por sustitucin
A B C D E F G H I J K L M N O P Q R S T U V W X Y Z
00 01 02 03 04 05 06 07 08 09 10 11 12 13 14 15 16 17 18 19 20 21 22 23 24 25 26
Cifrado: C
i
= (M
i
+ b) mod 27 Descifrado: M
i
= (C
i
b) mod 27
La letra ms frecuente del criptograma la hacemos coincidir con
la ms frecuente del lenguaje, la letra E, y encontramos as b.
C = LZAHL ZBTHW YBLIH XBLKL ILYOH ZLYCH ROKH
Frecuencias observadas en el criptograma: L (7); H (6); Z (3); B (3);
Y (3); I (2); K (2); O (2); A (1); T (1); W (1); X (1); C (1); R (1).
Luego, es posible que la letra E del lenguaje (la ms frecuente) se
cifre como L en el criptograma y que la letra A se cifre como H:
E + b mod 27 = L b = L - E mod 27 = 11 4 mod 27 = 7
A + b mod 27 = H b = H - A mod 27 = 7 0 mod 27 = 7
M = ESTA ES UNA PRUEBA QUE DEBERIA SER VALIDA
Jorge Rami Aguirre Madrid (Espaa) 2003
Seguridad Informtica y Criptografa. Tema 7: Sistemas de Cifra Clsicos
Diapositiva 325
Cifrador por sustitucin afn mod 27
Cifrado: C
i
= aM
i
+ b mod 27
Descifrado: M
i
= (C
i
b) a
-1
mod 27 donde a
-1
= inv (a, 27)
El factor de decimacin a deber ser primo relativo con el
cuerpo n (en este caso 27) para que exista el inverso.
El factor de desplazamiento puede ser cualquiera 0 b 27.
El ataque a este sistema es tambin muy elemental. Se relaciona el
elemento ms frecuente del criptograma a la letra E y el segundo a
la letra A, planteando un sistema de 2 ecuaciones. Si el texto tiene
varias decenas de caracteres este ataque prospera; caso contrario,
puede haber ligeros cambios en esta distribucin de frecuencias.
Jorge Rami Aguirre Madrid (Espaa) 2003
Seguridad Informtica y Criptografa. Tema 7: Sistemas de Cifra Clsicos
Diapositiva 326
Criptoanlisis a la cifra afn mod 27
C: NAQF EKNDP NCIVU FPUAN EJUIP FCNER NFRF UNPLN
AFPFQ TFPEI JRTE FPKI KTAPF LIKI AIPU RCUJI
PCIVU CUNER IRLNP TJIAF NEOI CFLNC NLUFA TEF
Caracteres ms frecuentes en criptograma: F = 14; N = 13; I = 12
Con E y A las ms frecuentes, el ataque falla. En un segundo
intento suponemos la letra A ms frecuente que la E, luego:
F = (aA + b) mod 27 (a0 + b) mod 27 = 5 b = 5
N = (aE + b) mod 27 (a4 + 5) mod 27 = 13
Entonces a = (13-5) inv (4, 27) mod 27 = 8 7 mod 27 = 2
Luego C
i
= (2M
i
+ 5) mod 27 M
i
= (C
i
5)inv (2, 27). luego:
M: EL GRAN PEZ SE MOVA SILENCIOSAMENTE A TRAVS DE
LAS AGUAS NOCTURNAS, PROPULSADO POR LOS RTMICOS
MOVIMIENTOS DE SU COLA EN FORMA DE MEDIA LUNA.
(Primer prrafo del libro Tiburn de P. Benchley).
Jorge Rami Aguirre Madrid (Espaa) 2003
Seguridad Informtica y Criptografa. Tema 7: Sistemas de Cifra Clsicos
Diapositiva 327
El cifrador de Vigenre
Este cifrador polialfabtico soluciona la debilidad del cifrado
del Csar de que una letra se cifre siempre igual. Usa una
clave K de longitud L y cifra carcter a carcter sumando
mdulo n el texto en claro con los elementos de esta clave.
C
i
= M
i
+ K
i
mod 27
Sea K = CIFRA y el mensaje M = HOLA AMIGOS
M = H O L A A M I G O S
K = C I F R A C I F R A sumando mod 27...
C = J W P R A P L G S
Ms de un alfabeto: la letra
O se cifra de forma distinta.
Observe que el criptograma P se obtiene de un texto L y de un texto I.
Jorge Rami Aguirre Madrid (Espaa) 2003
Seguridad Informtica y Criptografa. Tema 7: Sistemas de Cifra Clsicos
Diapositiva 328
Es Vigenre un algoritmo seguro?
Si la clave de Vigenre tiene mas de 6 caracteres distintos, se
logra una distribucin de frecuencias en el criptograma del
tipo normal, es decir ms o menos plana, por lo que se
difumina la redundancia del lenguaje.
Aunque pudiera parecer que usando una clave larga y de
muchos caracteres distintos y por tanto varios alfabetos de
cifrado, Vigenre es un sistema de cifra seguro, esto es falso.
La redundancia del lenguaje unido a tcnicas de criptoanlisis
muy sencillas, como los mtodos de Kasiski y del ndice de
Coincidencia, permiten romper la cifra y la clave de una
manera muy fcil y con mnimos recursos. Veamos un ataque
por el mtodo de Kasiski.
Jorge Rami Aguirre Madrid (Espaa) 2003
Seguridad Informtica y Criptografa. Tema 7: Sistemas de Cifra Clsicos
Diapositiva 329
Ataque por el mtodo de Kasiski
El mtodo de Kasiski consiste en buscar repeticiones de cadenas de
caracteres en el criptograma. Si estas cadenas son mayores o iguales a
tres caracteres y se repiten ms de una vez, lo ms probable es que esto
se deba a cadenas tpicas del texto en claro (trigramas, tetragramas, etc.,
muy comunes) que se han cifrado con una misma porcin de la clave.
Si se detectan estas cadenas, la distancia entre las mismas ser mltiplo
de la longitud de la clave. Luego, el mximo comn divisor entre esas
cadenas es un candidato a ser la longitud de la clave, digamos L.
Dividimos el criptograma en L subcriptogramas que entonces han sido
cifrados por una misma letra de la clave y en cada subcriptograma
hacemos un ataque simple ahora de tipo estadstico monoalfabtico.
La idea es buscar ahora a travs de los tres caracteres ms frecuentes en
cada subcriptograma las posiciones relativas de las letras A, E y O que
en castellano estn separadas por 4 y 11 espacios. La letra de la posicin
que ocupe la letra A (A = 0) ser entonces la letra clave correspondiente.
Jorge Rami Aguirre Madrid (Espaa) 2003
Seguridad Informtica y Criptografa. Tema 7: Sistemas de Cifra Clsicos
Diapositiva 330
Cadenas repetidas en ataque de Kasiski
Sea el criptograma C de 404 caracteres que vamos a criptoanalizar el siguiente:
PBVRQ VICAD SKAS DETSJ PSIED BGGMP SLRPW RPWY EDSDE DRDP CRCPQ MNPWK
UBZVS FNVRD MTIPW UEQVV CBOVN UEDIF QLONM WNUVR SEIKA ZYEAC EYEDS ETFPH
LBHGU ESOM EHLBX VAEEP UELI SEVEF WHUNM CLPQP MBRRN BPVI MTIBV VEID
ANSJA MTJOK MDODS ELPWI UFOZM QMVNF OHASE SRJWR SFQCO TWVMB JGRPW VSUEX
INQRS JEUEM GGRBD GNNIL AGSJI DSVSU EEINT GRUEE TFGGM PORDF OGTSS TOSEQ
OTGR RYVLP WJIFW XOTGG RPQRR JSKET XRNBL ZETGG NEMUO TXJAT ORVJH RSFHV
NUEJI BCHAS EHEUE UOTIE FFGYA TGGMP IKTBW UEEN IEEU.
Entre otras, se observan las siguientes cadenas (subrayadas) en el criptograma:
3 cadenas GGMP, separadas por 256 y 104 posiciones.
2 cadenas YEDS, separadas por 72 espacios.
2 cadenas HASE, separadas por 156 espacios.
2 cadenas VSUE, separadas por 32 espacios.
Luego el perodo de la clave puede ser mcd (256, 104, 72, 156, 32) = 4. La clave
tendr cuatro caracteres, por lo tanto tomaremos del criptograma el carcter 1, el
5, el 9, etc. para formar el primer subcriptograma C
A
; luego el 2, el 6, el 10,
etc. para formar el subcriptograma C
B
, y as hasta el subcriptograma C
D
.
Jorge Rami Aguirre Madrid (Espaa) 2003
Seguridad Informtica y Criptografa. Tema 7: Sistemas de Cifra Clsicos
Diapositiva 331
Paso a cifrado monoalfabtico en Kasiski
Tenemos 4 subcriptogramas que han sido cifrados con la misma letra de la clave:
C
A
= PQAAEPDMREEDCNUSRIECNIONSAAETLUOLAUIEULMNIIEAAOOLU
MNARSOMRSISERNAISIRTMDTOORLIORRENENOAVSNIAEOFAMTEI
C
B
= BVDTSBPPPDPPPBFDPQBUFNUEZCDFBMBESFNPBBBNMKDPF
QFSJFTBPUNJMBNGDUNUFPFSSRPFTPJTBTETTJFUBSUTFTPBE
C
C
= VISSSIGSWWSDCQWZNMWVOEQMVIYESPHEEXEEEWMQRPMVISTMSWO
MOEWQWJWEQEGDISSETEGOOSETYWWGQSXLGMXOHHECEEIGGIWEE
C
D
= RCKDJEGLRYDRRMKVVTUVVDLWRKEYEHGSHVPLVHCPRVTVDJJDEIZ
VHSRCVGVXRUGGLJVEGEGRGTQGVJXGRKRZGUJRRVJHHUEYGKUNU
La frecuencia relativa observada en cada uno de los subcriptogramas es:
A B C D E F G H I J K L M N O P Q R S T U V W X Y Z
11 0 2 3 12 1 0 0 11 0 0 5 6 9 1 10 2 1 9 7 4 5 1 0 0 0 0
0 14 1 6 4 12 1 0 0 4 1 0 3 6 8 6 14 2 1 6 9 7 1 0 0 0 1
0 0 1 2 18 0 7 3 7 1 0 1 7 1 0 0 2 6 1 12 3 0 3 12 3 2 1
0 0 3 5 7 0 12 6 1 7 5 4 1 1 0 6 2 1 13 2 3 7 14 0 2 3 2
C
A
C
B
C
C
C
D
La letra ms frecuente del subcriptograma debera corresponder a la letra E del
texto en claro, la segunda a la letra A y la tercera a la letra O.
Jorge Rami Aguirre Madrid (Espaa) 2003
Seguridad Informtica y Criptografa. Tema 7: Sistemas de Cifra Clsicos
Diapositiva 332
La regla AEO en el ataque de Kasiski
Si la posicin relativa de la letra A es el valor 0, entonces la letra E
est cuatro espacios a la derecha de la A (m+4 mod 27) y la letra O
est 15 espacios a la derecha de la letra A (m+15 mod 27).
Buscaremos en cada subcriptograma C
i
las tres letras ms frecuentes y
que cumplan adems con esta distribucin.
Es suficiente contar con estas tres letras para que el ataque prospere.
No obstante, podemos afinar ms el ataque si tomamos en cuenta la
siguiente letra frecuente en castellano (S) en posicin (m+19) mod 27.
En el ejemplo para C
A
se observa que la nica solucin que cumple con
esto es la que coincide la AEO (11, 12, 10) luego la letra clave sera la A.
Para C
B
elegimos BFP (14, 12, 14) por lo que la letra clave sera B. Para
C
C
elegimos EIS (18, 7, 12) por lo que la letra clave sera E. Para C
D
elegimos RVG (13, 14, 12) por lo que la letra clave sera R.
La clave ser K = ABER y M = Para que la cosa no me sorprenda....
Jorge Rami Aguirre Madrid (Espaa) 2003
Seguridad Informtica y Criptografa. Tema 7: Sistemas de Cifra Clsicos
Diapositiva 333
El ndice de coincidencia IC
Cuando encontramos una longitud L de la clave por el mtodo de Kasiski
y rompemos el criptogama en L subcriptogamas, podemos comprobar que
cada uno de ellos se trata efectivamente de un cifrado monoalfabtico
aplicando el concepto del ndice de coincidencia IC.
26
IC = p
i
2
para castellano mod 27: IC = p
A
2
+ p
B
2
+ ... + p
Z
2
= 0,072
i=0
Aunque el estudio de este ndice IC queda fuera del contexto de estos
apuntes, como para el castellano mod 27 el IC = 0,072, en el ataque de
Kasiski se comprueba que para cada subcriptograma su IC est cercano a
este valor. Si el IC es menor que 0,5 es muy probable que no estemos ante
un cifrador monoalfabtico sino uno polialfabtico de periodo 2 o mayor.
En el ejemplo anterior, una vez roto el criptograma en cuatro tenemos:
IC
CA
= 0,070; IC
CB
= 0,073; IC
CC
= 0,075; IC
CD
= 0,065.
Jorge Rami Aguirre Madrid (Espaa) 2003
Seguridad Informtica y Criptografa. Tema 7: Sistemas de Cifra Clsicos
Diapositiva 334
Cifrador poligrmico de Playfair
Los cifrados anteriores se hacan carcter a carcter, es decir eran
monogrmicos. Para aumentar la seguridad de la cifra podemos
cifrar por poligramas, bloques de caracteres.
Un cifrador inventado a finales del siglo XIX es el de Playfair que
trabaja con una matriz de 5x5 letras, cifrando por digramas. Si el
texto en claro tiene un nmero impar de elementos, se rellena con
una letra establecida, por ejemplo x.
Z Y X W V
U T S R Q
P O N/ M L
K I/J H G F
E D C B A
Si M
1
M
2
estn en la misma fila, C
1
C
2
son los dos caracteres de la derecha.
Si M
1
M
2
estn en la misma columna,
C
1
C
2
son los dos caracteres de abajo.
Si M
1
M
2
estn en filas y columnas
distintas, C
1
C
2
son los dos caracteres de
la diagonal, desde la fila de M
1
.
Jorge Rami Aguirre Madrid (Espaa) 2003
Seguridad Informtica y Criptografa. Tema 7: Sistemas de Cifra Clsicos
Diapositiva 335
Ejemplo de cifra con Playfair
Si la clave K = BEATLES, eliminando la letra , se pide cifrar el
mensaje M = With a little help from my friends.
Z Y X W V
U R Q P O
N M K I H
G F D C S
L T A E B
Se rompe la doble
MM agregando una
X y se rellena al
final con X
M = WI TH AL IT TL EH EL PF RO MX MY FR IE ND SX
C = EP BM TB ME LB BI AB RC UP KY RT MY PC KG DV
Estos sistemas tambin son criptoanalizables pues en el criptograma C
persisten algunas propiedades del lenguaje, en este caso la distribucin de
digramas tpicos del castellano como por ejemplo en, de, mb, etc.
Jorge Rami Aguirre Madrid (Espaa) 2003
Seguridad Informtica y Criptografa. Tema 7: Sistemas de Cifra Clsicos
Diapositiva 336
El cifrador de matrices de Hill
En 1929 Lester Hill propone un sistema de cifra usando una
matriz como clave, cifrando Ngramas de forma que:
C
1
k
11
k
12
k
13
... k
1N
M
1
C
2
k
21
k
22
k
23
... k
2N
M
2
C
3
k
31
k
32
k
33
... k
3N
M
3
.. .. .. .. .. ..
C
N
k
N1
k
N2
k
N3
... k
NN
M
N
=
X mod n
La matriz clave K debe tener inversa K
-1
en el cuerpo de cifra n.
Luego, como K
-1
= T
ADJ(K)
/|K| mod n, en donde ADJ(K) es la
matriz adjunta, T es la traspuesta y |K| el determinante, este ltimo
valor |K| no podr ser cero ni tener factores en comn con n puesto
que est en el denominador (concepto de inverso).
Si el texto en claro no es mltiplo del bloque N, se rellena con
caracteres predeterminados, por ejemplo la letra X o la Z.
Jorge Rami Aguirre Madrid (Espaa) 2003
Seguridad Informtica y Criptografa. Tema 7: Sistemas de Cifra Clsicos
Diapositiva 337
Ejemplo de cifrado de Hill
Sea M = AMIGO CONDUCTOR y la clave K la que se muestra:
C
1
16 4 11 0
C
2
8 6 18 12
C
3
15 19 15 8
=
X mod 27
M = AMI GOC OND UCT ORZ
C
1
= (160 + 412 + 118) mod 27 = 136 mod 27 = 1 = B
C
2
= (80 + 612 + 188) mod 27 = 216 mod 27 = 0 = A
C
3
= (150 + 1912 + 158) mod 27 = 348 mod 27 = 24 = X
C = BAX PMA BJE XAF EUM (compruebe Ud. los otros trigramas)
K = PELIGROSO ser la clave
simblica. Se cifrar el primer
trigrama: AMI = 0, 12, 8.
Para descifrar encontramos K
-1
= inv (K, 27) = K
-1
= T
ADJ(K)
/|K| mod n
|K| = 16(615 - 1918) 4(815 - 1518) + 11 (819 - 156) mod 27 = 4
Encontramos luego la matriz adjunta de K, la trasponemos cambiando
filas por columnas y la multiplicamos por inv (|K|, 27) = inv (4, 27) = 7
con lo que se obtiene la matriz que se indica (hgalo Ud.)
Jorge Rami Aguirre Madrid (Espaa) 2003
Seguridad Informtica y Criptografa. Tema 7: Sistemas de Cifra Clsicos
Diapositiva 338
Ejemplo de descifrado de Hill
18 26 15
M = K
-1
x C mod n y K
-1
= 24 6 13
11 24 10
C = BAXPMABJEXAFEUM y la clave K
-1
es la que se muestra:
M
1
18 26 15 1
M
2
24 6 13 0
M
3
11 24 10 24
=
X mod 27
Descifrado del primer trigrama
del criptograma: BAX = 1, 0, 24.
C = BAX PMA BJE XAF EUM
M
1
= (181 + 260 + 1524) mod 27 = 378 mod 27 = 0 = A
M
2
= (241 + 60 + 1324) mod 27 = 336 mod 27 = 12 = M
M
3
= (111 + 240 + 1024) mod 27 = 251 mod 27 = 8 = I
M = AMI GOC OND UCT ORZ (compruebe Ud. los otros trigramas)
Jorge Rami Aguirre Madrid (Espaa) 2003
Seguridad Informtica y Criptografa. Tema 7: Sistemas de Cifra Clsicos
Diapositiva 339
Es seguro el cifrador de Hill?
Si con el sistema de Hill se cifran bloques de 8 caracteres,
incluso en un cuerpo tan pequeo como n = 27 el espacio de
claves aumenta de forma espectacular, comparable con DES.
Si el mdulo de cifra es un primo p, entonces el nmero de
claves vlidas es cercano al mximo posible: p
x
donde x = d
2
,
con d el tamao de N-grama o de la matriz clave.
No obstante, el sistema no es seguro. Debido a su linealidad
ser muy fcil hacer un ataque con texto claro conocido segn
el mtodo de Gauss Jordan y encontrar as la matriz clave K.
Esto es debido a que aparecen los llamados vectores unitarios
en el criptograma o en el texto en claro, o bien los obtenemos
aplicando este mtodo.
Jorge Rami Aguirre Madrid (Espaa) 2003
Seguridad Informtica y Criptografa. Tema 7: Sistemas de Cifra Clsicos
Diapositiva 340
Ataque al cifrado de Hill por Gauss Jordan
El mtodo consiste en escribir una matriz 2N-grmica con los elementos
del texto en claro y los elementos del criptograma. En esta matriz
realizamos operaciones lineales (multiplicar filas por un nmero y restar
filas entre s) con el objeto de obtener los vectores unitarios.
Por ejemplo podemos romper la matriz clave K teniendo:
M = ENU NLU GAR DEL AMA NCH ADE CUY ONO ...
C = WVX IDQ DDO ITQ JGO GJI YMG FVC UT ...
E N U W V X 4 13 21 23 22 24
N L U I D Q 13 11 21 8 3 17
G A R D D O 6 0 18 3 3 15
D E L I T Q 3 4 11 8 20 17
A M A J G O = 0 12 0 9 6 15
N C H G J I 13 2 7 6 9 8
A D E Y M G 0 3 4 25 12 6
C U Y F V C 2 21 25 5 22 2
O N O U T 15 13 15 21 14 20
Jorge Rami Aguirre Madrid (Espaa) 2003
Seguridad Informtica y Criptografa. Tema 7: Sistemas de Cifra Clsicos
Diapositiva 341
Operaciones en la matriz de Gauss Jordan
Vamos a dejar en la primera columna un nmero uno en la fila
primera y todas las dems filas un cero. Luego multiplicamos el
vector (4 13 21 | 23 22 24) por el inv (4, 27) = 7. As obtenemos
7(4 13 21 | 23 22 24) mod 27 = (1 10 12 | 26 19 6). Si esto no se
puede hacer con la primera fila movemos los vectores. Hecho esto
vamos restando las filas respecto de esta primera como se indica:
4 13 21 23 22 24
13 11 21 8 3 17
6 0 18 3 3 15
3 4 11 8 20 17
0 12 0 9 6 15
13 2 7 6 9 8
0 3 4 25 12 6
2 21 25 5 22 2
15 13 15 21 14 20
a) 2 fila = 2 fila 131 fila mod 27
b) 3 fila = 3 fila 61 fila mod 27
c) 4 fila = 4 fila 31 fila mod 27
d) 5 fila ya tiene un 0
e) 6 fila = 6 fila 131 fila mod 27
f) 7 fila ya tiene un 0
g) 8 fila = 8 fila 21 fila mod 27
h) 9 fila = 9 fila 151 fila mod 27
Jorge Rami Aguirre Madrid (Espaa) 2003
Seguridad Informtica y Criptografa. Tema 7: Sistemas de Cifra Clsicos
Diapositiva 342
Matriz clave de Hill criptoanalizada
Repetimos este procedimiento ahora para algn vector en cuya
segunda columna tenga un nmero con inverso en 27 y lo mismo
para la tercera columna, moviendo si es preciso los vectores.
Como la mitad izquierda de la matriz 2N era el texto el claro, la
parte derecha de la matriz con vectores unitarios corresponder a
la traspuesta de la clave.
1 0 0 2 5 7
0 1 0 3 5 8
0 0 1 4 6 9
0 0 0 0 0 0
0 0 0 0 0 0
0 0 0 0 0 0
0 0 0 0 0 0
0 0 0 0 0 0
0 0 0 0 0 0
2 3 4
K = 5 5 6
7 8 9

Compruebe que la clave es


la utilizada en este cifrado.
Jorge Rami Aguirre Madrid (Espaa) 2003
Seguridad Informtica y Criptografa. Tema 7: Sistemas de Cifra Clsicos
Diapositiva 343
El cifrador de Vernam
En 1917 Gilbert Vernam (MIT) propone un cifrador por sustitucin
binaria con clave de un solo uso, basado en el cdigo Baudot de 5 bits:
La operacin de cifra es la funcin XOR.
Usa una secuencia cifrante binaria y aleatoria S que se obtiene de
una clave secreta K compartida por emisor y receptor.
El algoritmo de descifrado es igual al de cifrado por la involucin de
la funcin XOR.
La clave ser tan larga o ms que el mensaje y se usar una sola vez.
Criptograma M M
S S
Clave K Clave K
Algoritmo
Determinstico
Algoritmo
Determinstico

secuencia cifrante
MENSAJE MENSAJE
C
Jorge Rami Aguirre Madrid (Espaa) 2003
Seguridad Informtica y Criptografa. Tema 7: Sistemas de Cifra Clsicos
Diapositiva 344
Ejemplo de cifrado de Vernam
Usando el cdigo Baudot (vase captulo 18) se pide cifrar el
mensaje M = BYTES con la clave K = VERNAM.
Solucin:
BV = 1100111110 = 00111 = U
YE = 1010100001 = 10100 = H
TR = 1000001010 = 11010 = G
EN = 0000101100 = 01101 = F
SA = 0010100011 = 00110 = I
C = UHGFI
El sistema de Vernam es el nico que es matemticamente seguro e
imposible de criptoanalizar ya que la clave se usa una sola vez (one
time pad), es aleatoria y tanto o ms larga que el propio mensaje.
Jorge Rami Aguirre Madrid (Espaa) 2003
Seguridad Informtica y Criptografa. Tema 7: Sistemas de Cifra Clsicos
Diapositiva 345
En construccin ...
ESTIMADO/A LECTOR/A:
AL IGUAL QUE EN CIERTAS PGINAS WEB,
ESTE CAPTULO ESTAR EN CONSTRUCCIN
DURANTE MUCHO TIEMPO... ... mis disculpas!
NO OBSTANTE, SE SEGUIRN PUBLICANDO LOS TEMAS
SIGUIENTES DEL CURSO EN DIAPOSITIVAS SOBRE TCNICAS
DE CIFRA MODERNAS, DE MAYOR INTERS ACADMICO
DE ACUERDO CON EL AVANCE DE LA ASIGNATURA DE
SEGURIDAD INFORMTICA.
Pero si est muy interesado en el tema...
Jorge Rami Aguirre Madrid (Espaa) 2003
Seguridad Informtica y Criptografa. Tema 7: Sistemas de Cifra Clsicos
Diapositiva 346
El libro electrnico de criptografa clsica
Si est interesado en estos temas de criptografa clsica e
historia de las mquinas de cifrar, puede descargar el Libro
Electrnico de Criptografa Clsica hecho en ToolBook desde el
servidor de la Red Temtica Iberoamericana de Criptografa y
Seguridad de la Informacin, CriptoRed.
http://www.criptored.upm.es/paginas/software.htm#propio
El archivo de instalacin tiene 5.17 MB y es de libre distribucin
como gran parte del software y documentos de este servidor.
Su uso es verdaderamente sencillo. Podr adems comprobar y
practicar con estos algoritmos de cifra clsica puesto que el
libro incluye una seccin con software especfico para ello.
Tambin puede descargarse desde el mismo servidor un
programa para prcticas denominado CriptoClsicos.
Jorge Rami Aguirre Madrid (Espaa) 2003
Seguridad Informtica y Criptografa. Tema 7: Sistemas de Cifra Clsicos
Diapositiva 347
El libro de la asignatura y los exmenes
Tambin puede seguir con amplios detalles estos sistemas de
cifra clsicos en el libro de la asignatura Seguridad Informtica
Aplicaciones Criptogrficas, 2 edicin, Junio de 1999
Departamento de Publicaciones - Escuela Universitaria de
Informtica - Universidad Politcnica de Madrid (Espaa)
I.S.B.N.: 84-87238-57-2 Depsito Legal: M-24709-1999
Y en los casi 20 exmenes de dicha asignatura (incluyen siempre
un ejercicio bsico sobre este tipo de cifra) y sus soluciones
que podr encontrar en el servidor Web de CriptoRed
http://www.criptored.upm.es/paginas/docencia.htm#examenes
Fin del Tema 7
Jorge Rami Aguirre Madrid (Espaa) 2003
Seguridad Informtica y Criptografa. Tema 7: Sistemas de Cifra Clsicos
Diapositiva 348
Cuestiones y ejercicios (1 de 3)
LAS SIGUIENTES PREGUNTAS ESTN RELACIONADAS
CON ESTOS APUNTES, EL LIBRO ELECTRNICO DE
CRIPTOGRAFA CLSICA Y EL SOFTWARE DE PRCTICAS
CRIPTOCLSICOS QUE SE HA COMENTADO.
1. Qu significa cifrar por sustitucin y qu por transposicin?
2. Por qu que el mtodo esctala es un cifrado por permutacin?
3. Cul es la peor debilidad que tiene el sistema de cifra del Csar?
4. Ciframos el mensaje M = HOLA QUE TAL con un desplazamiento
de 6 caracteres, cul es el criptograma? Y si desplazamos 27?
5. Por qu no podemos cifrar en el cuerpo n = 27 con la funcin de
cifra C = (12M + 5) mod n? Qu condicin deber cumplirse?
6. Cmo podramos atacar un sistema de cifra tipo Csar? Y si la
cifra es de tipo afn como el de la pregunta anterior?
Jorge Rami Aguirre Madrid (Espaa) 2003
Seguridad Informtica y Criptografa. Tema 7: Sistemas de Cifra Clsicos
Diapositiva 349
Cuestiones y ejercicios (2 de 3)
7. Cifre el mensaje M = VAMOS A VER con un sistema afn siendo el
valor a = 5 y b = 2 usando slo operaciones modulares.
8. En un sistema de cifra de Vigenre la clave a usar puede ser CERO
o bien COMPADRE, cul de las dos usara y por qu?
9. Cifre segn Vigenre el mensaje M = UNA PRUEBA con la clave
K = OLA sin usar la tabla, slo con operaciones modulares.
10. Por qu se dice que Vigenre es un cifrador polialfabtico?
11. Cmo podramos atacar un cifrado polialfabtico peridico?
12. Cifre con el mtodo de Vernam binario en mensaje M = VIDA y
clave K = TACO suponiendo texto ASCII. Si la clave se cambia en
cada cifra y es aleatoria, cmo se comporta este cifrador?
13. Qu significa cifrar por homfonos? Qu es el cifrado de Beale?
Jorge Rami Aguirre Madrid (Espaa) 2003
Seguridad Informtica y Criptografa. Tema 7: Sistemas de Cifra Clsicos
Diapositiva 350
Cuestiones y ejercicios (3 de 3)
14. Indique las mquinas de cifrar que se usaron en la Segunda Guerra
Mundial y diga de forma sencilla cmo funcionaban.
15. Se cifra por permutaciones usando para ello una distribucin en
columnas con clave. Qu similitud tendr luego este sistema de
cifra con algunas operaciones hechas en el DES?
16. Cifre con el cifrador digrmico de Hill el mensaje ADIOS AMIGO.
Qu matriz simblica puede usar: GATO, GOTA, MISA o MESA?
17. Cifre y descifre con la matriz trigrmica simblica PELIGROSO el
mensaje HOY ES UN HERMOSO DIA.
18. Si la clave es tan grande, es segura la cifra de Hill? Por qu?
19. Qu significan los vectores unitarios? Es fcil encontrarlos?
20. Cmo funciona el ataque de Gauss Jordan? Obtenga la matriz clave
del ejercicio 17 mediante Gauss Jordan.
Tema 8
Sistemas de Cifra Modernos
Seguridad Informtica y Criptografa
Ultima actualizacin: 03/02/03
Archivo con 33 diapositivas
Jorge Rami Aguirre
Universidad Politcnica de Madrid
Material Docente de
Libre Distribucin
v 3.1
Este archivo forma parte de un curso sobre Seguridad Informtica y Criptografa. Se autoriza la
reproduccin en computador e impresin en papel slo con fines docentes o personales, respetando
en todo caso los crditos del autor. Queda prohibida su venta, excepto a travs del Departamento de
Publicaciones de la Escuela Universitaria de Informtica, Universidad Politcnica de Madrid, Espaa.
Curso de Seguridad Informtica y Criptografa JRA
Jorge Rami Aguirre Madrid (Espaa) 2003
Seguridad Informtica y Criptografa. Tema 8: Sistemas de Cifra Modernos
Diapositiva 352
Conceptos elementales
Un par de ideas bsicas
Los criptosistemas modernos, cuya cifra en bits est
orientada a todos los caracteres ASCII o ANSI usan por
lo general una operacin algebraica en Z
n
, un cuerpo
finito, sin que necesariamente este mdulo deba
corresponder con el nmero de elementos del alfabeto o
cdigo utilizado. Es ms, nunca coinciden; siempre ser
mucho mayor el cuerpo de trabajo que el alfabeto.
Su fortaleza est en la imposibilidad computacional de
descubrir una clave secreta nica, en tanto que el
algoritmo de cifra es (o debera ser) pblico.
Jorge Rami Aguirre Madrid (Espaa) 2003
Seguridad Informtica y Criptografa. Tema 8: Sistemas de Cifra Modernos
Diapositiva 353
Clasificacin de los criptosistemas
MTODOS DE CIFRA MODERNA
MTODOS DE CIFRA MODERNA
CIFRADO EN FLUJO CIFRADO EN BLOQUE
CLAVE SECRETA CLAVE PBLICA
EXPONENCIACIN SUMA/PRODUCTO
y algunos
ejemplos...
LFSRs A5
Telefona mvil
y tiempo real
DES; T-DES; CAST;
IDEA; RIJNDAEL ...
Cifrado propio de
la informacin
Intercambio de claves
y firma digital
MH (Proteccin
de SW va HW)
RSA , ELGAMAL
CE, MOCHILAS
Jorge Rami Aguirre Madrid (Espaa) 2003
Seguridad Informtica y Criptografa. Tema 8: Sistemas de Cifra Modernos
Diapositiva 354
Introduccin al cifrado de flujo
Usa el concepto de cifra propuesto por Vernam,
que cumple con las ideas de Shannon sobre
sistemas de cifra con secreto perfecto, esto es:
a) El espacio de las claves es igual o mayor que el
espacio de los mensajes.
b) Las claves deben ser equiprobables.
c) La secuencia de clave se usa una sola vez y luego
se destruye (sistema one-time pad).
DUDA: Es posible satisfacer la condicin a)?
Jorge Rami Aguirre Madrid (Espaa) 2003
Seguridad Informtica y Criptografa. Tema 8: Sistemas de Cifra Modernos
Diapositiva 355
Espacio de claves y del mensaje
Espacio de Claves Espacio de Mensajes?
1) La secuencia de bits de la clave deber enviarse al
destinatario a travs de un canal que sabemos es
inseguro (recuerde que an no conoce el protocolo
de intercambio de clave de Diffie y Hellman).
2) Si la secuencia es infinita, desbordaramos la
capacidad del canal de comunicaciones.
Qu solucin damos
a este problema?
Jorge Rami Aguirre Madrid (Espaa) 2003
Seguridad Informtica y Criptografa. Tema 8: Sistemas de Cifra Modernos
Diapositiva 356
El concepto de semilla en un generador
Si por el canal supuestamente seguro enviamos esa clave
tan larga ... por qu entonces no enviamos directamente
el mensaje en claro y nos dejamos de historias?
La solucin est en generar una secuencia de tipo
pseudoaleatoria con un algoritmo determinstico a
partir de una semilla de slo unas centenas de bits.
Podremos generar as secuencias con perodos del
orden de 2
n
, un valor ciertamente muy alto. Esta
semilla es la que se enva al receptor mediante un
sistema de cifra de clave pblica y un algoritmo de
intercambio de clave y no sobrecargamos el canal.
Jorge Rami Aguirre Madrid (Espaa) 2003
Seguridad Informtica y Criptografa. Tema 8: Sistemas de Cifra Modernos
Diapositiva 357
Tcnica de cifra en flujo
El mensaje en claro se leer bit a bit.
Se realizar una operacin de cifra, normalmente
la funcin XOR, con una secuencia cifrante de
bits S
i
que debe cumplir ciertas condiciones:
Un perodo muy alto.
Aleatoriedad en sus propiedades.
Lo veremos en
el captulo 9...
C
XOR
Secuencia cifrante S
i
Mensaje M
C
Bits del Criptograma
XOR
Mensaje M
Secuencia cifrante S
i
Jorge Rami Aguirre Madrid (Espaa) 2003
Seguridad Informtica y Criptografa. Tema 8: Sistemas de Cifra Modernos
Diapositiva 358
Introduccin a la cifra en bloque
El mensaje se agrupa en bloques, por lo
general de 8 bytes, antes de aplicar el
algoritmo de cifra a cada bloque de
forma independiente con la misma clave.
Cifrado con Clave Secreta
Hay algunos algoritmos muy conocidos por su uso
en aplicaciones bancarias (DES), correo electrnico
(IDEA, CAST) y comercio electrnico (Triple DES).
No obstante, tienen tres puntos dbiles.
Jorge Rami Aguirre Madrid (Espaa) 2003
Seguridad Informtica y Criptografa. Tema 8: Sistemas de Cifra Modernos
Diapositiva 359
Debilidades de la cifra con clave secreta
a) Mala gestin de claves. Crece el nmero de claves
secretas en un orden igual a n
2
para un valor n
grande de usuarios .
b) Mala distribucin de claves. No existe posibilidad
de enviar, de forma segura, una clave a travs de
un medio inseguro .
c) No tiene firma digital. Aunque s ser posible
autenticar el mensaje mediante una marca, no es
posible firmar digitalmente el mensaje .
Jorge Rami Aguirre Madrid (Espaa) 2003
Seguridad Informtica y Criptografa. Tema 8: Sistemas de Cifra Modernos
Diapositiva 360
Por qu usamos clave secreta?
a) Mala gestin de claves
b) Mala distribucin de claves
c) No tiene firma digital
Tiene algo de bueno la cifra
en bloque con clave secreta?
S: la velocidad de cifra es muy alta y por ello se usar
para realizar la funcin de cifra de la informacin. Adems,
con claves de slo unas centenas de bits obtendremos una
alta seguridad pues su no linealidad y algoritmo hace que el
nico ataque que puede prosperar es de el de la fuerza bruta.
Jorge Rami Aguirre Madrid (Espaa) 2003
Seguridad Informtica y Criptografa. Tema 8: Sistemas de Cifra Modernos
Diapositiva 361
Cifrado en bloque con clave pblica
Comienza a ser ampliamente conocido a travs de
su aplicacin en los sistemas de correo electrnico
seguro (PGP y PEM) permitiendo cifrar e incluir
una firma digital adjunta al documento o e-mail
enviado y tambin en navegadores Web.
Cada usuario tiene dos claves, una secreta o privada
y otra pblica, inversas dentro de un cuerpo.
Usan las funciones unidireccionales con trampa.
Jorge Rami Aguirre Madrid (Espaa) 2003
Seguridad Informtica y Criptografa. Tema 8: Sistemas de Cifra Modernos
Diapositiva 362
Funciones unidireccionales con trampa
Son funciones matemticas de un solo sentido
(one-way functions) y que nos permiten usar la
funcin en sentido directo o de clculo fcil para
cifrar y descifrar (usuarios legtimos) y fuerza el
sentido inverso o de clculo difcil para aquellos
impostores, hackers, etc. que lo que desean es
atacar o criptoanalizar la cifra.
f (M) = C es siempre fcil.
f
-1
(C) = M es difcil salvo que se tenga la trampa.
Jorge Rami Aguirre Madrid (Espaa) 2003
Seguridad Informtica y Criptografa. Tema 8: Sistemas de Cifra Modernos
Diapositiva 363
Funciones con trampa ms usadas
Problema de la factorizacin
Problema de la factorizacin
Clculo directo: producto de dos primos grandes pq = n
Clculo inverso: factorizacin de nmero grande n = pq
Problema del logaritmo discreto
Problema del logaritmo discreto
Clculo directo: exponenciacin discreta =
x
mod n
Clculo inverso: logaritmo discreto x = log

mod n
Jorge Rami Aguirre Madrid (Espaa) 2003
Seguridad Informtica y Criptografa. Tema 8: Sistemas de Cifra Modernos
Diapositiva 364
Otras funciones con trampa
Problema de la mochila
Problema de la mochila
Clculo directo: sumar elementos de mochila con trampa
Clculo inverso: sumar elementos de mochila sin trampa
Problema de la raz discreta
Problema de la raz discreta
Clculo directo: cuadrado discreto x = aa mod n
Clculo inverso: raz cuadrada discreta a = x mod n
Jorge Rami Aguirre Madrid (Espaa) 2003
Seguridad Informtica y Criptografa. Tema 8: Sistemas de Cifra Modernos
Diapositiva 365
Cifrado con clave pblica de destino
NUESTROS PROTAGONISTAS
origen destino
Claves: e
B
, n
B
, d
B
Claves: e
B
, n
B
, d
B
Claves: e
Si Benito realiza la
operacin con las
claves pblicas de
Adela (e
A
, n
A
), la
informacin que se
transmite mantiene la
confidencialidad:
slo ella puede verla.
Benito
Adela
A
, n
A
, d
A
Claves: e
A
, n
A
, d
A
e
B
, n
B
: pblicas
d
B
: privada
e
A
, n
A
: pblicas
d
A
: privada
e
B
y d
B
son
inversas dentro
de un cuerpo Z
B
e
A
y d
A
son
inversas dentro
de un cuerpo Z
A
C = E
eA
(M) mod n
A
Jorge Rami Aguirre Madrid (Espaa) 2003
Seguridad Informtica y Criptografa. Tema 8: Sistemas de Cifra Modernos
Diapositiva 366
Operacin de cifra con clave de destino
Cifrado:
Benito enva un mensaje M a Adela
Claves: e
B
, n
B
, d
B
Claves: e
A
, n
A
, d
A
Benito Adela
C = E
eA
(M) mod n
A
Claves pblicas
Descifrado:
Clave privada
M = E
dA
[E
eA
(M)] mod n
A
E
dA
y E
eA
son inversos
Se obtiene confidencialidad del mensaje
Jorge Rami Aguirre Madrid (Espaa) 2003
Seguridad Informtica y Criptografa. Tema 8: Sistemas de Cifra Modernos
Diapositiva 367
Y si usamos la clave pblica de origen?
Si en vez de utilizar la clave pblica de destino, el emisor
usa su propia clave pblica, la cifra no tiene sentido bajo
el punto de vista de sistemas de clave pblica ya que slo
l o ella sera capaz de descifrar el criptograma (deshacer
la operacin de cifra) con su propia clave privada.
Esto podra usarse para cifrar de forma
local uno o varios ficheros, por
ejemplo, pero para ello ya estn los
sistemas de clave secreta, mucho ms
rpidos y, por tanto, ms eficientes.
Jorge Rami Aguirre Madrid (Espaa) 2003
Seguridad Informtica y Criptografa. Tema 8: Sistemas de Cifra Modernos
Diapositiva 368
Y si usamos la clave privada de origen?
Si ahora el emisor usa su clave privada en la cifra sobre
el mensaje, se obtiene una firma digital que le autentica
como emisor ante el destinatario y, adems, a este ltimo
le permitir comprobar la integridad del mensaje.
Veamos antes un ejemplo de
algoritmo que usa un par de
claves entre dos usuarios...
Obviamente, el emisor nunca podr realizar la cifra del
mensaje M con la clave privada del receptor.
Jorge Rami Aguirre Madrid (Espaa) 2003
Seguridad Informtica y Criptografa. Tema 8: Sistemas de Cifra Modernos
Diapositiva 369
El algoritmo del mensaje en la caja
PROTOCOLO: A enva a B un mensaje M
1 A pone el mensaje M en la caja, la cierra con su
llavey la enva a B.
2 B recibe la caja, la cierra con su llavey enva a
A la caja con las dos cerraduras.
3 A recibe la caja, quita su llave y devuelve a B
la caja slo con la cerradura de B.
4 B recibe la caja, quita su cerradura y puede ver
el mensaje M que A puso en el interior de la caja.
Jorge Rami Aguirre Madrid (Espaa) 2003
Seguridad Informtica y Criptografa. Tema 8: Sistemas de Cifra Modernos
Diapositiva 370
Todo bien en el algoritmo de la caja?
Durante la transmisin, el mensaje est
protegido de cualquier intruso por lo
que existe integridad del mensaje y hay
proteccin contra una ataque pasivo.
Pero el usuario B no puede estar seguro
si quien le ha enviado el mensaje M es
el usuario A o un impostor. Por lo tanto
el algoritmo as implementado no nos
permite comprobar la autenticidad del
emisor pues no detecta la suplantacin
de identidad. No obstante...
Modificando algo el
algoritmo anterior,
podremos asegurar
tanto la integridad del
mensaje como la
autenticidad de emisor.
Jorge Rami Aguirre Madrid (Espaa) 2003
Seguridad Informtica y Criptografa. Tema 8: Sistemas de Cifra Modernos
Diapositiva 371
Cifrado con clave privada del origen
Claves: e
Si ahora Benito realiza
la operacin de cifra con
su clave privada d
B
en el
cuerpo n
B
Adela ser
capaz de comprobar esa
cifra ya que posee (entre
otras) la clave pblica de
Benito. Comprueba as
tanto la autenticidad del
mensaje como del autor.
e
B
y d
B
son
inversas dentro
de un cuerpo Z
B
e
A
y d
A
son
inversas dentro
de un cuerpo Z
A
origen destino
Benito
Adela
A
, n
A
, d
A
Claves: e
A
, n
A
, d
Claves: e
B
, n
B
, d
B
, d Claves: e
B
, n
B
A
B
e
B
, n
B
: pblicas
d
B
: privada
e
A
, n
A
: pblicas
d
A
: privada
C = E
dB
(M) mod n
B
Jorge Rami Aguirre Madrid (Espaa) 2003
Seguridad Informtica y Criptografa. Tema 8: Sistemas de Cifra Modernos
Diapositiva 372
Operacin de cifra con clave de origen
Cifrado:
Benito firma un mensaje M a Adela
Claves: e
B
, n
B
, d
B
Claves: e
A
, n
A
, d
A
Benito Adela
C = E
dB
(M) mod n
B
Clave privada
Descifrado:
Claves pblicas
M = E
eB
[E
dB
(M)] mod n
B
E
dB
y E
eB
son inversos
Se comprueba la integridad del origen
Jorge Rami Aguirre Madrid (Espaa) 2003
Seguridad Informtica y Criptografa. Tema 8: Sistemas de Cifra Modernos
Diapositiva 373
Uso de la criptografa asimtrica
Estas dos operaciones de cifra son posibles debido a la
caracterstica intrnseca de los sistemas de clave pblica: el
uso de una clave privada (secreta) inversa de una pblica.
Qu aplicacin tendrn entonces los sistemas de
criptografa de clave pblica o asimtrica?
Usando la clave pblica del destino se har el intercambio
de claves de sesin de una cifra con sistemas simtricos
(decenas a centenas de bits).
Usando la clave privada de origen, se firmar digitalmente
un resumen (decenas a centenas de bits) del mensaje
obtenido con una funcin hash.
Jorge Rami Aguirre Madrid (Espaa) 2003
Seguridad Informtica y Criptografa. Tema 8: Sistemas de Cifra Modernos
Diapositiva 374
La gestin de claves
Gestin de claves
Clave Secreta Clave Pblica
Hay que memorizar Slo es necesario
un nmero muy alto memorizar la clave
de claves: n
2
. privada del emisor.
En cuanto a la gestin de claves, sern mucho ms
eficientes los sistemas de cifra asimtricos pues los
simtricos no permiten una gestin lgica y eficiente
de estas claves.
Jorge Rami Aguirre Madrid (Espaa) 2003
Seguridad Informtica y Criptografa. Tema 8: Sistemas de Cifra Modernos
Diapositiva 375
El espacio de claves
Longitud y espacio de claves
Clave Secreta Clave Pblica
Debido al tipo de Por el algoritmo usado
cifrador usado, la en la cifra, la clave
clave ser del orden ser del orden de los
de la centena de bits. miles de bits.
En cuanto al espacio de claves, no son comparables los
sistemas simtricos con los asimtricos. Para atacar un
sistema asimtrico no se buscar en todo el espacio de
claves como debera hacerse en los sistemas simtricos.
128 1024
Jorge Rami Aguirre Madrid (Espaa) 2003
Seguridad Informtica y Criptografa. Tema 8: Sistemas de Cifra Modernos
Diapositiva 376
La vida de las claves
Vida de una clave
Clave Secreta Clave Pblica
La duracin es muy La duracin de la clave
corta. Normalmente pblica, que la entrega
se usa como una y gestiona un tercero,
clave de sesin. suele ser larga.
En cuanto a la vida de una clave, en los sistemas
simtricos sta es muchsimo menor que las usadas
en los asimtricos. La clave de sesin es aleatoria,
en cambio la asimtrica es propia del usuario.
Segundos
o minutos
Meses o
un ao
Jorge Rami Aguirre Madrid (Espaa) 2003
Seguridad Informtica y Criptografa. Tema 8: Sistemas de Cifra Modernos
Diapositiva 377
Vida de la clave y principio de caducidad
Si en un sistema de clave secreta, sta se usa como clave
de una sesin que dura muy poco tiempo...
y en este tiempo es imposible romperla...
para qu preocuparse entonces?
La confidencialidad de la informacin tiene
una caducidad. Si durante este tiempo
alguien puede tener el criptograma e intentar
un ataque por fuerza bruta, obtendr la clave
(que es lo menos importante) ...
pero tambin el mensaje secreto! ... puede ser muy peligroso.
Jorge Rami Aguirre Madrid (Espaa) 2003
Seguridad Informtica y Criptografa. Tema 8: Sistemas de Cifra Modernos
Diapositiva 378
El problema de la autenticacin
Condiciones de la autenticidad:
a) El usuario A deber protegerse ante mensajes
dirigidos a B que un tercer usuario desconocido C
introduce por ste. Es la suplantacin de identidad o
problema de la autenticacin del emisor.
b) El usuario A deber protegerse ante mensajes
falsificados por B que asegura haberlos recibido
firmados por A. Es la falsificacin de documento o
problema de la autenticacin del mensaje.
Jorge Rami Aguirre Madrid (Espaa) 2003
Seguridad Informtica y Criptografa. Tema 8: Sistemas de Cifra Modernos
Diapositiva 379
La autenticacin de origen y de destino
Autenticacin
Clave Secreta Clave Pblica
Se puede autenticar Al haber una clave
al mensaje pero no pblica y otra privada,
al emisor de forma se podr autenticar el
sencilla y clara. mensaje y al emisor.
En cuanto a la autenticacin, los sistemas simtricos
tienen una autenticacin ms pesada y con una tercera
parte de confianza. Los asimtricos permiten una
firma digital verdadera, eficiente y sencilla.
Jorge Rami Aguirre Madrid (Espaa) 2003
Seguridad Informtica y Criptografa. Tema 8: Sistemas de Cifra Modernos
Diapositiva 380
La velocidad de cifra
Velocidad de cifra
Clave Secreta Clave Pblica
La velocidad de La velocidad de cifra
cifra es muy alta. es muy baja. Se usa
Es el algoritmo de para el intercambio de
cifra del mensaje. clave y la firma digital.
En cuanto a la velocidad de cifra, los
sistemas simtricos son de 100 a 1.000
veces ms rpidos que los asimtricos.
En SW la velocidad de cifra es ms baja.
Cientos de
M Bytes/seg
en HW
Cientos de
K Bytes/seg
en HW
Jorge Rami Aguirre Madrid (Espaa) 2003
Seguridad Informtica y Criptografa. Tema 8: Sistemas de Cifra Modernos
Diapositiva 381
Resumen cifra simtrica v/s asimtrica
Cifrado Simtrico Cifrado Asimtrico
Confidencialidad
Autenticacin parcial
Sin firma digital
Claves:
Longitud pequea
Vida corta
Nmero elevado
Velocidad alta
Confidencialidad
Autenticacin total
Con firma digital
Claves:
Longitud grande
Vida larga
Nmero reducido
Velocidad baja
Fin del Tema 8
Jorge Rami Aguirre Madrid (Espaa) 2003
Seguridad Informtica y Criptografa. Tema 8: Sistemas de Cifra Modernos
Diapositiva 382
Cuestiones y ejercicios (1 de 2)
1. En un sistema de cifra se usa un cuerpo de trabajo n. Cmo es el
tamao de ese cuerpo comparado con el tamao del alfabeto usado?
2. Cmo se clasifican los criptosistemas en funcin del tratamiento
que hacemos del mensaje a cifrar?
3. Cmo se clasifican los criptosistemas en funcin de tipo de clave
que se usa en ambos extremos, emisor y receptor?
4. Por qu se dice que un sistema es simtrico y el otro asimtrico?
5. Es posible cumplir 100% con la condicin del cifrado de Vernam?
6. Por qu en los cifradores de flujo se usa la misma funcin XOR en
el extremo emisor y en el extremo receptor? Son inversas aqu las
claves usadas para cifrar y descifrar?
7. Indique y comente algunas debilidades de los sistemas de cifra en
bloque con clave secreta.
Jorge Rami Aguirre Madrid (Espaa) 2003
Seguridad Informtica y Criptografa. Tema 8: Sistemas de Cifra Modernos
Diapositiva 383
Cuestiones y ejercicios (2 de 2)
8. Si ciframos un nmero con la clave pblica del usuario receptor,
qu cree Ud. que estamos haciendo?
9. Por qu decimos que en un sistema asimtrico la gestin de claves
es mucho mejor que en un sistema simtrico?
10. Nos entregan un certificado digital (certificacin de clave pblica)
de 512 bits. Es hoy en da un valor adecuado? Por qu s o o?
11. Por qu decimos que con un sistema asimtrico es muy fcil
generar una firma digital en emisin y comprobarla en destino?
12. Compare los sistemas simtricos y asimtricos en cuanto a su
velocidad de cifra.
13. Qu es un cifrado hbrido? Por qu y cmo se usa la cifra hbrida
en el intercambio de informacin segura por ejemplo en Internet?
14. Qu relacin hay entre vida de una clave y principio de caducidad?
Tema 9
Sistemas de Cifra en Flujo
Seguridad Informtica y Criptografa
Ultima actualizacin: 03/03/03
Archivo con 48 diapositivas
Jorge Rami Aguirre
Universidad Politcnica de Madrid
Material Docente de
Libre Distribucin
v 3.1
Este archivo forma parte de un curso sobre Seguridad Informtica y Criptografa. Se autoriza la
reproduccin en computador e impresin en papel slo con fines docentes o personales, respetando
en todo caso los crditos del autor. Queda prohibida su venta, excepto a travs del Departamento de
Publicaciones de la Escuela Universitaria de Informtica, Universidad Politcnica de Madrid, Espaa.
Curso de Seguridad Informtica y Criptografa JRA
Jorge Rami Aguirre Madrid (Espaa) 2003
Seguridad Informtica y Criptografa. Tema 9: Sistemas de Cifra en Flujo
Diapositiva 386
Cifrador de flujo bsico
Siguiendo la propuesta de cifrador hecha en 1917 por
Vernam, los cifradores de flujo (clave secreta) usan:
Una cifra basada en la funcin XOR.
Una secuencia cifrante binaria y aleatoria S que se obtiene
de una clave secreta K compartida por emisor y receptor.
Un algoritmo de descifrado que es igual al de cifrado por la
involucin de la funcin XOR.
Clave K Clave K
Algoritmo
Determinstico
Algoritmo
Determinstico

secuencia cifrante
M M
S S
MENSAJE MENSAJE
C
Jorge Rami Aguirre Madrid (Espaa) 2003
Seguridad Informtica y Criptografa. Tema 9: Sistemas de Cifra en Flujo
Diapositiva 387
Caractersticas de la secuencia cifrante S
i
Condiciones para una clave segura
Perodo:
La clave deber ser tanto o ms larga que el mensaje.
En la prctica se usar una semilla de unos 120 a 250
bits para generar perodos superiores a 10
35
.
Distribucin de bits:
Distribucin uniforme de unos y ceros que represente
una secuencia pseudoaleatoria (Postulados Golomb).
Rachas de dgitos: uno o ms bits entre dos bits distintos.
Funcin de Autocorrelacin Fuera de Fase AC(k):
desplazamiento de k bits sobre la misma secuencia S.
Rachas y AC(k)
Jorge Rami Aguirre Madrid (Espaa) 2003
Seguridad Informtica y Criptografa. Tema 9: Sistemas de Cifra en Flujo
Diapositiva 388
Rachas de dgitos en una secuencia
Rachas de una secuencia S de perodo 15
1 1 1 1 0 1 0 1 1 0 0 1 0 0 0
1 1 1 1 0 1 0 1 1 0 0 1 0 0 0
Prximo
Bit anterior
Rachas de 1s
bit es un 1
es un 0
Rachas de 0s
Un 1 entre dos 0s
Un 0 entre dos 1s
Racha de 00s
Racha de 1111s
Un 00 entre dos 1s
Racha de 11s
Un 1111 entre dos 0s
Un 11 entre dos 0s
Racha de 000s
Un 000 entre dos 1s
Jorge Rami Aguirre Madrid (Espaa) 2003
Seguridad Informtica y Criptografa. Tema 9: Sistemas de Cifra en Flujo
Diapositiva 389
Distribucin de las rachas de dgitos
Las rachas, es decir la secuencia de dgitos iguales entre
dos dgitos distintos, debern seguir una distribucin
estadstica de forma que la secuencia cifrante S
i
tenga un
comportamiento de clave aleatoria o pseudoaleatoria.
Para que esto se cumpla, es obvio que habr ms rachas
cortas que rachas largas como en el ejemplo anterior.
Como veremos ms adelante, esta distribucin seguir
una progresin geomtrica. Por ejemplo una secuencia S
i
podra tener 8 rachas de longitud uno, 4 de longitud dos,
2 de longitud tres y 1 de longitud cuatro.
Jorge Rami Aguirre Madrid (Espaa) 2003
Seguridad Informtica y Criptografa. Tema 9: Sistemas de Cifra en Flujo
Diapositiva 390
Autocorrelacin fuera de fase AC(k)
Funcin de Autocorrelacin:
Autocorrelacin AC(k) fuera de fase de una secuencia
S
i
de perodo T desplazada k bits a la izquierda:
AC(k) = (A - F) / T
Aciertos bits iguales Fallos bits diferentes
Ejemplo
Si k = 1
S
i
1 1 1 1 0 1 0 1 1 0 0 1 0 0 0
A =
1 1 1 0 1 0 1 1 0 0 1 0 0 0 1 A=7; F=8
F = AC(1)= -1/15
Jorge Rami Aguirre Madrid (Espaa) 2003
Seguridad Informtica y Criptografa. Tema 9: Sistemas de Cifra en Flujo
Diapositiva 391
Autocorrelacin fuera de fase constante
1 1 1 1 0 1 0 1 1 0 0 1 0 0 0 S
i
Como ejercicio, compruebe que para esta secuencia cifrante
S
i
la Autocorrelacin Fuera de Fase AC(k) para todos los
valores de k (1 k 14) es constante e igual a -1/15.
Esto ser importante para la calidad de la clave.
Para que una secuencia cifrante sea considerada segura,
adems de cumplir con la distribucin de rachas, deber
tener una AC(k) constante como veremos ms adelante.
Jorge Rami Aguirre Madrid (Espaa) 2003
Seguridad Informtica y Criptografa. Tema 9: Sistemas de Cifra en Flujo
Diapositiva 392
Imprevisibilidad e implementacin de S
i
Imprevisibilidad:
Aunque se conozca una parte de la secuencia S
i
, la
probabilidad de predecir el prximo dgito no debe ser
superior al 50%.
Esto se define a partir de la Complejidad Lineal.
Facilidad de implementacin:
Debe ser fcil construir un generador de secuencia
cifrante con circuitos electrnicos y chips, con bajo
coste, alta velocidad, bajo consumo, un alto nivel de
integracin, etc.
Jorge Rami Aguirre Madrid (Espaa) 2003
Seguridad Informtica y Criptografa. Tema 9: Sistemas de Cifra en Flujo
Diapositiva 393
Primer postulado de Golomb G1
Postulado G1:
Deber existir igual nmero de ceros que de unos. Se
acepta como mximo una diferencia igual a la unidad.
S
1
1 1 1 1 0 1 0 1 1 0 0 1 0 0 0
En la secuencia S
1
de 15 bits, hay 8 unos y 7
ceros. Luego s cumple con el postulado G1.
S
2
1 1 0 1 0 1 0 1 0 0 0 1 0 0 0 1
En la secuencia S
2
de 16 bits, hay 7 unos y 9
ceros. Luego no cumple con el postulado G1.
Jorge Rami Aguirre Madrid (Espaa) 2003
Seguridad Informtica y Criptografa. Tema 9: Sistemas de Cifra en Flujo
Diapositiva 394
Significado del postulado G1
S
i
1 1 1 1 0 1 0 1 1 0 0 1 0 0 0
Qu significa esto?
Si una secuencia S
i
cumple con G1, quiere decir que la
probabilidad de recibir un bit 1 es igual a la de recibir un bit 0, es
decir un 50%.
Por lo tanto, a lo largo de una secuencia S
i
, independientemente
de los bits recibidos con anterioridad, en media ser igualmente
probable recibir un 1 que un 0, pues hay una mitad de
valores uno y otra mitad de valores cero.
Jorge Rami Aguirre Madrid (Espaa) 2003
Seguridad Informtica y Criptografa. Tema 9: Sistemas de Cifra en Flujo
Diapositiva 395
Segundo postulado de Golomb G2
Postulado G2:
En un perodo T, la mitad de las rachas de S
i
sern de
longitud 1, la cuarta parte de longitud 2, la octava parte
de longitud 3, etc.
1 1 1 1 0 1 0 1 1 0 0 1 0 0 0 S
i
En la secuencia S
i
de 15 bits, hay 4 rachas de longitud uno, 2
rachas de longitud dos, 1 racha de longitud tres y 1 racha de
longitud cuatro. Este tipo de distribucin en las rachas para
perodos impares, es tpica de las denominadas m-secuencias
como veremos ms adelante en el apartado generadores LFSR.
Las rachas de
esta secuencia
estn en una
diapositiva
anterior
Jorge Rami Aguirre Madrid (Espaa) 2003
Seguridad Informtica y Criptografa. Tema 9: Sistemas de Cifra en Flujo
Diapositiva 396
Significado del postulado G2
S
i
1 1 1 1 0 1 0 1 1 0 0 1 0 0 0
Qu significa esto?
Si una secuencia S
i
cumple con G2, quiere decir que la
probabilidad de recibir un bit 1 0 despus de haber recibido un
1 o un 0 es la misma, es decir un 50%.
Es decir, recibido por ejemplo un 1, la cadena 10 es
igualmente probable que la cadena 11. Lo mismo sucede con
un 0 al comienzo, un 00, 01, 10, 11, 000, 001, etc. Existe una
equiprobabilidad tambin en funcin de los bits ya recibidos.
Como comprobaremos ms adelante, esto va a significar que la
secuencia pasa por todos sus estados, es decir todos sus restos.
Jorge Rami Aguirre Madrid (Espaa) 2003
Seguridad Informtica y Criptografa. Tema 9: Sistemas de Cifra en Flujo
Diapositiva 397
Tercer postulado de Golomb G3 (1)
Postulado G3:
La autocorrelacin AC(k) deber ser constante
para todo valor de desplazamiento de k bits.
Secuencia original S
i
0 1 1 1 0 1 0 0
Desplazamiento de un bit a la izquierda
1 1 1 0 1 0 0 0
k=1 AC(1) = (4-4)/8 = 0
1 1 0 1 0 0 0 1
k=2 AC(2) = (4-4)/8 = 0
1 0 1 0 0 0 1 1
AC(3) = (2-6)/8 = -1/2 k=3
0 1 0 0 0 1 1 1
k=4
AC(4) = (4-4)/8 = 0
sigue
Jorge Rami Aguirre Madrid (Espaa) 2003
Seguridad Informtica y Criptografa. Tema 9: Sistemas de Cifra en Flujo
Diapositiva 398
Tercer postulado de Golomb G3 (2)
0 1 1 1 0 1 0 0
S
i
Secuencia original
1 0 0 0 1 1 1 0
k=5 AC(5) = (2-6)/8 = -1/2
0 0 0 1 1 1 0 1
k=6 AC(6) = (4-4)/8 = 0
0 0 1 1 1 0 1 0
AC(7) = (4-4)/8 = 0 k=7
0 1 1 1 0 1 0 0
k=8 Secuencia original en fase
La secuencia S
i
= 01110100 de 8 bits no cumple con G3.
S
i
= 10101100 s cumple. Compruebe que AC(k) = - .
Jorge Rami Aguirre Madrid (Espaa) 2003
Seguridad Informtica y Criptografa. Tema 9: Sistemas de Cifra en Flujo
Diapositiva 399
Significado del postulado G3
0 1 1 1 0 1 0 0
S
i
No cumple con G3
1 0 1 0 1 1 0 0
S
i
S cumple con G3
Qu significa esto?
Si una secuencia cumple con el postulado G3 quiere decir
que, independientemente del trozo de secuencia elegido por
el atacante, no habr una mayor cantidad de informacin que
en la secuencia anterior. As, ser imposible aplicar ataques
estadsticos a la secuencia recibida u observada al igual
como operbamos, por ejemplo y guardando las debidas
distancias, con el sistema Vigenre y el ataque de Kasiski.
Jorge Rami Aguirre Madrid (Espaa) 2003
Seguridad Informtica y Criptografa. Tema 9: Sistemas de Cifra en Flujo
Diapositiva 400
Generador de congruencia lineal
x
i+1
= (ax
i
b)(mod n) secuencia cifrante
Los valores a, b, n caracterizan al generador y
se utilizan como clave secreta.
El valor x
0
se conoce como semilla; es el que
inicia el proceso generador de la clave X
i
.
La secuencia se genera de i = 0 hasta i = n-1.
Tiene como debilidad que resulta relativamente
fcil atacar la secuencia, de forma similar a los
cifradores afines de la criptografa clsica.
Jorge Rami Aguirre Madrid (Espaa) 2003
Seguridad Informtica y Criptografa. Tema 9: Sistemas de Cifra en Flujo
Diapositiva 401
Ejemplo generador de congruencia lineal
x
i+1
= (ax
i
b)(mod n)
x
1
= (510+1) mod 16 = 3 x
2
= (53+1) mod 16 = 0
x
3
= (50+1) mod 16 = 1 x
4
= (51+1) mod 16 = 6
x
5
= (56+1) mod 16 = 15 x
6
= (515+1) mod 16 = 12
x
7
= (512+1) mod 16 = 13 x
8
= (513+1) mod 16 = 2
x
9
= (52+1) mod 16 = 11 x
10
= (511+1) mod 16 = 8
x
11
= (58+1) mod 16 = 9 x
12
= (59+1) mod 16 = 14
x
13
= (514+1) mod 16 = 7 x
14
= (57+1) mod 16 = 4
x
15
= (54+1) mod 16 = 5 x
16
= (55+1) mod 16 = 10
S
i
= 10, 3, 0, 1, 6, 15, 12, 13, 2, 11, 8, 9, 14, 7, 4, 5
Sea:
a = 5 b = 1
n = 16 x
0
= 10
Sea:
a = 5 b = 1
n = 16 x
0
= 10
Pero...
Jorge Rami Aguirre Madrid (Espaa) 2003
Seguridad Informtica y Criptografa. Tema 9: Sistemas de Cifra en Flujo
Diapositiva 402
Algo falla en este generador?
Qu sucede si
a = 11 b = 1
n = 16 x
0
= 7?
Qu sucede si
a = 11 b = 1
n = 16 x
0
= 7?
x
i+1
= (ax
i
b)(mod n)
Ejercicios
Qu sucede si
a = 5 b = 2
n = 16 x
0
= 10?
Qu sucede si
a = 5 b = 2
n = 16 x
0
= 10?
Qu sucede si
a = 5 b = 2
n = 16 x
0
= 1?
Qu sucede si
a = 5 b = 2
n = 16 x
Qu sucede si
a = 4 b = 1
n = 16 x
0
= 10?
0
= 1?
Qu sucede si
a = 4 b = 1
n = 16 x
0
= 10?
Saque sus propias conclusiones.
Como habr comprobado, este tipo de generadores de secuencia
cifrante no son criptogrficamente nada interesantes.
Una vez hecho esto personalmente, pase a la siguiente diapositiva.
Jorge Rami Aguirre Madrid (Espaa) 2003
Seguridad Informtica y Criptografa. Tema 9: Sistemas de Cifra en Flujo
Diapositiva 403
Debilidad en este tipo de generadores
S
i
= (117 + 1) mod 16
S
i
= 15, 7
El perodo que se genera es
slo de tamao dos ...
S
i
= (510 + 2) mod 16
S
i
= 4, 6, 0, 2, 12, 14, 8, 10
S
i
= (51 + 2) mod 16
S
i
= 7, 5, 11, 9, 15, 13, 3, 1
Se obtiene un perodo muy
bajo y slo valores pares e
impares. El primer caso es
igual que el de los apuntes
pero con b = 2 ...
S
i
= (410 + 1) mod 16
S
i
= 9, 5, 5, ....
Peor an, ya no se genera
una secuencia ...
Jorge Rami Aguirre Madrid (Espaa) 2003
Seguridad Informtica y Criptografa. Tema 9: Sistemas de Cifra en Flujo
Diapositiva 404
Registros de desplazamiento
Generador de secuencia cifrante con registros de desplazamiento
Realimentacin
g[a(t-1)a(t-2) ...a(t-n+1)]a(t-n)
S
n-1
S
i
Bit que se pierde
Desplazamiento
Genera una secuencia con un perodo mximo 2
n
a(t-1)
a(t-2)
a(t-3) a(t-4) a(t-n+1) a(t-n)
Registros de Desplazamiento Realimentados No Linealmente
Registros de Desplazamiento Realimentados Linealmente
NLFSR
LFSR
a(i) es el contenido de la celda i
S
i
es un bit 0 1
Conexiones de puertas
?
S
1
S
2
S
3
S
4
S
n
Jorge Rami Aguirre Madrid (Espaa) 2003
Seguridad Informtica y Criptografa. Tema 9: Sistemas de Cifra en Flujo
Diapositiva 405
Generador NLFSR de 4 celdas (1)
Generador de cuatro celdas (n = 4)
S
1
S
2
S
3
S
4
AND
XOR
NOT
OR
0 0
1
0
0 1 1 1
1
Este es el estado
de las celdas y
las operaciones
previas antes de
producirse el
desplazamiento
de un bit hacia a
la derecha.
S
1
S
2
S
3
S
4
Primera
operacin
S
i
Sea la semilla: S
1
S
2
S
3
S
4
= 0111 Operaciones
Jorge Rami Aguirre Madrid (Espaa) 2003
Seguridad Informtica y Criptografa. Tema 9: Sistemas de Cifra en Flujo
Diapositiva 406
Generador NLFSR de 4 celdas (2)
S
1
S
2
S
3
S
4
AND
XOR
NOT
OR
S
i
Semilla: S
1
S
2
S
3
S
4
= 0111
0 0
1
0
0 1 1 1
1
1 1 1 0 1
Observe que
primero se
transmite
S
4
S
3
S
2
S
1
S
1
S
2
S
3
S
4
S
i
= 1110 1100 1010 0001. T
mx
= 2
n
= 2
4
= 16. Se conoce
como secuencia de De Bruijn. El contenido de las celdas
pasa por todos los estados posibles: 0000 1111.
Jorge Rami Aguirre Madrid (Espaa) 2003
Seguridad Informtica y Criptografa. Tema 9: Sistemas de Cifra en Flujo
Diapositiva 407
Generadores lineales LFSR
a(t) = C
1
a(t-1) C
2
a(t-2) C
3
a(t-3) ... C
n
a(t-n)
C
i
= {1,0} conexin/no conexin celda C
n
= 1
Funcin nica: XOR T
mx
= 2
n
- 1
Polinomio asociado:
f(x) = C
n
x
n
+ C
n-1
x
n-1
+ .... + C
2
x
2
+ C
1
x + 1
S
1
S
2
S
3
S
4
XOR
S
i
Generador
LFSR de 4
etapas/celdas
Generador
LFSR de 4
etapas/celdas
C
1
C
2
C
3
C
4
Jorge Rami Aguirre Madrid (Espaa) 2003
Seguridad Informtica y Criptografa. Tema 9: Sistemas de Cifra en Flujo
Diapositiva 408
Tipos de generadores lineales LFSR
Observacin: como la nica funcin de realimentacin de un
LFSR es un XOR, no estar permitida la cadena de todos ceros.
En funcin del polinomio asociado tendremos:
LFSR con polinomios factorizables
No sern criptogrficamente interesantes.
LFSR con polinomios irreducibles
No sern criptogrficamente interesantes.
LFSR con polinomios primitivos
Segn los postulados de Golomb, este tipo de polinomio
que genera todos los estados lineales posibles del cuerpo
de trabajo n, ser el que nos entregue una secuencia
cifrante de inters criptogrfico con perodo T = 2
n
1.
Jorge Rami Aguirre Madrid (Espaa) 2003
Seguridad Informtica y Criptografa. Tema 9: Sistemas de Cifra en Flujo
Diapositiva 409
Generador LFSR con f(x) factorizable
Generador f(x) factorizable de cuatro celdas (n = 4)
S
1
S
2

S
3
S
4
Sea f(x) = x
4
+ x
2
+ 1
f(x) es factorizable porque:
Sea f(x
1
) = f(x
2
) = (x
2
+x+1)
f(x) = f(x
1
) f(x
2
)
f(x) = (x
2
+x+1) (x
2
+x+1)
f(x) = x
4
+2x
3
+3x
2
+2x+1
Tras la reduccin mdulo 2
Luego f(x) = x
4
+ x
2
+1
S
i
T depender de la semilla
T 2
n
- 1
Y adems, habr perodos
secundarios divisores de T.
Problema
Jorge Rami Aguirre Madrid (Espaa) 2003
Seguridad Informtica y Criptografa. Tema 9: Sistemas de Cifra en Flujo
Diapositiva 410
Ejemplo de LFSR con f(x) factorizable (1)
f(x) = x
4
+ x
2
+ 1
S
1
S
2
S
3
S
4

S
i
0 1 1 1
Sea ahora la semilla:
S
1
S
2
S
3
S
4
= 0111
Registro Bit S
i
Registro Bit S
i
1110 0
0111 1
Primer bit:
resultado de
la operacin
S
1
= S
2
S
4
1 0011 1111 1
semilla . . .
1 1001 0111 1
1100 0
S
i
= 111001 T = 6
Jorge Rami Aguirre Madrid (Espaa) 2003
Seguridad Informtica y Criptografa. Tema 9: Sistemas de Cifra en Flujo
Diapositiva 411
Ejemplo de LFSR con f(x) factorizable (2)
f(x) = x
4
+ x
2
+ 1
S
1
S
2
S
3
S
4

S
i
1 1 0 1
Sea la semilla:
S
1
S
2
S
3
S
4
= 1101
S
1
S
2
S
3
S
4
Registro Bit S
i
1101 1
T es un perodo
secundario y en
en este caso es
incluso menor
que la semilla.
Primer bit:
resultado de
la operacin
S
1
= S
2
S
4
S
i
= 101
T = 3
0 0110
1 1011
semilla . . .
1101 1
Jorge Rami Aguirre Madrid (Espaa) 2003
Seguridad Informtica y Criptografa. Tema 9: Sistemas de Cifra en Flujo
Diapositiva 412
Generador LFSR con f(x) irreducible
Generador f(x) irreducible de cuatro celdas (n = 4)
Sea f(x) = x
4
+ x
3
+ x
2
+ x + 1
S
1
S
2

S
3
S
4
Es imposible factorizar
en mdulo 2 la funcin
f(x) mediante dos
polinomios f(x
1
) y f(x
2
)
de grado menor
S
i
Ahora T ya no depende de la semilla
pero ser un factor de T
mx
= 2
n
1 y
no obtendremos un perodo mximo.
Problema
Jorge Rami Aguirre Madrid (Espaa) 2003
Seguridad Informtica y Criptografa. Tema 9: Sistemas de Cifra en Flujo
Diapositiva 413
Ejemplo de LFSR con f(x) irreducible
f(x) = x
4
+ x
3
+ x
2
+ x + 1
S
1
S
2
S
3
S
4

S
i
0 0 0 1
Sea ahora la semilla:
S
1
S
2
S
3
S
4
= 0001
Registro Bit S
i
Registro
0001 1
1000 0
1100 0
0110 0
Primer bit:
resultado de
la operacin
S
1
= S
2
S
4
0011 1
Bit S
i
semilla . . .
0001 1
S
i
= 100011 T = 5 siendo
T
mx
= 2
n
- 1 = 2
4
- 1 = 15
Jorge Rami Aguirre Madrid (Espaa) 2003
Seguridad Informtica y Criptografa. Tema 9: Sistemas de Cifra en Flujo
Diapositiva 414
Generador LFSR con f(x) primitivo
Generador f(x) primitivo de cuatro celdas (n = 4)
Sea f(x) = x
4
+ x + 1
S
1

S
2
S
3
S
4
f(x) no es factorizable
como f(x
1
)f(x
2
) en
mdulo dos. Es adems
un generador del grupo.
S
i
T ya no depender de la
semilla y ser un valor
mximo T
mx
= 2
n
- 1.
Se generan m-secuencias
Habr (2
n
- 1)/n
polinomios primitivos
Habr (2
n
- 1)/n
polinomios primitivos
Jorge Rami Aguirre Madrid (Espaa) 2003
Seguridad Informtica y Criptografa. Tema 9: Sistemas de Cifra en Flujo
Diapositiva 415
Ejemplo de LFSR con f(x) primitivo
Generador f(x) primitivo de cuatro celdas (n = 4)
S
1
S
2
S
3
S
4

S
i
1 0 0 1
S
1
= S
1
S
4
S
i
= 100100011110101
f(x) = x
4
+ x + 1
S
1
S
2
S
3
S
4
= 1001
Registro Bit S
i
1001 1
T = 2
n
- 1
T = 2
4
- 1
T = 15
0100 0 1110 0 1010 0
0010 0 1111 1 1101 1
0001 1 0111 1 0110 0
1000 0 1011 1 0011 1
1100 0 0101 1 1001 T = 15
Jorge Rami Aguirre Madrid (Espaa) 2003
Seguridad Informtica y Criptografa. Tema 9: Sistemas de Cifra en Flujo
Diapositiva 416
Secuencia S
i
de un LFSR con f(x) primitivo
Caractersticas
Secuencia mxima de 2
n
- 1 bits
Cumple con G1:
Hay 2n bits 1 y 2n-1 bits 0
Cumple con G2:
Distribucin de rachas de m-secuencia. El vector
binario de las celdas pasa por todos los estados
excepto la cadena de ceros que est prohibida.
Cumple con G3:
Los aciertos (A) sern iguales a 2
n-1
- 1
m-secuencia
Jorge Rami Aguirre Madrid (Espaa) 2003
Seguridad Informtica y Criptografa. Tema 9: Sistemas de Cifra en Flujo
Diapositiva 417
Rachas en S
i
de un LFSR con f(x) primitivo
Rachas de Longitud Rachas de Ceros Rachas de Unos
2
...
p
...
n-2
n-1
n
1
TOTAL
2
n-4
...
2
n-p-2
...
1
1
0
2
n-3
2
n-2
2
n-4
...
2
n-p-2
...
1
0
1
2
n-3
2
n-2
Rachas de una m-secuencia
Sin embargo, no es un generador ideal para la cifra
porque su Complejidad Lineal es muy baja.
Jorge Rami Aguirre Madrid (Espaa) 2003
Seguridad Informtica y Criptografa. Tema 9: Sistemas de Cifra en Flujo
Diapositiva 418
Debilidad de un LFSR con f(x) primitivo
Como este LFSR genera una secuencia de longitud
mxima, sta ser previsible y se puede encontrar la
secuencia completa S
i
de 2
n
- 1 bits ...
con slo conocer 2n bits !
Por ejemplo, si en un sistema de 8 celdas con un
perodo 2
8
-1 = 255 conocemos 28 = 16 bits seremos
capaces de encontrar las conexiones de las celdas o
valores de C
i
y generar as la secuencia completa S
i
.
Esta debilidad es la que usa el ataque conocido como
algoritmo de Berlekamp-Massey.
Jorge Rami Aguirre Madrid (Espaa) 2003
Seguridad Informtica y Criptografa. Tema 9: Sistemas de Cifra en Flujo
Diapositiva 419
Ejemplo de ataque de Berlekamp-Massey
Si conocemos 2n = 8 bits S
1
S
2
S
3
S
4
S
5
S
6
S
7
S
8
de un LFSR
de 4 celdas C
1
C
2
C
3
C
4
, tenemos el sistema de ecuaciones:
S
5
= C
1
S
1
C
2
S
2
C
3
S
3
C
4
S
4
S
6
= C
1
S
5
C
2
S
1
C
3
S
2
C
4
S
3
S
7
= C
1
S
6
C
2
S
5
C
3
S
1
C
4
S
2
S
8
= C
1
S
7
C
2
S
6
C
3
S
5
C
4
S
1

S
i S
1
S
2
S
3
S
4
C
1
C
2
C
3
C
4
=1
Si asignamos valores
de esos 2n = 8 bits
S
1
S
2
S
3
S
4
S
5
S
6
S
7
S
8
seremos capaces de
resolver este sistema
Primero se transmite
S
4
S
3
S
2
S
1
(semilla) y
luego bits S
5
S
6
S
7
S
8
.
Jorge Rami Aguirre Madrid (Espaa) 2003
Seguridad Informtica y Criptografa. Tema 9: Sistemas de Cifra en Flujo
Diapositiva 420
Solucin al ataque de Berlekamp-Massey
S
5
= C
1
S
1
C
2
S
2
C
3
S
3
C
4
S
4
S
6
= C
1
S
5
C
2
S
1
C
3
S
2
C
4
S
3
S
7
= C
1
S
6
C
2
S
5
C
3
S
1
C
4
S
2
S
8
= C
1
S
7
C
2
S
6
C
3
S
5
C
4
S
1
Si los 8 bits
S
1
S
2
S
3
S
4
S
5
S
6
S
7
S
8
son 1100 1000
S
1
= 0 S
5
= 1
S
2
= 0 S
6
= 0
S
3
= 1 S
7
= 0
S
4
= 1 S
8
= 0
1 = C
1
0 C
2
0 C
3
1 C
4
1
0 = C
1
1 C
2
0 C
3
0 C
4
1
C
1
= 1 C
2
= 0
0 = C
1
0 C
2
1 C
3
0 C
4
0
C
3
= 0 C
4
= 1
0 = C
1
0 C
2
0 C
3
1 C
4
0
Jorge Rami Aguirre Madrid (Espaa) 2003
Seguridad Informtica y Criptografa. Tema 9: Sistemas de Cifra en Flujo
Diapositiva 421
Conclusiones ataque Berlekamp-Massey
CONCLUSIONES:
Como se conoce la configuracin del generador LFSR, y
S
i
es una m-secuencia de perodo 2
n
- 1, entonces por el
conjunto de n celdas pasarn todos los restos del campo de
Galois de 2
n
, excepto la cadena de n ceros que sabemos
est prohibida en estos sistemas generadores lineales.
Para el ejemplo anterior, esto quiere decir que cualquier
grupo de 2n = 8 dgitos correlativos nos permite generar la
secuencia mxima, en este caso de 2
n
= 16 bits.
La solucin es aumentar la complejidad lineal del
generador por ejemplo conectando varios LFRs.
Jorge Rami Aguirre Madrid (Espaa) 2003
Seguridad Informtica y Criptografa. Tema 9: Sistemas de Cifra en Flujo
Diapositiva 422
Complejidad lineal LC
o Un LFSR con polinomio primitivo de n celdas tendr una
complejidad lineal LC igual a n; es decir con 2n bits se
puede generar la secuencia completa como hemos visto.
o Lo ideal es que si este LFSR entrega una secuencia S
i
con
un perodo igual a 2
n
1, su LC fuese cercana a este valor.
o Para aumentar esta LC podemos usar:
o Operaciones no lineales de las secuencias del LFSR
o Operaciones de suma
o Operaciones de multiplicacin
o Filtrado no lineal de los estados del LFSR.
Jorge Rami Aguirre Madrid (Espaa) 2003
Seguridad Informtica y Criptografa. Tema 9: Sistemas de Cifra en Flujo
Diapositiva 423
Operaciones no lineales con dos registros
LC = n
1
; T = 2
n1
-1
Generador primitivo con n
1
celdas
Generador primitivo con n
2
celdas

LC = n
2
T = 2
n2
-1
LC = n
1
+ n
2
T = mcm (2
n1
-1, 2
n2
-1)
Es el modelo usado por A5
S
i
LC = n
1
; T = 2
n1
-1
Generador primitivo con n
1
celdas
Generador primitivo con n
2
celdas

LC = n
2
T = 2
n2
-1
LC = n
1
n
2
T = mcm (2
n1
-1, 2
n2
-1)
S
i
Jorge Rami Aguirre Madrid (Espaa) 2003
Seguridad Informtica y Criptografa. Tema 9: Sistemas de Cifra en Flujo
Diapositiva 424
Generadores LFSR con filtrado no lineal
Generador de Geffe
LFSR 2
LFSR 3
LFSR 1
Selector
a
2
a
3
a
1
S
i
Si a
1
es un 0 S
i
es el bit de a
2
Si a
1
es un 1 S
i
es el bit de a
3
Luego: S
i
= a
2
a
1
a
2
a
1
a
3
0
1
LC = (n
1
+ 1)n
2
n
1
n
3
T = mcm (2
n1
-1, 2
n2
-1, 2
n3
-1)
Se mejora la LC e incluso se aumenta si ponemos ms LFSRs pero
este generador es dbil ante ataques por correlacin de bits.
Existen una infinidad de esquemas en esta lnea, entre ellos los de
Beth-Piper, Jennings, Gollmann y Massey-Rueppel. Encontrar
mayor informacin consultando la bibliografa del tema 18.
Jorge Rami Aguirre Madrid (Espaa) 2003
Seguridad Informtica y Criptografa. Tema 9: Sistemas de Cifra en Flujo
Diapositiva 425
Algoritmos de cifrado en flujo
Sistemas ms conocidos:
A5:
Algoritmo no publicado propuesto en 1994. Versiones
A5/1 fuerte (Europa) y A5/2 dbil (exportacin).
RC4:
Algoritmo de RSA Corp. (Rivest Cipher #4) desarrollado
en el ao 1987, usado en Lotus Notes y luego en el
navegador de Netscape desde 1999. No es pblico.
SEAL:
Algoritmo propuesto por IBM en 1994.
Jorge Rami Aguirre Madrid (Espaa) 2003
Seguridad Informtica y Criptografa. Tema 9: Sistemas de Cifra en Flujo
Diapositiva 426
El algoritmo de cifra A5
El uso habitual de este algoritmo lo encontramos en el
cifrado del enlace entre el abonado y la central de un
telfono mvil (celular) tipo GSM.
Con cerca de 130 millones de usuarios en Europa y
otros 100 millones de usuarios en el resto del mundo,
el sistema ha sucumbido a un ataque en diciembre de
1999 realizado por Alex Biryukov y Adi Shamir.
Esta es una consecuencia inevitable en el mundo de la
criptografa cuando los desarrolladores de algoritmos
no hacen pblico el cdigo fuente.
Jorge Rami Aguirre Madrid (Espaa) 2003
Seguridad Informtica y Criptografa. Tema 9: Sistemas de Cifra en Flujo
Diapositiva 427
Esquema del algoritmo de cifra A5/1

S
i
3 LFSR con
m-secuencia
R
1
n
1
= 19
R
2
n
2
= 22
R
3
n
3
= 23
Clave = 64 bits
f(x
1
) = x
19
+x
18
+x
17
+x
14
+1
C1 14 19 1
9: bit de reloj
C2 22
1
11: bit de reloj
C3 23 8 1
11: bit de reloj
R
1
R
2
R
3
Una funcin
mayora entre
C1, C2 y C3
hace que slo
los registros en
los que coincide
el bit con ese
valor produzcan
desplazamiento.
En cada paso
habr dos o tres
registros en
movimiento.
f(x
2
) = x
22
+x
21
+1
f(x
3
) = x
23
+x
22
+x
21
+x
8
+1
Jorge Rami Aguirre Madrid (Espaa) 2003
Seguridad Informtica y Criptografa. Tema 9: Sistemas de Cifra en Flujo
Diapositiva 428
Consideraciones sobre el perodo de A5/1
El perodo T vendr dado por el mximo comn mltiplo de los
tres perodos individuales:
T = mcm (2
n1
- 1, 2
n2
- 1, 2
n3
- 1)
Como n
1
, n
2
y n
3
son primos entre s, tambin lo sern los valores
(2
n1
-1), (2
n2
- 1) y (2
n3
- 1). Luego el perodo T ser el producto:
T = T
1
T
2
T
3
Entonces T = (2
19
-1)(2
22
-1)(2
23
-1) = 524.2874.194.3038.388.607
T = 18.446.702.292.280.803.327 < 2
64
que es un valor demasiado
bajo incluso para mediados de la dcada pasada.
Jorge Rami Aguirre Madrid (Espaa) 2003
Seguridad Informtica y Criptografa. Tema 9: Sistemas de Cifra en Flujo
Diapositiva 429
Registros y funcin mayora en A5/2
Usa los mismos tres registros de desplazamiento con polinomio
primitivo que A5/1:
f(x
1
) = x
19
+ x
18
+ x
17
+ x
14
+ 1
f(x
2
) = x
22
+ x
21
+ 1
f(x
3
) = x
23
+ x
22
+ x
21
+ x
8
+ 1
Adems, usa un cuarto registro R
4
con un polinomio primitivo:
f(x
4
) = x
17
+ x
12
+ 1
Usa cuatro copias de una funcin mayora F para cada uno de los
cuatro registros que se define como:
F(x
1
,x
2
,x
3
) = x
1
x
2
+ x
1
x
3
+ x
2
x
3
Jorge Rami Aguirre Madrid (Espaa) 2003
Seguridad Informtica y Criptografa. Tema 9: Sistemas de Cifra en Flujo
Diapositiva 430
Otras operaciones de A5/2
En R
1
las entradas a la funcin F
1
son las celdas 13, 15 y 16.
En R
2
las entradas a la funcin F
2
son las celdas 10, 14 y 17.
En R
3
las entradas a la funcin F
3
son las celdas 14, 17 y 19.
En R
4
las entradas a la funcin F
4
son las celdas 4, 8 y 11.
La salida de esta copia determina qu registros de R
1
,R
2
,R
3
se desplazarn en el ciclo.
Complementacin de celdas y sumas en salida de F:
En R
1
se complementa la celda 15 y se suma la celda 19 a F.
En R
2
se complementa la celda 17 y se suma la celda 22 a F.
En R
3
se complementa la celda 14 y se suma la celda 23 a F.
Fin del Tema 9
Jorge Rami Aguirre Madrid (Espaa) 2003
Seguridad Informtica y Criptografa. Tema 9: Sistemas de Cifra en Flujo
Diapositiva 431
Cuestiones y ejercicios (1 de 3)
1. Por qu en los sistemas de cifra en flujo se usa una funcin XOR
tanto en emisor como en receptor? Son las claves inversas?
2. Si tenemos una clave de 16 bits, cul es el perodo mximo que
podremos lograr en una secuencia cifrante? Por qu?
3. Qu rachas encuentra en la secuencia 110100111010100?
4. Por qu es lgico esperar ms rachas cortas que rachas largas?
5. Si en una secuencia cifrante se observa una correlacin fuera de fase
no constante, qu significa? Podramos hacer un ataque similar al
que permite romper el sistema de cifra polialfabtico Vigenre?
6. A nivel de probabilidades de ocurrencia de bits, qu significan los
postulados de Golomb G1 y G2?
7. Qu significa que una secuencia cifrante pase por todos los estados
o restos posibles? Cul sera en este caso su perodo?
Jorge Rami Aguirre Madrid (Espaa) 2003
Seguridad Informtica y Criptografa. Tema 9: Sistemas de Cifra en Flujo
Diapositiva 432
Cuestiones y ejercicios (2 de 3)
8. Qu secuencias se obtiene con un generador de congruencia lineal
en el que a = 7, b = 4 y n = 8? Y si ahora hacemos a = 3?
9. Qu tipo de ataque podramos intentar para romper una secuencia
cifrante obtenida con un generador de congruencia lineal?
10. Por qu en un registro de desplazamiento siempre se realimenta el
bit de la ltima celda, bit que sale a lnea?
11. En el generador NLFSR de los apuntes si se cambia la funcin AND
por una OR, qu sucede con la secuencia? Saque conclusiones.
12. Decimos que los generadores LFSR tienen asociado un polinomio
f(x) = a
n
x
n
+ a
n-1
x
n-1
+ a
n-2
x
n-2
+ ... + a
2
x
2
+ a
1
x + 1, donde a
i
toma los
valores de 0 1. Por qu f(x) termina en 1?
13. Qu polinomio elegira para un LFSR, un polinomio factorizable,
uno irreducible o uno primitivo? Por qu?
Jorge Rami Aguirre Madrid (Espaa) 2003
Seguridad Informtica y Criptografa. Tema 9: Sistemas de Cifra en Flujo
Diapositiva 433
Cuestiones y ejercicios (3 de 3)
14. Por qu no est permitida la cadena de n ceros en un generador
LFSR de n etapas y en cambio s en los NLFSR?
15. En el generador LFSR con f(x) primitivo de 4 etapas, la secuencia
pasa por todos los restos excepto 0000. Si usamos hora una semilla
distinta, debemos hacer otra vez todos los clculos o no? Por qu?
16. Justifique la distribucin de las rachas en una m-secuencia. Por qu
tiene ese comportamiento extrao al final de las rachas largas?
17. Qu debilidad de los sistemas LFSR con polinomios primitivos usa
el algoritmo de Berlekamp-Massey para realizar el ataque?
18. En un ataque de Berlekamp-Massey, importa en qu posicin de la
secuencia se encuentran los 2n bits? Por qu?
19. Por qu cree que el algoritmo A5 es dbil? Cul fue el peor error
cometido por sus creadores?
Tema 10
Cifrado Simtrico en Bloque
Seguridad Informtica y Criptografa
Ultima actualizacin: 03/03/03
Archivo con 87 diapositivas
Jorge Rami Aguirre
Universidad Politcnica de Madrid
Material Docente de
Libre Distribucin
v 3.1
Este archivo forma parte de un curso sobre Seguridad Informtica y Criptografa. Se autoriza la
reproduccin en computador e impresin en papel slo con fines docentes o personales, respetando
en todo caso los crditos del autor. Queda prohibida su venta, excepto a travs del Departamento de
Publicaciones de la Escuela Universitaria de Informtica, Universidad Politcnica de Madrid, Espaa.
Curso de Seguridad Informtica y Criptografa JRA
Jorge Rami Aguirre Madrid (Espaa) 2003
Seguridad Informtica y Criptografa. Tema 10: Cifrado Simtrico en Bloque
Diapositiva 436
Cifrado y descifrado genrico en bloque
MENSAJE (N bits)


FUNCIN f
k
i
X
i
A B
A X
i
B X
i
BLOQUE A BLOQUE B


FUNCIN f
N/2 N/2
k
i
X
i
A X
i
B X
i
Puesto que Y X
i
X
i
= Y
Jorge Rami Aguirre Madrid (Espaa) 2003
Seguridad Informtica y Criptografa. Tema 10: Cifrado Simtrico en Bloque
Diapositiva 437
Cifrado tipo Feistel
Horst Feistel: inventor (IBM) del algoritmo LUCIFER a comienzos de
los aos 70. El algoritmo fue utilizado por el Reino Unido. En 1974 se
propone a la NSA como estndar y en ese ao dar origen al DES.
Dado un bloque de N bits (tpico
64) ste se dividir en dos mitades.
Existir una funcin unidireccional F
(muy difcil de invertir).
Se realizan operaciones con la clave
k
i
slo con una mitad del bloque, y
se permutan en cada vuelta las dos
mitades, operacin que se repite
durante n vueltas.
Jorge Rami Aguirre Madrid (Espaa) 2003
Seguridad Informtica y Criptografa. Tema 10: Cifrado Simtrico en Bloque
Diapositiva 438
Un ejemplo bsico de cifrado tipo Feistel
El algoritmo usar bloques de tamao 8 caracteres. Tendr dos
vueltas y en cada vuelta realizar una operacin de sustitucin
S y una permutacin P sobre la 1 mitad.
Sustitucin: C
i
= (M
i
+1 ) mod 27
Permutacin: C
i
=
3241
(el carcter 1 pasa a la 4 posicin
en el criptograma, el 4 a la 3, el 2 a la 2 y el 3 a la 1)
Mensaje: M = STAR WARS, LA MISIN CONTINA
Jorge Rami Aguirre Madrid (Espaa) 2003
Seguridad Informtica y Criptografa. Tema 10: Cifrado Simtrico en Bloque
Diapositiva 439
Cifrado tipo Feistel en cuerpo n = 27
S
i
: +1 mod 27
P
i
:
3241
M = STAR WARS, LA MISIN CONTINA
M
1
= STAR WARS LAMI SION CONT INUA
S
1
= TUBS WARS MBNJ SION DPU INUA
P
1
= BUST WARS NBJM SION PUD INUA
M
2
= WARS BUST SION NBJM INUA PUD
S
2
= XBST BUST TJP NBJM JVB PUD
P
2
= SBTX BUST PJT NBJM VBJ PUD
M = STAR WARS, LA MISIN CONTINA
M
1
= STAR WARS LAMI SION CONT INUA
S
1
= TUBS WARS MBNJ SION DPU INUA
P
1
= BUST WARS NBJM SION PUD INUA
M
2
= WARS BUST SION NBJM INUA PUD
S
2
= XBST BUST TJP NBJM JVB PUD
P
2
= SBTX BUST PJT NBJM VBJ PUD
Primera
vuelta
Segunda
vuelta
C = SBTX BUST PJT NBJM VBJ PUD
Aunque le parezca increble, el DES har prcticamente lo mismo
trabajando con bits y con funciones un poco ms complejas.
Jorge Rami Aguirre Madrid (Espaa) 2003
Seguridad Informtica y Criptografa. Tema 10: Cifrado Simtrico en Bloque
Diapositiva 440
Cifradores de bloque ms conocidos
Algoritmo Bloque (bits) Clave (bits) Vueltas
Lucifer 128 128 16
DES 64 56 16
Loki 64 64 16
RC2 64 variable --
CAST 64 64 8
Blowfish 64 variable 16
IDEA 64 128 8
Skipjack 64 80 32
RIJNDAEL 128 128 o ms flexible
E

Jorge Rami Aguirre Madrid (Espaa) 2003


Seguridad Informtica y Criptografa. Tema 10: Cifrado Simtrico en Bloque
Diapositiva 441
Caractersticas de estos algoritmos
Lucifer: algoritmo original tipo Feistel que dar lugar al DES.
DES: algoritmo tipo Feistel que se convirti en estndar durante
casi treinta aos. Hoy es vulnerable por su longitud de clave.
Loki: algoritmo australiano similar al DES, tipo Feistel.
RC2: algoritmo propuesto por Ron Rivest y que se incluye en
navegadores de Internet desde 1999.
CAST: algoritmo tipo Feistel que se ofrece como cifrador por
defecto en ltimas versiones de PGP.
Blowfish: algoritmo tipo Feistel propuesto por Bruce Schneier.
IDEA: algoritmo europeo usado en el correo electrnico PGP.
Skipjack: propuesta de nuevo estndar en USA a finales de los
90 para comunicaciones oficiales (tiene puerta trasera).
RIJNDAEL: nuevo estndar mundial desde finales de 2001.
Jorge Rami Aguirre Madrid (Espaa) 2003
Seguridad Informtica y Criptografa. Tema 10: Cifrado Simtrico en Bloque
Diapositiva 442
Otros cifradores de bloque
Algoritmo Bloque (bits) Clave (bits) Vueltas
Twofish 128 variable variable
Khufu 64 512 16, 24, 32
Khafre 64 128 ms vueltas
Gost 64 256 32
RC5 variable variable variable
SAFER 64 64 64 8
Akelarre variable variable variable
FEAL 64 64 32
De stos, los ms conocidos son Twofish -uno de los candidatos a
AES y que lo encontraremos en ltimas versiones de PGP- y RC5.
Jorge Rami Aguirre Madrid (Espaa) 2003
Seguridad Informtica y Criptografa. Tema 10: Cifrado Simtrico en Bloque
Diapositiva 443
Caractersticas de estos algoritmos
Twofish: Propuesto por Bruce Schneir despus de Blowfish, de
tipo Feistel, diseo simple, sin claves dbiles y multiplataforma.
Khufu: algoritmo propuesto por Ralph Merkle con una clave
generada con un sistema de cajas S.
Khafre: algoritmo propuesto por Ralph Merkle en el que la
clave ya no depende de las cajas S.
Gost: algoritmo similar al DES con cajas S secretas propuesto
en la Unin Sovitica.
RC5: algoritmo propuesto por Ron Rivest; realiza operaciones
or exclusivo, suma modular y desplazamiento de bits.
SAFER 64: algoritmo propuesto por James Massey.
Akelarre: algoritmo espaol propuesto en 1996 por el CSIC,
Consejo Superior de Investigaciones Cientficas.
FEAL: algoritmo propuesto en Japn.
Jorge Rami Aguirre Madrid (Espaa) 2003
Seguridad Informtica y Criptografa. Tema 10: Cifrado Simtrico en Bloque
Diapositiva 444
Algunas tasas de cifra comparativas
Velocidad de cifra de algoritmos en un PC 486 a 33 MHz
Algoritmo Kbytes/seg Algoritmo Kbytes/seg
DES 35 Triple DES 12
IDEA 53 FEAL (32 v) 91
Khufu (16 v) 221 Khufu (32 v) 115
RC5 (8 v) 127 RC5 (16 v) 65
SAFER (6 v) 81 SAFER (12 v) 41
Blowfish (12 v) 182 Blowfish (20 v) 110
Estos son slo valores
indicativos ya que la
velocidad del PC es
algo baja (extrapolar).
Fuente: Criptografa Digital. Fundamentos y
Aplicaciones. Jos Pastor y Miguel Angel Sarasa,
Prensas Universitarias de Zaragoza (1998).
Jorge Rami Aguirre Madrid (Espaa) 2003
Seguridad Informtica y Criptografa. Tema 10: Cifrado Simtrico en Bloque
Diapositiva 445
Algoritmos DES, IDEA y AES
Profundizaremos en estas diapositivas en los algoritmos
DES, Triple DES, IDEA y RIJNDAEL. Por qu?
DES es un cifrador de Feistel, ha sido un estndar y en
aplicaciones bancarias se seguir usando durante tiempo.
DES es de muy fcil comprensin y usa cajas S como varios
algoritmos ms modernos y el actual estndar AES.
Triple DES sigue siendo un estndar en e-commerce.
IDEA es un algoritmo seguro que hace uso de los conceptos de
inversos en un cuerpo finito, como todos los algoritmos de
cifra modernos, y se usa entre otros en la aplicacin PGP.
RIJNDAEL es el nuevo estndar de cifra avanzada, AES.
Jorge Rami Aguirre Madrid (Espaa) 2003
Seguridad Informtica y Criptografa. Tema 10: Cifrado Simtrico en Bloque
Diapositiva 446
Modos de cifra
Todos los algoritmos pueden usarse aplicando diversos
modos de cifra, entre ellos:
ECB: Electronic CodeBook (libro electrnico de cdigos)
CBC: Cipher Block Chaining (encadenamiento de bloques)
CFB: Cipher FeedBack (realimentacin de bloques)
OFB: Output FeedBack (realimentacin bloque de salida)
Analizaremos cada uno de ellos para el caso del DES,
aunque el estudio es extensible a todos los dems ya que en
estos modos el cifrador se considera una caja negra.
Jorge Rami Aguirre Madrid (Espaa) 2003
Seguridad Informtica y Criptografa. Tema 10: Cifrado Simtrico en Bloque
Diapositiva 447
Data Encryption Standard DES
DES (Data Encryption Standard) ha sido el estndar utilizado
mundialmente durante 25 aos, generalmente en la banca. Hoy
presenta signos de envejecimiento y ha sucumbido a los diversos
criptoanlisis que contra l se viene realizando hace ya aos.
FECHAS DE INTERS
1973: En EEUU la NBS National Bureaux of Standards llama a
concurso pblico para buscar un algoritmo criptogrfico estndar.
1974: La NSA National Security Agency declara desierto el
primer concurso, publica unas segundas especificaciones y elige
Lucifer, algoritmo original de IBM (aos 70) con variaciones.
1976: El DES se adopta como estndar y se autoriza para ser
utilizado en las comunicaciones no clasificadas del gobierno.
Jorge Rami Aguirre Madrid (Espaa) 2003
Seguridad Informtica y Criptografa. Tema 10: Cifrado Simtrico en Bloque
Diapositiva 448
Especificaciones del algoritmo DES
Especificaciones del concurso
El nivel de seguridad computacional debe ser alto.
El algoritmo debe ser fcil de entender y deber estar
especificado en todos sus detalles.
La seguridad del sistema no debe verse afectada por la
publicacin y divulgacin del algoritmo.
Debe estar disponible para cualquier usuario.
Deber poder usarse en diferentes aplicaciones.
Fabricacin con dispositivos electrnicos de bajo costo.
Se debe poder usar como validacin.
Debe ser exportable.
No se cumplen en 1973 pero s en 1974, aunque ...
Jorge Rami Aguirre Madrid (Espaa) 2003
Seguridad Informtica y Criptografa. Tema 10: Cifrado Simtrico en Bloque
Diapositiva 449
El papel de la NSA en el DES
La NSA impone una limitacin en la longitud de la clave:
De los 128 bits de Lucifer,
NSA deja la clave en 64
bits. Al final, la clave slo
son 56 bits efectivos puesto
que al ser datos de 8 bits, se
conoce el bit de paridad.
Luego, el espacio de claves
es 2
56
= 7.2 10
16
, tan slo
72 mil billones de valores.
Jorge Rami Aguirre Madrid (Espaa) 2003
Seguridad Informtica y Criptografa. Tema 10: Cifrado Simtrico en Bloque
Diapositiva 450
Por qu esta reduccin?
Hay distintas versiones sobre esta
reduccin del espacio de claves: una
habla de la dificultad de disear
chips capaces de operar de forma
eficiente con una clave de 128 bits
en esos aos; la otra sobre una
poltica de seguridad interna para
proteger informacin sensible ante
ataques externos y ser capaces, no
obstante, de practicar criptoanlisis
en un tiempo razonable.
Jorge Rami Aguirre Madrid (Espaa) 2003
Seguridad Informtica y Criptografa. Tema 10: Cifrado Simtrico en Bloque
Diapositiva 451
Especificaciones tcnicas finales del DES
Bloque a cifrar: 64 bits
Clave: 8 bytes (con paridad, no caracteres ASCII)
Normas ANSI:
X3.92: Descripcin del algoritmo.
X3.108: Descripcin de los modos de operacin
(ECB, CBC, OFB).
Fcil implementacin en un circuito integrado.
Veremos su descripcin y modos de operacin.
Jorge Rami Aguirre Madrid (Espaa) 2003
Seguridad Informtica y Criptografa. Tema 10: Cifrado Simtrico en Bloque
Diapositiva 452
Visin general del DES
+ Cifrador de bloque
+ Tipo Feistel
+ Longitud de clave de 56 bits
+
+
Realiza
Realiza 16 vueltas.
+
+
La cifra del b
La cifra del bloque central usa
tcnicas de sustituciones y
permutaciones.
+ Para poder realizar las sumas or
exclusivo, usar permutaciones
con expansin y compresin para
igualar el nmero de bits.
En el descifrado se aplican claves y
desplazamientos en sentido inverso
Jorge Rami Aguirre Madrid (Espaa) 2003
Seguridad Informtica y Criptografa. Tema 10: Cifrado Simtrico en Bloque
Diapositiva 453
Permutacin inicial del DES: tabla IP
58 50 42 34 26 18 10 2
60 52 44 36 28 20 12 4
62 54 46 38 30 22 14 6
64 56 48 40 32 24 16 8
57 49 41 33 25 17 9 1
59 51 43 35 27 19 11 3
61 53 45 37 29 21 13 5
63 55 47 39 31 23 15 7

El bit 1 se lleva a la posicin 40
El bit 1 se lleva a la posicin 40
Tabla IP sobre bloque de texto
(no tiene inters criptogrfico)
Jorge Rami Aguirre Madrid (Espaa) 2003
Seguridad Informtica y Criptografa. Tema 10: Cifrado Simtrico en Bloque
Diapositiva 454
Bloques izquierdo y derecho de texto
58 50 42 34 26 18 10 2
60 52 44 36 28 20 12 4
62 54 46 38 30 22 14 6
64 56 48 40 32 24 16 8
57 49 41 33 25 17 9 1
59 51 43 35 27 19 11 3
61 53 45 37 29 21 13 5
63 55 47 39 31 23 15 7

L
0
= 58 50 42 34 26 18 10 02 60 52 44 36
28 20 12 04 62 54 46 38 30 22 14 06
64 56 48 40 32 24 16 08
R
0
= 57 49 41 33 25 17 09 01 59 51 43 35
27 19 11 03 61 53 45 37 29 21 13 05
63 55 47 39 31 23 15 07
Observe la distribucin correlativa que existe entre los
bits del bloque izquierdo L
0
y del bloque derecho R
0
de
texto. Este tipo de distribucin de los bits en tablas, a
simple vista caprichosa, ser muy comn en el DES.
Jorge Rami Aguirre Madrid (Espaa) 2003
Seguridad Informtica y Criptografa. Tema 10: Cifrado Simtrico en Bloque
Diapositiva 455
Permutacin final del DES: tabla IP
-1
40 8 48 16 56 24 64 32
39 7 47 15 55 23 63 31
38 6 46 14 54 22 62 30
37 5 45 13 53 21 61 29
36 4 44 12 52 20 60 28
35 3 43 11 51 19 59 27
34 2 42 10 50 18 58 26
33 1 41 9 49 17 57 25

El bit 40 vuelve a la posicin 1
y todos los dems bits a su
posicin inicial antes de IP.
El bit 40 vuelve a la posicin 1
y todos los dems bits a su
posicin inicial antes de IP.
Tabla IP
-1
Jorge Rami Aguirre Madrid (Espaa) 2003
Seguridad Informtica y Criptografa. Tema 10: Cifrado Simtrico en Bloque
Diapositiva 456
Operaciones en cada ciclo del DES
Se permuta la mitad derecha R
i
aplicando expansin a 48 bits
La clave de 56 bits se desplaza,
permuta y se seleccionan los 48
bits de K
i
La nueva mitad derecha R
i
y la
clave K
i
se suman XOR
Se reducen los 48 bits de salida
a 32 bits mediante las Cajas-S
Se permuta el resultado
El resultado se suma XOR con
la mitad izquierda L
i
En la prxima
vuelta, la mitad
derecha anterior
pasa ntegra como
mitad izquierda. La
mitad derecha de la
nueva vuelta ser el
ltimo resultado or
exclusivo del ciclo.
EN CADA CICLO:
Jorge Rami Aguirre Madrid (Espaa) 2003
Seguridad Informtica y Criptografa. Tema 10: Cifrado Simtrico en Bloque
Diapositiva 457
Mdulo de cifra en DES
16 bits repetidos
Columnas 1 y
2 repetidas
Esquema de la
funcin de cifra
f en cada ciclo
En las cajas S
se logra la
fortaleza del
algoritmo. Es
una funcin
unidireccional
y no lineal.
En las cajas S
se logra la
fortaleza del
algoritmo. Es
una funcin
unidireccional
y no lineal.
Jorge Rami Aguirre Madrid (Espaa) 2003
Seguridad Informtica y Criptografa. Tema 10: Cifrado Simtrico en Bloque
Diapositiva 458
Operacin de las cajas S en el DES
1 6 7 12 13 18 19 24 25 30 31 36 37 42 43 48
1 4 5 8 9 12 13 16 17 20 21 24 25 28 29 32
Permutacin con expansin a 48 bits (Tabla E)
k
i
(48 bits)
Operacin Or Exclusivo con la subclave k
i
S
1
S
2
S
3
S
4
S
5
S
6
S
7
S
8
Sustitucin con compresin a 32 bits (Cajas S)
Permutacin de Salida del ciclo (Tabla P)
De cada 6 bits de entrada se obtienen 4 bits de salida
No lineal y unidireccional. Hay cuatro soluciones de entrada para cada salida
Jorge Rami Aguirre Madrid (Espaa) 2003
Seguridad Informtica y Criptografa. Tema 10: Cifrado Simtrico en Bloque
Diapositiva 459
Valores de las cajas S
1
y S
2
del DES
C O L U M N A S
0 1 2 3 4 5 6 7 8 9 10 11 12 13 14 1 5
0
1
2
3
S
1
F
I
L
A
S
14 4 13 1 2 15 11 8 3 10 6 12 5 9 0 7
0 15 7 4 14 2 13 1 10 6 12 11 9 5 3 8
4 1 14 8 13 6 2 11 15 12 9 7 3 10 5 0
15 12 8 2 4 9 1 7 5 11 3 14 10 0 6 13
0 1 2 3 4 5 6 7 8 9 10 11 12 13 14 1 5
0
1
2
3
C O L U M N A S
S
2
F
I
L
A
S
15 1 8 14 6 11 3 4 9 7 2 13 12 0 5 10
3 13 4 7 15 2 8 14 12 0 1 10 6 9 11 5
0 14 7 11 10 4 13 1 5 8 12 6 9 3 2 15
13 8 10 1 3 15 4 2 11 6 7 12 0 5 14 9
Jorge Rami Aguirre Madrid (Espaa) 2003
Seguridad Informtica y Criptografa. Tema 10: Cifrado Simtrico en Bloque
Diapositiva 460
Valores de las cajas S
3
y S
4
del DES
C O L U M N A S
0 1 2 3 4 5 6 7 8 9 10 11 12 13 14 1 5
0
1
2
3
S
3
F
I
L
A
S
10 0 9 14 6 3 15 5 1 13 12 7 11 4 2 8
13 7 0 9 3 4 6 10 2 8 5 14 12 11 15 1
13 6 4 9 8 15 3 0 11 1 2 12 5 10 14 7
1 10 13 0 6 9 8 7 4 15 14 3 11 5 2 12
0 1 2 3 4 5 6 7 8 9 10 11 12 13 14 1 5
0
1
2
3
C O L U M N A S
S
4
F
I
L
A
S
7 13 14 3 0 6 9 10 1 2 8 5 11 12 4 15
13 8 11 5 6 15 0 3 4 7 2 12 1 10 14 9
10 6 9 0 12 11 7 13 15 1 3 14 5 2 8 4
3 15 0 6 10 1 13 8 9 4 5 11 12 7 2 14
Jorge Rami Aguirre Madrid (Espaa) 2003
Seguridad Informtica y Criptografa. Tema 10: Cifrado Simtrico en Bloque
Diapositiva 461
Valores de las cajas S
5
y S
6
del DES
C O L U M N A S
0 1 2 3 4 5 6 7 8 9 10 11 12 13 14 1 5
0
1
2
3
S
5
F
I
L
A
S
2 12 4 1 7 10 11 6 8 5 3 15 13 0 14 9
14 11 2 12 4 7 13 1 5 0 15 10 3 9 8 6
4 2 1 11 10 13 7 8 15 9 12 5 6 3 0 14
11 8 12 7 1 14 2 13 6 15 0 9 10 4 5 3
0 1 2 3 4 5 6 7 8 9 10 11 12 13 14 1 5
0
1
2
3
C O L U M N A S
S
6
F
I
L
A
S
12 1 10 15 9 2 6 8 0 13 3 4 14 7 5 11
10 15 4 2 7 12 9 5 6 1 13 14 0 11 3 8
9 14 15 5 2 8 12 3 7 0 4 10 1 13 11 6
4 3 2 12 9 5 15 10 11 14 1 7 6 0 8 13
Jorge Rami Aguirre Madrid (Espaa) 2003
Seguridad Informtica y Criptografa. Tema 10: Cifrado Simtrico en Bloque
Diapositiva 462
Valores de las cajas S
7
y S
8
del DES
C O L U M N A S
0 1 2 3 4 5 6 7 8 9 10 11 12 13 14 1 5
0
1
2
3
S
7
F
I
L
A
S
4 11 2 14 15 0 8 13 3 12 9 7 5 10 6 1
13 0 11 7 4 9 1 10 14 3 5 12 2 15 8 6
1 4 11 13 12 3 7 14 10 15 6 8 0 5 9 2
6 11 13 8 1 4 10 7 9 5 0 15 14 2 3 12
0 1 2 3 4 5 6 7 8 9 10 11 12 13 14 1 5
0
1
2
3
C O L U M N A S
S
8
F
I
L
A
S
13 2 8 4 6 15 11 1 10 9 3 14 5 0 12 7
1 15 13 8 10 3 7 4 12 5 6 11 0 14 9 2
7 11 4 1 9 12 14 2 0 6 10 13 15 3 5 8
2 1 14 7 4 10 8 13 15 12 9 0 3 5 6 11
Jorge Rami Aguirre Madrid (Espaa) 2003
Seguridad Informtica y Criptografa. Tema 10: Cifrado Simtrico en Bloque
Diapositiva 463
Ejemplo de operacin de cajas S del DES
Ejemplo:
Sean los bits 7 al 12 los siguientes: 101100
Los bits correspondern entonces a la entrada de la caja S
2
Para seleccionar la fila tomamos los bits extremos: 10
2
= 2
10
= 2
Para seleccionar la columna tomamos los bits centrales: 0110
2
= 6
10
= 6
La caja S
2
indica una salida igual a 13
10
= 1101
2
Ejemplo:
Sean los bits 7 al 12 los siguientes: 101100
Los bits correspondern entonces a la entrada de la caja S
2
Para seleccionar la fila tomamos los bits extremos: 10
2
= 2
10
= 2
Para seleccionar la columna tomamos los bits centrales: 0110
2
= 6
10
= 6
La caja S
2
indica una salida igual a 13
10
= 1101
2
explicacin
Entrada: 101100
Salida: 1101
Jorge Rami Aguirre Madrid (Espaa) 2003
Seguridad Informtica y Criptografa. Tema 10: Cifrado Simtrico en Bloque
Diapositiva 464
Clculo de subclaves en el DES (PC-1)
57 49 41 33 25 17 9
1 58 50 42 34 26 18
10 2 59 51 43 35 27
19 11 3 60 52 44 36
63 55 47 39 31 23 15
7 62 54 46 38 30 22
14 6 61 53 45 37 29
21 13 5 28 20 12 4

Se han eliminado los bits de paridad:
8, 16, 24, 32, 40, 48, 56, 64.
28 bits 28 bits
Tabla PC-1 (56 bits)
Jorge Rami Aguirre Madrid (Espaa) 2003
Seguridad Informtica y Criptografa. Tema 10: Cifrado Simtrico en Bloque
Diapositiva 465
Clculo de subclaves en el DES (PC-2)
4 17 11 24 1 5
3 28 15 6 21 10
23 19 12 4 26 8
16 7 27 20 13 2
41 52 31 37 47 55
30 40 51 45 33 48
44 49 39 56 34 53
46 42 50 36 29 32

Se han eliminado los bits:
9, 18, 22, 25, 35, 38, 43, 54.
28 bits 28 bits
48 bits
48 bits
48 bits
Tabla PC-2 (48 bits) k
1
, k
2
, ..., k
16
Jorge Rami Aguirre Madrid (Espaa) 2003
Seguridad Informtica y Criptografa. Tema 10: Cifrado Simtrico en Bloque
Diapositiva 466
Desplazamiento de subclaves en el DES
Se produce un desplazamiento total igual
a 28, todos los bits de cada bloque Ci y Di
28 bits 28 bits
LF
1
, LF
2
, ..., LF
16
Vuelta i Bits Desp. Izda.
1 1
2 1
3 2
4 2
5 2
6 2
7 2
8 2
Vuelta i Bits Desp. Izda.
9 1
10 2
11 2
12 2
13 2
14 2
15 2
16 1

Jorge Rami Aguirre Madrid (Espaa) 2003
Seguridad Informtica y Criptografa. Tema 10: Cifrado Simtrico en Bloque
Diapositiva 467
Operacin de descifrado en el DES
Se toman en sentido contrario:
k
16
, k
15
, k
14
, k
13
, k
12
, k
11
, k
10
,
k
9
, k
8
, k
7
, k
6
, k
5
, k
4
, k
3
, k
2
, k
1
Como se aplica un desplazamiento
de 28 bits en cada bloque de clave,
entonces D
16
= D
0
y C
16
= C
0
Los desplazamientos para el clculo de
las subclaves de descifrado son los
mismos de la tabla anterior pero ahora
se toman hacia la derecha, puesto que
los desplazamientos coinciden.
64 bits de criptograma
Jorge Rami Aguirre Madrid (Espaa) 2003
Seguridad Informtica y Criptografa. Tema 10: Cifrado Simtrico en Bloque
Diapositiva 468
Claves dbiles y semidbiles
Claves dbiles:
Una clave es dbil si se verifica que: E
k
2
(M) = M
Son claves k dbiles estas cuatro:
0101010101010101 FEFEFEFEFEFEFEFE
EOEOEOEOF1F1F1F1 1F1F1F1F0E0E0E0E
Nota: E0E0E0E0F1F1F1F1 = (en ANSI).
Compruebe esta clave dbil con el software CripMod de la asignatura.
Claves semidbiles:
Una clave es semidbil si se verifica que: E
k1
[E
k2
(M)] = M
Son claves k
1
,k
2
semidbiles las siguientes seis parejas:
(01FE01FE01FE01FE, FE01FE01FE01FE01)
(1FE01FE00EF10EF1, E01FE01FF10EF10E)
(01E001E001F101F1, E001E001F101F101)
(1FFE1FFE0EFE0EFE, FE1FFE1FFE0EFE0E)
(011F011F010E010E, 1F011F010E010E01)
(E0FEE0FEF1FEF1FE, FEE0FEE0FEF1FEF1)
Jorge Rami Aguirre Madrid (Espaa) 2003
Seguridad Informtica y Criptografa. Tema 10: Cifrado Simtrico en Bloque
Diapositiva 469
Ejemplo de clculo de claves en DES
Caso 1: a partir de las tablas PC-1 y PC-2, encuentre la secuencia de bits
de los registros C
1
y D
1
y la subclave k
1
.
Solucin:
i = 01 02 03 04 05 06 07 08 09 10 11 12 13 14 15 16 17 18 19 20 21 22 23 24 25 26 27 28
C
1
= 49 41 33 25 17 09 01 58 50 42 34 26 18 10 02 59 51 43 35 27 19 11 03 60 52 44 36 57
i = 29 30 31 32 33 34 35 36 37 38 39 40 41 42 43 44 45 46 47 48 49 50 51 52 53 54 55 56
D
1
= 55 47 39 31 23 15 07 62 54 46 38 30 22 14 06 61 53 45 37 29 21 13 05 28 20 12 04 63
k
1
= 10 51 34 60 49 17 33 57 02 09 19 42 03 35 26 25 44 58 59 01 36 27 18 41
22 28 39 54 37 04 47 30 05 53 23 29 61 21 38 63 15 20 45 14 13 62 55 31
Caso 2: si la clave es PRUEBALO, encuentre C
0
y D
0
y la subclave k
1
.
Para encontrar C
0
y D
0
escriba en ASCII la clave y elimine el ltimo bit
como si ste fuese el de paridad.
Solucin:
C
0
= 0000 0000 1111 1111 0000 0000 0000
C
1
= 1001 0010 1100 1100 1100 0000 0111
k
1
= 101000 001001 001001 000010 101101 010100 100111 100100
Jorge Rami Aguirre Madrid (Espaa) 2003
Seguridad Informtica y Criptografa. Tema 10: Cifrado Simtrico en Bloque
Diapositiva 470
Modo de cifra ECB
Lo dicho, estos modos son vlidos para todos los cifradores.
MODO ECB
Electronic CodeBook: cifra cada bloque con la clave k de
forma independiente. Por lo tanto, el resultado es como si se
codificase mediante un gran libro electrnico de cdigos.
Recuerde: codificar no es lo mismo que cifrar.
Debilidades:
Se podra reconstruir ese libro electrnico sin necesidad
de conocer la clave.
Aparece el problema denominado de comienzos y finales
fijos que permiten un tipo de ataque sencillo.
Se ataca a travs de la repeticin de bloques similares.
Jorge Rami Aguirre Madrid (Espaa) 2003
Seguridad Informtica y Criptografa. Tema 10: Cifrado Simtrico en Bloque
Diapositiva 471
Caractersticas del modo ECB en DES
Cada bloque de 64 bits del texto en claro se
pasa por el cifrador, usando la misma clave de
64 bits.
Para bloques de texto en claro iguales, se
obtiene siempre el mismo criptograma
Como a cada bloque de texto en claro le
corresponde un nico cdigo o texto cifrado de
salida y ste es constante, este modo de cifra
lleva por nombre Libro Electrnico de Cdigos.
Es como si tuvisemos un gran libro de cdigo
con un cdigo distinto para cada mensaje.
Jorge Rami Aguirre Madrid (Espaa) 2003
Seguridad Informtica y Criptografa. Tema 10: Cifrado Simtrico en Bloque
Diapositiva 472
Modo de cifra CBC en DES
Cipher Block Chaining
Cifra por encadenamiento
de bloques
Vector IV = I
0
Se encadenan los
bloques de texto en
claro con el bloque del
criptograma anterior.
Usa un vector de
inicializacin IV de 64
bits que se guarda en
secreto.
Jorge Rami Aguirre Madrid (Espaa) 2003
Seguridad Informtica y Criptografa. Tema 10: Cifrado Simtrico en Bloque
Diapositiva 473
Operaciones de cifra modo CBC en DES
Descifrado
Se descifra el primer bloque con
vector IV:
P
1
= D(C
1
) I
0
P
1
= D[E(P
1
I
0
)] I
0
Se guarda el bloque C
i-1
en un
registro. Se descifra el bloque C
i
y luego XOR entre esos bloques:
M
i
= D(C
i
) C
i-1
Cifrado
El vector IV se suma
XOR a los 64 bits de
texto en claro.
Se cifra con la clave K
esa suma.
El resultado C
i
se usa
como vector IV para el
nuevo bloque.
CARACTERSTICAS:
Evita el ataque por repeticin de bloque; enmascara el mensaje lo
mismo que la cifra en flujo; el espacio de claves es igual a 64 bits;
la propagacin de un error afecta a dos bloques contiguos.
Jorge Rami Aguirre Madrid (Espaa) 2003
Seguridad Informtica y Criptografa. Tema 10: Cifrado Simtrico en Bloque
Diapositiva 474
Modo de cifra CFB en DES
Cipher FeedBack
Cifra por realimentacin
de bloques
CIFRAR
CIFRAR
Se pueden cifrar
unidades de datos ms
pequeas que bloques,
por lo general un byte.
Se usa un registro de
desplazamiento RD de
64 bits como vector
inicial IV.
Jorge Rami Aguirre Madrid (Espaa) 2003
Seguridad Informtica y Criptografa. Tema 10: Cifrado Simtrico en Bloque
Diapositiva 475
Operaciones de cifra modo CFB en DES
Cifrado
Se suma XOR cada byte del
texto claro con bytes resultado
de la cifra de RD y la clave K.
El byte C
i
se enva al registro;
se desplaza 8 bits a la izquierda
hasta formar otro RD y se repite
el proceso de cifra.
Descifrado
Se cifra el registro RD.
Se obtienen de esta forma
los elementos de C
i-d
.
Se suma XOR los C
i-d
con
los C
i
del criptograma para
obtener P
i
.
Se realimenta C
i
al registro
RD y se repite el proceso.
CARACTERSTICAS:
Evita el ataque por repeticin de bloque; enmascara el mensaje como
en cifra en flujo, el espacio de claves es igual a 64 bits; la propagacin
de un error se limita a un bloque.
Jorge Rami Aguirre Madrid (Espaa) 2003
Seguridad Informtica y Criptografa. Tema 10: Cifrado Simtrico en Bloque
Diapositiva 476
Modo de cifra OFB en DES
El DES, la clave y el
Registro RD actan
como un generador de
secuencia cifrante.
Bits menos
significativos
Registro Desplazamiento (64 bits)
Byte
Mensaje
DES
K
C
i-1
B
i
C
i
La realimentacin de la
seal se realiza antes
de la operacin XOR.
Output FeedBack
Cifra por realimentacin
de bloques de salida
El byte se va desplazando por el registro
Si la cifra se realiza bit a bit, OFB se convierte en cifrador de flujo.
Jorge Rami Aguirre Madrid (Espaa) 2003
Seguridad Informtica y Criptografa. Tema 10: Cifrado Simtrico en Bloque
Diapositiva 477
Caractersticas del modo OFB en DES
O Evita el ataque por repeticin de bloque.
O Produce un enmascaramiento del mensaje similar al de
un cifrador de flujo.
O El espacio de claves es igual a 64 bits.
O La propagacin de un error afecta slo a un byte, el
que se realimenta en el registro de desplazamiento.
O Las operaciones de cifrado y descifrado son iguales.
A pesar de las propiedades interesantes de los
ltimos modos, el ms utilizado en los sistemas
de cifra de diversos protocolos es el CBC.
Jorge Rami Aguirre Madrid (Espaa) 2003
Seguridad Informtica y Criptografa. Tema 10: Cifrado Simtrico en Bloque
Diapositiva 478
Cifrado mltiple en un grupo
Si un sistema forma un grupo, entonces cifrar un mensaje M
con una clave k
1
y luego el resultado con una clave k
2
, es lo
mismo que cifrar el mensaje con una nica clave k
3
.
Por ejemplo, el cifrador de Vigenre es un grupo como se
demuestra a continuacin. Sea k
1
= PACO y k
2
= CINE y el
mensaje a cifrar M = ESTO ES UN GRUPO.
M
1
= ESTO ESUN GRUP O
k
1
= PACO PACO PACO P
C
1
= TSVD TSWB VRWE E
M
2
= TSVD TSWB VRWE E
k
2
= CINE CINE CINE C
C
2
= VAIH VAJF XZJI G
Obtendremos lo mismo si ciframos el mensaje M
con la clave k
3
= k
1
+ k
2
= PACO + CINE = RIOS.
Jorge Rami Aguirre Madrid (Espaa) 2003
Seguridad Informtica y Criptografa. Tema 10: Cifrado Simtrico en Bloque
Diapositiva 479
El DES no es un grupo
M
1
= ESTO ESUN GRUP O
k
1
= PACO PACO PACO P
C
1
= TSVD TSWB VRWE E
M
2
= TSVD TSWB VRWE E
k
2
= CINE CINE CINE C
C
2
= VAIH VAJF XZJI G
M
3
= ESTO ESUN GRUP O
k
3
= RIOS RIOS RIOS R
C
3
= VAIH VAJF XZJI G
Como ejercicio compruebe que a resultados similares llega si,
por ejemplo, usa ahora los siguientes pares de claves:
LAPALA y LANUCA; PASA y NADA; PAOS y TERMA.
Cules son las claves k
3
en cada caso?
El DES no ser un grupo y, por lo tanto, permitir el cifrado
mltiple. Esto aumentar el tamao efectivo de la clave.
Jorge Rami Aguirre Madrid (Espaa) 2003
Seguridad Informtica y Criptografa. Tema 10: Cifrado Simtrico en Bloque
Diapositiva 480
Cifrado DES mltiple
k
1
y k
2
son
claves n bits
k
1
k
2
Se duplica la longitud de la clave?
En este modelo, cabe esperar que la longitud efectiva de la
clave sea 2
2n
donde n representa la longitud de bits de las
claves k
1
y k
2
. No obstante esto no es cierto.
En realidad el tamao de la clave resultante en este caso es
equivalente a 2
n+1
, un aumento insignificante para n grande
y por esta razn no se usa.
DES DES
M C C
Jorge Rami Aguirre Madrid (Espaa) 2003
Seguridad Informtica y Criptografa. Tema 10: Cifrado Simtrico en Bloque
Diapositiva 481
Ataque por encuentro a medio camino
k
1
y k
2
son
claves n bits
k
1
k
2
DES DES M C
Y
a) Se descripta el criptograma C por fuerza bruta usando las 2
n
claves
posibles y realizando entonces 2
n
clculos. Se obtiene as Y.
b) Con los textos intermedios Y se forma una tabla ordenada de
textos cifrados con sus correspondientes valores k
2
.
c) Se cifran los textos en claro M elegidos con todas las claves k
1
y se
comparan con Y, realizando un mximo de 2
n
clculos.
d) Una de las claves ser la verdadera y se ha realizado un nmero
menor que 2
n
+ 2
n
= 2
n+1
clculos. Luego la clave real es igual a 2
n+1
.
Este ataque se conoce con el nombre de meet-in-the-middle.
Jorge Rami Aguirre Madrid (Espaa) 2003
Seguridad Informtica y Criptografa. Tema 10: Cifrado Simtrico en Bloque
Diapositiva 482
Triple DES
k
1
y k
2
son
claves n bits
k
1
k
2
k
1
En este caso se logra un valor efectivo de longitud de
clave igual a 2
2n
bits, es decir 2
2 56
= 2
112
bits.
El ejemplo anterior con slo dos claves (equivalente al
de tres claves) se usa por motivos de compatibilidad con
el DES de clave nica. Propuesto por Matyas y Meyer de
IBM, se denomina EDE: Encrypt-Decrypt-Encrypt.
Es inmune a ataques por encuentro a medio camino.
E (DES)
M C
D (DES) E (DES)
Jorge Rami Aguirre Madrid (Espaa) 2003
Seguridad Informtica y Criptografa. Tema 10: Cifrado Simtrico en Bloque
Diapositiva 483
Usos de Triple DES
k
1
k
2
E (DES)
M C
k
1
y k
2
claves de
64 bits
k
1
D (DES) E (DES)
Aunque el algoritmo DES haya sufrido diversos ataques y no se
haya vuelto a certificar por el NIST como estndar de cifrado, el
Triple DES s tiene una gran seguridad debido al tamao de su
clave de 112 bits efectivos y sigue siendo muy vlido en el ao
2002. De hecho, es el algoritmo propuesto en el protocolo SET y
se encuentra, entre otras aplicaciones, en el programa PGP.
Jorge Rami Aguirre Madrid (Espaa) 2003
Seguridad Informtica y Criptografa. Tema 10: Cifrado Simtrico en Bloque
Diapositiva 484
International Data Encryption Algorithm IDEA
Historia del IDEA
En 1990 Xuejia Lai y James Massey proponen el PES,
Proposed Encryption Standard.
En 1991 -debido a los avances de Biham y Shamir en el
criptoanlisis diferencial- los autores proponen el IPES,
Improved Proposed Encryption Standard.
En 1992 los autores proponen finalmente el algoritmo
IDEA, International Data Encryption Algorithm.
En 1999 el algoritmo IDEA, mucho ms seguro que el
DES y sus versiones, se comienza a usar ampliamente en
el sistema de correo electrnico seguro PGP.
Jorge Rami Aguirre Madrid (Espaa) 2003
Seguridad Informtica y Criptografa. Tema 10: Cifrado Simtrico en Bloque
Diapositiva 485
Estructura y esquema de IDEA
Todas sus operaciones
se realizan dentro de
un cuerpo finito
Cifra bloques de 64 bits
en 8 vueltas
Divide la entrada M en
cuatro bloques de 16 bits
Se generan 52 subclaves
de 16 bits a partir de la
clave maestra de 128 bits
Usa 6 claves por vuelta
Hay una transformacin
final con 4 claves para
invertir operacin inicial
Jorge Rami Aguirre Madrid (Espaa) 2003
Seguridad Informtica y Criptografa. Tema 10: Cifrado Simtrico en Bloque
Diapositiva 486
Operaciones matemticas en IDEA
Operaciones bsicas
XOR
Suma mdulo 2
16
(mod 65.536)
Multiplicacin mdulo 2
16
+1
(
(65.537)
Es primo y se
asegura el inverso
multiplicativo
Todas las operaciones se realizan con bloques de 16 bits y el
truco est en que los bloques cuyo valor sea 0 (16 bits) se
cambiarn por la constante 2
16
... de 17 bits! . Comprobar
aqu esto con tantos dgitos es complicado pero podemos ver
un ejemplo con nmeros pequeos que puede interpolarse.
Jorge Rami Aguirre Madrid (Espaa) 2003
Seguridad Informtica y Criptografa. Tema 10: Cifrado Simtrico en Bloque
Diapositiva 487
Operaciones +, y en grupo pequeo
Ejemplo dentro de un grupo n pequeo
Como 2
n
+ 1 debe ser primo, sea n = 2 ya que 2
2
= 4 y 2
2
+ 1 = 5
X Y X + Y X Y X Y
0 00 0 00 0 00 1 01 0 00
0 00 1 01 1 01 0 00 1 01
0 00 2 10 2 10 3 11 2 10
0 00 3 11 3 11 2 10 3 11
1 01 0 00 1 01 0 00 1 01
1 01 1 01 2 10 1 01 0 00
1 01 2 10 3 11 2 10 3 11
1 01 3 11 0 00 3 11 2 10
2 10 0 00 2 10 3 11 2 10
2 10 1 01 3 11 2 10 3 11
2 10 2 10 0 00 0 00 0 00
2 10 3 11 1 01 1 01 1 01
3 11 0 00 3 11 2 10 3 11
3 11 1 01 0 00 3 11 2 10
3 11 2 10 1 01 1 01 1 01
3 11 3 11 2 10 0 00 0 00

Operaciones: + mod 2
n
(mod 4), mod 2
n
+1 (mod 5), XOR (mod 2)
n = 2
dos bits
Veremos cmo
se opera con la
multiplicacin.
La suma y el or
exclusivo son
operaciones
similares.
Jorge Rami Aguirre Madrid (Espaa) 2003
Seguridad Informtica y Criptografa. Tema 10: Cifrado Simtrico en Bloque
Diapositiva 488
Ejemplo de operacin en IDEA
X Y X + Y X Y X Y
0 00 0 00 0 00 1 01 0 00
0 00 1 01 1 01 0 00 1 01
0 00 2 10 2 10 3 11 2 10
0 00 3 11 3 11 2 10 3 11
1 01 0 00 1 01 0 00 1 01
1 01 1 01 2 10 1 01 0 00
1 01 2 10 3 11 2 10 3 11
1 01 3 11 0 00 3 11 2 10
2 10 0 00 2 10 3 11 2 10
2 10 1 01 3 11 2 10 3 11
2 10 2 10 0 00 0 00 0 00
2 10 3 11 1 01 1 01 1 01
3 11 0 00 3 11 2 10 3 11
3 11 1 01 0 00 3 11 2 10
3 11 2 10 1 01 1 01 1 01
3 11 3 11 2 10 0 00 0 00

Operaciones: + mod 2
n
(mod 4), mod 2
n
+1 (mod 5), XOR (mod 2)
Recuerde que 0
es igual a 2
n
= 4
por lo que:
00 = 2
2
x 2
2
= 16 mod 5
= 1
Recuerde que 0
es igual a 2
n
= 4
por lo que:
00 = 2
2
x 2
2
= 16 mod 5
= 1
01 = 2
2
x 1 = 4
= 4 mod 5
= 4 = 0
(por definicin)
01 = 2
2
x 1 = 4
= 4 mod 5
= 4 = 0
(por definicin) 02 = 2
2
x 2 = 8
= 8 mod 5
= 3
02 = 2
2
x 2 = 8
= 8 mod 5
= 3
03 = 2
2
x 3 = 12
= 12 mod 5
= 2
03 = 2
2
x 3 = 12
= 12 mod 5
= 2
Los dems clculos con los diferentes valores de X e Y son todos similares
Jorge Rami Aguirre Madrid (Espaa) 2003
Seguridad Informtica y Criptografa. Tema 10: Cifrado Simtrico en Bloque
Diapositiva 489
Detalles del algoritmo IDEA
Operaciones inversas
al comienzo y al final
del algoritmo. Esto
permite usar el mismo
algoritmo para cifrar
que para descifrar.
MA
Operacin cifrado
Bloque principal
Jorge Rami Aguirre Madrid (Espaa) 2003
Seguridad Informtica y Criptografa. Tema 10: Cifrado Simtrico en Bloque
Diapositiva 490
Bloque principal de IDEA
Estas tres operaciones provocan confusin y no
cumplen las leyes distributiva ni asociativa.

La estructura que crea la


difusin es un bloque bsico
denominado Estructura MA
Multiplication / Addition.
Usa slo dos claves por cada
vuelta y sus entradas F
1
, F
2
as como sus salidas G
1
, G
2
estn conectadas por XOR.
Jorge Rami Aguirre Madrid (Espaa) 2003
Seguridad Informtica y Criptografa. Tema 10: Cifrado Simtrico en Bloque
Diapositiva 491
Generacin de claves en IDEA
A partir de una entrada de
128 bits, se generan las 52
subclaves de cifrado.
26
23 86
64 bits de ltimas claves
Con los
primeros
128 bits se
generan 8
subclaves
de 16 bits
cada una.
Los 64
ltimos
bits de
la fase 7
no se
usan.
Se produce un desplazamiento de 25
bits a la izquierda en cada una de las
7 fases de generacin de claves.
Jorge Rami Aguirre Madrid (Espaa) 2003
Seguridad Informtica y Criptografa. Tema 10: Cifrado Simtrico en Bloque
Diapositiva 492
Desplazamientos de la clave en IDEA
En cada operacin sobre la clave de 128 bits, se obtienen 8
claves de 16 bits de las que slo se usan 6 en cada vuelta.
Las claves restantes se guardan para la siguiente vuelta.

Clave Principal k = 128 bits
001 002 003 004 005 006 007 008 009 010 011 012 013 014 015 016 017 018 019 020 021 022 023 024 025 026 027 028 029 030 031 032
033 034 035 036 037 038 039 040 041 042 043 044 045 046 047 048 049 050 051 052 053 054 055 056 057 058 059 060 061 062 063 064
065 066 067 068 069 070 071 072 073 074 075 076 077 078 079 080 081 082 083 084 085 086 087 088 089 090 091 092 093 094 095 096
097 098 099 100 101 102 103 104 105 106 107 108 109 110 111 112 113 114 115 116 117 118 119 120 121 122 123 124 125 126 127 128

Primeros 16 bits de clave . Ultimos 16 bits de clave
1 001 002 003 004 005 006 007 008 009 010 011 012 013 014 015 016 . 113 114 115 116 117 118 119 120 121 122 123 124 125 126 127 128
2 026 027 028 029 030 031 032 033 034 035 036 037 038 039 040 041 . 010 011 012 013 014 015 016 017 018 019 020 021 022 023 024 025
3 051 052 053 054 055 056 057 058 059 060 061 062 063 064 065 066 . 035 036 037 038 039 040 041 042 043 044 045 046 047 048 049 050
4 076 077 078 079 080 081 082 083 084 085 086 087 088 089 090 091 . 060 061 062 063 064 065 066 067 068 069 070 071 072 073 074 075
5 101 102 103 104 105 106 107 108 109 110 111 112 113 114 115 116 . 085 086 087 088 089 090 091 092 093 094 095 096 097 098 099 100
6 126 127 128 001 002 003 004 005 006 007 008 009 010 011 012 103 . 110 111 112 113 114 115 116 117 118 119 120 121 122 123 124 125
7 023 024 025 026 027 028 029 030 031 032 033 034 035 036 037 038 . 007 008 009 010 011 012 013 014 015 016 017 018 019 020 021 022


Jorge Rami Aguirre Madrid (Espaa) 2003
Seguridad Informtica y Criptografa. Tema 10: Cifrado Simtrico en Bloque
Diapositiva 493
Claves usadas por IDEA en cada en vuelta
La distribucin de bits de subclaves
en cada vuelta sigue una lgica
Primera vuelta: k
1
k
2
k
3
k
4
k
5
k
6
B[196]
Segunda vuelta: k
7
k
8
k
9
k
10
k
11
k
12
B[97128; 2689]
Tercera vuelta: k
13
k
14
k
15
k
16
k
17
k
18
B[90128; 125; 5182]
Cuarta vuelta: k
19
k
20
k
21
k
22
k
23
k
24
B[83128; 150]
Quinta vuelta: k
25
k
26
k
27
k
28
k
29
k
30
B[76128; 143]
Sexta vuelta: k
31
k
32
k
33
k
34
k
35
k
36
B[4475; 101128; 136]
Sptima vuelta: k
37
k
38
k
39
k
40
k
41
k
42
B[37100; 126128; 129]
Octava vuelta: k
43
k
44
k
45
k
46
k
47
k
48
B[30125]
Transformacin: k
49
k
50
k
51
k
52
B[2386]
Jorge Rami Aguirre Madrid (Espaa) 2003
Seguridad Informtica y Criptografa. Tema 10: Cifrado Simtrico en Bloque
Diapositiva 494
Primeras claves en cada vuelta en IDEA
Las primeras claves de cada vuelta k
1
, k
7
, k
13
, k
19
, k
25
, k
31
, k
37
y
k
43
usan un conjunto diferente de bits. Excepto en las vueltas
primera y octava, los 96 bits de subclave usados en cada vuelta,
no son contiguos. Debido al desplazamiento en cada fase de 25
bits a la izquierda, se hace muy difcil el ataque a la clave.
K
1
: 001 002 003 004 005 006 007 008 009 010 011 012 013 014 015 016
K
7
: 097 098 099 100 101 102 103 104 105 106 107 108 109 110 111 112
K
13
: 090 091 092 093 094 095 096 097 098 099 100 101 102 103 104 105
K
19
: 083 084 085 086 087 088 089 090 091 092 093 094 095 096 097 098
K
25
: 076 077 078 079 080 081 082 083 084 085 086 087 088 089 090 091
K
31
: 044 045 046 047 048 049 050 051 052 053 054 055 056 057 058 059
k
37
: 037 038 039 040 041 042 043 044 045 046 047 048 049 050 051 052
k
43
: 030 031 032 033 034 035 036 037 038 039 040 041 042 043 044 045
Jorge Rami Aguirre Madrid (Espaa) 2003
Seguridad Informtica y Criptografa. Tema 10: Cifrado Simtrico en Bloque
Diapositiva 495
Ejemplo de clculo de claves en IDEA
Si la clave es IDEA es la clave, encuentre los 16 bits de la
segunda clave de la cuarta vuelta.
Solucin:
01 02 03 04 05 06 07 08 09 10 11 12 13 14 15 16 17 18 19 20 21 22 23 24 25 26 27 28 29 30
0 1 0 0 1 0 0 1 0 1 0 0 0 1 0 0 0 1 0 0 0 1 0 1 0 1 0 0 0 0
31 32 33 34 35 36 37 38 39 40 41 42 43 44 45 46 47 48 49 50 51 52 53 54 55 56 57 58 59 60
0 1 0 0 1 0 0 0 0 0 0 1 1 0 0 1 0 1 0 1 1 1 0 0 1 1 0 0 1 0
61 62 63 64 65 66 67 68 69 70 71 72 73 74 75 76 77 78 79 80 81 82 83 84 85 86 87 88 89 90
0 0 0 0 0 1 1 0 1 1 0 0 0 1 1 0 0 0 0 1 0 0 1 0 0 0 0 0 0 1
91 92 93 94 95 96 97 98 99 100 101 102 103 104 105 106 107 108 109 110 111 112 113 114
1 0 0 0 1 1 0 1 1 0 1 1 0 0 0 1 1 0 0 0 0 1 0 1
115 116 117 118 119 120 121 122 123 124 125 126 127 128
1 1 0 1 1 0 0 1 1 0 0 1 0 1
Como en cada vuelta se usan 6 subclaves, la segunda clave de la
cuarta vuelta ser la nmero 36+2 = 20. Como la clave 19
termina en el bit 98, la clave 20 sern los 16 bits siguientes, es
decir del 99 al 114: k
20
= 10110001 10000101.
Jorge Rami Aguirre Madrid (Espaa) 2003
Seguridad Informtica y Criptografa. Tema 10: Cifrado Simtrico en Bloque
Diapositiva 496
Descifrado con IDEA
El algoritmo IDEA, al igual que el DES, permite cifrar y
descifrar con la misma estructura. Como las operaciones se
hacen dentro de un cuerpo finito, en este caso las claves se
toman como los inversos de las operaciones XOR, suma
mod 2
16
y producto mod 2
16
+1, dependiendo de las
operaciones realizadas en la fase de cifrado.
INVERSOS
Inverso XOR: se aplica la misma funcin
Inverso aditivo: suma mod 2
16
Inverso multiplicativo: producto mod 2
16
+1
Jorge Rami Aguirre Madrid (Espaa) 2003
Seguridad Informtica y Criptografa. Tema 10: Cifrado Simtrico en Bloque
Diapositiva 497
Claves de descifrado en IDEA

d
1
= k
49
-1
d
2
= -k
50
d
3
= -k
51
d
4
= k
52
-1
d
5
= k
47
d
6
= k
48

d
7
= k
43
-1
d
8
= -k
45
d
9
= -k
44
d
10
= k
46
-1
d
11
= k
41
d
12
= k
42

d
13
= k
37
-1
d
14
= -k
39
d
15
= -k
38
d
16
= k
40
-1
d
17
= k
35
d
18
= k
36

d
19
= k
31
-1
d
20
= -k
33
d
21
= -k
32
d
22
= k
34
-1
d
23
= k
29
d
24
= k
30

d
25
= k
25
-1
d
26
= -k
27
d
27
= -k
26
d
28
= k
28
-1
d
29
= k
23
d
30
= k
24

d
31
= k
19
-1
d
32
= -k
21
d
33
= -k
20
d
34
= k
22
-1
d
35
= k
17
d
36
= k
18

d
37
= k
13
-1
d
38
= -k
15
d
39
= -k
14
d
40
= k
16
-1
d
41
= k
11
d
42
= k
12

d
43
= k
7
-1
d
44
= -k
9
d
45
= -k
8
d
46
= k
10
-1
d
47
= k
5
d
48
= k
6

d
49
= k
1
-1
d
50
= -k
2
d
51
= -k
3
d
52
= k
4
-1


Inversos del producto
Inversos de la suma
Inversos del XOR
Jorge Rami Aguirre Madrid (Espaa) 2003
Seguridad Informtica y Criptografa. Tema 10: Cifrado Simtrico en Bloque
Diapositiva 498
Operacin de descifrado con IDEA
Mdulo IDEA
Para descifrar, cada
bloque de criptograma
se dividir en cuatro
subbloques de 16 bits
Las operaciones
se hacen ahora
hacia arriba
Jorge Rami Aguirre Madrid (Espaa) 2003
Seguridad Informtica y Criptografa. Tema 10: Cifrado Simtrico en Bloque
Diapositiva 499
Uso de claves inversas en descifrado IDEA
d
1
= k
49
-1
d
2
= -k
50
d
3
= -k
51
d
4
= k
52
-1
d
5
= k
47
d
6
= k
48
d
7
= k
43
-1
d
8
= -k
45
d
9
= -k
44
d
10
= k
46
-1
d
11
= k
41
d
12
= k
42
d
13
= k
37
-1
d
14
= -k
39
d
15
= -k
38
d
16
= k
40
-1
d
17
= k
35
d
18
= k
36
d
19
= k
31
-1
d
20
= -k
33
d
21
= -k
32
d
22
= k
34
-1
d
23
= k
29
d
24
= k
30
d
25
= k
25
-1
d
26
= -k
27
d
27
= -k
26
d
28
= k
28
-1
d
29
= k
23
d
30
= k
24
d
31
= k
19
-1
d
32
= -k
21
d
33
= -k
20
d
34
= k
22
-1
d
35
= k
17
d
36
= k
18
d
37
= k
13
-1
d
38
= -k
15
d
39
= -k
14
d
40
= k
16
-1
d
41
= k
11
d
42
= k
12
d
43
= k
7
-1
d
44
= -k
9
d
45
= -k
8
d
46
= k
10
-1
d
47
= k
5
d
48
= k
6
d
49
= k
1
-1
d
50
= -k
2
d
51
= -k
3
d
52
= k
4
-1
Ultimas 6 claves
de descifrado
Jorge Rami Aguirre Madrid (Espaa) 2003
Seguridad Informtica y Criptografa. Tema 10: Cifrado Simtrico en Bloque
Diapositiva 500
Fortaleza del algoritmo IDEA
IDEA se muestra inmune ante un criptoanlisis diferencial. Sus
autores conocan esta debilidad del DES y lo hicieron resistente.
Joan Daemen descubre en 1992 una clase de claves dbiles. La
siguiente clave k = 0000,0000,0x00,0000,0000,000x,xxxx,x000
en hexadecimal es dbil, en el sentido de que un criptoanalista
podra identificarla en un ataque con texto en claro elegido. Las
posiciones x pueden ser cualquier nmero en hexadecimal.
La probabilidad de que se use este tipo de claves es slo de uno
en 2
96
y se puede, adems, eliminar por diseo.
A la fecha, ao 2003, no se conoce todava ningn sistema o
algoritmo de ataque que haya criptoanalizado el IDEA.
Joan Daemen y Vincent Rijmen crearn en 1997 el RIJNDAEL,
nuevo estndar mundial del NIST desde finales de 2001.
Jorge Rami Aguirre Madrid (Espaa) 2003
Seguridad Informtica y Criptografa. Tema 10: Cifrado Simtrico en Bloque
Diapositiva 501
Algoritmo RC2
Cifrador en bloque de clave variable propuesto por Ron Rivest.
El cdigo es secreto industrial de RSA Data Security Inc.
Tamao del bloque de texto: 64 bits.
Con una clave con tamao variable (de 8 a 1.024 bits) forma una
tabla de 128 bytes (1.024 bits) que depende de la clave inicial.
No usa cajas S y es casi tres veces ms rpido que DES.
Se usa en SMIME con longitudes de clave de 40, 64 y 128 bits.
Los algoritmos RC2 y RC4 se incluyen en productos para la
exportacin, como navegadores, limitando la clave a 40 bits.
Operaciones primitivas de cifra: suma en mdulo 2
32
, operacin
or exclusivo, complemento de bits, operacin AND y rotacin
circular a la izquierda.
Realiza 18 vueltas conocidas como mixing y mashing.
Jorge Rami Aguirre Madrid (Espaa) 2003
Seguridad Informtica y Criptografa. Tema 10: Cifrado Simtrico en Bloque
Diapositiva 502
Algoritmo RC5
RC5 es un cifrador en bloque de tamao variable de Ron Rivest.
Cifra bloques de texto de 32, 64 128 bits.
Tamao de clave hasta 2.048 bits, en funcin nmero de vueltas.
Nmero de vueltas de 0 a 255.
Versiones especficas: RC5 w/r/b donde w es el tamao de la
palabra (16, 32 64 bits) -RC5 cifra bloques de dos palabras-, r
es el nmero de vueltas y b es el tamao en octetos de la clave K.
El valor propuesto por Rivest como mnimo es RC5 32/12/16.
Rutina expansin de clave: se expande K para llenar una tabla.
Rutinas de cifrado y descifrado: usa primitivas de suma mdulo
2
w
, or exclusivo y rotacin circular a la izquierda.
Caractersticas: muy rpido, arquitectura simple, bajos requisitos
de memoria y alta seguridad. Las rotaciones dependientes de los
datos le fortalecen ante el criptoanlisis diferencial.
Jorge Rami Aguirre Madrid (Espaa) 2003
Seguridad Informtica y Criptografa. Tema 10: Cifrado Simtrico en Bloque
Diapositiva 503
Algoritmos SAFER 64 y 128
SAFER: Secure and Fast Encryption Routine (James Massey).
Cifra bloques de texto de 64 bits. Cada bloque a cifrar de texto se
divide en 8 bytes.
Tamao de clave: 64 128 bits.
Nmero de vueltas de 0 a 10; mnimo recomendable 6.
Operaciones de cifrado y descifrado distintas basadas en bytes,
que orientan su uso en aplicaciones de tarjetas inteligentes.
En cada vuelta hay operaciones or y sumas normales, potencias y
logaritmos discretos en p = 257, usando 45 como raz primitiva.
Al final del algoritmo hay tres niveles de operaciones lineales
conocidas como Pseudo Transformaciones de Hadamard, PTH,
cuyo objetivo es aumentar la difusin de los bits.
Existen versiones SAFER SK-64 y SK-128 ms seguras ante
claves dbiles que sus antecesoras.
Jorge Rami Aguirre Madrid (Espaa) 2003
Seguridad Informtica y Criptografa. Tema 10: Cifrado Simtrico en Bloque
Diapositiva 504
Algoritmo Blowfish
Cifrador tipo Feistel de clave variable (Bruce Schneier).
Cifra bloques de texto de 64 bits.
Tamao de clave: de 32 hasta 448 bits. Se generan 18 subclaves
de 32 bits y cuatro cajas S de 8x32 bits, en total 4.168 bytes.
Nmero de vueltas: 16, en cada una de ellas se realiza una
permutacin funcin de la clave y una sustitucin que es funcin
de la clave y los datos.
Operaciones bsicas: or exclusivo y suma mdulo 2
32
.
Cajas S: en cada vuelta hay cuatro con 256 entradas cada una.
Caractersticas: compacto porque necesita slo 5 K de memoria,
es muy rpido (5 veces ms veloz que DES), es conceptualmente
simple y su fortaleza puede variarse segn longitud de la clave.
Usa una funcin F con las cuatro cajas S y operaciones bsicas
de suma y or exclusivo que provocan un efecto de avalancha.
Jorge Rami Aguirre Madrid (Espaa) 2003
Seguridad Informtica y Criptografa. Tema 10: Cifrado Simtrico en Bloque
Diapositiva 505
Algoritmo CAST 128
Cifrador Feistel propuesto por C. Adams y S. Tavares (Canad).
Cifra bloques de texto de 64 bits con claves de 40 hasta 128 bits
en incrementos de octetos.
Cifra en 16 vueltas.
Usa ocho cajas S de 8 bits de entrada y 32 bits de salida con unas
funciones no lineales ptimas (funciones bent), cuatro cajas en
procesos de cifra y las otras cuatro para la generacin de claves.
Cada caja es un array de 32 columnas y 256 filas. Los 8 bits de
entrada seleccionan una fila y los 32 bits de sta es la salida.
Operaciones bsicas: suma y resta mdulo 2
32
, or exclusivo y
rotaciones circulares hacia la izquierda.
Caractersticas: inmune a ataques por criptoanlisis diferencial y
lineal; algoritmo estndar de cifra en ltimas versiones de PGP.
Jorge Rami Aguirre Madrid (Espaa) 2003
Seguridad Informtica y Criptografa. Tema 10: Cifrado Simtrico en Bloque
Diapositiva 506
Algoritmo Skipjack
Ha sido desarrollado por la NSA, National Security Agency, est
contenido en los chip Clipper y Capstone y su implementacin
slo est permitida en hardware.
Cifra bloques de 64 bits con una clave de 80 bits.
Los usuarios depositan sus claves secretas en diversas agencias
de gobierno.
Usa 32 vueltas en cada bloque de cifra.
Los detalles del algoritmo no son pblicos.
Caractersticas: imposicin de los EEUU para comunicaciones
con la administracin, tiene una puerta trasera que puede dejar
en claro la cifra, nadie puede asegurar que el algoritmo tenga la
suficiente fortaleza pero los Estados Unidos piensa usarlo en su
DMS, Defense Messaging System. Ha sido duramente criticado.
Jorge Rami Aguirre Madrid (Espaa) 2003
Seguridad Informtica y Criptografa. Tema 10: Cifrado Simtrico en Bloque
Diapositiva 507
El DES deja de ser un estndar
: El DES se adopta como estndar en 1976.
: El NIST certifica al DES en 1987 y luego en 1993.
: Durante esos aos se estandariza como algoritmo de cifra
en todo el mundo. Su uso principal lo encontramos en el
cifrado de la informacin intercambiada en transacciones
de dinero entre un cajero automtico y el banco respectivo.
: En 1997 NIST no certifica al DES y llama a un concurso
internacional para buscar un nuevo estndar mundial de
cifra denominado AES Advanced Encryption Standard.
: Precisamente entre 1997 y 1999 el DES se enfrenta a tres
ataques o desafos conocidos como DES Challenge que
impulsa y promociona la compaa RSA.
Jorge Rami Aguirre Madrid (Espaa) 2003
Seguridad Informtica y Criptografa. Tema 10: Cifrado Simtrico en Bloque
Diapositiva 508
DES Challenge I y II
e 29 enero 1997: DES Challenge I. Se rompe la clave en 96
das con 80.000 de ordenadores en Internet que evalan
7.000 millones de clave por segundo. Para encontrar la
clave se debe recorrer el 25% del espacio de claves .
e 13 enero 1998: DES Challenge II-1. Se rompe la clave en
39 das con un ataque tipo distribuido por distributed.net
que llega a evaluar 34.000 millones de claves por segundo
y debe recorrer el 88% del espacio de claves .
e 13 julio de 1998: DES Challenge II-2. Electronic Frontier
Foundation EFF crea el DES Cracker con una inversin
de US $ 200.000 y en 56 horas (2 das) rompe la clave
evaluando 90.000 millones de claves por segundo.
Jorge Rami Aguirre Madrid (Espaa) 2003
Seguridad Informtica y Criptografa. Tema 10: Cifrado Simtrico en Bloque
Diapositiva 509
DES Challenge III
e 18 enero 1999: DES Challenge III. Se unen la mquina
DES Cracker y distributed.net con 100.000 ordenadores
conectados en Internet para romper la clave en 22 horas,
menos de 1 da, evaluando 245.000 millones de claves
por segundo tras recorrer el 22% del espacio de claves.
e Se trata del ltimo desafo propuesto por RSA que pone
en evidencia la capacidad de ataque distribuido a travs
de los tiempos muertos de procesador de mquinas
conectadas a Internet que, con un programa cliente, van
resolviendo un pequeo trozo del espacio de claves,
comunicndose para ello con un servidor.
Jorge Rami Aguirre Madrid (Espaa) 2003
Seguridad Informtica y Criptografa. Tema 10: Cifrado Simtrico en Bloque
Diapositiva 510
El nuevo estndar en cifra AES
AES: Advanced Encryption Standard
El DES, estndar desde 1976, pasa la certificacin de la
NBS National Bureaux of Standards en 1987 y en 1993.
En 1997 el NIST National Institute of Standards and
Technology (antigua NBS) no certifica al DES y llama a
concurso pblico para un nuevo estndar: el AES.
En octubre del ao 2000 el NIST elige el algoritmo belga
RIJNDAEL como nuevo estndar para algoritmo de cifra
del siglo XXI. Es software de libre distribucin y est
disponible desde finales del ao 2001.
Jorge Rami Aguirre Madrid (Espaa) 2003
Seguridad Informtica y Criptografa. Tema 10: Cifrado Simtrico en Bloque
Diapositiva 511
Caractersticas del algoritmo RIJNDAEL
RIJNDAEL: autores Vincent Rijmen y Joan Daemen
No es de tipo Feistel.
Implementado para trabajar en los procesadores de 8 bits
usados en tarjetas inteligentes y en CPUs de 32 bits.
Tamao de clave variable: 128, 192 y 256 bits (estndar) o
bien mltiplo de 4 bytes.
Tamao del bloque de texto: 128 bits o mltiplo de 4 bytes.
Operaciones modulares a nivel de byte (representacin en
forma de polinomios) y de palabra de 4 bytes: 32 bits.
Nmero de etapas flexible segn necesidades del usuario.
Usa un conjunto de Cajas S similar al DES.
Jorge Rami Aguirre Madrid (Espaa) 2003
Seguridad Informtica y Criptografa. Tema 10: Cifrado Simtrico en Bloque
Diapositiva 512
Transformaciones o capas de RIJNDAEL
Hay tres transformaciones distintas llamadas capas en las
que se tratan los bits. Estas constan de:
Capa de Mezcla Lineal: en ella se busca la difusin de los
bits.
Capa No Lineal: se trata de una zona similar a las cajas S del
DES.
Capa Clave: operaciones con una funcin Xor de la subclave
y la informacin de esta etapa intermedia.
Las transformaciones realizadas en cada paso del algoritmo
se denominan estados y se representa por un array de 4
filas.
Puede ver un esquema detallado del algoritmo en la pgina Web
de su autor http://www.esat.kuleuven.ac.be/~rijmen/rijndael/ y
en http://home.ecn.ab.ca/~jsavard/crypto/co040401.htm.
Jorge Rami Aguirre Madrid (Espaa) 2003
Seguridad Informtica y Criptografa. Tema 10: Cifrado Simtrico en Bloque
Diapositiva 513
Operaciones con bytes en RIJNDAEL
Operaciones a nivel de byte en GF(2
8
)
Suma y multiplicacin. Son clculos en Campos de Galois
GF(2
8
) con 8 bits. Para la reduccin de exponente se usar
un polinomio primitivo p(x).
Producto por x. Esta operacin conocida como xtime(a) al
igual que en el caso anterior usa la reduccin de exponente.
Puede implementarse fcilmente con desplazamientos y
operaciones or exclusivo.
Ejemplos
Jorge Rami Aguirre Madrid (Espaa) 2003
Seguridad Informtica y Criptografa. Tema 10: Cifrado Simtrico en Bloque
Diapositiva 514
Ejemplo de suma en GF(2
8
)
Vamos a sumar los valores hexadecimales 57 y 83:
A = 57
16
= 0101 0111
2
B = 83
16
= 1000 0011
2
que expresados en polinomios dentro de GF(2
8
) sern:
A = 0101 0111
2
= x
6
+ x
4
+ x
2
+ x + 1
B = 1000 0011
2
= x
7
+ x + 1
Sumando: A+B = (x
6
+ x
4
+ x
2
+ x + 1) + (x
7
+ x + 1) mod 2
A+B = (x
7
+ x
6
+ x
4
+ x
2
+ 2x + 2) mod 2
A+B = x
7
+ x
6
+ x
4
+ x
2
= 1101 0100 = D4
16
Y lo mismo se obtiene con la suma Or exclusivo:
0101 0111 1000 0011 = 1101 0100 = D4
16
Jorge Rami Aguirre Madrid (Espaa) 2003
Seguridad Informtica y Criptografa. Tema 10: Cifrado Simtrico en Bloque
Diapositiva 515
Ejemplo de producto en GF(2
8
) (1)
Vamos a multiplicar los valores hexadecimales 57 y 83:
A = 57
16
= 0101 0111
2
B = 83
16
= 1000 0011
2
que expresados en polinomios dentro de GF(2
8
) sern:
A = 0101 0111
2
= x
6
+ x
4
+ x
2
+ x + 1
B = 1000 0011
2
= x
7
+ x + 1
Sea p(x) = x
8
+ x
4
+ x
3
+ x + 1 x
8
= x
4
+ x
3
+ x + 1
AB = (x
6
+ x
4
+ x
2
+ x + 1)(x
7
+ x + 1) mod 2
AB = x
13
+ x
11
+ x
9
+ x
8
+ 2x
7
+ x
6
+ x
5
+ x
4
+ x
3
+ 2x
2
+ 2x +1
AB = x
13
+ x
11
+ x
9
+ x
8
+ x
6
+ x
5
+ x
4
+ x
3
+ 1
Este resultado hay que reducirlo por p(x) = x
8
+ x
4
+ x
3
+ x + 1
Jorge Rami Aguirre Madrid (Espaa) 2003
Seguridad Informtica y Criptografa. Tema 10: Cifrado Simtrico en Bloque
Diapositiva 516
Ejemplo de producto en GF(2
8
) (2)
p(x): x
8
= x
4
+ x
3
+ x + 1
Estn fuera del cuerpo de 8 bits
AB = x
13
+ x
11
+ x
9
+ x
8
+ x
6
+ x
5
+ x
4
+ x
3
+ 1
x
13
= x
5
x
8
= x
5
(x
4
+ x
3
+ x + 1) = x
9
+ x
8
+ x
6
+ x
5
x
13
= x (x
4
+ x
3
+ x + 1) + (x
4
+ x
3
+ x + 1) + x
6
+ x
5
x
13
= (x
5
+ x
4
+ x
2
+ x) + (x
4
+ x
3
+ x + 1) + x
6
+ x
5
x
13
= x
6
+ x
3
+ x
2
+ 1
x
13
= x
6
+ x
3
+ x
2
+ 1
Jorge Rami Aguirre Madrid (Espaa) 2003
Seguridad Informtica y Criptografa. Tema 10: Cifrado Simtrico en Bloque
Diapositiva 517
Ejemplo de producto en GF(2
8
) (3)
p(x): x
8
= x
4
+ x
3
+ x + 1
Estn fuera del cuerpo de 8 bits
AB = x
13
+ x
11
+ x
9
+ x
8
+ x
6
+ x
5
+ x
4
+ x
3
+ 1
x
11
= x
3
x
8
= x
3
(x
4
+ x
3
+ x + 1)
x
11
= x
7
+ x
6
+ x
4
+ x
3
x
11
= x
7
+ x
6
+ x
4
+ x
3
x
9
= xx
8
= x(x
4
+ x
3
+ x + 1)
x
9
= x
5
+ x
4
+ x
2
+ x
x
9
= x
5
+ x
4
+ x
2
+ x
x
8
= x
4
+ x
3
+ x + 1
Jorge Rami Aguirre Madrid (Espaa) 2003
Seguridad Informtica y Criptografa. Tema 10: Cifrado Simtrico en Bloque
Diapositiva 518
Ejemplo de producto en GF(2
8
) (4)
x
13
= x
6
+ x
3
+ x
2
+ 1 x
11
= x
7
+ x
6
+ x
4
+ x
3
x
9
= x
5
+ x
4
+ x
2
+ x x
8
= x
4
+ x
3
+ x + 1
AB = x
13
+ x
11
+ x
9
+ x
8
+ x
6
+ x
5
+ x
4
+ x
3
+ 1
Reemplazando los clculo anteriores en la expresin:
AB = (x
6
+ x
3
+ x
2
+ 1) + (x
7
+ x
6
+ x
4
+ x
3
) + (x
5
+ x
4
+ x
2
+ x) +
+ (x
4
+ x
3
+ x + 1) + x
6
+ x
5
+ x
4
+ x
3
+ 1 mod 2
AB = x
7
+ x
6
+ 1 = 1100 0001 = C1
16
Fin del Tema 10
Jorge Rami Aguirre Madrid (Espaa) 2003
Seguridad Informtica y Criptografa. Tema 10: Cifrado Simtrico en Bloque
Diapositiva 519
Cuestiones y ejercicios (1 de 3)
1. Qu particularidad tiene el cifrado tipo Feistel?
2. Qu importante diferencia tiene el algoritmo Skipjack con respecto
a todos los dems? Razone si eso es bueno o malo en criptografa.
3. Cul es la razn principal de la debilidad del algoritmo DES?
4. Con cul de las dos versiones respecto a la reduccin de clave
aplicada al DES por la NSA se queda Ud.? Razone las dos
respuestas posibles.
5. Qu tamao de bloque de mensaje cifra el DES, con qu longitud
de clave y con cuntas vueltas?
6. Tiene algn inters criptogrfico la tabla de permutacin inicial IP
que se repite en sentido contrario al final en el DES? Por qu?
7. Qu distribucin especial observa en los dos bloques de texto a
cifrar L
0
y R
0
en DES? Qu separacin en bits hay entre ellos?
Jorge Rami Aguirre Madrid (Espaa) 2003
Seguridad Informtica y Criptografa. Tema 10: Cifrado Simtrico en Bloque
Diapositiva 520
Cuestiones y ejercicios (2 de 3)
8. Cmo se las arregla DES para realizar operaciones suma mdulo
dos con sub-bloques de texto de 32 bits y sub-claves de 56 bits?
9. Qu dos importantes funciones cumplen las cajas S en el DES?
10. En la caja S
3
del DES entra la secuencia de bits 101101, qu sale?
11. Si la clave DES en ASCII (no nmeros) es HOLAPACO, cules
sern la primera y segunda sub-claves de cifrado?
12. Por qu no debe usarse nunca el modo de cifra ECB?
13. Podemos usar el DES como un generador de secuencia cifrante?
14. Por qu decimos que el DES no es un grupo? Qu significa eso?
15. En qu consiste un ataque por encuentro a medio camino?
16. Por qu se usa en el triple DES un cifrado con slo dos claves y no
con tres como su nombre indica?
Jorge Rami Aguirre Madrid (Espaa) 2003
Seguridad Informtica y Criptografa. Tema 10: Cifrado Simtrico en Bloque
Diapositiva 521
Cuestiones y ejercicios (3 de 3)
17. Por qu en IDEA se usa una palabra de 16 bits y no de 32 bits? Se
podra usar una palabra de 8 bits 24 bits? Justifique su respuesta.
18. Encuentre los resultados de las tres operaciones bsicas para un
sistema simulado IDEA que trabaja con 4 bits.
19. Qu tamao de bloque cifra IDEA, con qu longitud de clave y con
cuntas vueltas?
20. Cmo se generan las sub-claves en IDEA?
21. Cules son las claves Z
9
y Z
10
en un sistema IDEA en el que la
clave maestra en ASCII es K = UnaClaveDePrueba.
22. Encuentre las claves de descifrado de las siguientes claves de cifra
en IDEA: k
12
= 3.256; k
13
= 34.517; k
14
= 45.592.
23. Sume y multiplique 31 y 18 en GF(2
8
) segn algoritmo RIJNDAEL.
Tema 11
Cifrado Asimtrico con Mochilas
Seguridad Informtica y Criptografa
Ultima actualizacin: 03/03/03
Archivo con 28 diapositivas
Jorge Rami Aguirre
Universidad Politcnica de Madrid
Material Docente de
Libre Distribucin
v 3.1
Este archivo forma parte de un curso sobre Seguridad Informtica y Criptografa. Se autoriza la
reproduccin en computador e impresin en papel slo con fines docentes o personales, respetando
en todo caso los crditos del autor. Queda prohibida su venta, excepto a travs del Departamento de
Publicaciones de la Escuela Universitaria de Informtica, Universidad Politcnica de Madrid, Espaa.
Curso de Seguridad Informtica y Criptografa JRA
Jorge Rami Aguirre Madrid (Espaa) 2003
Seguridad Informtica y Criptografa. Tema 11: Cifrado Asimtrico con Mochilas
Diapositiva 524
El problema de la mochila
El problema matemtico de la mochila
referido ahora a nmeros y no elementos
que entran en ella puede plantearse como
sigue:
Dada la siguiente secuencia de m nmeros enteros
positivos S = {S
1
, S
2
, S
3
, ..., S
m-2
, S
m-1
, S
m
} y un valor
u objetivo T, se pide encontrar un subconjunto de S
S
S
= {S
a
, S
b
, ..., S
j
} que cumpla con el objetivo:
T = S
S
= S
a
+ S
b
+ ... + S
j
Jorge Rami Aguirre Madrid (Espaa) 2003
Seguridad Informtica y Criptografa. Tema 11: Cifrado Asimtrico con Mochilas
Diapositiva 525
Solucin al problema de la mochila
Si los elementos de la mochila son nmeros grandes, no
estn ordenados y no siguen una distribucin supercreciente
-en este tipo de distribucin el elemento isimo S
i
de la
mochila es mayor que la suma de todos sus antecesores-, la
resolucin de este problema es de tipo no polinomial.
Se trata de encontrar los vectores V
i
de 0s y 1s de
forma que:
S
i
V
i
= T
i
Si se cumple esta relacin, la mochila tiene solucin.
En caso contrario, no existir solucin.
Jorge Rami Aguirre Madrid (Espaa) 2003
Seguridad Informtica y Criptografa. Tema 11: Cifrado Asimtrico con Mochilas
Diapositiva 526
Un ejemplo del problema de la mochila
Tenemos la mochila S = {20, 5, 7, 36, 13, 2} con m = 6 y el
valor T = 35. Se pide encontrar una solucin, si es que sta
existe, en una nica vuelta. En este momento no importa
que los valores de la mochila no estn ordenados.
SOLUCIN: Sin hacer ningn clculo mental, podemos
recorrer todos los valores (se puede descartar el elemento S
4
pues es mayor que el objetivo T) de la mochila S, bien de
izquierda a derecha o al revs (da igual el sentido elegido) y
restaremos el elemento isimo si es menor que el objetivo T
en esa etapa del algoritmo, como se indica:
Jorge Rami Aguirre Madrid (Espaa) 2003
Seguridad Informtica y Criptografa. Tema 11: Cifrado Asimtrico con Mochilas
Diapositiva 527
Solucin al ejemplo de la mochila
S = {S
1
, S
2
, S
3
, S
4
, S
5
, S
6
} = {20, 5, 7, 36, 13, 2} T = 35
S
1
= 20 Es menor que objetivo T = 35? S T = 35-20 = 15
S
2
= 5 Es menor que objetivo T = 15? S T = 15-5 = 10
S
3
= 7 Es menor que objetivo T = 10? S T = 10-7 = 3
S
4
= 36 Es menor que objetivo T = 3? No T = 3
S
5
= 13 Es menor que objetivo T = 3? No T = 3
S
6
= 2 Es menor que objetivo T = 3? S T = 3-2 = 1 0
Se ha recorrido toda la mochila y no se ha encontrado solucin.
En cambio s existe una solucin:
S
S
= {S
1
+S
5
+S
6
} = 20+13+2 = 35
V
i
= [1,0,0,0,1,1]
Jorge Rami Aguirre Madrid (Espaa) 2003
Seguridad Informtica y Criptografa. Tema 11: Cifrado Asimtrico con Mochilas
Diapositiva 528
Puede haber soluciones mltiples?
Si para la misma mochila S = {20, 5, 7, 36, 13, 2} buscamos
ahora el valor T = 27, encontramos tres soluciones vlidas:
S
S1
= {S
1
+S
3
} = 20+7 S
S2
= {S
1
+S
2
+S
6
} = 20+5+2
S
S3
= {S
2
+S
3
+S
5
+S
6
} = 5+7+13+2
Esto sera inadmisible en un sistema de cifra puesto que el
resultado de una operacin de descifrado debe ser nica ya
que proviene de un nico mensaje. La solucin ser el uso
de las denominadas mochilas simples en que la solucin al
problema de la mochila, si existe, es nica.

Jorge Rami Aguirre Madrid (Espaa) 2003
Seguridad Informtica y Criptografa. Tema 11: Cifrado Asimtrico con Mochilas
Diapositiva 529
Mochila simple o supercreciente
Una mochila es simple o
supercreciente si el elemento S
k
es mayor que la suma de los
elementos que le anteceden:
k-1
S
k
> S
j
j = 1
k-1
S
k
> S
j
j = 1
Por ejemplo, la mochila S = {2, 3, 7, 13, 28, 55, 110, 221}
con m = 8 elementos es supercreciente y la solucin para un
objetivo T = 148 es nica: V
i
= [S
2
+S
3
+S
5
+S
7
].
Para resolver cualquier valor T vlido para esta mochila,
sta se recorre de derecha a izquierda una sola vez con el
algoritmo ya visto.
Compruebe que para T = 289, 196 y 353 los vectores son
V
1
= 00010101; V
2
= 01001110; V
3
= 10110011.
Jorge Rami Aguirre Madrid (Espaa) 2003
Seguridad Informtica y Criptografa. Tema 11: Cifrado Asimtrico con Mochilas
Diapositiva 530
Operacin de cifra con mochila simple
Se representa la informacin en binario y se pasan los bits
por la mochila. Los bits 1s incluyen en la suma el elemento
al que apuntan y los bits 0s no.
Con la mochila S = {2, 4, 10, 19, 40} de m = 5 elementos
cifraremos el mensaje M = ADIOS.
SOLUCIN: Usando cdigo ASCII/ANSI: A = 01000001;
D = 01000100; I = 01001001; O = 01001111; S = 01010011
M = 01000 00101 00010 00100 10010 10011 11010 10011
C = (4), (10+40), (19), (10), (2+19), (2+19+40), (2+4+19), (2+19+40)
C = 4, 50, 19, 10, 21, 61, 25, 61
Jorge Rami Aguirre Madrid (Espaa) 2003
Seguridad Informtica y Criptografa. Tema 11: Cifrado Asimtrico con Mochilas
Diapositiva 531
Descifrado con mochila simple
C = 4, 50, 19, 10, 21, 61, 25, 61 S = {2, 4, 10, 19, 40}
La operacin de descifrado es elemental: pasamos por la
mochila los valores de C, encontramos el vector V
i
y por
ltimo agrupamos el resultado en grupos de 8 bits. En este
caso 4 V
i
= 01000, 50 V
i
= 00101, etc.
PROBLEMA: Es muy fcil cifrar y descifrar
pero tambin criptoanalizar el sistema de cifra
porque se usa una mochila simple.
Una posible solucin es usar mochilas de Merkle y Hellman.
Jorge Rami Aguirre Madrid (Espaa) 2003
Seguridad Informtica y Criptografa. Tema 11: Cifrado Asimtrico con Mochilas
Diapositiva 532
Mochila de Merkle y Hellman MH
En 1978 Ralph Merkle y Martin Hellman proponen un sistema
de cifra de clave pblica denominado Mochila con Trampa.
El algoritmo se basa en crear una mochila difcil a partir de una
mochila simple de forma que el cifrado se haga con la mochila
difcil y el descifrado con la mochila simple o fcil. Se puede
pasar fcilmente de la mochila simple a la difcil o viceversa
usando una trampa.
La trampa ser nuestra clave secreta.
La mochila difcil ser nuestra clave pblica.
Hay otros sistemas de cifra basados en mochilas algo ms
complejos pero, bsicamente, son iguales y no presentan una
fortaleza significativamente mayor. No tiene sentido su estudio.
Jorge Rami Aguirre Madrid (Espaa) 2003
Seguridad Informtica y Criptografa. Tema 11: Cifrado Asimtrico con Mochilas
Diapositiva 533
Diseo mochila de Merkle y Hellman (1)
1. Se selecciona una mochila supercreciente de m elementos
S = {S
1
, S
2
, ..., S
m
}.
2. Se elige un entero (mdulo de
trabajo) mayor que la suma de
los elementos de la mochila.
ms fcil:
2S
m

m
> S
i

i = 1
3. Se elige un entero primo
relativo con .
Se asegura
el inverso
mcd(,) = 1
Se recomienda que no tenga factores con los elementos de S
S
i
= S
i
mod
4. Se multiplica S por mod .
Obteniendo una mochila difcil S = {S
1
, S
2
, ..., S
m
}
Jorge Rami Aguirre Madrid (Espaa) 2003
Seguridad Informtica y Criptografa. Tema 11: Cifrado Asimtrico con Mochilas
Diapositiva 534
Diseo mochila de Merkle y Hellman (2)
5. Se calcula el inverso de en el cuerpo .
Clave privada: ,
-1
Clave privada: ,
-1

-1
= inv(,)
Esto se interpreta como
encontrar los vectores que
cumplan con un valor de T.
Clave pblica: mochila S
Clave pblica: mochila S
CIFRADO:
C = S M
como S = S mod
C = S M mod
DESCIFRADO:
M =
-1
C mod
Entonces obtenemos:
S M
Jorge Rami Aguirre Madrid (Espaa) 2003
Seguridad Informtica y Criptografa. Tema 11: Cifrado Asimtrico con Mochilas
Diapositiva 535
Cifrado mochila de Merkle y Hellman (1)
Vamos a cifrar el mensaje M = Sol usando la mochila simple y
supercreciente S = {3, 5, 12, 21}.
1. Eleccin de : 2S
4
221 = 45
2. Eleccin de : mcd(, ) = 1 = 32 (
-1
= 38)
3. Mochila S: S = S mod
S
1
= 32 3 mod 45 = 96 mod 45 = 6
S
2
= 32 5 mod 45 = 160 mod 45 = 25
S
3
= 32 12 mod 45 = 384 mod 45 = 24
S
4
= 32 21 mod 45 = 672 mod 45 = 42
Clave pblica: S = {6,25,24,42} Clave privada: =45,
-1
=38
Clave privada: =45, Clave pblica: S = {6,25,24,42}
-1
=38
Jorge Rami Aguirre Madrid (Espaa) 2003
Seguridad Informtica y Criptografa. Tema 11: Cifrado Asimtrico con Mochilas
Diapositiva 536
Cifrado mochila de Merkle y Hellman (2)
Clave pblica: S = {6,25,24,42} Clave privada: = 45,
-1
= 38
Clave privada: = 45,
-1
= 38 Clave pblica: S = {6,25,24,42}
Cifrado: M = Sol = 0101 0011 0110 1111 0110 1100
C = (25+42), (24+42), (25+24), (6+25+24+42), (25+24), (6+25)
C = 67, 66, 49, 97, 49, 31
Como m = 4, cifraremos bloques de 4 bits,
convirtiendo el mensaje a su equivalente
en binario del cdigo ASCII.
Jorge Rami Aguirre Madrid (Espaa) 2003
Seguridad Informtica y Criptografa. Tema 11: Cifrado Asimtrico con Mochilas
Diapositiva 537
Descifrado mochila de Merkle y Hellman
Clave pblica: S = {6,25,24,42} Clave privada: = 45,
-1
= 38
Clave privada: = 45,
-1
= 38 Clave pblica: S = {6,25,24,42}
Cifrado: M = Sol = 0101 0011 0110 1111 0110 1100
C = (25+42), (24+42), (25+24), (6+25+24+42), (25+24), (6+25)
C = 67, 66, 49, 97, 49, 31
Descifrado:
3867 mod 45=2546 mod 45 = 26 3897 mod 45=3686 mod 45 = 41
3866 mod 45=2508 mod 45 = 33 3849 mod 45=1862 mod 45 = 17
3849 mod 45=1862 mod 45 = 17 3831 mod 45=1178 mod 45 = 8
Como S = {3,5,12,21}
M = 0101 0011 0110 1111 0110 1100 = Sol
Jorge Rami Aguirre Madrid (Espaa) 2003
Seguridad Informtica y Criptografa. Tema 11: Cifrado Asimtrico con Mochilas
Diapositiva 538
Valores de diseo de mochilas M-H (1)
Merkle y Hellman proponen los siguientes parmetros:
a) Tamao de la mochila m 100
b) Mdulo uniforme en el siguiente intervalo:
Intervalo : [2
2m+1
+1, 2
2m+2
-1] 2m+2 bits
Si m = 100: todos los elementos de S son de 202 bits.
c) Valores de S
i
elegidos uniformemente en el intervalo:
Intervalo S
i
: [(2
i-1
-1)2
m
+1, 2
i-1
2
m
]
Si m = 100: 1 S
1
2
100
S
2
2
101
S
3
2
102
...
d) Elegir un valor x en el intervalo [2, -2]. El factor se
calcula como: = mcd (, x)
Jorge Rami Aguirre Madrid (Espaa) 2003
Seguridad Informtica y Criptografa. Tema 11: Cifrado Asimtrico con Mochilas
Diapositiva 539
Mochila con parmetros proporcionales (1)
a) Mochila con m = 6
b) Intervalo : [2
2m+1
+1, 2
2m+2
-1] = [2
26+1
+1, 2
26 +2
-1]
= [2
13
+1, 2
14
+1] = [8.193, 16.385] = 13.515
Todos los elementos sern de (2m+2) = 14 bits
c) Eleccin de los valores S
i
:
i=1 : [(2
1-1
-1)2
6
+1, (2
1-1
) 2
6
] 1 S
1
64
i=2 : [(2
2-1
-1)2
6
+1, (2
2-1
) 2
6
] 65 S
2
128
i=3 : [(2
3-1
-1)2
6
+1, (2
3-1
) 2
6
] 193 S
3
256
i=4 : [(2
4-1
-1)2
6
+1, (2
4-1
) 2
6
] 449 S
4
512
i=5 : [(2
5-1
-1)2
6
+1, (2
5-1
) 2
6
] 961 S
5
1.024
i=6 : [(2
6-1
-1)2
6
+1, (2
6-1
) 2
6
] 1.985 S
6
2.048
UNA ELECCIN
S
1
= 39
S
2
= 72
S
3
= 216
S
4
= 463
S
5
= 1.001
S
6
= 1.996
Jorge Rami Aguirre Madrid (Espaa) 2003
Seguridad Informtica y Criptografa. Tema 11: Cifrado Asimtrico con Mochilas
Diapositiva 540
Mochila con parmetros proporcionales (2)
d) Clculo del factor . Buscamos un valor x en el intervalo
[2, -2] = [2, 13.513], por ejemplo x = 9.805.
Como el mximo comn divisor entre = 13.515 y x = 9.805
es 265, luego = 9.805/265 = 37.
Vamos a elegir:
= 37 de forma que
-1
= 4.018 inv (37,13.515) = 4.018
Luego, la mochila simple y la clave privada sern:
S = {39, 72, 216, 463, 1.001, 1.996}
Mdulo: = 13.515
-1
= 4.018
Jorge Rami Aguirre Madrid (Espaa) 2003
Seguridad Informtica y Criptografa. Tema 11: Cifrado Asimtrico con Mochilas
Diapositiva 541
Mochila con parmetros proporcionales (3)
Mochila simple:
S = {39, 72, 216, 463, 1.001, 1.996} Mdulo: = 13.515
Factor multiplicador: = 37;
-1
= 4.018 Clave privada
S
1
= 3937 mod 13.515 = 1.443
S
2
= 7237 mod 13.515 = 2.664
S
3
= 21637 mod 13.515 = 7.992
S
4
= 46337 mod 13.515 = 3.616
S
5
= 1.00137 mod 13.515 = 10.007
S
6
= 1.99637 mod 13.515 = 6.277
Mochila difcil:
S = {1.443, 2.664, 7.992, 3.616, 10.007, 6.277} Clave pblica
Jorge Rami Aguirre Madrid (Espaa) 2003
Seguridad Informtica y Criptografa. Tema 11: Cifrado Asimtrico con Mochilas
Diapositiva 542
Fortaleza de las mochilas M-H
En el ao 1982 Adi Shamir y Richard Zippel encuentran
debilidades a las mochilas de Merkle-Hellman:
Si se conoce el mdulo (o puede deducirse) ...
Y si los dos primeros elementos (S
1
y S
2
) de la mochila
difcil se corresponden con los dos primeros elementos
(S
1
y S
2
) de la mochila simple y son primos con ...
Entonces podemos generar la mochila simple a partir de la
difcil ya que encontraremos la clave secreta
-1
...
Aunque como veremos existen fuertes restricciones, esta
debilidad no hace recomendable el uso de las mochilas de
M-H al menos para el cifrado de la informacin ...
Jorge Rami Aguirre Madrid (Espaa) 2003
Seguridad Informtica y Criptografa. Tema 11: Cifrado Asimtrico con Mochilas
Diapositiva 543
Criptoanlisis de Shamir y Zippel
El ataque para m = 100 supone:
a) Que los dos primeros elementos de S de 100 y 101 bits
son mucho ms pequeos que el mdulo de 202 bits.
b) Que podemos identificar los elementos S
1
y S
2
en la
mochila difcil y hacerlos corresponder con S
1
y S
2
.
c) Que conocemos el mdulo o podemos deducirlo.
Con estos datos se trata de encontrar los valores de S
1
y
S
2
adems del factor de multiplicacin .
Con estos valores podemos generar la mochila fcil S.
Jorge Rami Aguirre Madrid (Espaa) 2003
Seguridad Informtica y Criptografa. Tema 11: Cifrado Asimtrico con Mochilas
Diapositiva 544
Pasos del ataque de Shamir y Zippel (1)
1. Se calcula q = (S
1
/S
2
) mod
Como S
i
= S
i
mod entonces:
q = (S
1
/S
2
) mod = [S
1
inv(S
2
, )] mod
Esto implica una condicin: mcd (S
2
, ) = 1
2. Se calculan todos los mltiplos modulares del valor q con
multiplicadores en el rango [1, 2
m+1
] = [1, 2
101
]
CM = {1q mod , 2q mod , ..., 2
m+1
q mod }
3. El candidato para S
1
ser el valor ms pequeo de CM
puesto que ese elemento debe ser ms pequeo de la
mochila fcil S.
Jorge Rami Aguirre Madrid (Espaa) 2003
Seguridad Informtica y Criptografa. Tema 11: Cifrado Asimtrico con Mochilas
Diapositiva 545
Pasos del ataque de Shamir y Zippel (2)
4. Encontrado el candidato para S
1
se calcula:
= (S
1
/S
1
) mod = [S
1
inv(S
1
, )] mod
Esto implica una nueva condicin: mcd (S
1
, ) = 1
5. Conocido encontramos
-1
= inv(, ) y as calculamos
todos los elementos de la mochila S
i
= S
i

-1
mod que
debera ser del tipo supercreciente.
6. Si no se genera una mochila supercreciente, se elige el
siguiente valor ms pequeo del conjunto CM y as hasta
recorrer todos sus valores. Si con este conjunto CM no se
obtiene una mochila simple, se repite el punto 2 tomando
ahora valores en el rango 2
m+i
con i = 2, 3, etc. Por lo
general el ataque prospera con el primer conjunto CM.
Jorge Rami Aguirre Madrid (Espaa) 2003
Seguridad Informtica y Criptografa. Tema 11: Cifrado Asimtrico con Mochilas
Diapositiva 546
Ejemplo de ataque de Shamir y Zippel (1)
La clave pblica de un sistema de mochila Merkle-Hellman es:
S = {S
1
, S
2
, S
3
, S
4
, S
5
} = {3.241, 572, 2.163, 1.256, 3.531}
Si de alguna forma hemos conseguido conocer que el mdulo = 4.089,
se pide encontrar la mochila fcil S = {S
1
, S
2
, S
3
, S
4
, S
5
}.
Solucin:
q = S
1
/S
2
mod = S
1
inv (S
2
, ) mod . Calculamos ahora inv (S
2
, )
es decir inv (572, 4,089) = 309, luego q = 3.241309 mod 4.089 = 599.
Mltiplos CM = {1q mod , 2q mod , 3q mod , ..., 64q mod }
puesto que la mochila tiene m = 5 elementos y el intervalo ser [1, 2
5+1
].
Luego CM = [599, 1.198, 1.797, 2.396, 2.995, 3.594, 104, 703, 1.302,
1.901, 2.500, 3.099, 3.698, 208, 807, 1.406, 2.005, 2.604, 3.203, 3.802,
312, 911, 1.510, 2.109, 2.708, 3.307, 3.906, 416, 1.015, 1.614, 2.213,
2.812, 3.411, 4.010, 520, 1.119, 1.718, 2.317, 2.916, 3.515, 25, 624,
1.223, 1.822, 2.421, 3.020, 3.619, 129, 728, 1.327, 1.926, 2.525, 3.124,
3.723, 233, 832, 1.431, 2.030, 2.629, 3.228, 3.827, 337, 936, 1.535].
Jorge Rami Aguirre Madrid (Espaa) 2003
Seguridad Informtica y Criptografa. Tema 11: Cifrado Asimtrico con Mochilas
Diapositiva 547
Ejemplo de ataque de Shamir y Zippel (2)
Suponemos que el nmero ms pequeo de CM es candidato a S
1
= 25.
El factor de multiplicacin sera = (S
1
/S
1
) = S
1
inv (S
1
, ) mod .
Como inv (S
1
, ) = inv (25, 4,089) = 2.617, el factor de multiplicacin
= 3.2412.617 mod 4.089 = 1.111.
Por lo tanto su valor inverso ser
-1
= inv (, ) = inv (1.111, 4.089).
Luego
-1
= 622.
Multiplicamos ahora los valores S de la mochila difcil por
-1
a ver si
obtenemos una mochila supercreciente S (S
i
= S
i

-1
mod ):
S
1
= 25 (valor elegido como candidato del conjunto CM)
S
2
= S
2

-1
mod = 572 622 mod 4.089 = 41
S
3
= S
3

-1
mod = 2.163 622 mod 4.089 = 105
S
4
= S
4

-1
mod = 1.256 622 mod 4.089 = 233
S
5
= S
5

-1
mod = 3.531 622 mod 4.089 = 489
Como la mochila S = {25, 41, 105, 233, 489} es supercreciente, el
ataque ha prosperado y hemos encontrado la clave privada.
Jorge Rami Aguirre Madrid (Espaa) 2003
Seguridad Informtica y Criptografa. Tema 11: Cifrado Asimtrico con Mochilas
Diapositiva 548
Usos de proteccin con mochilas
Existen varios algoritmos propuestos como sistemas de cifra
usando el problema de la mochila: el de Graham-Shamir,
Chor-Rivest, etc.
No obstante todos han sucumbido a los criptoanlisis y en la
actualidad en el nico entorno que se usan es en la proteccin
de diversos programas de aplicacin, en forma de hardware
que se conecta en la salida paralela del computador para
descifrar el cdigo ejecutable de esa aplicacin dejando, sin
embargo, activa la salida a impresora. De esta manera slo en
aquel sistema con la mochila instalada se puede ejecutar el
programa. No se usa en comunicaciones.
Fin del Tema 11
Jorge Rami Aguirre Madrid (Espaa) 2003
Seguridad Informtica y Criptografa. Tema 11: Cifrado Asimtrico con Mochilas
Diapositiva 549
Cuestiones y ejercicios (1 de 2)
1. Recorra de izquierda a derecha y de derecha a izquierda la mochila
S = {13, 6, 1, 3, 4, 9, 10} para T = 24. Tiene solucin rpida?
2. Para la mochila de la pregunta anterior, hay una o ms soluciones?
3. Interesa usar en criptografa el problema de la mochila con una
solucin no nica? Por qu s o no?
4. Qu significa que una mochila sea supercreciente? Es la mochila
S = {3, 4, 9, 18, 32, 73} supercreciente? Por qu?
5. A partir de la mochila S = {3, 5, 10, 21, 43} obtenga la mochila
MH difcil S. Para y use los valores mnimos posibles.
6. Si la mochila fcil es S = {1, 2, 4, 8, 16, 32, 64, 128} con = 257 y
= 21, cifre con una mochila de MH el mensaje en ASCII de 10
caracteres M = Hola amigo (recuerde que el espacio se cifra).
7. Descifre el criptograma obtenido en la pregunta anterior.
Jorge Rami Aguirre Madrid (Espaa) 2003
Seguridad Informtica y Criptografa. Tema 11: Cifrado Asimtrico con Mochilas
Diapositiva 550
Cuestiones y ejercicios (2 de 2)
8. Qu valores mnimos de diseo propusieron Merkle y Hellman
para su sistema de cifrado con mochila? Por qu?
9. Disee una mochila de MH con parmetros proporcionales si m = 5.
10. No es un buen criterio elegir m = 4, m = 8 o m = 16. Por qu?
11. En qu consiste el ataque de Shamir y Zippel a la mochila de MH?
12. En el ejemplo de los apuntes, cuntas operaciones ha tenido que
hacer nuestro algoritmo para romper la clave privada?
13. Con el software de mochilas de la asignatura genere una mochila
difcil a partir de la mochila fcil S = {122, 250, 506, 1.018, 2.042,
4.090, 8.186}, con = 59.369 y = 59.361. Cifre algo con ella y
luego haga un ataque. Cuntas mochilas S encuentra? Comntelo.
14. Usara un sistema de mochila para cifrar informacin en un entorno
como Internet? Y en una intranet para respuestas a un examen?
Tema 12
Cifrado Asimtrico Exponencial
Seguridad Informtica y Criptografa
Ultima actualizacin: 03/03/03
Archivo con 56 diapositivas
Jorge Rami Aguirre
Universidad Politcnica de Madrid
Material Docente de
Libre Distribucin
v 3.1
Este archivo forma parte de un curso sobre Seguridad Informtica y Criptografa. Se autoriza la
reproduccin en computador e impresin en papel slo con fines docentes o personales, respetando
en todo caso los crditos del autor. Queda prohibida su venta, excepto a travs del Departamento de
Publicaciones de la Escuela Universitaria de Informtica, Universidad Politcnica de Madrid, Espaa.
Curso de Seguridad Informtica y Criptografa JRA
Jorge Rami Aguirre Madrid (Espaa) 2003
Seguridad Informtica y Criptografa. Tema 12: Cifrado Asimtrico Exponencial
Diapositiva 552
Cifrado exponencial con clave del receptor
En el cifrado del mensaje E
e
(M) = C y en el descifrado
del criptograma E
d
(C) = M, se usa una exponenciacin.
En la operacin de cifrado, el subndice e significa el uso
de la clave pblica del receptor (R) en el extremo emisor
y el subndice d el uso de la clave privada del receptor
(R) en el extremo receptor.
C = E
eR
(M) = M
eR
mod n
R
M = E
dR
(C) = C
dR
mod n
R
M debe ser un elemento del CCR de n
R
.
Esta operacin se usar para realizar el intercambio de
una clave de sesin entre un emisor y un receptor.
Jorge Rami Aguirre Madrid (Espaa) 2003
Seguridad Informtica y Criptografa. Tema 12: Cifrado Asimtrico Exponencial
Diapositiva 553
Cifrado exponencial con clave del emisor
En la operacin de cifrado el subndice d significa el uso
de la clave privada del emisor (E) en el extremo emisor y
el subndice e el uso de la clave pblica del emisor (E) en
el extremo receptor.
C = E
dE
(M) = M
dE
mod n
E
M = E
eE
(C) = C
eE
mod n
E
M debe ser un elemento del CCR de n
E
.
Esta operacin se usar para autenticar la identidad de un
usuario mediante una firma digital al mismo tiempo que
la integridad del mensaje.
Jorge Rami Aguirre Madrid (Espaa) 2003
Seguridad Informtica y Criptografa. Tema 12: Cifrado Asimtrico Exponencial
Diapositiva 554
Cifrado exponencial genrico tipo RSA
Sea el grupo de trabajo n = pq (n) = (p-1)(q-1)
Se eligen una clave pblica e y una privada d de forma que:
ed mod (n) = 1 ed = k(p-1)(q-1) + 1.
Si ed = k(n) + 1
Por el Teorema del Resto
Chino se tiene que:
M
ed
= M mod n
ssi M
ed
= M mod p
M
ed
= M mod q
y ...
Por el Teorema de Euler
se tiene que:
M
k(n)
mod n = 1
para todo M primo con n
Luego, el sistema de cifra es
vlido para cualquier valor de M
Jorge Rami Aguirre Madrid (Espaa) 2003
Seguridad Informtica y Criptografa. Tema 12: Cifrado Asimtrico Exponencial
Diapositiva 555
Operacin de descifrado exponencial
Al cifrar el mensaje M con una clave pblica e (en este caso
para intercambio de clave, aunque es igual de vlido con una
clave d en caso de firma digital) tenemos:
Cifrado: C = M
e
mod n
Descifrado: C
d
mod n = (M
e
)
d
mod n = M
ed
mod n
C
d
mod n = M
k(n)+1
mod n = MM
k(n)
mod n
C
d
mod n = M1 mod n = M mod n
Por lo tanto, la operacin C
d
mod n recupera el mensaje M.
Jorge Rami Aguirre Madrid (Espaa) 2003
Seguridad Informtica y Criptografa. Tema 12: Cifrado Asimtrico Exponencial
Diapositiva 556
Comprobacin de recuperacin de texto
Sea n = pq = 511 = 55 (n) = (5-1)(11-1) = 40
Mensaje M = 50 = 25
2
(debe ser un elemento de n)
Se elige e = 3 d = inv[e, (n)] = inv (3,40) = 27
ed mod (n) = 327 mod 40 = 81 mod 40 = 1
C = M
e
mod n = 50
3
mod 55 = (25
2
)
3
mod 55
C = [(2)
3
mod 55 (5
2
)
3
mod 55] mod 55 - por reducibilidad +-
M = C
d
mod n = {[(2)
3
mod 55 (5
2
)
3
mod 55] mod 55}
27
mod 55
M = [(2)
327
mod 55 (5
2
)
327
mod 55] mod 55
M = [2
2(n)+1
5
2(n)+1
5
2(n)+1
] mod 55
Por el Teorema de Euler y del Resto Chino
= 2 5 5 mod 55 = 50
Jorge Rami Aguirre Madrid (Espaa) 2003
Seguridad Informtica y Criptografa. Tema 12: Cifrado Asimtrico Exponencial
Diapositiva 557
Intercambio de clave de Diffie y Hellman
El comienzo de los sistemas de clave pblica se debe al estudio
hecho por Whitfield Diffie y Martin Hellman (1976).
Protocolo de Intercambio de Claves de Diffie y Hellman
A y B seleccionan un grupo multiplicativo (con inverso) p
y un generador de dicho primo, ambos valores pblicos
A genera un nmero aleatorio a y enva a B
a
mod p
B genera un nmero aleatorio b y enva a A
b
mod p
B calcula (
a
)
b
mod p =
ab
mod p y luego destruye b
A calcula (
b
)
a
mod p =
ba
mod p y luego destruye a
El secreto compartido por A y B es el valor
ab
mod p
Jorge Rami Aguirre Madrid (Espaa) 2003
Seguridad Informtica y Criptografa. Tema 12: Cifrado Asimtrico Exponencial
Diapositiva 558
Ejemplo de intercambio de clave de DH
Adela (A) y Benito (B) van a intercambiar una clave de
sesin dentro del cuerpo p = 1.999, con = 33. El
usuario A elegir a = 47 y el usuario B elegir b = 117.
Algoritmo:
A calcula
a
mod p = 33
47
mod 1.999 = 1.343 y se lo enva a B.
B calcula
b
mod p = 33
117
mod 1.999 = 1.991 y se lo enva a A.
B recibe 1.343 y calcula 1.343
117
mod 1.999 = 1.506.
A recibe 1.991 y calcula 1.991
47
mod 1.999 = 1.506.
La clave secreta compartida por (A) y (B) ser 1.506
Jorge Rami Aguirre Madrid (Espaa) 2003
Seguridad Informtica y Criptografa. Tema 12: Cifrado Asimtrico Exponencial
Diapositiva 559
Puede un intruso atacar la clave DH?
Un intruso que conozca las claves pblicas p y e
intercepte el valor
a
mod p que ha transmitido A y
el valor
b
mod p transmitido por B no podr
descubrir los valores de a, b ni
ab
mod p ...
salvo que se enfrente al Problema del Logaritmo
Discreto (PLD) que, como ya hemos visto, se
vuelve computacionalmente intratable para valores
del primo p grandes.
Jorge Rami Aguirre Madrid (Espaa) 2003
Seguridad Informtica y Criptografa. Tema 12: Cifrado Asimtrico Exponencial
Diapositiva 560
Es vulnerable el protocolo de DH?
A elige un nmero a con 1 < a < p-1 y enva a B
a
mod p
C intercepta este valor, elige un nmero c con 1 < c < p-1 y
enva a B
c
mod p
B elige un nmero b con 1 < b < p-1 y enva a A
b
mod p
C intercepta este valor y enva a A
c
mod p (valor anterior)
A y B calculan sus claves k
A
= (
c
)
a
mod p, k
B
= (
c
)
b
mod p
C calcula tambin las claves:
k
CA
= (
a
)
c
mod p
k
CB
= (
b
)
c
mod p
Por lo tanto, a partir de ahora C tiene luz verde y puede
interceptar todos los mensajes que se intercambian A y B.
Qu hacer?
La solucin a
este problema
es el sellado
de tiempo
Jorge Rami Aguirre Madrid (Espaa) 2003
Seguridad Informtica y Criptografa. Tema 12: Cifrado Asimtrico Exponencial
Diapositiva 561
Intercambio de clave DH entre n usuarios
El protocolo DH se puede generalizar para n usuarios: sea n = 3.
A, B y C seleccionan un grupo p y un generador
A genera un nmero aleatorio a y enva
a
mod p a B
B genera un nmero aleatorio b y enva
b
mod p a C
C genera un nmero aleatorio c y enva
c
mod p a A
A recibe
c
mod p y calcula (
c
)
a
mod p y se lo enva a B
B recibe
a
mod p y calcula (
a
)
b
mod p y se lo enva a C
C recibe
b
mod p y calcula (
b
)
c
mod p y se lo enva a A
A recibe
bc
mod p y calcula (
bc
)
a
mod p =
bca
mod p
B recibe
ca
mod p y calcula (
ca
)
b
mod p =
cab
mod p
C recibe
ab
mod p y calcula (
ab
)
c
mod p =
abc
mod p
El secreto compartido por A, B y C es el valor
abc
mod p
Jorge Rami Aguirre Madrid (Espaa) 2003
Seguridad Informtica y Criptografa. Tema 12: Cifrado Asimtrico Exponencial
Diapositiva 562
Condiciones del intercambio de clave D-H
CONDICIONES DEL PROTOCOLO:
El mdulo p debe ser un primo grande, al menos 1.024 bits.
Interesa que p-1 debe tener factores primos grandes.
El generador debe ser una raz primitiva del mdulo p.
Si el mdulo es un primo pequeo, se puede hacer un ataque por
fuerza bruta dentro de un tiempo razonable.
Si el generador no es una raz primitiva del grupo p, entonces la
operacin
i
mod p (1 i p-1) no genera todos los restos del
grupo y esto facilita el ataque por fuerza bruta.
Veamos un ejemplo
Jorge Rami Aguirre Madrid (Espaa) 2003
Seguridad Informtica y Criptografa. Tema 12: Cifrado Asimtrico Exponencial
Diapositiva 563
Raz incorrecta (falsa)
MALA ELECCIN DE LOS PARMETROS:
Sean el grupo de trabajo p = 13 y un valor = 3 ... entonces
3
1
mod 13 = 3 3
2
mod 13 = 9 3
3
mod 13 = 1 1 vamos mal
3
4
mod 13 = 3 3
5
mod 13 = 9 3
6
mod 13 = 1
3
7
mod 13 = 3 3
8
mod 13 = 9 3
9
mod 13 = 1
3
10
mod 13 = 3 3
11
mod 13 = 9 3
12
mod 13 = 1 (
p-1
mod p = 1)
Slo debera
darse unidad
en este caso.
Se repiten los restos 3, 9 y 1 porque 3 no es un generador de Z
13
.
Observe que 3
4
mod 13 = (3
3
)(3)
1
mod 13 = 1(3)
1
mod 13 = 3.
Un ataque por fuerza bruta deber buscar slo en una tercera parte
del espacio de claves y, lo que es peor, la probabilidad de xito de
encontrar un valor verdadero b en
b
mod p aumenta de 1/12 a 1/3.
Jorge Rami Aguirre Madrid (Espaa) 2003
Seguridad Informtica y Criptografa. Tema 12: Cifrado Asimtrico Exponencial
Diapositiva 564
Raz correcta
Y si ahora = 2 ?
Primero intente calcularlo... y luego para comprobar sus resultados, avance.
2
1
mod 13 = 2 2
2
mod 13 = 4 2
3
mod 13 = 8
2
4
mod 13 = 3 2
5
mod 13 = 6 2
6
mod 13 = 12
2
7
mod 13 = 11 2
8
mod 13 = 9 2
9
mod 13 = 5
2
10
mod 13 = 10 2
11
mod 13 = 7 2
12
mod 13 = 1
Ahora s estn todos los restos multiplicativos del cuerpo Z
13
porque
el resto 2 es un generador dentro de este cuerpo.
Observe que el valor unidad slo se obtiene para
p-1
mod p.
Y si ahora = 2 ?
Como vimos en el captulo de Teora de Nmeros,
en p = 13 sern generadores g = 2, 6, 7, 11.
Jorge Rami Aguirre Madrid (Espaa) 2003
Seguridad Informtica y Criptografa. Tema 12: Cifrado Asimtrico Exponencial
Diapositiva 565
Algoritmo de cifra asimtrica RSA
En febrero de 1978 Ron Rivest, Adi Shamir y Leonard Adleman
proponen un algoritmo de cifra de clave pblica: RSA
Algoritmo
1. Cada usuario elige un grupo n = pq (pueden ser distintos).
2. Los valores p y q no se hacen pblicos.
3. Cada usuario calcula (n) = (p-1)(q-1).
4. Cada usuario elige una clave pblica e que sea parte del cuerpo n
y que cumpla: mcd [e, (n)] = 1.
5. Cada usuario calcula la clave privada d = inv [e,(n)].
6. Se hace pblico el grupo n y la clave e.
7. Se guarda en secreto la clave d.
Podran destruirse
ahora p, q y (n).
Jorge Rami Aguirre Madrid (Espaa) 2003
Seguridad Informtica y Criptografa. Tema 12: Cifrado Asimtrico Exponencial
Diapositiva 566
Ejemplo de cifrado y descifrado con RSA
Grupo n = 91 = 713; (n) = (713) = (7-1)(13-1) = 72 M = 48
Elegimos e = 5 pues mcd (5,72) = 1 d = inv(5,72) = 29
CIFRADO:
C = M
e
mod n = 48
5
mod 91 = 5245.803.968 mod 91 = 55
DESCIFRADO:
M = C
d
mod n = 55
29
mod 91 = 48 ... 55
29
ya es nmero grande
55
29
es un nmero con 51 dgitos...
55
29
= 295473131755644748809642476009391248226165771484375
Cmo podemos acelerar esta operacin?

1 opcin: usar reducibilidad 2 opcin: algoritmo exp. rpida


`
Opcin ptima: usar el Teorema del Resto Chino
Jorge Rami Aguirre Madrid (Espaa) 2003
Seguridad Informtica y Criptografa. Tema 12: Cifrado Asimtrico Exponencial
Diapositiva 567
Uso del Teorema del Resto Chino en RSA
Normalmente la clave pblica e de RSA es un valor bastante bajo,
por ejemplo 2
16
+ 1. Luego, en el proceso de cifra (no en la firma)
no tendremos problemas con la velocidad de cifrado porque el
exponente e es relativamente bajo.
Como el cuerpo de trabajo n = pq es mucho mayor, del orden de
2
1024
si hablamos de claves de 1024 bits, entonces la clave privada
d ser por lo general mucho mayor que el valor de e. Por lo tanto,
puede ser costoso para el receptor descifrar algo con su clave
privada o bien firmar digitalmente un documento con dicha clave.
La solucin est en aplicar el Teorema del Resto Chino. En vez de
trabajar en n, lo haremos en p y q. Entonces las exponenciaciones
modulares se harn en p y q, mucho ms rpido que hacerlo en n.
Jorge Rami Aguirre Madrid (Espaa) 2003
Seguridad Informtica y Criptografa. Tema 12: Cifrado Asimtrico Exponencial
Diapositiva 568
Descifrado RSA aplicando el TRC
M = C
d
mod n Aplicando el Teorema del Resto Chino en n:
M = {A
p
[C
p
dp
(mod p)] + A
q
[C
q
dq
(mod q)]} mod n
con C
p
= C mod p C
q
= C mod q
d
p
= d mod (p-1) d
q
= d mod (q-1)
A
p
= q [inv (q,p)] = q
p-1
mod n
A
q
= p [inv (p,q)] = p
q-1
mod n
Recuerde que p y q son la trampa que slo conoce el dueo de la clave
Jorge Rami Aguirre Madrid (Espaa) 2003
Seguridad Informtica y Criptografa. Tema 12: Cifrado Asimtrico Exponencial
Diapositiva 569
Ejemplo de descifrado RSA usando el TRC
Sea: p = 89, q = 31, n = pq = 8931 = 2.759, (n) = 8830 = 2.640
Elegimos e = 29 d = inv [e, (n)] = inv [29, 2.640] = 2.549
Si el bloque de mensaje ya codificado es M = 1.995, entonces:
C = M
e
mod n = 1.995
29
mod 2.759 = 141
M = C
d
mod n = 141
2.549
mod 2.759 = 1.995
A
p
= q
p-1
mod n = 31
88
mod 2.759 = 713 A
q
= p
q-1
mod n = 89
30
mod 2.759 = 2.047
C
p
= C mod p = 141 mod 89 = 52 C
q
= C mod q = 141 mod 31 = 17
d
p
= d mod (p-1) = 2.549 mod 88 = 85 d
q
= d mod (q-1) = 2.549 mod 30 = 29
Reemplazando en: M = {A
p
[C
p
dp
(mod p)] + A
q
[C
q
dq
(mod q)]} mod n
M = {713[52
85
mod 89] + 2.047[17
29
mod 31]} mod 2.759
M = {71337 + 2.04711} mod 2.759 = (26.381 + 22.517) mod 2.759 = 1.995
Jorge Rami Aguirre Madrid (Espaa) 2003
Seguridad Informtica y Criptografa. Tema 12: Cifrado Asimtrico Exponencial
Diapositiva 570
El problema en la eleccin del valor de n
Si p y q son muy cercanos, es fcil factorizar n.
Si p q y suponemos que p > q, entonces (p-q)/2 es un
entero muy pequeo y por otra parte (p+q)/2 es
ligeramente superior a n.
Adems se cumplir que: n = (p+q)/4 - (p-q)/4. Esto
lo podemos escribir como n = x - y y = x - n
Elegimos enteros x > n hasta que (x - n) sea cuadrado
perfecto. En este caso x = (p+q)/2; y = (p-q)/2. Por lo
tanto rompemos el valor n: p = (x+y); q = (x-y).
`
Jorge Rami Aguirre Madrid (Espaa) 2003
Seguridad Informtica y Criptografa. Tema 12: Cifrado Asimtrico Exponencial
Diapositiva 571
Ejemplo de mala eleccin del valor de n
Sea p = 181; q = 251 n = 181251 = 45.431
Como 45.431 = 213,14 buscaremos valores enteros
de x mayores que 213 de forma que (x - 45.431) sea
un cuadrado perfecto +
1. x = 214 x 45.431 = 365 365 = 19,10
2. x = 215 x 45.431 = 794 794 = 28,17
3. x = 216 x 45.431 = 1.225 1.225 = 35
Entonces: p = x y = 216 35 = 181
q = x + y = 216 + 35 = 251
`
Para evitar otros problemas, es recomendable
usar los denominados primos seguros.
Jorge Rami Aguirre Madrid (Espaa) 2003
Seguridad Informtica y Criptografa. Tema 12: Cifrado Asimtrico Exponencial
Diapositiva 572
Eleccin de los nmeros primos
Los valores primos deben elegirse apropiadamente:
Sistema RSA
a) p y q deben diferir en unos pocos dgitos.
b) p y q no deben ser primos muy cercanos.
c) Longitud mnima de p y q: 500 bits.
d) Valores de (p-1) y (q-1) del Indicador de
Euler deben tener factores primos grandes.
e) El mcd entre p-1 y q-1 debe ser pequeo.
Cmo?
Jorge Rami Aguirre Madrid (Espaa) 2003
Seguridad Informtica y Criptografa. Tema 12: Cifrado Asimtrico Exponencial
Diapositiva 573
Clculo de nmeros primos seguros
Primos seguros: se elige r un primo grande de modo que:
p = 2r + 1; q = 2p + 1 sean primos y mcd (p-1, q-1) pequeo.
EJEMPLO: Si r es el primo de 4 dgitos 1.019:
p = 21.019 + 1 = 2.039 Es primo `
q = 22.039 + 1 = 4.079 Es primo `
Luego: p-1 = 2.038; q-1 = 4.078
p-1 = 21.019; q-1 = 22.039 mcd (p-1, q-1) = 2
Luego los primos p y q cumplen la condicin de primos seguros
El mdulo ser n = pq = 8.317.081
Jorge Rami Aguirre Madrid (Espaa) 2003
Seguridad Informtica y Criptografa. Tema 12: Cifrado Asimtrico Exponencial
Diapositiva 574
El problema de las claves privadas parejas
Una clave pareja d, permite descifrar el criptograma C
resultado de una cifra con la clave pblica e sin que d sea
el inverso de la clave pblica e. En el sistema RSA habr
como mnimo una clave d pareja de la clave pblica e.
Ejemplo:
Si p = 5; q = 13; n = 65, (n) = 48 = 2
4
x3, elegimos e = 5.
Calculamos d = inv (5, 48) = 29 que es nico. Si ciframos
el mensaje M = 59, obtenemos C = 59
5
mod 65 = 24.
Luego sabemos que M = C
d
mod n = 24
29
mod 65 = 59
Pero tambin lo desciframos con d = 5, 17, 41 y 53!

Jorge Rami Aguirre Madrid (Espaa) 2003


Seguridad Informtica y Criptografa. Tema 12: Cifrado Asimtrico Exponencial
Diapositiva 575
Nmero de claves privadas parejas
Si: = mcm (p-1),(q-1) sea d

= e
-1
mod = inv (e, )
La clave pblica e tendr claves parejas de forma que:
d
i
= d

+ i 1 < d
i
< n
i = 0, 1, ... -1 = (n - d

)/
En el ejemplo anterior tenemos que = mcm (4, 12) = 12.
Luego: d

= inv (5, 12) = 5, as d


i
= d

+ i = 5 + i12.
Es decir d
i
= 5, 17, 29, 41, 53; el nmero de claves que
corresponden al valor = (n - d

)/ = (65 5)/12) = 5.
En este caso hablamos de debilidad de la clave, al igual que suceda
con las claves dbiles y semidbiles por ejemplo en DES e IDEA.
Jorge Rami Aguirre Madrid (Espaa) 2003
Seguridad Informtica y Criptografa. Tema 12: Cifrado Asimtrico Exponencial
Diapositiva 576
Minimizando el nmero de claves parejas
Para que sea lo ms pequeo posible ( = 1) deberemos
elegir los primos p y q como primos seguros.
Ejemplo:
Sea r = 5 p = 2r + 1 = 11 (es primo `)
q = 2p + 1 = 23 (es primo `)
En estas condiciones con n = 253 y (n) = 220, sea e = 7
Luego = mcm (10, 22) = 110 y d

= inv (7, 110) = 63


Entonces = (n - d

)/ = (253 63)/110 = 1
As: d
i
= d

+ i = 63 + i110 = 63, 173 (con i = 0, 1)


En efecto 63 = inv [e, (n)] = inv (7, 220) y lo cifrado con
e = 7 tambin se descifra con d = 173. Slo dos claves.
Jorge Rami Aguirre Madrid (Espaa) 2003
Seguridad Informtica y Criptografa. Tema 12: Cifrado Asimtrico Exponencial
Diapositiva 577
Mensajes no cifrables
+ Si M
e
mod n = Mse dice que el mensaje es no cifrable.
Aunque la clave e sea vlida, ste se enva en claro .
+ En RSA habr como mnimo 9 mensajes no cifrables.
Ejemplo:
Sea el cuerpo n = 35 (p = 5, q = 7), con (n) = 24 y e = 11.
Dentro de los mensajes posibles {0, 34} sern no cifrables:
{6, 14, 15, 20, 21, 29, 34} adems de los obvios {0, 1}.
+ En el caso ms crtico, todos los mensajes del cuerpo n
pueden ser no cifrables como veremos ms adelante.
Cmo se obtienen estos mensajes no cifrables?
Jorge Rami Aguirre Madrid (Espaa) 2003
Seguridad Informtica y Criptografa. Tema 12: Cifrado Asimtrico Exponencial
Diapositiva 578
Nmero de mensajes no cifrables
El nmero de mensajes no cifrables dentro de un cuerpo n viene dado por:

n
= [1 + mcd (e-1, p-1)][1 + mcd (e-1, q-1)]
Los mensajes no cifrables sern:
M = [q{inv (q, p)}M
p
+ p{inv (p, q)}M
q
] mod n
con: M
p
las soluciones de M
e
mod p = M
M
q
las soluciones de M
e
mod q = M
Esto ltimo debido al TRC puesto que M
e
mod n = M
En el ejemplo anterior se da el caso mnimo:

n
= [1 + mcd (10, 4)][1 + mcd (10, 6)] = (1+2)(1+2) = 9
M
11
mod 5 = M M
5
= {0, 1, 4} M
11
mod 7 = M M
7
= {0, 1, 6}
M = [7{inv (7, 5)}M
p
+ 5 {inv (5,7)}M
q
] mod 35
M = [73 M
p
+ 53 M
q
] mod 35 = [21{0, 1, 4} + 15 {0, 1, 6}] mod 35
M = {0, 15, 90, 21, 36, 111, 84, 99, 175} mod 35 ordenando...
M = {0, 1, 6, 14, 15, 20, 21, 29, 34}
Jorge Rami Aguirre Madrid (Espaa) 2003
Seguridad Informtica y Criptografa. Tema 12: Cifrado Asimtrico Exponencial
Diapositiva 579
Ejemplo de mensajes no cifrables (1)
Sea p = 13; q = 17; n = pq = 221
Elegimos e = 7 por lo que d = inv (7, 192) = 55, luego:

n
= [1 + mcd (e-1, p-1)][1 + mcd (e-1, q-1)]

221
= [1 + mcd (6, 12)][1 + mcd (6, 16)] = (1+6)(1+2) = 21
Soluciones de M
7
mod 13 = M M
p
= {0, 1, 3, 4, 9, 10, 12}
Soluciones de M
7
mod 17 = M M
q
= {0, 1, 16}
Los mensajes no cifrables sern:
M = [q{inv (q, p)}M
p
+ p{inv (p, q)}M
q
] mod n
M = [17{inv (17, 13)}M
p
+ 13{inv (13, 17)}M
q
] mod 221
M = [{1710}M
p
+ {134}M
q
] mod 221
M = [170M
p
+ 52M
q
] mod 221
Jorge Rami Aguirre Madrid (Espaa) 2003
Seguridad Informtica y Criptografa. Tema 12: Cifrado Asimtrico Exponencial
Diapositiva 580
Ejemplo de mensajes no cifrables (2)
Tenamos M
p
= {0, 1, 3, 4, 9, 10, 12}
M
q
= {0, 1, 16}
M = [170M
p
+ 52M
q
] mod 221 luego:
M = [170{0, 1, 3, 4, 9, 10, 12} + 52{0, 1, 16}] mod 221
Como 52{0, 1, 16} = {0, 52, 832} entonces:
M = [0+0, 0+52, 0+832, 170+0, 170+52, 170+832, ...] mod 221
M = [0, 52, 832, 170, 222, 1.002, 510, 562, 1.342, 680, 732,
1.512, 1.530, 1.582, 2.362, 1.700, 1.752, 2.531, 2.040,
2.092, 2.872] mod 221
M = [0, 52, 169, 170, 1, 118, 68, 120, 16, 17, 69, 186, 204, 35,
152, 153, 205, 101, 51, 103, 220] ordenando:
M = [0, 1, 16, 17, 35, 51, 52, 68, 69, 101, 103, 118, 120, 152,
153, 169, 170, 186, 204, 205, 220] los 21 mensajes de
221
Jorge Rami Aguirre Madrid (Espaa) 2003
Seguridad Informtica y Criptografa. Tema 12: Cifrado Asimtrico Exponencial
Diapositiva 581
Nmero mnimo de mensajes no cifrables
Para que el nmero de mensajes no cifrables sea el mnimo posible,
es decir 9, deberemos elegir la clave pblica e de forma que:
mcd (e-1, p-1) = 2 y mcd (e-1, q-1) = 2
Entonces:
n
= [1 + 2][1 + 2] = 9
Esto se logra usando primos seguros:
p = 2r + 1 y q = 2p + 1 con r, p y q primos grandes
ya que: mcd (e-1, p-1) = mcd (e-1, (2r +1)-1) mcd = 2 o bien r
mcd (e-1, q-1) = mcd (e-1, (2p +1)-1) mcd = 2 o bien p
Luego:
n
= {9, 3(r+1), 3(p+1), (r+1)(p+1)}
Hay que comprobar en diseo que no se den valores del mcd igual a
r o p pues tendramos un nmero muy alto de este tipo de mensajes.
Jorge Rami Aguirre Madrid (Espaa) 2003
Seguridad Informtica y Criptografa. Tema 12: Cifrado Asimtrico Exponencial
Diapositiva 582
Nmero mximo de mensajes no cifrables
En el peor de los casos, mcd (e-1, p-1) = p-1 y mcd (e-1, q-1) = q-1
Entonces:
n
= [1 + mcd(e-1, p-1)][1 + mcd(e-1, q-1)]

n
= pq = n ... todas las cifras irn en claro!
Si en el ejemplo anterior con p = 13, q = 17, hubisemos elegido
como clave e = 49, con d = inv (49, 192) = 145, observamos que:
mcd (e-1, p-1) = mcd (48, 12) = 12
mcd (e-1, q-1) = mcd (48, 16) = 16

n
= [1 + 12][1 + 16] = 1317 = 221 = pq = n
Por lo tanto, cualquier mensaje en el cuerpo n = 221 ser no cifrable
para la clave pblica e = 49. Compruebe que en este caso esto se
cumple si e = (n)/k +1 (k = 1, 2 y 4), es decir e = 193, 97 y 49.
Nunca podr usarse e = (n) + 1 ya que la clave de descifrado es 1
ni e = (n)/2 + 1 pues tampoco se cifrarn todos los mensajes de n.
Jorge Rami Aguirre Madrid (Espaa) 2003
Seguridad Informtica y Criptografa. Tema 12: Cifrado Asimtrico Exponencial
Diapositiva 583
Ataque a la clave por factorizacin de n
Qu fortaleza tendr este algoritmo ante ataques?
El intruso que desee conocer la clave secreta d a
partir de los valores pblicos n y e se enfrentar al
Problema de la Factorizacin de Nmeros Grandes
(PFNG) puesto que la solucin para conocer esa
clave privada pasa por deducir el valor del Indicador
de Euler (n) = (p-1)(q-1) para as poder encontrar el
inverso de la clave pblica d = inv [e,(n)].
La complejidad asociada al PFNG viene dada por la
ecuacin e
ln(n) ln ln(n)
, donde ln es logaritmo natural.
Jorge Rami Aguirre Madrid (Espaa) 2003
Seguridad Informtica y Criptografa. Tema 12: Cifrado Asimtrico Exponencial
Diapositiva 584
Tiempo necesario para afrontar el PFNG
1,0 x 10
6
3,8 x 10
8
200 665
2,5 x 10
2
9,4 x 10
4
133 442
2,5 9,0 x 10
2
109 363
- 1,0 77 256
- 1,5 x 10
-5
36 120
- 1,7 x 10
-8
18 60
Aos Das N de dgitos N de bits (n)
Para un procesador de 2x10
8
instrucciones por segundo.
Fuente: Criptografa Digital, Jos Pastor. Prensas Univ. de Zaragoza, 1998.
Existen, no obstante, otros tipos de ataques a este
sistema que no pasan por la factorizacin de n.

Jorge Rami Aguirre Madrid (Espaa) 2003
Seguridad Informtica y Criptografa. Tema 12: Cifrado Asimtrico Exponencial
Diapositiva 585
Ataque al secreto de M por cifrado cclico
Se puede encontrar el mensaje en claro M sin necesidad de
conocer d, la clave privada del receptor.
Como C = M
e
mod n, realizaremos cifrados sucesivos de los
criptogramas C
i
resultantes con la misma clave pblica hasta
obtener nuevamente el cifrado C original.
C
i
= C
e
i-1
mod n (i = 1, 2, ...) y C
0
= C
Si en el cifrado isimo se encuentra el criptograma C inicial,
entonces es obvio que el cifrado (i-1) ser el mensaje
buscado.
Esto se debe a que RSA es un grupo mutiplicativo. Para
evitarlo hay que usar primos seguros de forma que los
subgrupos de trabajo sean bastante altos.
Jorge Rami Aguirre Madrid (Espaa) 2003
Seguridad Informtica y Criptografa. Tema 12: Cifrado Asimtrico Exponencial
Diapositiva 586
Ejemplo de ataque por cifrado cclico
Sea p = 13, q = 19, n = 247, (n) = 216, e = 29 (d = 149, no conocido)
El mensaje a cifrar ser M = 123 C = 123
29
mod 247 = 119
Tabla de cifra i C
i
i = 0 C
0
= 119
i = 1 C
1
= 119
29
mod 247 = 6
i = 2 C
2
= 6
29
mod 247 = 93
i = 3 C
3
= 93
29
mod 247 = 175
i = 4 C
4
= 175
29
mod 247 = 54
i = 5 C
5
= 54
29
mod 247 = 123
i = 6 C
6
= 123
29
mod 247 = 119
El ataque ha prosperado muy rpidamente: como hemos obtenido otra
vez el criptograma C = 119, es obvio que el paso anterior con C = 123
se corresponda con el texto en claro. Y si usamos primos seguros?
Jorge Rami Aguirre Madrid (Espaa) 2003
Seguridad Informtica y Criptografa. Tema 12: Cifrado Asimtrico Exponencial
Diapositiva 587
Ataque cifrado cclico: caso primos seguros
Sea p = 11, q = 23, n = 253, (n) = 220, e = 17 (d = 134, no conocido)
Sea el mensaje secreto M = 123 C = 123
17
mod 253 = 128
i C
i
i C
i
i = 0 C
0
= 128 i = 12 C
12
= 167
17
mod 253 = 150
i = 1 C
1
= 128
17
mod 253 = 6 i = 13 C
13
= 150
17
mod 253 = 193
i = 2 C
2
= 6
17
mod 253 = 173 i = 14 C
14
= 193
17
mod 253 = 118
i = 3 C
3
= 173
17
mod 253 = 101 i = 15 C
15
= 118
17
mod 253 = 200
i = 4 C
4
= 101
17
mod 253 = 95 i = 16 C
16
= 200
17
mod 253 = 73
i = 5 C
5
= 95
17
mod 253 = 39 i = 17 C
17
= 73
17
mod 253 = 94
i = 6 C
6
= 39
17
mod 253 = 96 i = 18 C
18
= 94
17
mod 253 = 41
i = 7 C
7
= 96
17
mod 253 = 2 i = 19 C
19
= 41
17
mod 253 = 123 -
i = 8 C
8
= 2
17
mod 253 = 18 i = 20 C
20
= 123
17
mod 253 = 128
i = 9 C
9
= 18
17
mod 253 = 215
i = 10 C
10
= 215
17
mod 253 = 151
i = 11 C
11
= 151
17
mod 253 = 167
Hemos tenido que recorrer un
espacio mucho mayor dentro
de un cuerpo de cifra similar.
Jorge Rami Aguirre Madrid (Espaa) 2003
Seguridad Informtica y Criptografa. Tema 12: Cifrado Asimtrico Exponencial
Diapositiva 588
Ataque a la clave por paradoja cumpleaos
La paradoja del cumpleaos se ver en el prximo captulo.
Algoritmo propuesto por Merkle y Hellman en 1981:
El atacante elige dos nmeros aleatorios distintos i, j
dentro del cuerpo de cifra n. Elige adems un mensaje M
cualquiera.
Para i = i+1 y para j = j+1 calcula M
i
mod n y M
j
mod n.
Cuando encuentra una coincidencia de igual resultado
para una pareja (i, j), puede encontrar d.
Ejemplo: sea p = 7; q = 13, n = pq = 81, e = 11, d = 59.
El atacante elige los valores i = 10, j = 50 y parte con el
mensaje M = 20. Slo conoce n = 81 y e = 11.
Jorge Rami Aguirre Madrid (Espaa) 2003
Seguridad Informtica y Criptografa. Tema 12: Cifrado Asimtrico Exponencial
Diapositiva 589
Ejemplo de ataque paradoja cumpleaos
i C
i
j C
i
i = 10 C
10
= 20
10
mod 81 = 34 i = 50 C
50
= 20
50
mod 81 = 13
i = 11 C
11
= 20
11
mod 81 = 32 i = 51 C
51
= 20
51
mod 81 = 17
i = 12 C
12
= 20
12
mod 81 = 73 i = 52 C
52
= 20
52
mod 81 = 16
i = 13 C
13
= 20
13
mod 81 = 2 i = 53 C
53
= 20
53
mod 81 = 77
i = 14 C
14
= 20
14
mod 81 = 40 i = 54 C
54
= 20
54
mod 81 = 1
i = 15 C
15
= 20
15
mod 81 = 71 i = 55 C
55
= 20
55
mod 81 = 20
i = 16 C
16
= 20
16
mod 81 = 43 i = 56 C
56
= 20
56
mod 81 = 76
i = 17 C
17
= 20
17
mod 81 = 50 i = 57 C
57
= 20
57
mod 81 = 62
i = 18 C
18
= 20
18
mod 81 = 28 i = 58 C
58
= 20
58
mod 81 = 25
i = 19 C
19
= 20
19
mod 81 = 74 i = 59 C
59
= 20
59
mod 81 = 14
i = 20 C
20
= 20
20
mod 81 = 22 i = 60 C
60
= 20
60
mod 81 = 37
i = 21 C
21
= 20
21
mod 81 = 35 i = 61 C
61
= 20
61
mod 81 = 11
i = 22 C
22
= 20
22
mod 81 = 52 i = 62 C
62
= 20
62
mod 81 = 58
i = 23 C
23
= 20
23
mod 81 = 68 i = 63 C
63
= 20
63
mod 81 = 26
i = 24 C
24
= 20
24
mod 81 = 64 i = 64 C
64
= 20
64
mod 81 = 34
Jorge Rami Aguirre Madrid (Espaa) 2003
Seguridad Informtica y Criptografa. Tema 12: Cifrado Asimtrico Exponencial
Diapositiva 590
Resultado del ataque paradoja cumpleaos
NOTA: Puesto que la explicacin matemtica de este algoritmo podra
resultar algo compleja y larga para exponerla en estos apuntes, se dar
slo la solucin numrica al problema que, como ver, es muy sencilla.
La primera coincidencia se encuentra para i = 10; j = 64. As, el atacante
conociendo la clave pblica e = 11, calcula:
w = (10-64) / mcd (11, 10-64) = -54 / mcd (11, -54) = - 54.
Entonces debern existir valores s,t de forma que se cumpla lo siguiente:
ws + et = 1 -54s + 11t = 1
Las posibles soluciones a la ecuacin son: ws mod e = 1; et mod w = 1
-54s = 1 mod 11 s = inv (-54, 11) = 1
11t = 1 mod 54 t = inv (11, 54) = 5
El valor t = 5 ser una clave secreta d pareja de d = 59. Compruebe que
se verifica ws + et = 1. Observe que 5 mod 54 = 59 mod 54 = 5.
Jorge Rami Aguirre Madrid (Espaa) 2003
Seguridad Informtica y Criptografa. Tema 12: Cifrado Asimtrico Exponencial
Diapositiva 591
La otra historia del algoritmo RSA
Rivest, Shamir y Adleman son los autores de RSA pero
el desarrollo de un algoritmo de cifra asimtrico que
usara el problema de factorizar nmeros grandes como
funcin unidireccional fue descubierto mucho antes.
En el ao 1969 el GCHQ (Government Communications
Headquarters) en Gran Bretaa comienza a trabajar en
la idea de poder distribuir claves a travs de una cifra no
simtrica. En 1973, el matemtico Clifford Cocks llegar
a la misma conclusin que los creadores de RSA.
Desgraciadamente este trabajo fue considerado como
alto secreto por el gobierno britnico por lo que no se
hace pblico ni se patenta como invento, algo que s
hacen Diffie y Hellman en 1976 con su intercambio de
claves y en 1978 otro tanto los creadores de RSA.
Jorge Rami Aguirre Madrid (Espaa) 2003
Seguridad Informtica y Criptografa. Tema 12: Cifrado Asimtrico Exponencial
Diapositiva 592
Cifrado Pohlig y Hellman con clave secreta
Stephen Pohlig y Martin Hellman proponen en enero de 1978 un
algoritmo de cifra de clave secreta y que basa su seguridad en el
problema del logaritmo discreto. Hablamos de slo un mes antes
que el algoritmo RSA... algo que tambin llama la atencin.
+ Se elige un grupo multiplicativo Z
p
*, p es un primo grande.
+ Cada usuario elige una clave e, que sea primo relativo con el grupo
(p) = p-1 y calcula d = inv (e, (p)].
+ La clave secreta sern los valores e y d.
El sistema carece de
firma digital y no puede
competir en velocidad
con los algoritmos de
clave secreta por lo
que no tiene uso.
Operaciones de cifra
CIFRADO
C = M
e
mod p
DESCIFRADO
M = C
d
mod p
Jorge Rami Aguirre Madrid (Espaa) 2003
Seguridad Informtica y Criptografa. Tema 12: Cifrado Asimtrico Exponencial
Diapositiva 593
Ejemplo de cifrado Pohlig y Hellman
A cifra un mensaje M que enva a B
p = 263 (p) = 262; e = 15 d = inv(15,262) = 35
Sea M = Adis = 65 100 105 243 115
Como se usa el cdigo ANSI, podremos cifrar en bloques
de un carcter pues el mdulo p es algo mayor que 256.
C = M
e
mod p = 65
15
mod 263, 100
15
mod 263,
105
15
mod 263, 243
15
mod 263, 115
15
mod 263
C = 245, 143, 179, 86, 101
Operacin Cifrado:
Jorge Rami Aguirre Madrid (Espaa) 2003
Seguridad Informtica y Criptografa. Tema 12: Cifrado Asimtrico Exponencial
Diapositiva 594
Ejemplo de descifrado Pohlig y Hellman
B descifra el criptograma C enviado por A
p = 263; d = inv (15, 262) = 35
C = 245, 143, 179, 86, 101
Operacin Descifrado:
M = C
d
mod p = 245
35
mod 263, 143
35
mod 263,
179
35
mod 263, 86
35
mod 263, 101
35
mod 263
M = 065, 100, 105, 243, 115
Convirtindolo al cdigo ANSI: M = Adis
Jorge Rami Aguirre Madrid (Espaa) 2003
Seguridad Informtica y Criptografa. Tema 12: Cifrado Asimtrico Exponencial
Diapositiva 595
Algoritmo de cifra asimtrica de ElGamal
Taher ElGamal propone en 1985 un algoritmo de cifra
que hace uso del Problema del Logaritmo Discreto PLD.
Caractersticas
Se elige un grupo multiplicativo Z
p
*, p es primo grande.
Del grupo p se elige una raz , generador del grupo.
Cada usuario elige un nmero aleatorio dentro de p.
Esta ser la clave privada.
Cada usuario calcula

mod p.
Junto con p es la clave pblica.
Para descubrir la clave privada
el atacante deber enfrentarse
al Problema del Logaritmo
Discreto para un valor p alto.
Para descubrir la clave privada
el atacante deber enfrentarse
al Problema del Logaritmo
Discreto para un valor p alto.
Jorge Rami Aguirre Madrid (Espaa) 2003
Seguridad Informtica y Criptografa. Tema 12: Cifrado Asimtrico Exponencial
Diapositiva 596
Operacin de cifra con ElGamal
Operacin Cifrado: A cifra un mensaje M que enva a B
El usuario B ha elegido su clave privada b dentro del
cuerpo del nmero primo p que es pblico.
El usuario B ha hecho pblica su clave
b
mod p.
El emisor A genera un nmero aleatorio de sesin y
calcula

mod p.
Con la clave pblica de B (
b
) el emisor A calcula:
(
b
)

mod p y M(
b
)

mod p
A enva a B el par: C = [

mod p, M (
b
)

mod p]
Jorge Rami Aguirre Madrid (Espaa) 2003
Seguridad Informtica y Criptografa. Tema 12: Cifrado Asimtrico Exponencial
Diapositiva 597
Operacin de descifrado con ElGamal
Operacin Descifrado: B descifra el criptograma C que enva A
El usuario B recibe C = [

mod p, M (
b
)

mod p].
B toma el valor

mod p y calcula (

)
b
mod p.
B descifra el criptograma C haciendo la siguiente divisin:
[M (
b
)

mod p ] / [(

)
b
mod p] (
b
)

= (

)
b
El paso anterior es posible hacerlo porque existir el
inverso de (

)
b
en el grupo p al ser p un primo. Luego:
[M (
b
)

{inv (

)
b
, p}] mod p = M
Jorge Rami Aguirre Madrid (Espaa) 2003
Seguridad Informtica y Criptografa. Tema 12: Cifrado Asimtrico Exponencial
Diapositiva 598
Ejemplo de cifrado con ElGamal
Adela (A) enviar a Benito (B) el mensaje M = 10 cifrado
dentro del cuerpo p = 13 que usa Benito.
CIFRADO
Claves pblicas de Benito: p = 13, = 6, (
b
) mod p = 2
Adela A elige por ejemplo = 4 y calcula:
(

) mod p = 6
4
mod 13 = 9
(
b
)
v
mod p = 2
4
mod 13 = 3
M(
b
)
v
mod p = 103 mod 13 = 4
Enva a B (

) mod p, M(
b
)
v
mod p = [9, 4]
Jorge Rami Aguirre Madrid (Espaa) 2003
Seguridad Informtica y Criptografa. Tema 12: Cifrado Asimtrico Exponencial
Diapositiva 599
Ejemplo de descifrado con ElGamal
DESCIFRADO
La clave privada de Benito es b = 5
Benito recibe: [(

) mod p, M(
b
)
v
mod p] = [9, 4]
Benito calcula:
(

)
b
mod p = 9
5
mod 13 = 3
[M(
b
)
v
] inv[(

)
b
, p] = 4 inv (3, 13) = 4 9
M = 4 9 mod 13 = 10 (se recupera el mensaje)
Recuerde que debe ser una raz de p. Como ya hemos
visto, si no es una raz, aunque s puede hacerse la cifra,
se facilitar el ataque al Problema del Logaritmo Discreto.
Jorge Rami Aguirre Madrid (Espaa) 2003
Seguridad Informtica y Criptografa. Tema 12: Cifrado Asimtrico Exponencial
Diapositiva 600
Consideraciones sobre el bloque
Si el mensaje M fuese mayor que el mdulo de trabajo
(n = pq para RSA y p para ElGamal)
cmo se generaran los bloques del mensaje a cifrar?
El mensaje M se transforma en
nmeros y stos se dividen en bloques
de de g-1 dgitos, siendo g el nmero
de dgitos del mdulo de trabajo: el
valor n para RSA y p para ElGamal.
Nota: en la prctica esto no ocurrir ya que el
cuerpo de cifra es como mnimo de 512 bits y el
mensaje a cifrar tendr slo una centena de bits.
Ejemplo
Jorge Rami Aguirre Madrid (Espaa) 2003
Seguridad Informtica y Criptografa. Tema 12: Cifrado Asimtrico Exponencial
Diapositiva 601
Ejemplo de eleccin del bloque con RSA
Se representar el mensaje en su valor ANSI decimal.
n = pq = 89127 = 11.303 bloques de cuatro dgitos
(n) = 11.088; e = 25; d = inv (25, 11.088) = 10.201
M = Ol = 079 108 233 M = 0791 0823 3
Se recupera el mensaje agrupando en
bloques de 4 dgitos excepto el ltimo
CIFRADO DESCIFRADO
C
1
= 791
25
mod 11.303 = 7.853
C
2
= 823
25
mod 11.303 = 2.460
C
3
= 3
25
mod 11.303 = 6.970
M
1
= 7.853
10201
mod 11.303 = 0791
M
2
= 2.460
10201
mod 11.303 = 0823
M
3
= 6.970
10201
mod 11.303 = 3
Jorge Rami Aguirre Madrid (Espaa) 2003
Seguridad Informtica y Criptografa. Tema 12: Cifrado Asimtrico Exponencial
Diapositiva 602
Fortaleza de la cifra exponencial
El problema de la Factorizacin de Nmeros Grandes PFNG
tiene una complejidad similar al del Logaritmo Discreto PLD:
ambos suponen un tiempo de ejecucin no polinomial.
Nmero de pasos que debe realizar el algoritmo PFNG:
e
{ln(n)ln[ln(n)]}
n = 60 dgitos 2,710
11
pasos
n = 100 dgitos 2,310
15
pasos
n = 200 dgitos 1,210
23
pasos
Sistema que consuma 1 seg por paso:
3 das
74 aos
3,810
9
aos
El PLD es matemticamente similar:
nmero de pasos e
{ln(p)ln[ln(p)]}
Fin del Tema 12
Jorge Rami Aguirre Madrid (Espaa) 2003
Seguridad Informtica y Criptografa. Tema 12: Cifrado Asimtrico Exponencial
Diapositiva 603
Cuestiones y ejercicios (1 de 4)
1. A partir de la ecuacin ed = k(n) + 1, compruebe que las claves
RSA e = 133 y d = 38.797 son inversas en el cuerpo n = 40.501.
2. En el ejemplo de intercambio de clave DH del libro con p = 1.999,
podran haber elegido Adela y Benito = 34, 35, 36 37?
3. Carmela (C) intercepta la clave de sesin DH que se intercambian
Adela (A) y Benito (B) dentro del cuerpo p = 127. Si se usa como
generador = 19 y a = 3, b = 12 y c = 7, desarrolle el algoritmo que
permite a C interceptar la comunicacin y engaar a A y B.
4. Los usuarios A, B, C y D desean intercambiar una clave usando el
mtodo DH. Proponga un protocolo genrico que solucione este
problema y presente un ejemplo en el cuerpo p = 23 y elija .
5. Disee un sistema RSA en el que p = 53, q = 113. Elija como clave
pblica el mnimo valor posible de 2 dgitos.
Jorge Rami Aguirre Madrid (Espaa) 2003
Seguridad Informtica y Criptografa. Tema 12: Cifrado Asimtrico Exponencial
Diapositiva 604
Cuestiones y ejercicios (2 de 4)
6. Para los datos de diseo del ejercicio 5, cifre el mensaje M = 121 y
luego descfrelo. Use el algoritmo de exponenciacin rpida.
7. Vuelva a descifrar el criptograma usando ahora el Teorema del
Resto Chino. Aunque en este caso haya hecho ms clculos por
qu es interesante usar aqu el Teorema del Resto Chino?
8. Si p = 353 y q = 1.103, cifre el mensaje ASCII de cuatro caracteres
M = HOLA en bloques de tamao eficiente (mnimo) para e = 17.
9. Para los datos del ejercicio anterior, encuentre la clave privada d con
el algoritmo extendido de Euclides.
10. Por qu se usa una clave pblica e de un valor relativamente bajo
y, por contrapartida, la clave privada d es alta? Qu utilidad tiene?
11. Cmo atacara un sistema RSA mal diseado con primos cercanos
y cuyo mdulo es n = 205.027? Encuentre los valores de p y q.
Jorge Rami Aguirre Madrid (Espaa) 2003
Seguridad Informtica y Criptografa. Tema 12: Cifrado Asimtrico Exponencial
Diapositiva 605
Cuestiones y ejercicios (3 de 4)
12. En el sistema RSA con p = 11 y q = 19, se elige como clave pblica
e = 31. Cuntas y cules son las claves privadas parejas?
13. Para los mismos datos del ejercicio anterior, cuntos y cules son
los mensajes no cifrables? Y si ahora e = 33?
14. Cmo puede minimizarse el nmero de claves privadas parejas y el
de mensajes no cifrables? D un ejemplo con primos de dos dgitos.
15. Atacamos un sistema RSA con un cifrado cclico. Qu es lo que
vulneramos, el secreto de la clave privada o secreto del mensaje?
16. Se ataca por cifrado cclico el sistema RSA en el que p = 23, q = 41
y e = 17. Cuntos cifrados hay que hacer hasta encontrar M?
17. Cifre M = La Puerta de Alcal (de 19 caracteres ANSI) con un
sistema de Pohlig y Hellman. Use el primer primo que le permita
cifrar bloques de 2 bytes. Descifre ahora el criptograma C.
Jorge Rami Aguirre Madrid (Espaa) 2003
Seguridad Informtica y Criptografa. Tema 12: Cifrado Asimtrico Exponencial
Diapositiva 606
Cuestiones y ejercicios (4 de 4)
18. Vamos a cifrar con ElGamal el mensaje en ASCII decimal de la
letra A. Cules son los valores mnimos del cuerpo de cifra, del
generador , de la clave pblica y del valor local v?
19. Descifre el criptograma obtenido en el ejercicio anterior.
20. Se va a cifrar con RSA el mensaje M = Hola. Si p = 53 y q = 97, de
cuntos dgitos ser el bloque ptimo de cifra?
21. En un sistema real, tiene sentido hablar de bloques con un nmero
de dgitos ptimos? Por qu? Justifique su respuesta.
22. Cuntos pasos debe realizar una factorizacin de un nmero n de
120 dgitos? Si tenemos un sistema que consume 1 seg por paso,
cuntos aos tardaramos ms o menos en factorizar ese nmero?
23. El valor encontrado en el ejercicio anterior, es siempre fijo o es
slo una aproximacin? Justifique su respuesta.
Tema 13
Funciones Hash en Criptografa
Seguridad Informtica y Criptografa
Ultima actualizacin: 03/03/03
Archivo con 30 diapositivas
Jorge Rami Aguirre
Universidad Politcnica de Madrid
Material Docente de
Libre Distribucin
v 3.1
Este archivo forma parte de un curso sobre Seguridad Informtica y Criptografa. Se autoriza la
reproduccin en computador e impresin en papel slo con fines docentes o personales, respetando
en todo caso los crditos del autor. Queda prohibida su venta, excepto a travs del Departamento de
Publicaciones de la Escuela Universitaria de Informtica, Universidad Politcnica de Madrid, Espaa.
Curso de Seguridad Informtica y Criptografa JRA
Jorge Rami Aguirre Madrid (Espaa) 2003
Seguridad Informtica y Criptografa. Tema 13: Funciones Hash en Criptografa
Diapositiva 608
Uso de las funciones hash en criptografa
Una de las aplicaciones ms interesantes de la
criptografa es la posibilidad real de incluir en un
mensaje una firma digital. Todo esto comienza en el
ao 1976 cuando Diffie y Hellman presentan un
modelo de cifrado asimtrico con clave pblica. Con
los sistemas de clave simtrica esto era inviable.
No obstante, dado que los sistemas de clave pblica
son muy lentos, en vez de firmar digitalmente el
mensaje completo, en un sistema criptogrfico se
incluir como firma digital una operacin con la clave
privada sobre un resumen o hash de dicho mensaje
representado por slo una centena de bits.
Jorge Rami Aguirre Madrid (Espaa) 2003
Seguridad Informtica y Criptografa. Tema 13: Funciones Hash en Criptografa
Diapositiva 609
Funciones hash
Mensaje = M Funcin Resumen = H(M)
Firma (rbrica): r = E
dE
{H(M)}
dE es la clave privada del emisor que firmar H(M)
Cmo se comprueba la identidad en destino?
Se descifra la rbrica r con la clave pblica del
emisor eE. Al mensaje en claro recibido M (se
descifra si viene cifrado) se le aplica la misma
funcin hash que en emisin. Si los valores son
iguales, la firma es autntica y el mensaje ntegro:
Calcula: E
eE
(r) = H(M)
Compara: H(M) = H(M)?
Qu seguridad nos da
un resumen de k bits?
Jorge Rami Aguirre Madrid (Espaa) 2003
Seguridad Informtica y Criptografa. Tema 13: Funciones Hash en Criptografa
Diapositiva 610
Seguridad asociada a una funcin hash
Suponga que hemos creado una funcin hash de forma que el resumen es
slo de 4 bits, independientemente del tamao del mensaje de entrada.
La pregunta es: cul es la probabilidad de que dos mensajes distintos
tengan igual funcin hash? Si esta probabilidad fuese muy baja (en este
caso 1/16: hash desde 0000 hasta 1111) podra darse el siguiente caso:
alguien modifica nuestro mensaje firmado, y enva ese mensaje falso con
la firma del primero ya que en ambos casos son los mismos 4 bits...
Mensaje 1: Rechazamos el contrato por no interesarnos nada hash: 1101
Mensaje 2: Firma todo lo que te pongan porque nos interesa hash: 1101
Observe que ambos mensajes tienen 47 caracteres. As, podramos crear
una gran cantidad de mensajes diferentes que digan casi lo mismo, incluso
con igual nmero de caracteres... hasta que los dos hash coincidan!
Por este motivo las funciones hash para que sean interesantes en
criptografa, deben cumplir un conjunto de propiedades.
Jorge Rami Aguirre Madrid (Espaa) 2003
Seguridad Informtica y Criptografa. Tema 13: Funciones Hash en Criptografa
Diapositiva 611
Propiedades de las funciones hash
H(M) ser segura si tiene las siguientes caractersticas:
1. Unidireccionalidad. Conocido un resumen H(M), debe
ser computacionalmente imposible encontrar M a
partir de dicho resumen.
2. Compresin. A partir de un mensaje de cualquier
longitud, el resumen H(M) debe tener una longitud
fija. Lo normal es que la longitud de H(M) sea menor.
3. Facilidad de clculo. Debe ser fcil calcular H(M) a
partir de un mensaje M.
4. Difusin. El resumen H(M) debe ser una funcin
compleja de todos los bits del mensaje M. Si se
modifica un bit del mensaje M, el hash H(M) debera
cambiar aproximadamente la mitad de sus bits.
Jorge Rami Aguirre Madrid (Espaa) 2003
Seguridad Informtica y Criptografa. Tema 13: Funciones Hash en Criptografa
Diapositiva 612
Colisiones: resistencia dbil y fuerte
5. Colisin simple. Conocido M, ser
computacionalmente imposible encontrar otro M tal
que H(M) = H(M). Se conoce como resistencia dbil
a las colisiones.
6. Colisin fuerte. Ser computacionalmente difcil
encontrar un par (M, M) de forma que H(M) = H(M).
Se conoce como resistencia fuerte a las colisiones.
Ataque por la paradoja del cumpleaos
Para tener confianza en encontrar dos mensajes con el mismo
resumen H(M) -probabilidad 50%- no habr que buscar en
2
n
(p.e. 2
128
), bastar una bsqueda en el espacio 2
n/2
(2
64
).
La complejidad algortmica se reduce de forma drstica!
Jorge Rami Aguirre Madrid (Espaa) 2003
Seguridad Informtica y Criptografa. Tema 13: Funciones Hash en Criptografa
Diapositiva 613
La paradoja del cumpleaos
Problema: Cul ser la confianza (probabilidad > 50%) de que en un
aula con 365 personas -no se tiene en cuenta el da 29/02 de los aos
bisiestos- dos de ellas al azar cumplan aos en la misma fecha.
Solucin: Se escribe en la pizarra los 365 das del ao y entran al aula
de uno en uno, borrando el da de su cumpleaos de la pizarra. Para
alcanzar esa confianza, basta que entren 23 personas al aula, un valor
muy bajo, en principio inimaginable y de all el nombre de paradoja.
Explicacin: El primero en entrar tendr una probabilidad de que su
nmero no est borrado igual a n/n = 1, el segundo de (n-1)/n, etc. La
probabilidad de no coincidencia ser p
NC
= n!/(n-k)n
k
. Para k = 23 se
tiene p
NC
= 0,493 y as la probabilidad de coincidencia es p
C
= 0,507.
En nuestro caso, esto es equivalente a sacar dos mensajes de dos
conjunto disjuntos y comparar sus hash; si los hash no son iguales
sacamos otros dos mensajes, ...etc., hasta que haya una colisin.
Jorge Rami Aguirre Madrid (Espaa) 2003
Seguridad Informtica y Criptografa. Tema 13: Funciones Hash en Criptografa
Diapositiva 614
Algoritmos de resumen en criptografa
MD5: Ron Rivest 1992. Mejoras al MD4 y MD2 (1990), es ms lento
pero con mayor nivel de seguridad. Resumen de 128 bits.
SHA-1: Del NIST, National Institute of Standards and Technology,
1994. Similar a MD5 pero con resumen de 160 bits. Existen otras
nuevas propuestas conocidas como SHA-256 y SHA-512.
RIPEMD: Comunidad Europea, RACE, 1992. Resumen de 160 bits.
N-Hash: Nippon Telephone and Telegraph, 1990. Resumen: 128 bits.
Snefru: Ralph Merkle, 1990. Resmenes entre 128 y 256 bits. Ha sido
criptoanalizado y es lento.
Tiger: Ross Anderson, Eli Biham, 1996. Resmenes de hasta 192 bits.
Optimizado para mquinas de 64 bits (Alpha).
Panama: John Daemen, Craig Clapp, 1998. Resmenes de 256 bits de
longitud. Trabaja en modo funcin hash o como cifrador de flujo.
Haval: Yuliang Zheng, Josef Pieprzyk y Jennifer Seberry, 1992.
Admite 15 configuraciones diferentes. Hasta 256 bits.
Jorge Rami Aguirre Madrid (Espaa) 2003
Seguridad Informtica y Criptografa. Tema 13: Funciones Hash en Criptografa
Diapositiva 615
Message Digest 5, MD5
Algoritmo bsico MD5
a) Un mensaje M se convierte en un bloque mltiplo de 512
bits, aadiendo bits si es necesario al final del mismo.
b) Con los 128 bits de cuatro vectores iniciales ABCD de 32
bits cada uno y el primer bloque del mensaje de 512 bits, se
realizan diversas operaciones lgicas entre ambos bloques.
c) La salida de esta operacin (128 bits) se convierte en el
nuevo conjunto de 4 vectores ABCD y se realiza la misma
funcin con el segundo bloque de 512 bits del mensaje y
as hasta el ltimo bloque del mensaje.
d) Al terminar, el algoritmo entrega un resumen que
corresponde a los ltimos 128 bits de estas operaciones.
Jorge Rami Aguirre Madrid (Espaa) 2003
Seguridad Informtica y Criptografa. Tema 13: Funciones Hash en Criptografa
Diapositiva 616
Etapas de MD5
a) Aadir bits para congruencia mdulo 512, reservando
los ltimos 64 bits para un indicador de longitud.
b) Aadir indicacin de la longitud del mensaje en los 64
bits reservados.
c) Inicializar el vector ABCD de claves.
d) Procesar bloques de 512 bits, entregando una salida de
128 bits que formarn nuevamente el vector ABCD.
e) Obtener el resumen de los ltimos 128 bits.
Bloques funcionales de MD5
Esquema
Jorge Rami Aguirre Madrid (Espaa) 2003
Seguridad Informtica y Criptografa. Tema 13: Funciones Hash en Criptografa
Diapositiva 617
Esquema de la funcin MD5
Mensaje
1000...
Relleno de 1 a 448 bits
K mod 2
64
(64 bits)
Mensaje de K bits
K
L512 bits = N 32 bits
N palabras de 32 bits
Y
q
H
MD5
Y
L-1
H
MD5
RESUMEN
de 128 bits
ABCD
Y
1
Y
2
512 bits
A
16
= 01234567
B
16
= 89ABCDEF
C
16
= FEDCBA98
D
16
= 76543210
A
16
= 01234567
B
16
= 89ABCDEF
C
16
= FEDCBA98
D
16
= 76543210
H
MD5
H
MD5
Primer resumen
Jorge Rami Aguirre Madrid (Espaa) 2003
Seguridad Informtica y Criptografa. Tema 13: Funciones Hash en Criptografa
Diapositiva 618
Bloque principal de MD5
Qu hacen las funciones F y FF...?
Bloque principal
1
er
bloque de 512
bits del mensaje M
Vuelta
1
Funciones
F y FF
Vuelta
2
Funciones
G y GG
Vuelta
3
Funciones
H y HH
Vuelta
4
Funciones
I e II
+
+
+
+
Nuevo Vector ABCD
de 128 bits para el
prximo bloque
A
Vector
inicial
A
B
B
C
C
D
D
SUMA MDULO 2
32
+
Jorge Rami Aguirre Madrid (Espaa) 2003
Seguridad Informtica y Criptografa. Tema 13: Funciones Hash en Criptografa
Diapositiva 619
Esquema funciones F, G, H, I en MD5
a b c d b c d
Vector inicial ABCD
128 bits
A
16
= 01234567
B
16
= 89ABCDEF
C
16
= FEDCBA98
D
16
= 76543210
A
16
= 01234567
B
16
= 89ABCDEF
C
16
= FEDCBA98
D
16
= 76543210
funcin no lineal
x, y, z b, c, d
F (x, y, z)
(x AND y) OR (NOT x AND z)
G (x, y, z)
(x AND z) OR (y AND NOT z)
H (x, y, z)
x XOR y XOR z
I (x, y, z)
y XOR (x OR NOT z)
F (x, y, z)
(x AND y) OR (NOT x AND z)
G (x, y, z)
(x AND z) OR (y AND NOT z)
H (x, y, z)
x XOR y XOR z
I (x, y, z)
y XOR (x OR NOT z)
F (b, c, d)
(b AND c) OR (NOT b AND d)
G (b, c, d)
(b AND d) OR (c AND NOT d)
H (b, c, d)
b XOR c XOR d
I (b, c, d)
c XOR (b OR NOT d)
F (b, c, d)
(b AND c) OR (NOT b AND d)
G (b, c, d)
(b AND d) OR (c AND NOT d)
H (b, c, d)
b XOR c XOR d
I (b, c, d)
c XOR (b OR NOT d)
Jorge Rami Aguirre Madrid (Espaa) 2003
Seguridad Informtica y Criptografa. Tema 13: Funciones Hash en Criptografa
Diapositiva 620
Algoritmo de las funciones en MD5
a b c d b c a d
Desplazamiento del registro
+
+
+
32 bits
32 bits
Situacin luego del desplazamiento
funcin no lineal
Se repite el proceso para
M
j+1
hasta 16 bloques del
texto. En las vueltas 2, 3 y 4
se repite el proceso ahora
con funciones G, H e I.
M
j
t
j
El algoritmo realiza
416 = 64 vueltas
para cada uno de los
bloques de 512 bits
s
j
bits a la
izquierda
<<< s
j
+
+
Suma
mod 2
32
Jorge Rami Aguirre Madrid (Espaa) 2003
Seguridad Informtica y Criptografa. Tema 13: Funciones Hash en Criptografa
Diapositiva 621
Funciones no lineales en MD5
Vector de 128 bits
Funciones no lineales
en cada vuelta
a b c d
1 Vuelta:
FF(a,b,c,d,M
j
,t
j
,s) a = b + ((a + F(b,c,d) + M
j
+ t
j
) <<< s)
2 Vuelta:
GG(a,b,c,d,M
j
,t
j
,s) a = b + ((a + G(b,c,d) + M
j
+ t
j
) <<< s)
3 Vuelta:
HH(a,b,c,d,M
j
,t
j
,s) a = b + ((a + H(b,c,d) + M
j
+ t
j
) <<< s)
4 Vuelta:
II(a,b,c,d,M
j
,t
j
,s) a = b + ((a + I(b,c,d) + M
j
+ t
j
) <<< s)
Jorge Rami Aguirre Madrid (Espaa) 2003
Seguridad Informtica y Criptografa. Tema 13: Funciones Hash en Criptografa
Diapositiva 622
Algoritmo y desplazamiento en MD5
Vector de 128 bits
a b c d
Sea f la funcin F, G, H o I segn la vuelta.
El algoritmo ser:
Para j = 0 hasta 15 hacer:
TEMP = [(a + f(b,c,d) + M
j
+ t
j
) <<<s
j
]
a = d
d = c
c = b
b = a
a = TEMP
Jorge Rami Aguirre Madrid (Espaa) 2003
Seguridad Informtica y Criptografa. Tema 13: Funciones Hash en Criptografa
Diapositiva 623
Operaciones en 1 y 2 vueltas en MD5
FF (a, b, c, d, M
j
, t
j
, s)
FF(a, b, c, d, M
0
, D76AA478, 7)
FF(d, a, b, c, M
1
, E8C7B756, 12)
FF(c, d, a, b, M
2
, 242070DB, 17)
FF(b, c, d, a, M
3
, C1BDCEEE, 22)
FF(a, b, c, d, M
4
, F57C0FAF, 7)
FF(d, a, b, c, M
5
, 4787C62A, 12)
FF(c, d, a, b, M
6
, A8304613, 17)
FF(b, c, d, a, M
7
, FD469501, 22)
FF(a, b, c, d, M
8
, 698098D8, 7)
FF(d, a, b, c, M
9
, 8B44F7AF, 12)
FF(c, d, a, b, M
10
, FFFF5BB1, 17)
FF(b, c, d, a, M
11
, 895CD7BE, 22)
FF(a, b, c, d, M
12
, 6B901122, 7)
FF(d, a, b, c, M
13
, FD987193, 12)
FF(c, d, a, b, M
14
, A679438E, 17)
FF(b, c, d, a, M
15
, 49B40821, 22)
P
r
i
m
e
r
a

v
u
e
l
t
a
GG (a, b, c, d,
GG(a, b, c, d, M
GG(d, a, b, c, M
GG(c, d, a, b, M
GG(b, c, d, a, M
GG(a, b, c, d, M
GG(d, a, b, c, M
GG(c, d, a, b, M
GG(b, c, d, a, M
GG(a, b, c, d, M
GG(d, a, b, c, M
GG(c, d, a, b, M
GG(b, c, d, a, M
GG(a, b, c, d, M
GG(d, a, b, c, M
GG(c, d, a, b, M
GG(b, c, d, a, M
S
e
g
u
n
d
a

v
u
e
l
t
a
M
j
, t
j
, s)
1
, F61E2562, 5)
6
, C040B340, 9)
11
, 265E5A51, 14)
0
, E9B6C7AA, 20)
5
, D62F105D, 5)
10
, 02441453, 9)
15
, D8A1E681, 14)
4
, E7D3FBC8, 20)
9
, 21E1CDE6, 5)
14
, C33707D6, 9)
3
, F4D50D87, 14)
8
, 455A14ED, 20)
13
, A9E3E905, 5)
2
, FCEFA3F8, 9)
7
, 676F02D9, 14)
12
, 8D2A4C8A, 20)
Jorge Rami Aguirre Madrid (Espaa) 2003
Seguridad Informtica y Criptografa. Tema 13: Funciones Hash en Criptografa
Diapositiva 624
Operaciones en 3 y 4 vueltas en MD5
HH (a, b, c, d, M
j
, t
j
, s)
HH(a, b, c, d, M
5
, FFFA3942, 4)
HH(d, a, b, c, M
8
, 8771F681, 11)
HH(c, d, a, b, M
11
, 6D9D6122, 16)
HH(b, c, d, a, M
14
, FDE5380C, 23)
HH(a, b, c, d, M
1
, A4BEEA44, 4)
HH(d, a, b, c, M
4
, 4BDECFA9, 11)
HH(c, d, a, b, M
7
, F6BB4B60, 16)
HH(b, c, d, a, M
10
, BEBFBC70, 23)
HH(a, b, c, d, M
13
, 289B7EC6, 4)
HH(d, a, b, c, M
0
, EAA127FA, 11)
HH(c, d, a, b, M
3
, D4EF3085, 16)
HH(b, c, d, a, M
6
, 04881D05, 23)
HH(a, b, c, d, M
9
, D9D4D039, 4)
HH(d, a, b, c, M
12
, E6DB99E5, 11)
HH(c, d, a, b, M
15
, 1FA27CF8, 16)
HH(b, c, d, a, M
2
, C4AC5665, 23)
T
e
r
c
e
r
a

v
u
e
l
t
a
II (a, b, c, d,
II(a, b, c, d, M
II(d, a, b, c, M
II(c, d, a, b, M
II(b, c, d, a, M
II(a, b, c, d, M
II(d, a, b, c, M
II(c, d, a, b, M
II(b, c, d, a, M
II(a, b, c, d, M
II(d, a, b, c, M
II(c, d, a, b, M
II(b, c, d, a, M
II(a, b, c, d, M
II(d, a, b, c, M
II(c, d, a, b, M
II(b, c, d, a, M
C
u
a
r
t
a

v
u
e
l
t
a
M
j
, t
j
, s)
0
, F4292244, 6)
7
, 411AFF97, 10)
14
, AB9423A7, 15)
5
, FC93A039, 21)
12
, 655B59C3, 6)
3
, 8F0CCC92, 10)
10
, FFEFF47D, 15)
1
, 85845DD1, 21)
8
, 6FA87E4F, 6)
15
, FE2CE6E0, 10)
6
, A3014314, 15)
13
, 4E0811A1, 21)
4
, F7537E82, 6)
11
, BD3AF235, 10)
2
, 2AD7D2BB, 15)
9
, EB86D391, 21)
Jorge Rami Aguirre Madrid (Espaa) 2003
Seguridad Informtica y Criptografa. Tema 13: Funciones Hash en Criptografa
Diapositiva 625
Funcin de resumen SHA-1
Un resumen de 128 bits tiene una complejidad algortmica de
slo 2
64
, un valor en la actualidad muy comprometido...

La funcin SHA-1, Secure Hash Algorithm, entregar un
resumen de 160 bits una complejidad algortmica de 2
80
.

Vector Inicial : A = 67452301 B = EFCDAB89


C = 98BADCFE D = 10325476 E = C3D2E1F0
Esta forma de tomar los bits se ver ms adelante
SHA-1
Algoritmo:
Es muy similar a MD5. El vector inicial tiene una palabra ms
de 32 bits (E) por lo que el resumen ser de 160 bits. A cada
bloque de 512 bits del mensaje se le aplicarn 80 vueltas.
Jorge Rami Aguirre Madrid (Espaa) 2003
Seguridad Informtica y Criptografa. Tema 13: Funciones Hash en Criptografa
Diapositiva 626
Esquema del resumen SHA-1
Despus de esta ltima operacin, se
produce el desplazamiento del
registro hacia la derecha
+
+
+
W
t
K
t
32 bits
32 bits
A
16
= 67452301
B
16
= EFCDAB89
C
16
= 98BADCFE
D
16
= 10325476
E
16
= C3D2E1F0
A
16
= 67452301
B
16
= EFCDAB89
C
16
= 98BADCFE
D
16
= 10325476
E
16
= C3D2E1F0
+
<<< 5
Bloques del texto a partir
del bloque de 16 palabras
Una constante en cada
una de las cuatro vueltas
Funcin
no lineal
a b c d e c d b
Vector inicial ABCDE
Registro de 160 bits
<<< 30
Suma
mod 2
32
+
Vase la prxima diapositiva...
Jorge Rami Aguirre Madrid (Espaa) 2003
Seguridad Informtica y Criptografa. Tema 13: Funciones Hash en Criptografa
Diapositiva 627
Vueltas funciones F, G, H, I en SHA-1
ae ba c b dc e d
Desplazamiento del registro
Se repite el proceso con la
funcin F para las restantes 15
palabras de 32 bits del bloque
actual hasta llegar a 20. En
vueltas 2, 3 y 4 se repite el
proceso con funciones G, H e I.

Tenemos 420 = 80 pasos por cada bloque de 512 bits.
Pero ... cmo es posible repetir 80 veces un bloque que slo
cuenta con 16 bloques de texto de 32 bits cada uno?
F (b, c, d) vueltas t = 0 a 19
(b AND c) OR ((NOT b) AND d)
G (b, c, d) vueltas t = 20 a 39
b XOR c XOR d
H (b, c, d) vueltas t = 40 a 59
(b AND c) OR (b AND d) OR
(c AND d)
I (b, c, d) vueltas t = 60 a 79
b XOR c XOR d
(b AND c) OR ((NOT b) AND d)
G (b, c, d) vueltas t = 20 a 39
b XOR c XOR d
H (b, c, d) vueltas t = 40 a 59
(b AND c) OR (b AND d) OR
(c AND d)
b XOR c XOR d
F (b, c, d) vueltas t = 0 a 19
I (b, c, d) vueltas t = 60 a 79
Jorge Rami Aguirre Madrid (Espaa) 2003
Seguridad Informtica y Criptografa. Tema 13: Funciones Hash en Criptografa
Diapositiva 628
Las 80 vueltas en SHA-1
Vector de 160 bits
a b c d e
Cada bloque de 16 palabras del mensaje (M
0
... M
15
) se
expandir en 80 palabras (W
0
... W
79
) segn el algoritmo:
W
t
= M
t
(para t = 0, ..., 15)
W
t
= (W
t-3
W
t-8
W
t-14
W
t-16
) <<<1 (para t = 16, ..., 79)
y adems: K
t
= 5A827999 para t = 0, ..., 19
K
t
= 6ED9EBA1 para t = 20, ..., 39
K
t
= 8F1BBCDC para t = 40, ..., 59
K
t
= CA62C1D6 para t = 60, ..., 79
Jorge Rami Aguirre Madrid (Espaa) 2003
Seguridad Informtica y Criptografa. Tema 13: Funciones Hash en Criptografa
Diapositiva 629
Algoritmo y desplazamiento en SHA-1
Vector de 160 bits
a b c d e
El algoritmo para cada bloque de 512 bits ser:
Para t = 0 hasta 79 hacer:
TEMP = (a <<<5) + f
t
(b,c,d) + e + W
t
+ K
t
a = e
e = d
d = c
c = b <<<30
b = a
a = TEMP
Jorge Rami Aguirre Madrid (Espaa) 2003
Seguridad Informtica y Criptografa. Tema 13: Funciones Hash en Criptografa
Diapositiva 630
Comparativa entre MD5 y SHA-1
SHA-1 genera una salida de 160 bits de longitud mientras
que MD5 genera slo 128 bits.
La dificultad de generar un mensaje que tenga un resumen
dado es del orden de 2
128
operaciones para MD5 y 2
160
para
SHA-1.
La dificultad de generar dos mensajes aleatorios distintos y
que tengan el mismo resumen (ataques basados en paradoja
del cumpleaos) es del orden de 2
64
operaciones para MD5 y
2
80
para SHA-1.
Esta diferencia de 16 bits a favor de SHA-1 lo convierte en
ms seguro y resistente a ataques por fuerza bruta que el
algoritmo MD5. Aunque es ms lento que MD5, hoy es el
estndar como funcin hash.
Jorge Rami Aguirre Madrid (Espaa) 2003
Seguridad Informtica y Criptografa. Tema 13: Funciones Hash en Criptografa
Diapositiva 631
Pasos y tasas de cifra en MD5 y SHA-1
Ambos algoritmos procesan bloques de 512 bits y emplean 4
funciones primitivas para generar el resumen del mensaje, pero...
SHA-1 realiza un mayor nmero de pasos que MD5 (80 frente
a los 64 que realiza MD5).
SHA-1 debe procesar 160 bits de buffer en comparacin con
los 128 bits de MD5.
Por estos motivos la ejecucin del algoritmo SHA-1 es ms
lenta que la de MD5 usando un mismo hardware. Por ejemplo
en un Pentium a 266 MHz un programa realizado en C entrega
para SHA-1 una tasa del orden de 20 Mbits/seg y para MD5
esta tasa llega a los 60 Mbits/seg.
Jorge Rami Aguirre Madrid (Espaa) 2003
Seguridad Informtica y Criptografa. Tema 13: Funciones Hash en Criptografa
Diapositiva 632
Ms diferencias entre MD5 y SHA-1
La longitud mxima del mensaje para SHA-1 debe ser
menor de 2
64
bits, mientras que MD5 no tiene limitaciones
de longitud.
MD5 emplea 64 constantes (una por cada paso), mientras
que SHA-1 slo emplea 4 (una para cada 20 pasos).
MD5 se basa en la arquitectura little-endian, mientras que
SHA-1 se basa en la arquitectura big-endian. Por ello el
vector ABCD inicial en MD5 y SHA-1 son iguales:
A = 01234567 (MD5) 67452301 (SHA-1)
B = 89ABCDEF (MD5) EFCDAB89 (SHA-1)
C = FEDCBA98 (MD5) 98BADCFE (SHA-1)
D = 76543210 (MD5) 10325476 (SHA-1)
Jorge Rami Aguirre Madrid (Espaa) 2003
Seguridad Informtica y Criptografa. Tema 13: Funciones Hash en Criptografa
Diapositiva 633
Arquitecturas little-endian v/s big-endian
Arquitectura little-endian:
Esta es la arquitectura empleada en procesadores Intel de la
familia 80xxx y Pentium.
Para almacenar una palabra en memoria, el byte menos
significativo de los que forman dicha palabra se guarda en
la posicin ms baja de la memoria.
Arquitectura big-endian:
Empleada por otras arquitecturas como SUN.
Para almacenar una palabra en memoria, el byte ms
significativo de los que forman dicha palabra se guarda en
la posicin ms baja de memoria.
Ejemplo
Jorge Rami Aguirre Madrid (Espaa) 2003
Seguridad Informtica y Criptografa. Tema 13: Funciones Hash en Criptografa
Diapositiva 634
Ejemplo little-endian v/s big-endian
Supongamos que queremos almacenar en memoria la siguiente
palabra de 32 bits (4 bytes) representada en hexadecimal:
76543210
10 32 54 76
Si consideramos que las posiciones de memoria ms bajas se
encuentran a la izquierda y las ms altas a la derecha:
67 45 23 01 En formato little-endian se representa:
01 23 45 67 En formato big-endian se representa:
Jorge Rami Aguirre Madrid (Espaa) 2003
Seguridad Informtica y Criptografa. Tema 13: Funciones Hash en Criptografa
Diapositiva 635
Funciones hash para la autenticacin
Las funciones hash vistas (MD5, SHA-1, etc.) pueden usarse
adems para autenticar a dos usuarios.
Como carecen de una clave privada no pueden usarse de
forma directa para estos propsitos. No obstante, existen
algoritmos que permiten incluirles esta funcin.
Entre ellos est HMAC, una funcin que usando los hash
vistos y una clave secreta, autentica a dos usuarios mediante
sistemas de clave secreta. Las funciones MAC, Message
Authentication Code, y HMAC se tratarn en el prximo
captulo dedicado a la autenticacin y firma digital.
HMAC se usa en plataformas IP seguras como por ejemplo en
Secure Socket Layer, SSL.
Fin del Tema 13
Jorge Rami Aguirre Madrid (Espaa) 2003
Seguridad Informtica y Criptografa. Tema 13: Funciones Hash en Criptografa
Diapositiva 636
Cuestiones y ejercicios
1. Qu propiedades debe tener una funcin hash para que su uso en
criptografa sea de inters? Vale cualquier funcin reductora?
2. Por qu prospera un ataque basado en la paradoja del cumpleaos
con un tiempo significativamente menor que un ataque elemental?
3. Se va a aplicar la funcin MD5 a un mensaje de longitud 250 bytes.
Cmo se rellena y cmo queda el ltimo bloque?
4. Por qu razn decimos que la funcin SHA-1 es actualmente un
estndar ms seguro que la funcin MD5?
5. Cmo puede la funcin SHA-1 hacer 80 vueltas con bloques de 32
bits partiendo de un bloque de texto o mensaje de slo 512 bits?
6. Qu funcin hash es ms rpida, MD5 o SHA-1? Por qu?
7. Si en un mensaje cambiamos un bit, en cunto debera cambiar su
hash aproximadamente con respecto a su resumen anterior?
Tema 14
Autenticacin y Firma Digital
Seguridad Informtica y Criptografa
Ultima actualizacin: 03/03/03
Archivo con 50 diapositivas
Jorge Rami Aguirre
Universidad Politcnica de Madrid
Material Docente de
Libre Distribucin
v 3.1
Este archivo forma parte de un curso sobre Seguridad Informtica y Criptografa. Se autoriza la
reproduccin en computador e impresin en papel slo con fines docentes o personales, respetando
en todo caso los crditos del autor. Queda prohibida su venta, excepto a travs del Departamento de
Publicaciones de la Escuela Universitaria de Informtica, Universidad Politcnica de Madrid, Espaa.
Curso de Seguridad Informtica y Criptografa JRA
Jorge Rami Aguirre Madrid (Espaa) 2003
Seguridad Informtica y Criptografa. Tema 14: Autenticacin y Firma Digital
Diapositiva 638
Confidencialidad v/s integridad
Confidencialidad
Para lograrla se cifra el mensaje M obteniendo un
criptograma C.
Integridad
Para lograrla se firma un hash del mensaje H(M),
aadiendo una marca al mensaje o criptograma.
Si bien en ciertos escenarios es muy importante
mantener el secreto de la informacin, si sta lo
requiere, en muchos casos tiene quizs ms
trascendencia el poder certificar la autenticidad
entre cliente y servidor como ocurre en Internet.
Jorge Rami Aguirre Madrid (Espaa) 2003
Seguridad Informtica y Criptografa. Tema 14: Autenticacin y Firma Digital
Diapositiva 639
Algunos problemas de integridad
a) Autenticidad del emisor
Cmo comprueba Benito (B) que el mensaje recibido del emisor
que dice ser Adela (A) es efectivamente de esa persona?
b) Integridad del mensaje
Cmo comprueba Benito (B) que el mensaje recibido del emisor
Adela (A) es el autntico y no un mensaje falso?
c) Actualidad del mensaje
Cmo comprueba Benito (B) que el mensaje recibido del emisor
Adela (A) es actual, no un mensaje de fecha anterior reenviado?
Jorge Rami Aguirre Madrid (Espaa) 2003
Seguridad Informtica y Criptografa. Tema 14: Autenticacin y Firma Digital
Diapositiva 640
Ms problemas de integridad
d) No repudio del emisor
Cmo comprueba Benito (B) que el mensaje enviado por el emisor
Adela (A) -y que niega haberlo enviado- efectivamente ha llegado?
e) No repudio del receptor
Cmo comprueba Benito (B) que el mensaje enviado al receptor
Adela (A) -y que niega haberlo recibido- efectivamente se envi?
d) Usurpacin de identidad del emisor/receptor
Cmo comprueba Benito (B) que Adela (A), Carmela (C) u otros
usuarios estn enviando mensajes firmados como Benito (B)?
Jorge Rami Aguirre Madrid (Espaa) 2003
Seguridad Informtica y Criptografa. Tema 14: Autenticacin y Firma Digital
Diapositiva 641
Primer escenario de integridad
A enva un mensaje M a B:
A cifra M con la clave K
A

E
KA
(M) y lo enva al Juez.
Este comprueba la integridad
de A, lo descifra y enva a B,
cifrado con K
B
, el mensaje M,
la identidad de A y la firma
E
KA
(M): E
KB
{M, A, E
KA
(M)}.
Ambos confan en el Juez y
ante cualquier duda ste puede
desvelar la identidad de A
descifrando E
KA
(M).
1
er
escenario de desconfianza
1 Solucin. Uso de un Juez.
El Juez tendr una clave K
A
con la que se comunica con
A y una clave K
B
con la que
se comunica con B.
Usa criptografa simtrica
Jorge Rami Aguirre Madrid (Espaa) 2003
Seguridad Informtica y Criptografa. Tema 14: Autenticacin y Firma Digital
Diapositiva 642
Segundo escenario de integridad
2 escenario de desconfianza
No est claro que pueda
convertirse en un estndar.
Permanece la figura de la
Autoridad de Certificacin
... es decir
2 Solucin. Prescindir de la
figura del Juez y aceptar la
autenticidad e integridad por
convencimiento propio y la
confianza en los algoritmos.
Ambos confiarn en un protocolo seguro
y se autenticarn a travs de una
Autoridad de Certificacin AC.
Ambos confiarn en un protocolo seguro
y se autenticarn a travs de una
Autoridad de Certificacin AC.
Jorge Rami Aguirre Madrid (Espaa) 2003
Seguridad Informtica y Criptografa. Tema 14: Autenticacin y Firma Digital
Diapositiva 643
Funciones de autenticacin
Autenticacin mediante el cifrado de mensajes con
criptografa simtrica
La cifra de datos puede servir como autenticacin.
Autenticacin con MAC Message Code Authentication o
checksum
Una funcin pblica y una clave secreta producen un valor
de longitud fija vlida como autenticador.
Autenticacin mediante funciones hash
Una funcin pblica reduce el mensaje a una longitud de
valor hash que sirve como autenticador.
Autenticacin mediante firma digital del mensaje con
criptografa asimtrica
Una funcin pblica y un par de claves, pblica y privada
inversas en un cuerpo, permiten la autenticacin completa.
Jorge Rami Aguirre Madrid (Espaa) 2003
Seguridad Informtica y Criptografa. Tema 14: Autenticacin y Firma Digital
Diapositiva 644
Autenticacin con sistemas simtricos
Si la clave de un sistema simtrico es segura, es decir no
est en entredicho, podemos afirmar que, adems de la
confidencialidad que nos entrega dicha cifra, se
obtienen tambin simultneamente la integridad del
emisor y autenticidad del mensaje, en tanto que slo el
usuario emisor (en quien se confa por el modelo de
cifra) puede generar ese mensaje.
Con los sistemas de cifra simtricos no podremos
realizar una autenticacin completa (emisor y mensaje).
Ahora bien, podremos hacer algo similar con algunos
procedimientos ms o menos complejos que usen
sistemas de cifra simtrica como es el caso de Kerberos.
Jorge Rami Aguirre Madrid (Espaa) 2003
Seguridad Informtica y Criptografa. Tema 14: Autenticacin y Firma Digital
Diapositiva 645
Los problemas de la autenticacin simtrica
No obstante, subyacen los problemas caractersticos de
un criptosistema: cmo asegurar que la clave simtrica
entre emisor y receptor es segura? o lo que es lo mismo,
cmo intercambiar claves de forma segura?
El intercambio de claves de forma segura ya hemos visto que
se logra eficientemente slo a travs de sistemas asimtricos.
Las herramientas ms usuales para autenticacin sern los
cdigos de autenticacin de mensajes MACs y el sistema
Kerberos con cifras simtricas. Kerberos tambin permite el
intercambio seguro de una clave de sesin aunque es ms
complejo y largo que el algoritmo de Diffie Hellman.
Jorge Rami Aguirre Madrid (Espaa) 2003
Seguridad Informtica y Criptografa. Tema 14: Autenticacin y Firma Digital
Diapositiva 646
Autenticacin con MAC o Checksum
Los dos extremos de la comunicacin A y B comparten
una nica clave secreta que no est en entredicho.
El MAC o checksum ser una funcin que se aplica al
mensaje M junto a una clave secreta K C
K
(M).
A enva el mensaje en claro y el Messsage Authentication
Code (MAC) o Checksum C
K
(M) a B.
Integridad: el receptor B puede estar seguro de que nadie
ha modificado el mensaje durante la transmisin pues el
valor C
K
(M) en destino coincide con el enviado por A.
Autenticacin: como solamente el emisor A y el receptor B
comparten la clave secreta K, se asegura la autenticacin
de ambos usuarios la clave K debe ser muy segura.
Jorge Rami Aguirre Madrid (Espaa) 2003
Seguridad Informtica y Criptografa. Tema 14: Autenticacin y Firma Digital
Diapositiva 647
Message Authentication Code con DES
Se inserta un cdigo al final del mensaje M transmitido en
claro, consistente en la cifra con clave secreta de los
ltimos bytes del texto, por ejemplo 64 bits de DES.
En destino, con el mismo algoritmo y clave secreta, se
realiza la cifra y se comparan estos ltimos bloques.
Como la cifra es por encadenamiento o realimentacin,
esos bytes cifrados dependen de todo el mensaje por lo que
cualquier modificacin ser detectada al no coincidir los
resultados del cifrado de M en emisor y receptor.
Esquema
Jorge Rami Aguirre Madrid (Espaa) 2003
Seguridad Informtica y Criptografa. Tema 14: Autenticacin y Firma Digital
Diapositiva 648
Esquema de autenticacin MAC con DES
Mensaje M
Cifrado del ltimo bloque con encadenamiento
M
1
M
2
M
n
... E (K,M)
M
1
M
2
M
n
...
receptor
Am
emisor
K
E (K,M) = Am M
R
M
R
: espacio de marcas de autenticacin
Cules son las
debilidades de
este modelo?
La Marca Am son 16, 32 64 bits.
Es un tamao muy pequeo y podra
dar lugar a colisiones: mensajes
distintos con iguales MACs.
Jorge Rami Aguirre Madrid (Espaa) 2003
Seguridad Informtica y Criptografa. Tema 14: Autenticacin y Firma Digital
Diapositiva 649
Autenticacin con funciones hash HMAC
Las funciones hash vistas (MD5, SHA-1, etc.) no han sido
diseadas para la autenticacin al carecer de clave secreta.
No obstante, son interesantes puesto que su velocidad es mayor
que muchos cifradores de bloque, su cdigo fuente es abierto y
sus propiedades y funcionamiento son muy conocidos.
La RFC 2104 propone el uso de una autenticacin en entornos
seguros como SSL mediante una operacin MAC en la que
intervenga una funcin hash: su nombre es HMAC.
HMAC usa entonces una funcin hash (MD5, SHA-1, etc.)
como una caja negra, una clave K en lo posible mayor que el
tamao del hash en bits, una funcin xor y operaciones de
relleno de bits, tal como se muestra en el siguiente esquema.
Jorge Rami Aguirre Madrid (Espaa) 2003
Seguridad Informtica y Criptografa. Tema 14: Autenticacin y Firma Digital
Diapositiva 650
Esquema de HMAC

ipad K
+
b bits
Hash
n bits
IV Relleno hasta b bits
S
0
n bits
IV
Hash HMAC
K
(M)
S
1
Y
0
Y
1
Y
L-1

opad K
+
b bits
b bits
H(S
i
||M)
n bits
n bits
Parmetros
esquema en
la siguiente
diapositiva.
Y
i
bloques del mensaje
Jorge Rami Aguirre Madrid (Espaa) 2003
Seguridad Informtica y Criptografa. Tema 14: Autenticacin y Firma Digital
Diapositiva 651
Parmetros, valores tpicos y salida HMAC
M = mensaje de entrada incluyendo el relleno.
H = alguna funcin hash como MD5 (128 bits) o SHA-1 (160 bits).
Y
i
= bloque isimo de M.
L = nmero de bloques en M.
b = nmero de bits en cada bloque (512).
n = longitud del resumen del hash ocupado en el sistema (128 / 160 bits).
K = clave secreta (160 bits) aunque se recomienda sea mayor que n. Si la
clave K es mayor que b, esta clave se hace pasar antes por la funcin hash
para reducirla a una clave de n bits.
K
+
= clave K con ceros aadidos a la izquierda para alcanzar b bits.
ipad = 00110110 octeto repetido b/8 (64) veces.
opad = 01011010 octeto repetido b/8 (64) veces.
Salida HMAC: HMAC
K
= H{(K
+
opad) || H[(K
+
ipad) || M ]}
Jorge Rami Aguirre Madrid (Espaa) 2003
Seguridad Informtica y Criptografa. Tema 14: Autenticacin y Firma Digital
Diapositiva 652
Operaciones y seguridad de HMAC
Aadir ceros a la izquierda de K hasta obtener K
+
, una cadena de b bits.
Sumar or exclusivo K
+
con la cadena ipad para obtener S
1
de b bits.
Aadir a S
1
el mensaje M como bloques Y
i
de b bits cada uno.
Aplicar la funcin hash elegida a la cadena de bits antes formada, con el
vector inicial IV de n bits para generar un hash de n bits.
Sumar or exclusivo K
+
con la cadena opad para obtener S
0
de b bits.
Aadir a S
0
el hash anterior, rellenando este ltimo hasta b bits.
Aplicar la funcin hash elegida a los dos bloques de b bits generados en
el paso anterior, con vector IV de n bits para generar un hash de n bits.
El resultado de este ltimo hash es el HMAC del mensaje M con la
clave K, es decir HMAC
K
(M).
Aunque la seguridad de HMAC est directamente relacionada con el hash
utilizado, el modelo presentado no es seguro. Hay otras configuraciones
ms desarrolladas que mejoran esta caracterstica.
Jorge Rami Aguirre Madrid (Espaa) 2003
Seguridad Informtica y Criptografa. Tema 14: Autenticacin y Firma Digital
Diapositiva 653
Autenticacin con cifra simtrica y un KDC
Caractersticas
Utilizacin de un KDC (Key Distribution Centre) o
Distribuidor de Claves de Confianza, que comparte una
clave llamada maestra con los clientes.
Cada parte, usuario o entidad de la red comparte una clave
secreta y nica o clave maestra con el KDC.
El KDC se ocupa de distribuir una clave de sesin que va a
ser utilizada en la conexin entre dos partes.
La clave de sesin se protege con la clave maestra de los
participantes de una comunicacin.
Jorge Rami Aguirre Madrid (Espaa) 2003
Seguridad Informtica y Criptografa. Tema 14: Autenticacin y Firma Digital
Diapositiva 654
Propuesta de Needham y Schroeder (1978)
1. A KDC: ID
A
||ID
B
||N
1
2. KDC A: E
KA
[K
S
||ID
B
||N
1
||E
KB
[K
S
||ID
A
]]
3. A B: E
KB
[K
S
||ID
A
]
4. B A: E
KS
[N
2
]
5. A B: E
KS
[f (N
2
)]
f (N2) es una funcin
conocida por A y B.
Qu sucede si no se
realizan los pasos 4 y 5?
ID
A
= Nombre de A o identificador de A
ID
B
= Nombre de B o identificador de B
N
X
= Valor nonce
K
S
= Clave de sesin
E
KX
= Cifrado con clave secreta de X
K
X
= Clave secreta de X (A B)
Un intruso podra capturar
el mensaje en el paso 3 y
repetirlo, interfiriendo as
las operaciones de B.
Jorge Rami Aguirre Madrid (Espaa) 2003
Seguridad Informtica y Criptografa. Tema 14: Autenticacin y Firma Digital
Diapositiva 655
Ms debilidades de la propuesta N-S
Algo ms improbable es que un intruso X tenga una clave
de sesin antigua y repita el mensaje del paso 3 enviado a
B, quien no tiene porqu recordar todas las claves de
sesin usadas previamente con A.
Si X captura el mensaje del paso 4, podr suplantar la
respuesta de A del paso 5, enviando a B mensajes que
parecen venir de A con clave de sesin autenticada.
Denning propone la inclusin de un valor timestamp en los
pasos 2 y 3. La utilizacin de timestamps requiere la
sincronizacin de relojes, pero la utilizacin de valores
nonce es tambin vulnerable. Se propone como solucin
ptima dar una duracin a la clave de sesin (tickets).
Jorge Rami Aguirre Madrid (Espaa) 2003
Seguridad Informtica y Criptografa. Tema 14: Autenticacin y Firma Digital
Diapositiva 656
Solucin propuesta por Denning y tickets
1. A KDC: ID
A
||ID
B
2. KDC A: E
KA
[K
S
||ID
B
||T||E
KB
[K
S
||ID
A
||T]]
3. A B: E
KB
[K
S
||ID
A
||T]
4. B A: E
KS
[N
1
]
5. A B: E
KS
[f (N
1
)]
Denning
T = Timestamp
| Clock t | < t
1
+ t
2
1. A B: ID
A
||N
A
2. B KDC: ID
B
||N
B
||E
KB
[ID
A
||N
A
||T
B
]
3. KDC A: E
KA
[ID
B
||N
A
||K
S
||T
B
]||E
KB
[ID
A
||K
S
||T
B
]||N
B
4. A B: E
KB
[ID
A
||K
S
||T
B
]||E
KS
[N
B
]
Tickets
Tickets
Solucin al problema del timestamp
Jorge Rami Aguirre Madrid (Espaa) 2003
Seguridad Informtica y Criptografa. Tema 14: Autenticacin y Firma Digital
Diapositiva 657
Autenticacin en un sentido
El correo electrnico es un ejemplo de aplicacin que
requiere este tipo de autenticacin:
No es necesario que el emisor y el receptor estn conectados
al mismo tiempo.
El receptor necesita confirmar de alguna forma que el emisor
del mensaje es quien dice ser.
1. A KDC: ID
A
||ID
B
||N
1
2. KDC A: E
KA
[K
S
||ID
B
||N
1
||E
KB
[K
S
||ID
A
]]
3. A B: E
KB
[K
S
||ID
A
]||E
KS
[M]
Se garantiza as que slo el receptor puede leer el mensaje
y autentica adems al emisor. Es vulnerable a repeticiones.
Jorge Rami Aguirre Madrid (Espaa) 2003
Seguridad Informtica y Criptografa. Tema 14: Autenticacin y Firma Digital
Diapositiva 658
Autenticacin con Kerberos (1988)
Servicio de autenticacin
desarrollado en el Massachusetts
Institute of Technology (MIT)
Perro de tres cabezas y cola de serpiente segn mitologa
griega, guardin de la entrada del Templo de Hades.
Tres componentes guardarn la puerta: Autenticacin,
Contabilidad y Auditora. Las dos ltimas cabezas nunca
han sido implementadas.
Jorge Rami Aguirre Madrid (Espaa) 2003
Seguridad Informtica y Criptografa. Tema 14: Autenticacin y Firma Digital
Diapositiva 659
Caractersticas y fases de Kerberos
Est basado en el protocolo de distribucin de
claves de Needham & Schroeder.
Usa un intercambio de claves con una tercera
parte de confianza.
Fases de autenticacin:
Obtencin de credenciales
Tickets
Autenticadores
Peticin de autenticacin frente un servicio.
Presentacin de las credenciales al servidor final.
Jorge Rami Aguirre Madrid (Espaa) 2003
Seguridad Informtica y Criptografa. Tema 14: Autenticacin y Firma Digital
Diapositiva 660
Elementos del dilogo de autenticacin
Usuario, cliente y servidor: persona o mquina que necesita
autenticarse en la red.
Principal: entidad o cliente que usa Kerberos y que ha sido
previamente autenticado por un servidor de autenticacin.
Servicio y servidor: servicio es una entidad abstracta (por
ejemplo correo) y servidor el proceso que lo ejecuta.
Clave y password: funcin aplicada a la clave de usuario.
Credenciales: lo que se utiliza para autenticarse.
Maestros y esclavos: la mquina que hospeda la base de
datos es el master y esclavos son las mquinas que poseen
copias de sta.
Dominio: nombre de entidad administrativa que mantiene
los datos de autenticacin.
Jorge Rami Aguirre Madrid (Espaa) 2003
Seguridad Informtica y Criptografa. Tema 14: Autenticacin y Firma Digital
Diapositiva 661
El ticket en la credencial de Kerberos
Existen dos tipos de credenciales utilizadas en el modelo de
Kerberos: tickets y autenticadores.
Es necesario un ticket para pasar de una forma segura la
identidad del cliente entre el servidor de autenticacin y el
servidor final.
[s, c, addr, timestamp, life, K
S,C
]K
S
s = nombre del servidor
c = nombre del cliente
addr = direccin Internet del cliente
timestamp = fecha de iniciacin del ticket
life = duracin
K
S
= clave de sesin aleatoria
Jorge Rami Aguirre Madrid (Espaa) 2003
Seguridad Informtica y Criptografa. Tema 14: Autenticacin y Firma Digital
Diapositiva 662
El autenticador en la credencial de Kerberos
El autenticador contiene informacin adicional que,
comparada con el ticket, prueba que el cliente que presenta
el ticket es el mismo a quien se le entreg,
[c, addr, timestamp]K
S,C
c = nombre del cliente
addr = direccin Internet de la estacin de trabajo
timestamp = fecha de iniciacin del ticket
K
S,C
= clave de sesin que es parte del ticket
Los objetivos de las credenciales son minimizar el nmero de veces
que un usuario tiene que introducir la password as como eliminar el
problema de enviar la password en texto plano por la red.
Jorge Rami Aguirre Madrid (Espaa) 2003
Seguridad Informtica y Criptografa. Tema 14: Autenticacin y Firma Digital
Diapositiva 663
Ejemplo de autenticacin con Kerberos (1)
Se autenticar el usuario A ante el usuario B teniendo
como tercera parte de confianza al KDC.
Usarn el algoritmo DES.
A B
KDC
k
A
k
B
Paso 1. A pide una credencial a KDC.
A KDC ID(A), ID(B), NA
Enva el nmero aleatorio NA para que
nadie pueda suplantar su identidad.
Paso 2. KDC genera una clave de sesin K
S
con perodo de validez L
y una credencial c para el usuario B.
m = E
kA
[K
S
, L, NA, ID(B)]; c = E
kB
[K
S
, L, ID(A)]
KDC A (m, c)
Jorge Rami Aguirre Madrid (Espaa) 2003
Seguridad Informtica y Criptografa. Tema 14: Autenticacin y Firma Digital
Diapositiva 664
Ejemplo de autenticacin con Kerberos (2)
Paso 3. A descifra con su clave k
A
el valor m:
D
kA
[K
S
, L, NA, ID(B)] = K
S
, L, NA, ID(B)
Y luego con la clave de sesin entregada por KDC generar un
autenticador a para el usuario B junto con un sello temporal T
A
.
a = E
KS
[ID(A), T
A
]
A B (c, a)
Paso 4. B descifra con su clave k
B
c:
D
kB
[K
S
, L, ID(A)] = K
S
, L, ID(A)
Ahora que tiene K
S
descifra el valor a:
D
KS
[ID(A), T
A
] = ID(A), T
A
Comprueba que coinciden los identificadores ID(A) del usuario A
y que T
A
est dentro del rango de validez L dado por KDC.
Jorge Rami Aguirre Madrid (Espaa) 2003
Seguridad Informtica y Criptografa. Tema 14: Autenticacin y Firma Digital
Diapositiva 665
Ejemplo de autenticacin con Kerberos (3)
Paso 5. B calcula una funcin acordada con A por ejemplo (T
A
+ 1)
y con la clave de sesin K
S
se lo enva cifrado.
B A h = E
KS
[T
A
+ 1]
Paso 6. A descifra h con la clave de sesin K
S
:
D
KS
[T
A
+ 1] = T
A
+ 1
Comprueba que coincide con lo acordado lo que le demuestra que
B ha recibido correctamente la clave de sesin K
S
.
El valor de T
A
permite ms autenticaciones dentro del perodo de
validez L sin la intervencin de la tercera parte de confianza KDC.
Jorge Rami Aguirre Madrid (Espaa) 2003
Seguridad Informtica y Criptografa. Tema 14: Autenticacin y Firma Digital
Diapositiva 666
Resumen del ejemplo de autenticacin
Los valores de c y m aseguran la confidencialidad en
la transmisin de la clave de sesin K
S
.
Los valores de a y h aseguran la confirmacin de la
recepcin correcta de la clave de sesin K
S
por B.
En este protocolo slo se autentica A ante B.
Existen extensiones del algoritmo que permiten una
autenticacin doble entre A y B.
Otra opcin es que los usuarios A y B compartan una
clave K
S
sin que su valor sea conocido por KDC.
Jorge Rami Aguirre Madrid (Espaa) 2003
Seguridad Informtica y Criptografa. Tema 14: Autenticacin y Firma Digital
Diapositiva 667
El hash en la autenticacin asimtrica
H(M) es el resultado de un algoritmo que con una entrada
M de cualquier tamao, produce salida de tamao fijo.
El emisor A enva el mensaje M y la funcin hash H(M) a
B, quien aplica la misma funcin al mensaje M recibido.
Si los mensajes son iguales entonces se asegura que los
hash tambin son iguales. No obstante, el hecho de que los
hash coincidan no significa que los mensajes M y M sean
iguales (ya visto en el captulo correspondiente).
Integridad: el receptor B puede confiar en que nadie ha
modificado el mensaje durante la transmisin pues el valor
H(M) en destino coincide con el enviado por A.
Autenticacin: Es imposible la autenticacin de usuario,
salvo que se use un modelo con clave tipo HMAC ya visto.
Jorge Rami Aguirre Madrid (Espaa) 2003
Seguridad Informtica y Criptografa. Tema 14: Autenticacin y Firma Digital
Diapositiva 668
Autenticacin con sistemas asimtricos
Al existir una clave pblica y otra privada que
son inversas, se autentica el mensaje y al emisor.
Permite la firma digital, nica para cada mensaje
Problema:
Los sistemas de cifra asimtricos son muy lentos y el mensaje
podra tener miles o millones de bytes ...
Solucin:
Se genera un resumen del mensaje, representativo del mismo, con
una funcin hash imposible de invertir. La funcin hash comprime
un mensaje de longitud variable a uno de longitud fija y pequea.
Jorge Rami Aguirre Madrid (Espaa) 2003
Seguridad Informtica y Criptografa. Tema 14: Autenticacin y Firma Digital
Diapositiva 669
Caractersticas de una firma digital
Requisitos de la Firma Digital:
a) Debe ser fcil de generar.
b) Ser irrevocable, no rechazable por su propietario.
c) Ser nica, slo posible de generar por su propietario.
d) Ser fcil de autenticar o reconocer por su propietario y los
usuarios receptores.
e) Debe depender del mensaje y del autor.
Esta ltima
propiedad es
muy importante!
Son condiciones ms fuertes
que la de una firma manuscrita.
Son condiciones ms fuertes
que la de una firma manuscrita.
Jorge Rami Aguirre Madrid (Espaa) 2003
Seguridad Informtica y Criptografa. Tema 14: Autenticacin y Firma Digital
Diapositiva 670
Firma digital RSA de A hacia B
Clave Pblica (n
A
, e
A
) Clave Privada (d
A
)
Rbrica: r
A
H(M) = H(M)
dA
mod n
A
Algoritmo:
A enva el mensaje M en claro (o cifrado) al destinatario B
junto a la rbrica: {M, r
A
H(M)}
El destinatario B tiene la clave pblica e
A
,n
A
de
A y descifra r
A
H(M) {(H(M)
dA
)
eA
mod n
A
}
obteniendo as H(M). Como recibe el mensaje
M, calcula la funcin hash H(M) y compara:
Si H(M) = H(M) se acepta la firma.

Benito
Adela
Jorge Rami Aguirre Madrid (Espaa) 2003
Seguridad Informtica y Criptografa. Tema 14: Autenticacin y Firma Digital
Diapositiva 671
Valores y tamaos tpicos de firmas
En los siguientes ejemplos, por limitacin del tamao de los
primos elegidos, se firmarn slo bloques de una cantidad
determinada de bits en funcin del cuerpo de trabajo.
No obstante, en los sistemas reales esto no es as puesto que
las funciones hash ya vistas entregarn -por lo general-
resmenes comprendidos entre 128 y 160 bits y, por otra
parte, el cuerpo de trabajo de la cifra asimtrica para la
firma digital ser como mnimo de 512 bits (si bien en la
actualidad se recomienda al menos 1.024). Por lo tanto el
resumen que se firma es menor que el cuerpo de cifra o, lo
que es lo mismo, es parte del conjunto de restos del grupo.
Jorge Rami Aguirre Madrid (Espaa) 2003
Seguridad Informtica y Criptografa. Tema 14: Autenticacin y Firma Digital
Diapositiva 672
Ejemplo de firma digital RSA (B A)
Hola. Te envo el documento. Saludos, Beni.
Sea H(M) = F3A9 (16 bits)
Claves Benito
n
B
= 65.669
e
B
= 35, d
B
= 53.771
2
16
< 65.669 < 2
17
Forzaremos firmar
bloques de 16 bits
Benito Adela
Claves Adela
n
A
= 66.331
e
A
= 25, d
A
= 18.377
Firma
H(M) = F3A9
16
= 62.377
10
r
H(M)
= H(M)
dB
mod n
B
r
H(M)
= 62.377
53.771
mod 65.669 = 24.622
Benito enva el par (M, r) = (M, 24.622)
Nota: los primos
que usa Benito
son 97, 677 y
Adela 113, 587
Jorge Rami Aguirre Madrid (Espaa) 2003
Seguridad Informtica y Criptografa. Tema 14: Autenticacin y Firma Digital
Diapositiva 673
Comprobacin la firma RSA por A
Claves Benito
n
B
= 65.669
e
B
= 35, d
B
= 53.771
Claves Adela
n
A
= 66.331
e
A
= 25, d
A
= 18.377
Tenamos que: H(M) = F3A9
16
= 62.377
10
r
H(M)
= H(M)
dB
mod n
B
r
H(M)
= 62.377
53.771
mod 65.669 = 24.622
Benito haba enviado el par (M, r) = (M, 24.622)
Adela recibe un mensaje M junto con una rbrica r = 24.622:
Calcula r
eB
mod n
B
= 24.622
35
mod 65.669 = 62.377.
Calcula el resumen de M es decir H(M) y lo compara con H(M).
Si los mensajes M y M son iguales, entonces H(M) = H(M) y se
acepta la firma como vlida.
NOTA: No obstante, H(M) = H(M) no implica que M = M.
Benito Adela
Jorge Rami Aguirre Madrid (Espaa) 2003
Seguridad Informtica y Criptografa. Tema 14: Autenticacin y Firma Digital
Diapositiva 674
Firma digital ElGamal de A hacia B
ElGamal: El usuario A generaba un nmero
aleatorio a (clave privada) del cuerpo p. La
clave pblica es
a
mod p, con generador.
Algoritmo de firma:
1 El usuario A genera un nmero aleatorio h, que ser primo
relativo con (p): h / mcd {h, (p)} = 1
2 Calcula h
-1
= inv {h, (p)}
3 Calcula r =
h
mod p
4 Resuelve la siguiente congruencia:
M = ar + hs mod (p)
s = (M - ar)inv[h,(p)] mod (p)
M = ar + hs mod (p)
s = (M - ar)inv[h,(p)] mod (p)
Firma: (r, s)
Firma: (r, s)
Adela
Jorge Rami Aguirre Madrid (Espaa) 2003
Seguridad Informtica y Criptografa. Tema 14: Autenticacin y Firma Digital
Diapositiva 675
Comprobacin de firma ElGamal por B
Algoritmo comprobacin de firma:
1 El usuario B recibe el par (r, s) y calcula:
r
s
mod p y (
a
)
r
mod p
2 Calcula k = [(
a
)
r
r
s
] mod p
Como r era igual a
h
mod p entonces:
k = [(
ar

hs
] mod p =
(ar + hs)
mod p =

mod p
3 Como M = (ar + hs) mod (p) y es una raz
primitiva de p se cumple que:

ssi = mod (p-1)


4 Comprueba que k =
M
mod p
Benito
Si k = [(
a
)
r
r
s
] mod p
es igual a
M
mod p ...
Si k = [(
a
)
r
r
s
] mod p
es igual a
M
mod p ...
Se acepta la firma
Se acepta la firma
Conoce: p y (
a
) mod p
Jorge Rami Aguirre Madrid (Espaa) 2003
Seguridad Informtica y Criptografa. Tema 14: Autenticacin y Firma Digital
Diapositiva 676
Ejemplo de firma ElGamal (B A)
Hola otra vez! Soy Benito de nuevo. Saludos.
Sea H(M) = A69B (16 bits)
Claves Benito
p
B
= 79.903 = 10

b
mod p = 3.631
b = 20, h = 31
2
16
< 79.903 < 2
17
Forzaremos firmar
bloques de 16 bits
Benito Adela
Firma
1) h
-1
= inv[h, (p)] = inv (31, 79.902) = 5.155
2) r =
h
mod p = 10
31
mod 79.903 = 11.755
3) s = [H(M) - br][inv(h,(p)] mod (p) H(M) = A69B
16
= 42.651
10
4) s = [42.651-2011.755]5.155 mod 79.902
5) s = 68.539
(r, s) = (11.755, 68.539)
(r, s) = (11.755, 68.539)
Luego, la firma ser
Jorge Rami Aguirre Madrid (Espaa) 2003
Seguridad Informtica y Criptografa. Tema 14: Autenticacin y Firma Digital
Diapositiva 677
Comprobacin de firma ElGamal por A
Adela Benito
Claves Benito
p
B
= 79.903 = 10

b
mod p = 3.631
b = 20, h = 31
H(M) = A69B = 42.651
Adela recibe el par (r, s) = (11.755, 68.539)
Comprobacin de la firma:
1) r
s
mod p = 11.755
68.539
mod 79.903 = 66.404
2) (
b
)
r
mod p = 3.631
11.755
mod 79.903 = 12.023
3) (
b
)
r
r
s
mod p = (12.023 66.404) mod 79.903 = 64.419 = k
4)
H(M)
mod p = 10
42.651
mod 79.903 = 64.419
Como hay igualdad
se acepta la firma
Jorge Rami Aguirre Madrid (Espaa) 2003
Seguridad Informtica y Criptografa. Tema 14: Autenticacin y Firma Digital
Diapositiva 678
Importancia de en la firma de ElGamal
Claves Benito
p
B
= 79.903 = 10

b
mod p = 3.631
b = 20, h = 31
= 10 es un generador
del cuerpo p = 79.903
puesto que:
p-1 = 79.902 = 23
2
23193
q
1
= 2; q
2
= 3; q
3
= 23; q
4
= 193
y se cumple 10
(p-1)qi
mod p 1
10
39.951
mod 79.903 = 79.902
10
26.634
mod 79.903 = 71.324
10
3.474
mod 79.903 = 2.631
10
414
mod 79.903 = 41.829
Benito
Si se elige = 11 que no es raz, para el exponente 39.951 se
obtiene el valor 1. No nos servir para la firma porque ser
imposible comprobarla mediante la ecuacin k =
M
mod p.
Jorge Rami Aguirre Madrid (Espaa) 2003
Seguridad Informtica y Criptografa. Tema 14: Autenticacin y Firma Digital
Diapositiva 679
Estndares de firma digital
1991: National Institute of Standards and Technology (NIST)
propone el DSA, Digital Signature Algorithm, una
variante de los algoritmos de ElGamal y Schnoor.
1994: Se establece como estndar el DSA y se conoce como
DSS, Digital Signature Standard.
1996: La administracin de los Estados Unidos permite la
exportacin de Clipper 3.11 en donde viene inmerso el
DSS, que usa una funcin hash de tipo SHS, Secure
Hash Standard.
El peor inconveniente de la firma propuesta por ElGamal es que
duplica el tamao del mensaje M al enviar un par (r, s) en Z
p
y (p).
No obstante, se solucionar con el algoritmo denominado DSS.
Jorge Rami Aguirre Madrid (Espaa) 2003
Seguridad Informtica y Criptografa. Tema 14: Autenticacin y Firma Digital
Diapositiva 680
Digital Signature Standard DSS
Parmetros pblicos de la firma:
Un nmero primo grande p (512 bits)
Un nmero primo q (160 bits) divisor de p-1
Un generador de orden q del grupo p
Generador de orden q es aquella raz en el cuerpo Z
p
de
forma que q es el entero ms pequeo que verifica:

q
mod p = 1
En este caso se cumple para todo t que:

t
=
t (mod q)
mod p
y eso qu es?
Jorge Rami Aguirre Madrid (Espaa) 2003
Seguridad Informtica y Criptografa. Tema 14: Autenticacin y Firma Digital
Diapositiva 681
Generacin de firma DSS (A B)
GENERACIN DE LA FIRMA POR PARTE DE A
Claves pblicas de A: primos p, q y el generador
Clave secreta de la firma: a (1 < a < q) aleatorio
Clave pblica de la firma: y =
a
mod p
Para firmar un mensaje 1 < M < p, el firmante A elige un
valor aleatorio 1 < h < q y calcula:
r = (
h
mod p) mod q
s = [(M + ar) inv (h,q)] mod q
La firma digital de M ser el par (r, s)
Jorge Rami Aguirre Madrid (Espaa) 2003
Seguridad Informtica y Criptografa. Tema 14: Autenticacin y Firma Digital
Diapositiva 682
Comprobacin de firma DSS por B
COMPROBACIN DE LA FIRMA DE A POR B
B recibe el par (r, s)
Luego calcula:
w = inv (s, q)
u = M w mod q
v = r w mod q
Comprueba que se cumple la relacin:
r = (
u
y
v
mod p) mod q
Si se cumple, se acepta la firma como vlida.
La firma tendr en este caso
un tamao menor que q, es
decir, menos bits que los del
mdulo de firma p ya que se
elige por diseo p >> q
Jorge Rami Aguirre Madrid (Espaa) 2003
Seguridad Informtica y Criptografa. Tema 14: Autenticacin y Firma Digital
Diapositiva 683
Ejemplo de firma DSS (B A)
Hola Adela, soy Benito y lo firmo con DSS.
Adela
Benito
Sea H(M) = 1101000 = 104 (un elemento de p
B
)
Claves Benito
p
B
= 223 q
B
= 37 = 17
y =
b
mod p = 30
b = 25, h = 12
2
8
< p
B
= 223 < 2
7
Forzaremos firmar
bloques de 7 bits
Firma
1) inv (h,q) = inv (12, 37) = 34
2) r = (
h
mod p) mod q = (17
12
mod 223) mod 37 = 171 mod 37 = 23
3) s = [H(M)+br][inv (h,q)] mod q = [104+2523]34 mod 37 = 35
4) La firma digital de H(M) = 104 ser: (r, s) = (23, 35)
5) Benito transmite a Adela el bloque (M, r, s) = (M, 23, 35)
Jorge Rami Aguirre Madrid (Espaa) 2003
Seguridad Informtica y Criptografa. Tema 14: Autenticacin y Firma Digital
Diapositiva 684
Comprobacin de la firma DSS por A
1) w = inv (s, q) = inv (35, 37) = 18
2) u = Mw mod q = 10418 mod 37 = 22
3) v = rw mod q = 2318 mod 37 = 7
4) (
u
y
v
mod p) mod q = r ?
5) [(17
22
30
7
) mod 223] mod 37 = 23
En caso afirmativo,
se acepta la firma
Y el tamao ser menor que
q
B
= 37 es decir << p
B
= 223
que era precisamente el punto
dbil del sistema ElGamal.
Claves Benito
p
B
= 223 q
B
= 37 = 17
y =
b
mod p = 30
b = 25, h = 12
Adela recibe:
(M, r, s) = (M, 23, 35)
Igualdad?
Fin del Tema 14
Benito Adela
Comprobacin de firma
Jorge Rami Aguirre Madrid (Espaa) 2003
Seguridad Informtica y Criptografa. Tema 14: Autenticacin y Firma Digital
Diapositiva 685
Cuestiones y ejercicios (1 de 2)
1. Por qu cree que un escenario de integridad en la que hay siempre
una tercera parte de confianza activa no sera adecuado en Internet?
2. Si una Autoridad de Certificacin es la tercera parte de confianza,
acta de forma activa o pasiva en la comunicacin? Explquelo.
3. Qu importancia tiene la redundancia del lenguaje en un sistema de
autenticacin? Qu sucedera si no hubiese esa redundancia?
4. Hacemos una autenticacin de mensaje mediante un MAC en el que
el algoritmo es DES. Sera acertado usar modo ECB? Por qu?
5. En un sistema de autenticacin mediante Kerberos, cmo sabe el
que comienza el protocolo (A) que no hay un impostor que intenta
suplantarle? Cmo sabe el que comienza el protocolo (A) que el
segundo usuario (B) o elemento del sistema ha recibido bien la clave
de sesin entregada por el centro de distribucin de claves KDC?
Jorge Rami Aguirre Madrid (Espaa) 2003
Seguridad Informtica y Criptografa. Tema 14: Autenticacin y Firma Digital
Diapositiva 686
Cuestiones y ejercicios (2 de 2)
6. Cmo podramos usar una funcin hash para comprobar que un
archivo no se ha modificado o que est libre de virus?
7. Nos afirman que podemos autenticar un mensaje usando para ello
slo una funcin hash. Es esto verdadero? Por qu?
8. Por qu se dice que una firma digital tiene condiciones ms fuertes
que una firma manuscrita?
9. Por qu es importante que la firma digital dependa del mensaje?
10. Firme digitalmente con RSA el mensaje M = 121 si los datos del
firmante son p = 19; q = 31, d = 149. Compruebe la firma.
11. Con p = 331 y clave privada 15, vamos a firmar digitalmente con
ElGamal el mensaje M = 250 . Para ello, elija los valores ms
pequeos posibles de los parmetros y h. Compruebe la firma.
12. Repita el ejercicio 10 con DSS y valores propuestos por Ud.
Tema 15
Certificados Digitales
Seguridad Informtica y Criptografa
Ultima actualizacin: 03/03/03
Archivo con 11 diapositivas
Jorge Rami Aguirre
Universidad Politcnica de Madrid
Material Docente de
Libre Distribucin
v 3.1
Este archivo forma parte de un curso sobre Seguridad Informtica y Criptografa. Se autoriza la
reproduccin en computador e impresin en papel slo con fines docentes o personales, respetando
en todo caso los crditos del autor. Queda prohibida su venta, excepto a travs del Departamento de
Publicaciones de la Escuela Universitaria de Informtica, Universidad Politcnica de Madrid, Espaa.
Curso de Seguridad Informtica y Criptografa JRA
Jorge Rami Aguirre Madrid (Espaa) 2003
Seguridad Informtica y Criptografa. Tema 15: Certificados Digitales
Diapositiva 688
Nota del autor
El contenido de este archivo
corresponde a una primera
introduccin bsica sobre
certificados digitales, en
particular X.509, y el
intercambio de stos entre
cliente y servidor para una completa autenticacin. En
este ao 2003 se ir ampliando este captulo de forma
que en la prxima versin de este libro se pueda contar
con ya con un tema de certificacin digital y Autoridades
de Certificacin como debe corresponder a un curso de
criptografa y seguridad informtica.
Jorge Rami Aguirre Madrid (Espaa) 2003
Seguridad Informtica y Criptografa. Tema 15: Certificados Digitales
Diapositiva 689
Qu son los certificados digitales?
Un certificado digital es un documento que
contiene diversos datos, entre ellos el nombre de
un usuario y su clave pblica, y que es firmado
por una Autoridad de Certificacin (AC).
Como emisor y receptor confiarn en esa AC, el
usuario que tenga un certificado expedido por
ella se autenticar ante el otro, en tanto que su
clave pblica est firmada por dicha autoridad.
Jorge Rami Aguirre Madrid (Espaa) 2003
Seguridad Informtica y Criptografa. Tema 15: Certificados Digitales
Diapositiva 690
Certificado digital X.509
X.509 est basado en criptografa asimtrica y firma digital.
En X.509 se define un framework (una capa de abstraccin)
para suministrar servicios de autenticacin a los usuarios del
directorio X.500.
La autenticacin se realiza mediante el uso de certificados.
- Un certificado contiene: el nombre de la AC, el nombre
del usuario, la clave pblica del usuario y cualquier otra
informacin como puede ser un un indicador de tiempo
o timestamp.
- El certificado se cifra con la clave privada de la AC.
- Todos los usuarios poseen la clave pblica del AC.
Jorge Rami Aguirre Madrid (Espaa) 2003
Seguridad Informtica y Criptografa. Tema 15: Certificados Digitales
Diapositiva 691
Formato del certificado digital X.509
Funcin hash que se cifra con
la clave privada de la AC
N de serie
Algoritmo
Parmetros
Autoridad de Certificacin
Inicio de la validez
Caducidad de la validez
Nombre del usuario
Algoritmo
Parmetros
Clave pblica del usuario
Identificador del algoritmo
Perodo de validez
Clave pblica que se firma
Versin
Firma de la AC
Jorge Rami Aguirre Madrid (Espaa) 2003
Seguridad Informtica y Criptografa. Tema 15: Certificados Digitales
Diapositiva 692
Campos del certificado digital X.509
V: Versin del certificado (actualmente V3).
SN: Nmero de serie.
AI: identificador del algoritmo de firma que sirve para identificar el
algoritmo usado para firmar el paquete X.509.
CA: Autoridad certificadora.
T
A
: Periodo de validez.
A: Propietario de la clave pblica que se est firmando.
P: Clave pblica ms identificador de algoritmo utilizado y ms
parmetros si son necesarios.
Y{I}:Firma digital de Y por I usando la clave privada de la unidad
certificadora.
CA<<A>> = CA { V, SN, AI, CA, T
A
, A, AP }
Y<<X>> es el certificado del usuario X expedido por la autoridad certificadora Y.
Jorge Rami Aguirre Madrid (Espaa) 2003
Seguridad Informtica y Criptografa. Tema 15: Certificados Digitales
Diapositiva 693
Autoridades de Certificacin
B
Autoridad de Certificacin AC
c
l
a
v
e

p

b
l
i
c
a

A
C
c
l
a
v
e

p

b
l
i
c
a

A
C
AC

A
Autoridad de Certificacin es un
ente u organismo que, de acuerdo
con unas polticas y algoritmos,
certificar -por ejemplo- claves
pblicas de usuarios o servidores.
certificado de B
certificado de A
El usuario A enviar al usuario B
su certificado (la clave pblica
firmada por AC) y ste comprobar
con esa autoridad su autenticidad.
Lo mismo en sentido contrario.
Jorge Rami Aguirre Madrid (Espaa) 2003
Seguridad Informtica y Criptografa. Tema 15: Certificados Digitales
Diapositiva 694
Elementos de una AC
El sistema de autenticacin debe tener:
Una poltica de certificacin
Un certificado de la CA
Los certificados de los usuarios (X.509)
Los protocolos de autenticacin, gestin y obtencin
de certificados:
Se obtienen de bases de datos (directorio X.500)
O bien directamente del usuario en tiempo de
conexin (WWW con SSL)
Jorge Rami Aguirre Madrid (Espaa) 2003
Seguridad Informtica y Criptografa. Tema 15: Certificados Digitales
Diapositiva 695
Algunas caractersticas de diseo de la AC
Deber definirse una poltica de certificacin
Ambito de actuacin y estructura
Relaciones con otras ACs
Deber definirse el procedimiento de certificacin
para la emisin de certificados:
Verificacin on-line
Verificacin presencial
Deber generarse una Lista de Certificados Revocados
Jorge Rami Aguirre Madrid (Espaa) 2003
Seguridad Informtica y Criptografa. Tema 15: Certificados Digitales
Diapositiva 696
Funcionamiento de una AC
Puesta en marcha de la AC:
Generar su par de claves
Proteger la clave privada con una passphrase
Generar el certificado de la propia AC
Distribucin del certificado de la AC:
A travs del directorio X.500
Por medio de pginas Web
Podr certificar a servidores y a clientes
Fin del Tema 15
Jorge Rami Aguirre Madrid (Espaa) 2003
Seguridad Informtica y Criptografa. Tema 15: Certificados Digitales
Diapositiva 697
Cuestiones y ejercicios
1. Qu es lo que certifica un certificado digital?
2. Solicitamos un certificado digital a una empresa. Est nuestra clave
privada entre los datos que nos solicita para el alta?
3. Si un certificado digital pierde la validez, qu debemos hacer?
4. Por qu una Autoridad de Certificacin firma una funcin hash?
5. Si Ud. fuese una Autoridad de Certificacin, qu condicin pondra
para expedir con seguridad un certificado a un usuario?
6. Una Autoridad de Certificacin emite certificados digitales de hasta
1.024 bits y duracin un ao. Si ella tiene un certificado digital raz
de 2.048 bits, sera lgico plantear una validez de ste por 10 aos?
7. Qu es y cundo se solicita la revocacin de un certificado digital?
8. Qu significa que la Autoridad de Certificacin deba gestionar una
lista de certificados revocados?
Tema 16
Aplicaciones Criptogrficas
Seguridad Informtica y Criptografa
Ultima actualizacin: 03/03/03
Archivo con 83 diapositivas
Jorge Rami Aguirre
Universidad Politcnica de Madrid
Material Docente de
Libre Distribucin
v 3.1
Este archivo forma parte de un curso sobre Seguridad Informtica y Criptografa. Se autoriza la
reproduccin en computador e impresin en papel slo con fines docentes o personales, respetando
en todo caso los crditos del autor. Queda prohibida su venta, excepto a travs del Departamento de
Publicaciones de la Escuela Universitaria de Informtica, Universidad Politcnica de Madrid, Espaa.
Curso de Seguridad Informtica y Criptografa JRA
Jorge Rami Aguirre Madrid (Espaa) 2003
Seguridad Informtica y Criptografa. Tema 16: Aplicaciones Criptogrficas
Diapositiva 700
Resumen de los sistemas de clave secreta
Pros y contras de los Sistemas de Clave Secreta
El emisor y el receptor comparten una misma clave.
La seguridad depende slo del secreto de la clave.
La velocidad de cifra es muy alta y los sistemas con un
espacio de clave grande son muy seguros.
Permiten autenticar los mensajes con MACs.
`
... pero
Es imposible establecer un sistema de distribucin y
gestin de claves eficiente entre emisor y receptor.
Carecen de una firma digital en sentido amplio.

Jorge Rami Aguirre Madrid (Espaa) 2003
Seguridad Informtica y Criptografa. Tema 16: Aplicaciones Criptogrficas
Diapositiva 701
Resumen de los sistemas de clave pblica
Pros y contras de los Sistemas de Clave Pblica
Emisor y receptor generan un par de claves, pblica y
privada, relacionadas por una funcin con trampa.
Emisor y receptor de un mensaje usan claves diferentes
para las operaciones de cifrado, descifrado y firma.
La seguridad del sistema va asociada a la resolucin de un
problema matemtico difcil.
Tiene firma digital: autenticacin de mensaje y del emisor.
`
... pero
Es necesario contar con mecanismos de certificacin
para asegurar la veracidad de las claves pblicas: ACs.
Son sistemas de cifra muy lentos.

Jorge Rami Aguirre Madrid (Espaa) 2003
Seguridad Informtica y Criptografa. Tema 16: Aplicaciones Criptogrficas
Diapositiva 702
El correo electrnico seguro
A comienzos de los aos 90 hacen su aparicin
dos sistemas de correo electrnico seguro:
PEM (Private Enhanced Mail)
PGP (Pretty Good Privacy)
De los dos, ha sido PGP quien se ha convertido en un
estndar de hecho en clientes del e-mail seguro. Por
lo tanto veremos slo algunos aspectos genricos de
PEM y analizaremos ms en profundidad PGP.
Su estudio nos permitir ver una aplicacin real de
los sistemas de cifra y firma analizados en el curso.
Jorge Rami Aguirre Madrid (Espaa) 2003
Seguridad Informtica y Criptografa. Tema 16: Aplicaciones Criptogrficas
Diapositiva 703
Private Enhanced Mail PEM
Es una propuesta de la IETF Internet Engineering Task
Force en 1985. El documento tcnico se publica en 1993.
Las especificaciones tcnicas estn en las RFCs Request
For Comments nmeros 1421, 1422, 1423 y 1424.
Se usa conjuntamente con el protocolo SMTP Simple Mail
Internet Protocol.
Cifrado de la informacin: DES modo CBC.
Generacin y gestin de claves: RSA de 508 a 1024 bits.
Estructura de certificados segn la norma X.509.
Clave de sesin: DES modo ECB, TripleDES-EDE.
Firma digital: RSA, MD2, MD5.
Jorge Rami Aguirre Madrid (Espaa) 2003
Seguridad Informtica y Criptografa. Tema 16: Aplicaciones Criptogrficas
Diapositiva 704
Implementacin de PEM
Es compatible con otros modelos de mensajera como, por
ejemplo, X.400.
PEM se implementa en el nivel de aplicacin:
es independiente de los protocolos de los niveles OSI o
TCP/IP inferiores.
es independiente de los sistemas operativos o del
ordenador.
Se puede implementar como un mdulo independiente que
trabaje con el cliente de correo habitual para el usuario.
Jorge Rami Aguirre Madrid (Espaa) 2003
Seguridad Informtica y Criptografa. Tema 16: Aplicaciones Criptogrficas
Diapositiva 705
Servicios de seguridad en PEM
Servicios de seguridad contemplados:
Autenticacin del origen.
Confidencialidad.
Integridad del mensaje.
No repudio del origen cuando se utiliza gestin de clave
con algoritmo de clave asimtrica.
Servicios de seguridad no contemplados:
Control de acceso.
Confidencialidad del trfico de mensajes.
No repudio del mensaje por parte del receptor.
Jorge Rami Aguirre Madrid (Espaa) 2003
Seguridad Informtica y Criptografa. Tema 16: Aplicaciones Criptogrficas
Diapositiva 706
Formato e implementacin de PEM
TIS/PEM
Plataformas UNIX. Trusted Information
System. Cdigo fuente disponible para
los ciudadanos o empresas de Estados
Unidos y Canad. Usa una jerarqua de
certificacin mltiple.
RIPEM
Implementa parte de los protocolos PEM
sin certificados para autenticacin de
claves. Gratuito para aplicaciones no
comerciales. Exportacin prohibida fuera
de Estados Unidos. Existen versiones
utilizadas en todo el mundo.
Jorge Rami Aguirre Madrid (Espaa) 2003
Seguridad Informtica y Criptografa. Tema 16: Aplicaciones Criptogrficas
Diapositiva 707
Pretty Good Privacy PGP
Philip Zimmermann publica la versin 1.0 de PGP en
1991 con mnimos requisitos de hardware y software.
En 1992 aparece la versin 2.0 en la que ya participan
programadores de todo el mundo. Su cdigo se escribe
fuera de USA para evitar las leyes restrictivas respecto al
software criptogrfico y sus problemas legales.
En 1993 aparece la versin 2.3a muy popular en sitios FTP
y vlida para varias plataformas de sistemas operativos.
En 1994 participa en el proyecto el Massachusetts Institute
of Technology MIT y aparecen las versiones 2.4, 2.5 y 2.6.
La versin 2.6.3i se populariza a nivel mundial.
Jorge Rami Aguirre Madrid (Espaa) 2003
Seguridad Informtica y Criptografa. Tema 16: Aplicaciones Criptogrficas
Diapositiva 708
Nota aclaratoria sobre versiones de PGP
Aunque hay ms de una oferta de software para correo seguro que
el programa PGP, ste se ha convertido en un estndar de hecho.
Si bien las ltimas versiones del programa orientadas a entornos
Windows presentan altas prestaciones, las operaciones bsicas
siguen siendo las mismas que en la conocida versin 2.6.3i.
Las nuevas versiones de PGP en entorno Windows cambian muy
rpidamente por lo que resulta muy difcil tener unos apuntes
permanentemente actualizados. Por ello, se usar la versin 2.6.3i
como versin simple para la explicacin de las operaciones de
cifra y firma con PGP y, posteriormente, haremos un repaso de las
caractersticas de la versiones 6.5.1 y 8.0, la ltima a la fecha.
La filosofa de las nuevas versiones es exactamente la misma
Jorge Rami Aguirre Madrid (Espaa) 2003
Seguridad Informtica y Criptografa. Tema 16: Aplicaciones Criptogrficas
Diapositiva 709
Tutorial de PGP 2.6.3i
Si no conoce PGP o no ha trabajado nunca con este
entorno, le recomiendo que descargue desde la pgina
Web de la Red Temtica CriptoRed el archivo del
Tutorial de PGP 2.6.3i en formato HTML.
Esta aplicacin, obra de D. David Lin Zayas, alumno
que realiz este proyecto como su Tesis Fin de Carrera
tutorizado por el autor de estos apuntes, le servir para
aprender rpidamente los comandos y prestaciones de
esta versin de PGP, muy similar a las actuales. Adems
PGP 2.6.3i ocupa menos que un disquete de 1,4 MB.
http://www.criptored.upm.es/paginas/software.htm
Jorge Rami Aguirre Madrid (Espaa) 2003
Seguridad Informtica y Criptografa. Tema 16: Aplicaciones Criptogrficas
Diapositiva 710
Caractersticas de PGP 2.6.3i
PGP, en su versin 2.6.3i (internacional) se convirti a
mediados de la dcada de los 90 en un estndar de hecho.
De hecho, muchos usuarios siguen fieles a esta versin.
Cifra todo tipo de datos en entornos MS-DOS y UNIX. Su
orientacin principal es el cifrado de los datos y la firma
digital en correo electrnico.
Los algoritmos bsicos que usa son:
IDEA para cifrar con sistema de clave secreta.
RSA para intercambio de claves y firma digital.
MD5 para obtener la funcin hash de la firma digital.
Jorge Rami Aguirre Madrid (Espaa) 2003
Seguridad Informtica y Criptografa. Tema 16: Aplicaciones Criptogrficas
Diapositiva 711
Algoritmos usados en PGP 2.6.3i
Compresin ZIP
Se comprime el mensaje en claro y la firma para
almacenarlo o transmitirlo.
Generacin de claves RSA, MD5
Genera una clave pblica y otra privada, encontrando
dos nmeros primos muy grandes. El valor privado se
guarda cifrado con IDEA usando como clave un resumen
MD5 de la frase de paso secreta.
Cifrado Convencional IDEA
Cifra el mensaje con una clave de sesin de 128 bits
(nica) generada en el emisor de forma aleatoria.
Intercambio de claves IDEA, RSA
Cifra la clave de sesin IDEA con la clave pblica del
destinatario con RSA y la aade en el criptograma.
Firma Digital MD5, RSA
La funcin hash MD5 genera un resumen de 128 bits,
que representa al mensaje en claro completo, y que se
cifra en RSA con la clave privada del emisor. Se aade
al mensaje enviado.
Compatibilidad e-mail Base-64
Permite transmitir el mensaje a todo tipo de aplicaciones
e-mail. Convierte los octetos en caracteres imprimibles.
Segmentacin
Divide el criptograma final en bloques de menos de
50.000 bytes para su correcta transmisin en Internet y
su recuperacin.

- Operacin - - Algoritmo - - Descripcin de su funcin -
Jorge Rami Aguirre Madrid (Espaa) 2003
Seguridad Informtica y Criptografa. Tema 16: Aplicaciones Criptogrficas
Diapositiva 712
Caractersticas del cifrado local
O Esta operacin sirve para mantener los archivos
protegidos, por ejemplo en el disco duro.
O El acceso al texto en claro slo ser posible si se
conoce una clave o contrasea que es la frase de
paso usada al cifrar.
O Recuerde que si despus de cifrar el archivo borra
fsicamente el texto en claro -operacin que
realiza una grabacin de unos y ceros aleatorios
en la zona de almacenamiento del disco- le ser
imposible recuperarlo si olvida la contrasea.
Jorge Rami Aguirre Madrid (Espaa) 2003
Seguridad Informtica y Criptografa. Tema 16: Aplicaciones Criptogrficas
Diapositiva 713
Pasos del cifrado local con IDEA
Pasos:
1. PGP solicita una frase de paso: sta debe ser lo
suficientemente larga como para evitar ataques por
combinaciones.
2. Se aplica el algoritmo de resumen MD5 a esa
contrasea, generando as una clave de 128 bits.
3. PGP cifra el documento con el algoritmo IDEA y le
pone como extensin .pgp.
4. Permite luego hacer un borrado fsico del archivo
en claro.
Jorge Rami Aguirre Madrid (Espaa) 2003
Seguridad Informtica y Criptografa. Tema 16: Aplicaciones Criptogrficas
Diapositiva 714
Esquema de cifrado local con IDEA
Mensaje en
claro
Clave Local
de 128 bits
Mensaje
cifrado
El documento se comprime
con el algoritmo ZIP
El archivo cifrado
puede guardarse,
por ejemplo, en
disco.
Borrado del texto
en claro opcional.
La contrasea es una frase de paso.
Se recomienda que tenga espacios,
signos y caracteres de puntuacin
CONTRASEA
Cada nuevo cifrado requiere una contrasea. Esta puede ser igual o distinta.
Jorge Rami Aguirre Madrid (Espaa) 2003
Seguridad Informtica y Criptografa. Tema 16: Aplicaciones Criptogrficas
Diapositiva 715
Operaciones con claves asimtricas
Las operaciones de PGP para cifrar, descifrar, firmar y la
comprobacin posterior de la firma digital, usan los
algoritmos de funciones hash, de clave pblica y de clave
secreta ya vistos en captulos anteriores.
Para poder enviar y recibir correo seguro, es necesario
contar al menos con las siguientes claves:
Clave pblica del destinatario.
Par de claves asimtricas del emisor.
Generacin de claves con RSA
Jorge Rami Aguirre Madrid (Espaa) 2003
Seguridad Informtica y Criptografa. Tema 16: Aplicaciones Criptogrficas
Diapositiva 716
Generacin de claves asimtricas tipo RSA
Una vez instalado PGP, se procede a la generacin de
claves asimtricas del usuario propietario.
Se elige el tamao del mdulo n, por ejemplo 1.024 bits.
PGP generar un par de nmeros primos e (clave pblica)
y d (clave privada) de forma que ed mod (n) = 1.
Para una mayor facilidad en el descifrado en destino, el
valor de la clave pblica e ser pequeo (un valor tpico es
el nmero primo 65.537 = 2
16
+1).
PGP pedir una contrasea o passphrase y con ella y MD5
generar una clave de 128 bits con la que cifrar la clave
privada antes de almacenarla en el disco.
Jorge Rami Aguirre Madrid (Espaa) 2003
Seguridad Informtica y Criptografa. Tema 16: Aplicaciones Criptogrficas
Diapositiva 717
Anillos de claves asimtricas
Con las claves pblica y privada generadas y otras
claves pblicas que podr importar de otros usuarios,
se crean dos anillos de claves:
Anillo de claves pblicas: archivo pubring.pgp
en el que se guardan las claves pblicas del
usuario propietario (puede tener ms de una
identidad) y las claves pblicas de importadas.
Anillo de claves privadas: archivo secring.pgp
en el que se guarda la o las claves privadas del
usuario propietario.
Nota: estos anillos cambiarn su extensin en
las nuevas versiones.
Jorge Rami Aguirre Madrid (Espaa) 2003
Seguridad Informtica y Criptografa. Tema 16: Aplicaciones Criptogrficas
Diapositiva 718
Estructura del anillo de claves privadas
Sellado de tiempo Clave ID* Clave pblica Clave privada cifrada ID usuario
T
1
e
1
mod 2
64
Clave pb. 1 Clave priv. 1 Usuario 1
--- --- --- --- ---
T
i
e
i
mod 2
64
e
i
E
H(FPi)
(d
i
) Usuario i
--- --- --- --- ---
T
n
e
n
mod 2
64
Clave pb. n Clave priv. n Usuario n
Descripcin de los campos
(*) Se usa este campo para la indexacin de la tabla en ambos anillos
Jorge Rami Aguirre Madrid (Espaa) 2003
Seguridad Informtica y Criptografa. Tema 16: Aplicaciones Criptogrficas
Diapositiva 719
Campos de los anillos de claves
Sellado de tiempo:
Fecha y hora de la generacin del par de claves.
Clave ID:
Identificador de clave (ltimos 64 bits de la clave pblica e).
Clave pblica:
Nmero primo e, inverso del primo d en el cuerpo (n).
Clave privada cifrada:
Cifra E
H(FPi)
de la clave privada d con IDEA y la funcin hash
de la frase de paso del propietario como clave secreta.
ID usuario:
Identificacin del usuario, normalmente direccin de email.
Jorge Rami Aguirre Madrid (Espaa) 2003
Seguridad Informtica y Criptografa. Tema 16: Aplicaciones Criptogrficas
Diapositiva 720
Estructura del anillo de claves pblicas (1)
Sellado de tiempo Clave ID* Clave pblica Confianza propietario ID usuario
T
1
e
1
mod 2
64
Clave pb. 1 flag_confianza 1 Usuario 1
--- --- --- --- ---
T
i
e
i
mod 2
64
e
i
flag_confianza i Usuario i
--- --- --- --- ---
T
n
e
n
mod 2
64
Clave pb. n Clave priv. n Usuario n
...
...
...
...
contina en prxima diapositiva
(*) Se usa este campo para la indexacin de la tabla en ambos anillos
Jorge Rami Aguirre Madrid (Espaa) 2003
Seguridad Informtica y Criptografa. Tema 16: Aplicaciones Criptogrficas
Diapositiva 721
Estructura del anillo de claves pblicas (2)
Legitimacin de clave Firma(s) Confianza de Firmas
flag_confianza 1
--- --- ---
flag_confianza i
--- --- ---
flag_confianza n
...
...
...
...
viene de la diapositiva anterior
Con la clave pblica del destinatario ya podemos
enviar el correo cifrado y/o firmado. Pero ...
cmo se gestionan las claves en PGP?
Jorge Rami Aguirre Madrid (Espaa) 2003
Seguridad Informtica y Criptografa. Tema 16: Aplicaciones Criptogrficas
Diapositiva 722
Gestin del anillo de claves pblicas
?
?
?
A1 Nivel A
B1 B2 B3 Nivel B
C2 C4 C5 C1 C3 Nivel C
D1
Nivel D
X Y
X es firmado por Y
Propietario del anillo
A1 cree en el propietario de la clave para firmar otra clave
ms
A1 cree parcialmente en el propietario de la clave para firmar otra clave
La clave est firmada por un
usuario o Autoridad que no
est en anillo de claves de A1
A1 cree en legitimidad de clave
?
A1 no cree que la clave sea legtima
Jorge Rami Aguirre Madrid (Espaa) 2003
Seguridad Informtica y Criptografa. Tema 16: Aplicaciones Criptogrficas
Diapositiva 723
Otros escenarios de confianza en PGP
D1
?
A1
B1 B3
C2 C1
D2
X Y
X es firmado por Y
B2
?
C1
Otras situaciones en anillo de claves
D1 D1
Nodo hurfano
con firmas no
reconocibles
C1 D1
estados iniciales
?
Observe cambios
en estados finales
A1 cree en el propietario de la clave para firmar otra clave
A1 cree parcialmente en el propietario de la clave para firmar otra clave
PGP hace que A1 crea en la legitimidad de las claves pues tienen al menos dos
firmas parciales (B1-B2)o una completa (C2) pero no da confianza para firmar
A1 no cree que la clave sea legtima
Jorge Rami Aguirre Madrid (Espaa) 2003
Seguridad Informtica y Criptografa. Tema 16: Aplicaciones Criptogrficas
Diapositiva 724
Problema en estos escenarios de confianza
La gestin de claves en PGP se
basa en la confianza mutua:
los amigos de tus amigos son
mis amigos!
- En un sistema abierto en Internet como puede ser el
comercio electrnico, esta situacin y otras ms que
pueden darse en este sistema de gestin de claves de
confianza mutua, resulta inaceptable.
- La solucin, que PGP ya contempla en sus ltimas
versiones, es la aceptacin de las Autoridades de
Certificacin como certificadores de claves pblicas.
Jorge Rami Aguirre Madrid (Espaa) 2003
Seguridad Informtica y Criptografa. Tema 16: Aplicaciones Criptogrficas
Diapositiva 725
Pasos cifrado con clave pblica de destino
Pasos:
1. PGP genera un nmero aleatorio de 128 bits que
ser la clave de sesin.
2. Se cifra el mensaje con dicha clave usando IDEA.
3. Se cifra la clave de sesin con la clave pblica RSA
del destinatario y se aade al criptograma.
4. Se aade el identificador ID de la clave pblica del
destinatario a la clave de sesin cifrada en el paso 3
como indicativo de la identidad del receptor.
Recuerde que el correo electrnico no es en general una comunicacin
en tiempo real por lo que, aunque se enva una clave para descifrar el
criptograma en recepcin, no se trata de una clave de sesin en los
mismos trminos que se usa, por ejemplo, en una comunicacin SSL.
Jorge Rami Aguirre Madrid (Espaa) 2003
Seguridad Informtica y Criptografa. Tema 16: Aplicaciones Criptogrficas
Diapositiva 726
Cifrado con clave pblica de destino
Mensaje en
claro
Clave pblica
del destinatario
Mensaje
cifrado
Clave de
sesin
cifrada
Clave
de
sesin
Se busca en el anillo de
claves pblicas del emisor
Necesitamos una
clave de sesin...
Por compatibilidad de
sistemas clientes de
correo, se le aade
armadura (Base 64)
antes de transmitirlo
El documento se comprime
antes con el algoritmo ZIP
Jorge Rami Aguirre Madrid (Espaa) 2003
Seguridad Informtica y Criptografa. Tema 16: Aplicaciones Criptogrficas
Diapositiva 727
Pasos descifrado con clave privada destino
Pasos:
1. PGP busca en la cabecera del criptograma el
identificador de usuario ID (receptor) que se ha
aadido en la clave de sesin cifrada.
2. Se busca la clave privada del identificador ID en el
anillo de claves privadas del receptor.
3. Se accede a la clave privada en claro, descifrndola
con IDEA al introducir el propietario ID su frase de
paso. Slo en ese momento est en claro.
4. Con la clave privada se descifra la clave de sesin.
5. Con la clave de sesin se descifra el criptograma.
Jorge Rami Aguirre Madrid (Espaa) 2003
Seguridad Informtica y Criptografa. Tema 16: Aplicaciones Criptogrficas
Diapositiva 728
Descifrado con la clave privada de destino
Mensaje
cifrado
Clave de
sesin
cifrada
Clave privada
destino cifrada
Clave privada
descifrada
Clave
de
sesin
Mensaje en
claro
Se ha quitado la armadura y se descomprime
Se busca en el anillo de
claves privadas del receptor
CONTRASEA
Jorge Rami Aguirre Madrid (Espaa) 2003
Seguridad Informtica y Criptografa. Tema 16: Aplicaciones Criptogrficas
Diapositiva 729
Firma digital RSA
Mensaje en
claro
Bloque de
firma
digital
Mensaje en
claro
Clave privada
cifrada IDEA
Clave privada
descifrada
Se va a firmar un mensaje en claro
Necesitamos nuestra
clave privada...
Si se desea se puede
enviar tambin cifrado
CONTRASEA
Jorge Rami Aguirre Madrid (Espaa) 2003
Seguridad Informtica y Criptografa. Tema 16: Aplicaciones Criptogrficas
Diapositiva 730
Comprobacin de la firma digital RSA
Mensaje en
claro recibido
Se calcula en destino la funcin hash del mensaje y comparamos
IGUALES ?
Firma
correcta
Firma
incorrecta
S
No
H(M)
enviado
H(M)
calculado
Clave pblica
del emisor
Bloque de
firma digital
Se busca la clave pblica del
emisor para descifrar la firma
Jorge Rami Aguirre Madrid (Espaa) 2003
Seguridad Informtica y Criptografa. Tema 16: Aplicaciones Criptogrficas
Diapositiva 731
Formato de un mensaje PGP dirigido a B
NOMBRE DEL FICHERO
IDENTIFICADOR DE CLAVE PUBLICA DEL RECEPTOR
CLAVE DE SESION
SELLO DE TIEMPO
IDENTIFICADOR DE CLAVE PUBLICA DEL EMISOR
RESUMEN DEL MENSAJE
TEXTO DEL USUARIO
COMPONENTES
DE LA FIRMA
COMPONENTES
DEL MENSAJE
DOS PRIMEROS OCTETOS DEL RESUMEN
SELLO DE TIEMPO
COMPONENTES DE
CLAVE DE SESION
Orden de las operaciones
R64
E(e
B
)
ZIP
Jorge Rami Aguirre Madrid (Espaa) 2003
Seguridad Informtica y Criptografa. Tema 16: Aplicaciones Criptogrficas
Diapositiva 732
Los tiempos cambian, pero ...
La mtica versin de PGP 2.6.3 del MIT se convierte rpidamente
en el software de libre distribucin freeware ms popular en el
mundo de los PCs y especialmente en entornos de correo
electrnico: usa cifra y firma con criptografa calificada como fuerte.
Las versiones en entorno Windows a travs de Network Associates
presentan opciones avanzadas, servicios de red para seguimiento
de paquetes y autenticacin mediante Autoridades de Certificacin.
Existe una versin freeware para usos no comerciales .
Las versiones 5 y 6 tuvieron su cdigo fuente abierto, en la 7 el
cdigo deja de ser pblico y la nueva versin 8.0 (diciembre
2002) ahora con PGP Corporation ha liberado otra vez el cdigo.
Ha vuelto otra vez la cordura? Aqu puede haber varias opiniones....
Jorge Rami Aguirre Madrid (Espaa) 2003
Seguridad Informtica y Criptografa. Tema 16: Aplicaciones Criptogrficas
Diapositiva 733
Algoritmos en nuevas versiones de PGP
Generacin de claves
RSA: 1.024, 1.536, 2.048 bits
Diffie y Hellman: 1.024, 1.536, 2.048, 3.072, 4.096 bits
Firma digital
DSS Digital Signature Standard 1.024 bits
Cifrado
CAST, IDEA, TripleDES, AES, Twofish
Resumen
SHA-1 (160 bits) y MD5 (128 bits)
Jorge Rami Aguirre Madrid (Espaa) 2003
Seguridad Informtica y Criptografa. Tema 16: Aplicaciones Criptogrficas
Diapositiva 734
Algunas versiones de PGP en Windows
Desde la versin 5.0 hasta la actual 8.0 en el momento de escribir este
libro (febrero de 2003) los esquemas de cifra y firma digital han cambiado
muy poco aunque presentan mayores prestaciones. No obstante, recuerde
que algunas prestaciones slo estarn activadas en versiones comerciales.
PGP 7.0.3
PGP 6.5.1 PGP 8.0
Veremos algunas operaciones de estas tres versiones con mayor detalle.
Recuerde, eso s, que la versin 7.0.3 no tiene su cdigo fuente abierto.
Jorge Rami Aguirre Madrid (Espaa) 2003
Seguridad Informtica y Criptografa. Tema 16: Aplicaciones Criptogrficas
Diapositiva 735
Instalacin de la versin PGP 6.5.1
PGP 6.5.1 internacional aparece en el ao 1999. Puede
considerarse como una de las versiones seguras mejor
optimizadas desde la primera en entorno Windows.
Puede descargar
estas versiones
desde el servidor
http://www.pgpi.org
Jorge Rami Aguirre Madrid (Espaa) 2003
Seguridad Informtica y Criptografa. Tema 16: Aplicaciones Criptogrficas
Diapositiva 736
Carpetas y programas de PGP 6.5.1
Encontrar aqu
un excelente
manual de PGP
en formato PDF
Tal vez el programa ms
importante de PGP
La instalacin crear esta carpeta
Jorge Rami Aguirre Madrid (Espaa) 2003
Seguridad Informtica y Criptografa. Tema 16: Aplicaciones Criptogrficas
Diapositiva 737
Opciones generales de PGP 6.5.1
- La generacin rpida
de claves slo puede
hacerse para valores
DH de una longitud
predeterminada.
- Se puede limitar el
tiempo de descifrado
de la frase de paso en
memoria cach.
- El borrado fsico de
datos y ficheros se
hace escribiendo 1s y
0s aleatorios en los
cluster, desde 8 hasta
32 veces.
Jorge Rami Aguirre Madrid (Espaa) 2003
Seguridad Informtica y Criptografa. Tema 16: Aplicaciones Criptogrficas
Diapositiva 738
Opciones de ficheros de PGP 6.5.1
- Los archivos donde
guarda las claves
pblicas y claves
privadas siguen
llamndose pubring y
secring pero ahora, a
diferencia de versiones
anteriores, usa como
extensiones pkr.
- El archivo de semilla
permite generar
nmeros aleatorios
para crear claves.
Jorge Rami Aguirre Madrid (Espaa) 2003
Seguridad Informtica y Criptografa. Tema 16: Aplicaciones Criptogrficas
Diapositiva 739
Opciones de e-mail de PGP 6.5.1
- El PGP/MIME slo
funciona con plugins.
- Se puede pedir que
cifre, firme o descifre
y compruebe firma por
defecto al enviar o
abrir mensajes.
- Si usa Secure Viewer,
al descifrar un archivo
ste slo se muestra en
la pantalla usando para
ello una tcnica de
enmascarado que evita
los ataques por captura
de radiofrecuencias del
teclado, TEMPEST.
Jorge Rami Aguirre Madrid (Espaa) 2003
Seguridad Informtica y Criptografa. Tema 16: Aplicaciones Criptogrficas
Diapositiva 740
Opciones de atajos de PGP 6.5.1
- La opcin de usar
teclas para atajos es
poco interesante pero
puede activarse si se
desea.
Jorge Rami Aguirre Madrid (Espaa) 2003
Seguridad Informtica y Criptografa. Tema 16: Aplicaciones Criptogrficas
Diapositiva 741
Opciones de servidores de PGP 6.5.1
- A estos servidores se
puede enviar nuestra
clave pblica para que
los usuarios accedan
ms fcilmente a ella.
- Es interesante estar
sincronizado con el
servidor por el tema de
las claves revocadas.
Jorge Rami Aguirre Madrid (Espaa) 2003
Seguridad Informtica y Criptografa. Tema 16: Aplicaciones Criptogrficas
Diapositiva 742
Opciones de ACs de PGP 6.5.1
Tambin
VeriSign OnSite
y Entrust
Jorge Rami Aguirre Madrid (Espaa) 2003
Seguridad Informtica y Criptografa. Tema 16: Aplicaciones Criptogrficas
Diapositiva 743
Opciones avanzadas de PGP 6.5.1
- IDEA era en versiones
anteriores el algoritmo
de cifra por defecto.
- El aviso sobre uso de
Additional Decryption
Key (ADK) significa
que el administrador
del sistema puede usar
una clave extra que le
permita descifrar lo
cifrado, en caso de
necesidad o por un
requerimiento judicial.
- El formato compatible
es el cdigo base 64.
Jorge Rami Aguirre Madrid (Espaa) 2003
Seguridad Informtica y Criptografa. Tema 16: Aplicaciones Criptogrficas
Diapositiva 744
El programa PGPtray de acceso directo
Si PGPnet est instalado, en la
barra de tareas, al lado del
candado de PGPtray, nos
aparecer un icono vertical
como se ve en la figura.
- La ventana actual cifra
texto y no documentos
con formato.
- El uso de portapapeles
es la solucin si no
tenemos el plugin para
correo electrnico.
Barra de tareas del PC
Jorge Rami Aguirre Madrid (Espaa) 2003
Seguridad Informtica y Criptografa. Tema 16: Aplicaciones Criptogrficas
Diapositiva 745
Barra flotante de PGPtools
PGPkeys
Encrypt
Sign
Encrypt & Sign
Decrypt/Verify
Wipe
Freespace Wipe
No es muy cmodo y resulta mejor usar
el men contextual con el botn derecho
del ratn. En este caso
sobre el archivo Kerberos_borrador.doc
Jorge Rami Aguirre Madrid (Espaa) 2003
Seguridad Informtica y Criptografa. Tema 16: Aplicaciones Criptogrficas
Diapositiva 746
Generacin de claves con PGPkeys 6.5.1
Esta es la primera pantalla que aparece una vez instalado PGP.
Jorge Rami Aguirre Madrid (Espaa) 2003
Seguridad Informtica y Criptografa. Tema 16: Aplicaciones Criptogrficas
Diapositiva 747
Nombre del usuario y su correo
- No es necesario que
la direccin de
correo sea la real.
No obstante sirve
para los que se
comunican con
nosotros sepan que
esa clave pertenece
a esa direccin de
email.
Jorge Rami Aguirre Madrid (Espaa) 2003
Seguridad Informtica y Criptografa. Tema 16: Aplicaciones Criptogrficas
Diapositiva 748
Eleccin del tipo de clave asimtrica
- El estndar para la
generacin de las
claves asimtricas
es en la actualidad
Diffie y Hellman
junto con la Digital
Signature Standard,
y se representar
como DH/DSS.
- Tambin puede usar
claves RSA que son
compatibles con las
versiones anteriores
de PGP.
Habr nuevas
sorpresas en
otras versiones
Jorge Rami Aguirre Madrid (Espaa) 2003
Seguridad Informtica y Criptografa. Tema 16: Aplicaciones Criptogrficas
Diapositiva 749
Eleccin de la longitud de la clave
- Si genera claves de
longitud distinta a
los valores que se
proponen, puede
tardar bastante
tiempo generarlas.
Algo similar sucede
si no usa la opcin
generacin rpida
de claves.
- Es recomendable
que use una clave
de 2.048 bits. Esta
se generar slo en
esta fase.
`
Jorge Rami Aguirre Madrid (Espaa) 2003
Seguridad Informtica y Criptografa. Tema 16: Aplicaciones Criptogrficas
Diapositiva 750
Clave sin caducidad
- Puede optar porque
su clave no caduque
nunca eligiendo esa
opcin.
Jorge Rami Aguirre Madrid (Espaa) 2003
Seguridad Informtica y Criptografa. Tema 16: Aplicaciones Criptogrficas
Diapositiva 751
Clave con caducidad
- Puede optar que la
clave caduque de
acuerdo con un
calendario que nos
muestra PGP. En el
ejemplo la clave era
vlida desde el 4 de
junio de 2000 al 4
de junio de 2001.
Jorge Rami Aguirre Madrid (Espaa) 2003
Seguridad Informtica y Criptografa. Tema 16: Aplicaciones Criptogrficas
Diapositiva 752
Frase de paso para cifrar la clave privada
- La frase de paso
debe tener varias
palabras para que
sea difcil un ataque
por diccionario.
- Si quita la opcin
Hide Typing podr
ver lo que escribe.
- Por seguridad, no
est permitido usar
el portapapeles para
copiar la frase de
arriba en la casilla
de confirmacin.
No muy buena
Jorge Rami Aguirre Madrid (Espaa) 2003
Seguridad Informtica y Criptografa. Tema 16: Aplicaciones Criptogrficas
Diapositiva 753
Generacin de los nmeros primos
- PGP genera dos
primos tanto para
las claves RSA (los
valores p y q) como
para DH/DSS, en
este caso el primo p
para el intercambio
de clave y el primo
q para la firma DSS.
- Normalmente tarda
pocos segundos si
se eligen los valores
estndar que nos
propone PGP.
Jorge Rami Aguirre Madrid (Espaa) 2003
Seguridad Informtica y Criptografa. Tema 16: Aplicaciones Criptogrficas
Diapositiva 754
Envo de la clave al servidor de claves
- Se puede enviar la
clave a un servidor.
Jorge Rami Aguirre Madrid (Espaa) 2003
Seguridad Informtica y Criptografa. Tema 16: Aplicaciones Criptogrficas
Diapositiva 755
Generacin de clave concluida
- Se ha concluido
satisfactoriamente la
generacin del par
de claves pblica y
privada que se
guardarn en los
anillos pubring.pkr
y secring.pkr.
- La clave privada se
guarda cifrada con
una clave de sesin
que se genera al
aplicar una funcin
hash a la frase de
paso del propietario.
Jorge Rami Aguirre Madrid (Espaa) 2003
Seguridad Informtica y Criptografa. Tema 16: Aplicaciones Criptogrficas
Diapositiva 756
Visualizacin de la clave de Benito
Por defecto, el propietario se firma la clave pblica con su clave privada.
Jorge Rami Aguirre Madrid (Espaa) 2003
Seguridad Informtica y Criptografa. Tema 16: Aplicaciones Criptogrficas
Diapositiva 757
Exportacin de la clave pblica de Benito
Jorge Rami Aguirre Madrid (Espaa) 2003
Seguridad Informtica y Criptografa. Tema 16: Aplicaciones Criptogrficas
Diapositiva 758
Claves de dos usuarios para un ejemplo
U1
U2
Frase de paso para descifrar la clave privada de este usuario
Jorge Rami Aguirre Madrid (Espaa) 2003
Seguridad Informtica y Criptografa. Tema 16: Aplicaciones Criptogrficas
Diapositiva 759
Inclusin de fotografa en clave pblica
Usuario al que se
le aade una
fotografa en
propiedades.
Se firma la fotografa
En negrita el usuario por defecto
Jorge Rami Aguirre Madrid (Espaa) 2003
Seguridad Informtica y Criptografa. Tema 16: Aplicaciones Criptogrficas
Diapositiva 760
Las claves pblicas del ejemplo
Jorge Rami Aguirre Madrid (Espaa) 2003
Seguridad Informtica y Criptografa. Tema 16: Aplicaciones Criptogrficas
Diapositiva 761
Definir un usuario por defecto
El usuario en negrita
es aquel que se ha
definido por defecto.
En cursiva: usuario
cuya clave ha sido
revocada o bien ha
caducado su validez.
Jorge Rami Aguirre Madrid (Espaa) 2003
Seguridad Informtica y Criptografa. Tema 16: Aplicaciones Criptogrficas
Diapositiva 762
Ejemplo de cifra y firma con portapapeles
Era el usuario
por defecto
El texto se escribe
con WordPad o con
el Bloc de Notas y
luego se copia al
portapapeles.
Si lo desea tambin
puede crear el texto
con la opcin Edit y
copiarlo luego al
portapapeles.
Las operaciones
(slo sobre textos)
se realizarn en el
portapapeles por lo
que en este entorno
no crear archivos.
Jorge Rami Aguirre Madrid (Espaa) 2003
Seguridad Informtica y Criptografa. Tema 16: Aplicaciones Criptogrficas
Diapositiva 763
Destinatario y frase de paso para la firma
Al trabajar con el portapapeles,
el archivo cifrado siempre
estar en formato base 64.
Se elige el destino y se
arrastra hacia la zona de
destinatarios. Puede ser
ms de uno y hacerlo con
doble clic. Para quitar a
un receptor se hace doble
clic sobre l.
Como se va a firmar, PGP
pedir la frase de paso del
emisor, la agente Scully.
Jorge Rami Aguirre Madrid (Espaa) 2003
Seguridad Informtica y Criptografa. Tema 16: Aplicaciones Criptogrficas
Diapositiva 764
Documento portapapeles formato base 64
Todo el texto que hay en
el portapapeles, incluido
BEGIN PGP y END PGP,
puede ahora copiarse en
el cuerpo del cliente de
correo electrnico.
Si desea enviar por el
cliente de correo un
archivo adjunto cifrado
y/o firmado, deber crear
primero ese documento
con cualquier programa
como Word, Excel, etc. y
aplicar sobre el archivo el
men contextual del
botn derecho del ratn.
Jorge Rami Aguirre Madrid (Espaa) 2003
Seguridad Informtica y Criptografa. Tema 16: Aplicaciones Criptogrficas
Diapositiva 765
Descifrado del criptograma por destinatario
Introducimos la clave solicitada
para descifrar...
Aunque el usuario por defecto era
Scully, al pedir descifrar lo del
portapapeles, PGP detecta que el
receptor es el usuario Mulder.
Jorge Rami Aguirre Madrid (Espaa) 2003
Seguridad Informtica y Criptografa. Tema 16: Aplicaciones Criptogrficas
Diapositiva 766
Mensaje en claro y firma comprobada
Comprobacin de la firma
Descifrado del
criptograma
Texto en claro
Jorge Rami Aguirre Madrid (Espaa) 2003
Seguridad Informtica y Criptografa. Tema 16: Aplicaciones Criptogrficas
Diapositiva 767
Otras operaciones con PGP
PGP permite hacer otras operaciones interesantes:
Dividir (split) una clave privada en varias subclaves, de
forma que para deshacer una operacin de cifra o firmar un
documento se requiere un umbral de estas subclaves dadas
por diseo. Est basado en el esquema de Blakely-Shamir.
Firmar las claves pblicas de otros usuarios con distintos
niveles de confianza.
Revocar una clave, habilitar o deshabilitar una clave.
Enviar, buscar y actualizar claves desde servidores.
Cifrar con la opcin slo para tus ojos, crear grupos, etc.
Le recomiendo que stas y otras operaciones las realice
a modo de ejercicio, instalando en su PC el programa.
Jorge Rami Aguirre Madrid (Espaa) 2003
Seguridad Informtica y Criptografa. Tema 16: Aplicaciones Criptogrficas
Diapositiva 768
PGP versin 7.0.3
Es bsicamente el mismo programa de la versin 6.5.1 pero
con ligeras diferencias:
Los mayores cambios se observan en men PGP Options.
Incluye opcin de descifrado automtico muy interesante.
Incluye dos nuevos algoritmos: AES y Twofish
La creacin de claves es de forma automtica DH/DSS con
1.024 bits. Si queremos crear claves RSA, debemos entrar
obligatoriamente en la opcin experto.
Aade opciones de configuracin de VPNs.
El peor inconveniente es que su cdigo no es pblico por lo que nadie
puede asegurar que el programa haga exactamente lo que dice que hace,
por ejemplo la fortaleza de la cifra, proteccin ante ataques tempest, etc.
Jorge Rami Aguirre Madrid (Espaa) 2003
Seguridad Informtica y Criptografa. Tema 16: Aplicaciones Criptogrficas
Diapositiva 769
Opciones de VPN en PGP 7.0.3
Jorge Rami Aguirre Madrid (Espaa) 2003
Seguridad Informtica y Criptografa. Tema 16: Aplicaciones Criptogrficas
Diapositiva 770
Autenticacin VPN en PGP 7.0.3
Jorge Rami Aguirre Madrid (Espaa) 2003
Seguridad Informtica y Criptografa. Tema 16: Aplicaciones Criptogrficas
Diapositiva 771
Opciones VPN avanzadas en PGP 7.0.3
Jorge Rami Aguirre Madrid (Espaa) 2003
Seguridad Informtica y Criptografa. Tema 16: Aplicaciones Criptogrficas
Diapositiva 772
Cifrado local de ficheros con PGP 7.0.3
Podemos usar el
botn derecho del
ratn sobre archivo
Leame.doc...
Elegimos la opcin cifrado convencional
Jorge Rami Aguirre Madrid (Espaa) 2003
Seguridad Informtica y Criptografa. Tema 16: Aplicaciones Criptogrficas
Diapositiva 773
Descifrado de ficheros con PGP 7.0.3
El archivo queda cifrado, con icono de
PGP y extensin pgp. Observe que el
archivo original permanece porque no
hemos activado la opcin wipe original.
Pinchando dos veces sobre el icono...
Si olvidamos la clave y usamos wipe,
nunca podremos recuperar el archivo!
Jorge Rami Aguirre Madrid (Espaa) 2003
Seguridad Informtica y Criptografa. Tema 16: Aplicaciones Criptogrficas
Diapositiva 774
Cifrado en modo SDA con PGP 7.0.3
Opcin SDA: Self
Decrypting Archive
Jorge Rami Aguirre Madrid (Espaa) 2003
Seguridad Informtica y Criptografa. Tema 16: Aplicaciones Criptogrficas
Diapositiva 775
Descifrado SDA con PGP 7.0.3
Este archivo
ejecutable se descifra
de forma automtica
en recepcin sin
necesidad de que el
usuario tenga
instalado PGP.
Jorge Rami Aguirre Madrid (Espaa) 2003
Seguridad Informtica y Criptografa. Tema 16: Aplicaciones Criptogrficas
Diapositiva 776
Borrado fsico de archivos con PGP 7.0.3
Jorge Rami Aguirre Madrid (Espaa) 2003
Seguridad Informtica y Criptografa. Tema 16: Aplicaciones Criptogrficas
Diapositiva 777
PGP versin 8.0
Las operaciones sobre archivos para cifra y firma digital
siguen siendo muy similares a las versiones anteriores. La
oferta de producto, adems de la versin freeware contempla:
PGP Desktop
PGP Desktop Upgrade to Enterprise
PGP Enterprise
PGP Mobile
PGP Personal
Adems de las carpetas de instalacin, veremos algunas
de las opciones de configuracin con diferencias notables
respecto a las versiones anteriores.
Jorge Rami Aguirre Madrid (Espaa) 2003
Seguridad Informtica y Criptografa. Tema 16: Aplicaciones Criptogrficas
Diapositiva 778
Carpeta e iconos de PGP versin 8.0
La versin 8.0 se instala en
la carpeta PGP Corporation
que cuelga de la carpeta de
C:/Archivos de Programa.
Incluye un nuevo
programa: PGPdisk
El programa PGPmail
es el antiguo PGPtools
Jorge Rami Aguirre Madrid (Espaa) 2003
Seguridad Informtica y Criptografa. Tema 16: Aplicaciones Criptogrficas
Diapositiva 779
Programa PGPdisk
El programa PGPdisk
permite montar una
seccin del disco como si
se tratase de una unidad
ms en su PC.
De esta forma toda la
informacin que all se
almacene estar protegida
por una clave.
Desgraciadamente esta
opcin no viene incluida
en la edicin freeware.
Jorge Rami Aguirre Madrid (Espaa) 2003
Seguridad Informtica y Criptografa. Tema 16: Aplicaciones Criptogrficas
Diapositiva 780
Opciones generales de PGP 8.0
Incluye al igual que en la
versin 7.03 la opcin de
Single Sign On. Consiste
en permitir la firma
digital de documentos
durante un tiempo dado,
sin tener que introducir
en cada uno de ellos la
frase de paso para
acceder a clave privada.
Las dems opciones son
las mismas, con ligeras
modificaciones.
Jorge Rami Aguirre Madrid (Espaa) 2003
Seguridad Informtica y Criptografa. Tema 16: Aplicaciones Criptogrficas
Diapositiva 781
Opciones avanzadas de PGP 8.0
El algoritmo por defecto
es el nuevo estndar
AES (Rijndael) y cambia
Blowfish por Twofish.
Incluye la opcin de usar
tarjetas inteligentes para
almacenar y gestionar
claves.
Se puede configurar un
backup automtico del
anillo de claves al cerrar
el programa.
Jorge Rami Aguirre Madrid (Espaa) 2003
Seguridad Informtica y Criptografa. Tema 16: Aplicaciones Criptogrficas
Diapositiva 782
Acceso a Wipe Free Space desde PGPmail
Desde PGPtray se
accede a PGPmail
Jorge Rami Aguirre Madrid (Espaa) 2003
Seguridad Informtica y Criptografa. Tema 16: Aplicaciones Criptogrficas
Diapositiva 783
Free Space Wipe con PGP 8.0
Seleccionamos la unidad en la que
vamos a borrar archivos temporales y
el espacio libre al final del cluster de
cada archivo del disco.
Se elige el nmero de pasos que
har el programa de borrado.
Nota: es una accin que toma
bastante tiempo. En este caso
de unidad A:\ y con slo dos
archivos, tres pasadas han
significado ms de 7 minutos
Jorge Rami Aguirre Madrid (Espaa) 2003
Seguridad Informtica y Criptografa. Tema 16: Aplicaciones Criptogrficas
Diapositiva 784
Estndar PKCS
PKCS: Public-Key Cryptography Standards. Conjunto de
especificaciones tcnicas desarrolladas por Netscape, RSA y
otros desarrolladores de informtica con el objeto de
uniformizar las tcnicas de criptografa pblica
Publicacin de la primera versin 1.0 en el ao 1991.
PKCS forma parte de distintos estndares de hecho como ANSI
X9, PKIX, SET, S/MIME y SSL.
A la fecha existen 14 documentos con ttulos genricos que van
desde PKCS #1 a PKCS #15. Los puede descargar desde el
servidor http://www.rsasecurity.com/rsalabs/pkcs/.
Mantendremos los ttulos originales en su versin en ingls de
RSA Security Inc. Public-Key Cryptography Standards PCKS.
Jorge Rami Aguirre Madrid (Espaa) 2003
Seguridad Informtica y Criptografa. Tema 16: Aplicaciones Criptogrficas
Diapositiva 785
Documentos del estndar PKCS (2002)
PKCS #1: RSA Cryptography Standard
PKCS #2: Incluido ahora en PKCS #1
PKCS #3: Diffie-Hellman Key Agreement Standard
PKCS #4: Incluido ahora en PKCS #1
PKCS #5: Password-Based Cryptography Standard
PKCS #6: Extended-Certificate Syntax Standard
PKCS #7: Cryptographic Message Syntax Standard
PKCS #8: Private-Key Information Syntax Standard
PKCS #9: Selected Attribute Types
PKCS #10: Certification Request Syntax Standard
PKCS #11: Cryptographic Token Interface Standard
PKCS #12: Personal Information Exchange Syntax Standard
PKCS #13: Elliptic Curve Cryptography Standard
PKCS #15: Cryptographic Token Information Format Standard
Veremos slo un
breve resumen del
contenido del
estndar PKCS #1
Jorge Rami Aguirre Madrid (Espaa) 2003
Seguridad Informtica y Criptografa. Tema 16: Aplicaciones Criptogrficas
Diapositiva 786
PCKS #1 v2.1 del 14 de Junio 2002 (1)
PKCS #1: RSA Cryptography Standard
Tipos de claves: definicin de claves pblica y privada.
Conversin de primitivas: I2OSP (Integer-to-Octet-String primitive) y
OS2IP (Octet-String-to-Integer primitive).
Primitivas criptogrficas cifra: RSAEP (RSA encryption primitive) y
RSADP (RSA decryption primitive). En este ltimo caso se especifica
la operacin tpica para n = pq y la operacin para n = r
1
r
2
r
3
...r
u
.
Primitivas criptogrficas firma: RSASP1 (RSA signature primitive 1)
especificando la operacin tpica para n = pq y la operacin en caso
de que n = r
1
r
2
r
3
...r
u
y RSAVP1 (RSA verification primitive 1).
Esquemas de cifrado: RSAES-OAEP (RSA encryption scheme usando
Optimal Asymmetric Encryption Padding). Especificacin de las
operaciones de cifrado y descifrado.
Jorge Rami Aguirre Madrid (Espaa) 2003
Seguridad Informtica y Criptografa. Tema 16: Aplicaciones Criptogrficas
Diapositiva 787
PCKS #1 v2.1 del 14 de Junio 2002 (2)
Esquemas de firma con apndice: RSASSA-PSS (RSA signature
scheme with appendix - Probabilistic Signature Scheme). Define la
firma y su verificacin.
Mtodos de codificacin para asignaturas con apndices: EMSA-PSS
(Encoding Method for Signatures with Appendix - Probabilistic
Signature Scheme). Define operaciones de codificacin y verificacin.
Sintaxis ASN.1: define los identificadores de objetos ANS.1 para las
claves pblica y privada RSA, RSAES-OAEP, RSASSA-PSS, etc.
Tcnicas soportadas: algoritmos funciones hash MD2, MD5, SHA-1 y
los propuestos SHA-256, SHA-384 y SHA-512 as como funciones de
generacin de mscaras: MGF1 (Mask Generation Function 1).
La patente de RSA ha expirado en septiembre de 2000 no as el nuevo
sistema de cifra RSA con mltiples primos.
Fin del Tema 16
Jorge Rami Aguirre Madrid (Espaa) 2003
Seguridad Informtica y Criptografa. Tema 16: Aplicaciones Criptogrficas
Diapositiva 788
Cuestiones y ejercicios (1 de 4)
1. Usando la tabla correspondiente represente en base 64 los siguientes
mensajes ASCII: M
1
= AMIGOS, M
2
= Qu pasa?
2. Instale una versin de PGP, vaya a la carpeta del programa y luego
imprima el documento que ver en la carpeta documentation.
3. Con cualquier versin de PGP cifre de forma local y con armadura
(base 64) un archivo que haya creado con el bloc de notas y observe
los rellenos que introduce en el criptograma y al final de l. Aada
una letra al texto en claro y vuelva a comparar los textos ASC.
4. Cifre el documento TXT anterior y observe la salida ASC. Vuelva a
cifrarlo y observe la salida. Coinciden? Qu ha pasado?
5. Es posible que PGP cifre un documento y ste salga en claro?
6. Por qu siempre se comprime el mensaje antes de cifrar?
7. Qu puede decir de la gestin de claves pblicas que ofrece PGP?
Jorge Rami Aguirre Madrid (Espaa) 2003
Seguridad Informtica y Criptografa. Tema 16: Aplicaciones Criptogrficas
Diapositiva 789
Cuestiones y ejercicios (2 de 4)
8. Qu tipo de esquema de cifra es el que utiliza PGP?
9. Despus de instalar PGP en nuestro computador, qu es lo primero
que nos sugiere?
10. Qu diferencia hay entre elegir una clave DH/DSS y RSA?
11. Si creamos un nuevo par de claves asimtricas, queda el nuevo
usuario como usuario por defecto?
12. Cree tres nuevos usuarios Hugo, Paco y Luis con diferentes tipos y
longitudes de clave. Haga que entre ellos se firmen sus claves.
13. Incluya en cada uno una fotografa en sus propiedades. Puede ser
cualquier archivo con formato de imagen.
14. Qu sucede si creamos que un nuevo usuario Ana con una clave
tipo DH/DSS con una longitud exacta de 4.000 bits? Podemos
crear una clave RSA de 4.000 bits?
Jorge Rami Aguirre Madrid (Espaa) 2003
Seguridad Informtica y Criptografa. Tema 16: Aplicaciones Criptogrficas
Diapositiva 790
Cuestiones y ejercicios (3 de 4)
15. Revoque una clave y observe sus propiedades. Se puede recuperar
esa clave? Puede deshabilitar una clave? Qu significa esto?
16. Cree el grupo Sobrinos con los usuarios Hugo, Paco y Luis. Enve
un mensaje a ese grupo eligiendo desde PGPkeys Show Groups.
17. Cree el usuario FirmaEspecial con frase de paso UnaFirmaEspecial
en la que intervengan 4 usuarios, cada uno con una porcin igual de
la clave y umbral 3. Cifre y firme un documento con dicha clave.
Podra alguien tener ms de una participacin de la clave?
18. Mediante el botn derecho del ratn cifre de forma local un archivo
por ejemplo de Word, en modo formato compatible. Vuelva a cifrar
el archivo original con otro nombre pero ahora sin formato base 64.
Cmo son ambos criptogramas? Cmo son sus tamaos en bytes?
19. Si se cifra un archivo txt con la opcin Secure Viewer, se guarda el
archivo descifrado? Es seguro? Puede hacerse con archivo Word?
Jorge Rami Aguirre Madrid (Espaa) 2003
Seguridad Informtica y Criptografa. Tema 16: Aplicaciones Criptogrficas
Diapositiva 791
Cuestiones y ejercicios (4 de 4)
20. Cree el nuevo usuario Pepillo y exporte su clave a un archivo. Borre
ahora este usuario y desde PGPkey importe ese archivo de clave
pblica. Se aade el usuario al anillo de claves pblicas?
21. Qu pasa en un cifrado local de un archivo con self decrypting?
Comprubelo con un archivo cualquiera. Podramos usar esta
opcin si adems de cifrar vamos a firmar ese documento?
22. Qu significa actualizar una clave desde PGPkeys?
23. Aada un nuevo nombre a una clave. Para qu puede servir esto?
24. Qu son el KeyID y el Fingerprint? Qu utilidad puede tener que
la huella dactilar tambin est dada como un conjunto de palabras?
25. Si una clave est revocada, puede recibir archivos cifrados con su
clave pblica? Puede firmar nuevos documentos? Puede descifrar
y/o comprobar documentos anteriores a la fecha de revocacin?
Tema 17
Protocolos y Esquemas Criptogrficos
Seguridad Informtica y Criptografa
Ultima actualizacin: 03/03/03
Archivo con 72 diapositivas
Jorge Rami Aguirre
Universidad Politcnica de Madrid
Material Docente de
Libre Distribucin
v 3.1
Este archivo forma parte de un curso sobre Seguridad Informtica y Criptografa. Se autoriza la
reproduccin en computador e impresin en papel slo con fines docentes o personales, respetando
en todo caso los crditos del autor. Queda prohibida su venta, excepto a travs del Departamento de
Publicaciones de la Escuela Universitaria de Informtica, Universidad Politcnica de Madrid, Espaa.
Curso de Seguridad Informtica y Criptografa JRA
Jorge Rami Aguirre Madrid (Espaa) 2003
Seguridad Informtica y Criptografa. Tema 17: Protocolos y Esquemas Criptogrficos
Diapositiva 794
Definicin de protocolo criptogrfico
Veamos 10 ejemplos
Protocolo: es el conjunto de acciones
coordinadas que realizan dos o ms partes
o entidades con el objeto de llevar a cabo
un intercambio de datos o informacin.
Protocolos criptogrficos sern aquellos
que cumplen esta funcin usando para ello
algoritmos y mtodos criptogrficos.
Permiten dar una solucin a distintos
problemas de la vida real, especialmente
en aquellos en donde puede existir un
grado de desconfianza entre las partes.
Qu es un
protocolo?
Jorge Rami Aguirre Madrid (Espaa) 2003
Seguridad Informtica y Criptografa. Tema 17: Protocolos y Esquemas Criptogrficos
Diapositiva 795
Ejemplos de protocolos criptogrficos (1)
1.- El problema de la identificacin del usuario
Cmo permitir que un usuario se identifique y autentique
ante una mquina -y viceversa- sin que sea posible la
obtencin por un tercero de la clave o password?
2.- El problema del lanzamiento de la moneda
Cmo permitir que dos usuarios realicen una prueba con
probabilidad -como es el lanzamiento de una moneda-
si stos no se encuentran fsicamente frente a frente y,
a la vez, asegurar que ninguno de los dos hace trampa?
Jorge Rami Aguirre Madrid (Espaa) 2003
Seguridad Informtica y Criptografa. Tema 17: Protocolos y Esquemas Criptogrficos
Diapositiva 796
Ejemplos de protocolos criptogrficos (2)
3.- El problema de la firma de contratos
Cmo permitir que dos o ms usuarios que se encuentran
fsicamente alejados puedan realizar la firma de un contrato,
asegurando que ninguno de ellos va a modificar las
condiciones ni negarse a ltima hora a dicha firma?
4.- El problema del descubrimiento mnimo de un secreto
Cmo poder demostrar y convencer a otra persona o
sistema que uno est en posesin de un secreto, sin por ello
tener que desvelarlo ni a ella ni a un tercero?
Jorge Rami Aguirre Madrid (Espaa) 2003
Seguridad Informtica y Criptografa. Tema 17: Protocolos y Esquemas Criptogrficos
Diapositiva 797
Ejemplos de protocolos criptogrficos (3)
5.- El problema del juego de pker mental o por telfono
Cmo permitir que varios usuarios puedan jugar a travs de
la red un juego de pker -o cualquier otro- si no estn
fsicamente en una misma mesa de juego y asegurando, al
mismo tiempo, que ninguno de ellos va a hacer trampa?
6.- El problema de la divisin de un secreto o del umbral
Si tenemos un secreto nico y por tanto muy vulnerable,
cmo permitir que ese secreto se divida en n partes, de
forma que juntando k partes sea posible reconstruirlo y, en
cambio, con k-1 partes imposible su reconstruccin?
Jorge Rami Aguirre Madrid (Espaa) 2003
Seguridad Informtica y Criptografa. Tema 17: Protocolos y Esquemas Criptogrficos
Diapositiva 798
Ejemplos de protocolos criptogrficos (4)
7.- El problema del esquema electoral o voto electrnico
Cmo realizar unas elecciones a travs de una red, de forma
que pueda asegurarse que el voto es nico y secreto, que los
votantes estn autenticados y que ese voto se contabiliza
adecuadamente en el cmputo final?
8.- El problema de la transmisin por canales subliminales
Dos usuarios desean intercambiar informacin a travs de un
tercero del cual desconfan. Cmo pueden hacerlo sin cifrar
la informacin de forma que este tercero slo vea un mensaje
con texto en claro aparentemente inocente?
Jorge Rami Aguirre Madrid (Espaa) 2003
Seguridad Informtica y Criptografa. Tema 17: Protocolos y Esquemas Criptogrficos
Diapositiva 799
Ejemplos de protocolos criptogrficos (5)
9.- El problema del millonario
Dos usuarios desean conocer cul de los dos tiene ms
dinero en su cuenta corriente. Cmo pueden hacerlo de
forma que, una vez terminado el protocolo, ambos sepan
quin de los dos es ms rico sin desvelar la cantidad de
dinero del otro?
10.- El problema del correo electrnico con acuse de recibo
Cmo hacer que una vez recibido un correo electrnico,
ste slo pueda ser ledo (abierto) si el receptor enva, con
anterioridad al emisor, un acuse de recibo como sucede -de
forma similar- con el correo normal certificado?
Jorge Rami Aguirre Madrid (Espaa) 2003
Seguridad Informtica y Criptografa. Tema 17: Protocolos y Esquemas Criptogrficos
Diapositiva 800
Transferencia inconsciente o trascordada
Algoritmo de TI propuesto por Michael Rabin en 1981.
Un usuario A transfiere a un usuario B un dato o
secreto con un cifrado probabilstico del 50%.
El usuario B recibe el dato y tiene una probabilidad
del 50% de descubrir el secreto. Una vez que ha
recibido el dato, B sabe si ste es el secreto o no.
No obstante, el usuario A no tiene forma de saber si el
usuario B ha recibido el secreto o no.
Esta incertidumbre mutua forzar a
los protagonistas a que terminen el
protocolo sin hacer trampas.
Jorge Rami Aguirre Madrid (Espaa) 2003
Seguridad Informtica y Criptografa. Tema 17: Protocolos y Esquemas Criptogrficos
Diapositiva 801
Algoritmo de TI de Rabin (1)
Paso 1 A elige dos primos (p y q), calcula n = pq y
enva el valor n a B.
Paso 2 B elige un nmero aleatorio x del CCR(n) de
forma que que mcd (x,n) = 1, y devuelve a A
el valor K = x
2
mod n.
Paso 3 A calcula las cuatro races de x
2
mod n y
enva a B una de ellas. Las races de x
2
mod n
sern: x, n-x, y, n-y. Slo A puede hacerlo
porque conoce los valores de p y q.
Paso 4 B intenta descubrir el valor de p o q.
Jorge Rami Aguirre Madrid (Espaa) 2003
Seguridad Informtica y Criptografa. Tema 17: Protocolos y Esquemas Criptogrficos
Diapositiva 802
Conclusin del algoritmo de TI de Rabin
Si B recibe x o n-x no ser capaz de encontrar p o q.
No tiene ms informacin que la que tena porque:
x y n-x son valores que conoce (B ha elegido x).
Si B recibe y o n-y, podr encontrar p o q.
En este caso, como x
2
mod n = y
2
mod n, entonces:
(x
2
- y
2
) mod n = (x+y)(x-y) mod n = 0
Luego (x+y)(x-y) = kn y se cumplir que:
p = mcd (x+y, n) y
q = mcd (x-y, n)
Para entenderlo mejor ... veamos un ejemplo
Jorge Rami Aguirre Madrid (Espaa) 2003
Seguridad Informtica y Criptografa. Tema 17: Protocolos y Esquemas Criptogrficos
Diapositiva 803
Ejemplo de algoritmo de TI de Rabin (1)
A Adela tiene como nmeros secretos p y q, valores que
corresponden a la factorizacin del valor n.
B Benito conoce el valor n y deber descubrir, a partir
del protocolo de transferencia inconsciente, p o q.
Ejemplo con valores:
Sea p = 7; q = 13. Luego, n = pq = 713 = 91.
1.- A enva a B el valor n = 91.
2.- B elige al azar del CCR(91) el valor x = 15 y calcula
K = 15
2
mod 91 = 225 mod 91 = 43. Se lo enva a A.
3.- A recibe K = 43 y calcula las 4 races de x
2
mod n.
Jorge Rami Aguirre Madrid (Espaa) 2003
Seguridad Informtica y Criptografa. Tema 17: Protocolos y Esquemas Criptogrficos
Diapositiva 804
Clculo de races de la TI de Rabin
A calcula las dos races de x
2
mod n = K de en p y q:
x
1
2
= K mod p = 43 mod 7 = 1 x
1
= 1
x
2
2
= K mod q = 43 mod 13 = 4 x
2
= 2
Con estos valores usa ahora el Teorema del Resto Chino
Si no recuerda el Teorema del Resto
Chino, repase el archivo SItema05.
Tenamos que: x
1
= 1 y x
2
= 2.
Aplicando entonces la ecuacin del TRC:
No siempre
ser tan fcil
el clculo de
estas races
como se ver
ms adelante
Jorge Rami Aguirre Madrid (Espaa) 2003
Seguridad Informtica y Criptografa. Tema 17: Protocolos y Esquemas Criptogrficos
Diapositiva 805
Aplicacin del TRC en la TI de Rabin
y
1
= inv (n/p, p) = inv (91/7, 7) = inv (13,7) y
1
= 6
y
2
= inv (n/q, q) = inv (91/13, 13) = inv (7,13) y
2
= 2
x = [(n/p)y
1
x
1
+ (n/q)y
2
x
2
] mod n
x = (136x
1
+ 72x
2
) mod 91
Luego para todas las combinaciones x
i
, p y q se tiene:
{x
1
, x
2
} [1,2] x = 15
{x
1
, q-x
2
} [1,13-2] = [1,11] x = 50
{p-x
1
, x
2
} [7-1,2] = [6,2] x = 41
{p-x
1
, q-x
2
} [7-1,13-2] = [6,11] x = 76
15
2
mod 91 = 50
2
mod 91 = 41
2
mod 91 = 76
2
mod 91 = 43.
Adems se cumple que 15 + 76 = 91 = n y 50 + 41 = 91 = n.
Jorge Rami Aguirre Madrid (Espaa) 2003
Seguridad Informtica y Criptografa. Tema 17: Protocolos y Esquemas Criptogrficos
Diapositiva 806
Conclusin del algoritmo de TI de Rabin
A recibir cualquiera de estos cuatro valores:15, 50, 41, 76.
Si A recibe el nmero 15 (el valor que haba enviado a B) o
bien n-15 = 91-15 = 76 (que llamaremos valores x) no tiene
ms datos que los que tena al comienzo del protocolo y no
podr factorizar n.
Si A recibe cualquiera de los otros dos valores enviados por B
(50 41) valores que llamaremos y, podr factorizar n usando
la expresin mcd (x+y, n) con x el valor elegido por A al
comienzo del protocolo, es decir 15.
Si y = 50 mcd (50+15, 91) = mcd (65, 91) = 13 q = 13
Si y = 41 mcd (41+15, 91) = mcd (56, 91) = 7 p = 7
Jorge Rami Aguirre Madrid (Espaa) 2003
Seguridad Informtica y Criptografa. Tema 17: Protocolos y Esquemas Criptogrficos
Diapositiva 807
Eleccin de p y q en algoritmo de Rabin
Para facilitar el clculo de las races de x
2
mod p y x
2
mod q, el usuario
A elegir los valores de p y q de forma que cumplan:
- El valor (p+1) sea divisible por 4.
- El valor (q+1) sea divisible por 4.
Si x
2
mod p = a mod p dos soluciones: x
1
y (p x
1
)
Si x
2
mod q = a mod q dos soluciones: x
2
y (q x
2
)
Estas soluciones se obtienen aplicando el TRC, no obstante si (p+1)
es divisible por 4 entonces para este primo p si x = a
(p+1)/4
se cumple:
(a
(p+1)/4
)
2
mod p = a
(p+1)/2
mod p = a(a
(p-1)/2
) mod p = a
Esto es vlido porque : a
(p-1)/2
mod p = 1. Lo mismo sucede con q.
Luego: x
1
= a
(p+1)/4
mod p y x
2
= a
(q+1)/4
mod q
Jorge Rami Aguirre Madrid (Espaa) 2003
Seguridad Informtica y Criptografa. Tema 17: Protocolos y Esquemas Criptogrficos
Diapositiva 808
Problema lanzamiento de la moneda (1)
Algoritmo propuesto por Mario Blum en 1982.
Se trata de resolver una apuesta entre dos
personas A y B distantes entre s mediante el
lanzamiento de una moneda (cara o cruz).
Situaciones si A lanza la moneda al aire:
Caso 1
1 A lanza la moneda.
2 B hace su apuesta y se lo dice a A.
3 A le dice a B que ha salido justo lo contrario
... independientemente de lo que haya salido.
En este caso el usuario A hace trampa ...
A
Jorge Rami Aguirre Madrid (Espaa) 2003
Seguridad Informtica y Criptografa. Tema 17: Protocolos y Esquemas Criptogrficos
Diapositiva 809
Problema lanzamiento de la moneda (2)
Caso 2
1 A lanza la moneda.
2 B hace su apuesta y se lo dice a A.
3 No sale lo apostado por B y A se lo notifica.
4 B se desmiente y dice que esa era su apuesta.
Ahora es el usuario B quien hace trampa ...
Si A y B estn distantes y no hay un testigo de fe, cmo
puede desarrollarse el algoritmo para que ninguno de los
dos pueda hacer trampa y, si lo hace, el otro lo detecte?
Esquema de Blum
B
Jorge Rami Aguirre Madrid (Espaa) 2003
Seguridad Informtica y Criptografa. Tema 17: Protocolos y Esquemas Criptogrficos
Diapositiva 810
El problema de la moneda segn Blum
Soluciones al problema del lanzamiento de la moneda:
Usar el protocolo de la transferencia inconsciente de Rabin
con probabilidad del 50% ya visto, o bien...
Usar el Esquema General de Blum:
1 A partir de un conjunto de nmeros que la mitad son pares
y la otra impares y una funcin unidireccional f : xy, el
usuario A elige un valor x, calcula y = f (x) y lo enva a B.
2 El usuario B apuesta por la paridad de x.
3 A le muestra a B el verdadero valor de x y su paridad.
Jorge Rami Aguirre Madrid (Espaa) 2003
Seguridad Informtica y Criptografa. Tema 17: Protocolos y Esquemas Criptogrficos
Diapositiva 811
Condiciones del esquema general de Blum
B tendr igual probabilidad de recibir un nmero
par o impar.
A deber tener una probabilidad igual (50%) de
recibir una apuesta par o impar por parte B.
Ninguno de los dos podr hacer trampa.
Bsqueda de esa funcin f?
Antes deberemos explicar qu se entiende
por restos cuadrticos y enteros de Blum
Jorge Rami Aguirre Madrid (Espaa) 2003
Seguridad Informtica y Criptografa. Tema 17: Protocolos y Esquemas Criptogrficos
Diapositiva 812
Restos cuadrticos de Blum
Buscamos una funcin unidireccional con trampa que
cumpla las caractersticas del protocolo anterior.
El valor a es un resto cuadrtico de Blum R
2
mod n si:
x
2
mod n = a
siendo mcd (a,n) = 1
Algn problema? S No sigue la paridad deseada.
Por ejemplo, el resto cuadrtico R
2
= 4 en mod 11 se
obtiene para x = 2 (par) y x = 9 (impar) ya que:
2
2
mod 11 = 4 mod 11 = 4 y 9
2
mod 11 = 81 mod 11 = 4
solucin
Jorge Rami Aguirre Madrid (Espaa) 2003
Seguridad Informtica y Criptografa. Tema 17: Protocolos y Esquemas Criptogrficos
Diapositiva 813
Enteros de Blum
Un entero de Blum es un nmero resultado del producto de dos
primos p y q, ambos congruentes con 3 mdulo 4.
En este caso se cumplir que:
y = x
2
mod n mantendr la paridad con z = y
2
mod n x Z
n
Ejemplo: sea n = 1119 = 209 y el valor x = 24
11 mod 4 = 3; 19 mod 4 = 3 (cumplen congruencia 3 mod 4 `)
y = x
2
mod n = 24
2
mod 209 = 576 mod 209 = 158
z = y
2
mod n = 158
2
mod 209 = 24.964 mod 209 = 93
Como se observa, en este caso y es par y z es impar.
Luego, para todos los restos principales de y = 158 (par) que se obtengan con
valores de x diferentes, el resto cuadrtico z
2
ser siempre el valor 93 (impar).
Jorge Rami Aguirre Madrid (Espaa) 2003
Seguridad Informtica y Criptografa. Tema 17: Protocolos y Esquemas Criptogrficos
Diapositiva 814
Paridad en enteros de Blum
Es importante recalcar que:
Existir igual nmero de soluciones y (pares o
impares) que de soluciones z (pares o impares).
Esto no suceder con enteros que no sean de Blum.
Por lo tanto, esta igualdad de paridad en los valores de
los restos de z y de y, har que desde el punto de vista
del usuario B que recibe como dato el valor z o resto
R
2
enviado por A, exista una equiprobabilidad.
El siguiente cuadro indica la paridad de R
2
para
algunos mdulos enteros y no enteros de Blum.
Jorge Rami Aguirre Madrid (Espaa) 2003
Seguridad Informtica y Criptografa. Tema 17: Protocolos y Esquemas Criptogrficos
Diapositiva 815
Ejemplo de paridad en enteros de Blum

n p q y (pares) y (impares) z (pares) z (impares)
21 3 7 10 10 10 10
33 3 11 12 20 12 20
57 3 19 24 32 24 32
69 3 23 36 32 36 32
77 7 11 36 40 36 40

Paridad de elementos de R
2
para mdulos enteros de Blum
Observe que se obtiene igual cantidad de valores y pares que
de z pares. De la misma forma, se obtiene igual cantidad de
valores y impares que de z impares.
Jorge Rami Aguirre Madrid (Espaa) 2003
Seguridad Informtica y Criptografa. Tema 17: Protocolos y Esquemas Criptogrficos
Diapositiva 816
Ejemplo de paridad en no enteros de Blum

n p q y (pares) y (impares) z (pares) z (impares)
15 3 5 8 6 6 8
35 5 7 14 20 8 26
39 3 13 22 16 16 22

Paridad de elementos de R
2
para mdulos no enteros de Blum
En este caso no se obtienen cantidades iguales de valores y, z.
Como ejercicio, compruebe que los nmeros 21, 33, 57, 69 y 77
del ejemplo anterior son enteros de Blum y que, por el
contrario, 15, 35 y 39 no lo son.
Jorge Rami Aguirre Madrid (Espaa) 2003
Seguridad Informtica y Criptografa. Tema 17: Protocolos y Esquemas Criptogrficos
Diapositiva 817
El algoritmo de Blum
1) A elige dos primos p y q de forma que n = pq es un
entero de Blum (p y q son congruentes con 3 mod 4)
2) A elige un elemento x de Z
n
y calcula y = x
2
mod n.
Luego calcula z = y
2
mod n, valor que enva a B.
3) B recibe z y apuesta por la paridad del valor y.
4) A le informa a B si ha acertado o no en su apuesta.
Le muestra tambin el valor x elegido y el valor de y.
Adems le comprueba que n es un entero de Blum.
5) B comprueba que y = x
2
mod n y que z = y
2
mod n.
6) A y B han actuado con una probabilidad del 50% en
los pasos 2 y 3, respectivamente.
Jorge Rami Aguirre Madrid (Espaa) 2003
Seguridad Informtica y Criptografa. Tema 17: Protocolos y Esquemas Criptogrficos
Diapositiva 818
Ejemplo del algoritmo de Blum
Sean los primos p = 7 y q = 19
Luego, n = pq = 719 = 133
Comprobacin de que n = 133 es un entero de Blum:
7 mod 4 = 3; 19 mod 4 = 3 `
A elige el valor x = 41 y calcula:
y = x
2
mod n
y = 41
2
mod 133 = 1.681 mod 133 = 85
z = y
2
mod n
z = 85
2
mod 133 = 7.225 mod 133 = 43
A enva a B el valor z = 43.
B debe apostar por la paridad de y.
Jorge Rami Aguirre Madrid (Espaa) 2003
Seguridad Informtica y Criptografa. Tema 17: Protocolos y Esquemas Criptogrficos
Diapositiva 819
Conclusin del ejemplo de Blum
Situacin 1 (B acierta)
Si B acierta y dice que y es impar, A no puede negarle que ha
ganado. A debe mostrarle a B los valores x e y. Adems debe
demostrarle a B que n era un entero de Blum.
Situacin 2 (B no acierta)
Si B no acierta y dice que y es par, A le dice a B que ha
perdido, le demuestra que n era un entero de Blum y le
muestra el valor x elegido as como el valor y.
Compruebe que a iguales valores de resto principal y resto cuadrtico
se llega para x = 22, x = 92 y x = 111. Es decir, si se recibe z = 43
(impar) la nica posibilidad es que el valor de y sea 85 (impar) y que
A haya elegido como valor x alguno de stos: 22, 41, 92 111.
Jorge Rami Aguirre Madrid (Espaa) 2003
Seguridad Informtica y Criptografa. Tema 17: Protocolos y Esquemas Criptogrficos
Diapositiva 820
La firma de contratos
Dos personas desean firmar un
contrato sin un ministro de fe.
- Deben cumplirse dos condiciones:
Que los firmantes queden obligados a culminar la
firma slo a partir de un punto del protocolo. Esto
se conoce como compromiso de los contratantes.
Que la firma no pueda falsificarse y que, adems,
pueda ser comprobada por la otra parte.
Un posible algoritmo
Jorge Rami Aguirre Madrid (Espaa) 2003
Seguridad Informtica y Criptografa. Tema 17: Protocolos y Esquemas Criptogrficos
Diapositiva 821
Algoritmo bsico de firma de contratos (1)
1. El usuario A elige dos claves i
A
y j
A
en un sistema de clave
pblica y calcula sus claves privadas i
A
-1
y j
A
-1
.
2. El usuario B elige una clave secreta K
B
.
3. A enva a B sus dos claves pblicas i
A
y j
A
.
4. B elige una de las dos claves recibidas y con ella cifra su
clave K
B
, enviando el resultado al usuario A.
5. A elige al azar una de sus dos claves privadas i
A
-1
y j
A
-1
y
descifra con dicha clave el valor recibido en el punto 4.
6. A cifra el primer bloque del mensaje de firma usando el
valor elegido en el punto 5 como clave y lo enva a B.
7. B descifrar con la clave recibida el bloque de firma.
Jorge Rami Aguirre Madrid (Espaa) 2003
Seguridad Informtica y Criptografa. Tema 17: Protocolos y Esquemas Criptogrficos
Diapositiva 822
Algoritmo bsico de firma de contratos (2)
Observe que los siete pasos
anteriores corresponden
bsicamente al algoritmo de
transferencia inconsciente
entre los usuarios A y B.
Finalizacin
del protocolo
8. A repite la operacin de los pasos 5 y 6 para cada uno de
los bloques de su firma y B el paso 7.
9. Terminados los bloques de su firma, A repite el paso 6
utilizando ahora su otra clave privada y B el paso 7.
Jorge Rami Aguirre Madrid (Espaa) 2003
Seguridad Informtica y Criptografa. Tema 17: Protocolos y Esquemas Criptogrficos
Diapositiva 823
Algoritmo bsico de firma de contratos (3)
Si A y B han elegido al azar la misma clave con una
probabilidad del 50% para cada uno, B descifrar un
mensaje con sentido en la primera vuelta. En caso
contrario, B recibe un texto sin sentido y deber
esperar hasta recibir el ltimo bloque de la segunda
vuelta para obtener el texto en claro.
Sin embargo, A no tiene cmo saber en cul de los dos
pasos (en la primera o la segunda vuelta) ha logrado B
descifrar el criptograma y obtener un texto con sentido
lo que fuerza a ambas partes a terminar el algoritmo.
Jorge Rami Aguirre Madrid (Espaa) 2003
Seguridad Informtica y Criptografa. Tema 17: Protocolos y Esquemas Criptogrficos
Diapositiva 824
Firma de contratos: algoritmo de Even (1)
En el ao 1985 Even, Goldreinch y Lempel proponen el uso
de sistemas de cifra simtricos para la firma de contratos.
1. A elige un conjunto de 2n claves en un sistema simtrico:
C
1
, C
2
, ..., C
n
, C
n+1
, ..., C
2n
. Las claves se tomarn como
parejas, esto es (C
1
, C
n+1
), (C
2
, C
n+2
), ..., (C
n
,C
2n
) aunque
no tengan ninguna relacin entre s.
2. A cifra un mensaje estndar M
A
conocido por B con 2n
claves E
C1
(M
A
), E
C2
(M
A
), ..., E
C2n
(M
A
) y le enva a B
ordenados los 2n criptogramas.
3. A se comprometer ms adelante a la firma del contrato
si B puede presentarle para algn i el par (C
i
, C
n+i
).
Jorge Rami Aguirre Madrid (Espaa) 2003
Seguridad Informtica y Criptografa. Tema 17: Protocolos y Esquemas Criptogrficos
Diapositiva 825
Firma de contratos: algoritmo de Even (2)
4. B elige tambin un conjunto de 2n claves de un sistema
simtrico: D
1
, D
2
,..., D
n
, D
n+1
, ..., D
2n
y las claves se
tomarn como parejas (D
1
, D
n+1
), (D
2
, D
n+2
), ..., (D
n
,D
2n
).
B cifra un mensaje estndar M
B
conocido por A con las
2n claves E
D1
(M
B
), E
D2
(M
B
), ..., E
D2n
(M
B
) y enva a A 2n
criptogramas ordenados. B se comprometer a la firma en
los mismos trminos que lo hizo A en el punto anterior.
5. A enva a B cada par (C
i
, C
n+i
) ordenados mediante una
transferencia inconsciente; es decir enviando C
i
o C
n+i
con igual probabilidad. Lo mismo hace B enviando a A
ordenadamente uno de los dos valores del par (D
i
, D
n+i
).
En este punto A y B tienen la mitad de las claves del otro.
Jorge Rami Aguirre Madrid (Espaa) 2003
Seguridad Informtica y Criptografa. Tema 17: Protocolos y Esquemas Criptogrficos
Diapositiva 826
Firma de contratos: algoritmo de Even (3)
6. Si la longitud de cada clave C
i
o D
i
es de L bits, A y B
realizan el siguiente bucle con 1 i 2n para la clave C
i
y D
i
que no han usado en los pasos anteriores:
for 1 j L
begin
A enva a B el bit jsimo de todas esas claves C
i
B enva a A el bit jsimo de todas esas claves D
i
end (Esto se conoce como compromiso bit a bit)
7. Al realizar el bucle completo, A y B tienen las 2n claves
del otro y se supone firmado el contrato.
A y B pueden generar mensajes del tipo Esta es mi mitad izquierda
i de mi firma para cifrar con la clave C
i
y D
i
y Esta es mi mitad
derecha i de mi firma para cifrar con la clave C
n+i
y D
n+i
Jorge Rami Aguirre Madrid (Espaa) 2003
Seguridad Informtica y Criptografa. Tema 17: Protocolos y Esquemas Criptogrficos
Diapositiva 827
Protocolo de firma ciega
Supongamos que Adela desea que Benito le firme algo pero sin que
Benito se entere de qu es lo que est firmando. En este caso Benito
acta como un ministro de fe, autenticando a Adela.
Protocolo:
Adela pone un documento dentro de un sobre.
Adela cierra el sobre y se lo enva a Benito.
Benito firma el sobre autenticando a Adela y se lo devuelve.
Adela abre el sobre y demuestra que Benito al firmar en el sobre
cerrado tambin ha firmado el documento que estaba en su interior.
En el anterior algoritmo, si Benito necesita una comprobacin de la
identidad Adela, sta sencillamente incluye una firma digital suya en el
sobre que le permita a Benito comprobar su autenticidad.
Jorge Rami Aguirre Madrid (Espaa) 2003
Seguridad Informtica y Criptografa. Tema 17: Protocolos y Esquemas Criptogrficos
Diapositiva 828
Algoritmo de firma ciega RSA (Chaum)
Adela desea que Benito le firme un documento M
Adela (A) conoce las claves pblicas de Benito (B: n
B
, e
B
)
A elige un valor k de forma que mcd (k, n
B
) = 1, calcula
k
-1
= inv (k, n
B
) y luego enmascara su mensaje mediante
la siguiente operacin en n
B
:
t
A
= M k
eB
mod n
B
y lo enva a B
B firma el valor: t
B
= t
A
dB
mod n
B
y lo enva a A
A quita la mscara haciendo s = t
B
inv (k, n
B
) mod n
B
El resultado es que A tiene M
dB
mod n
B
, la firma de B.
Comprobacin: t
B
= (M k
eB
)
dB
mod n
B
= M
dB
k mod n
B
Luego: [M
dB
k inv (k, n
B
)] mod n
B
= M
dB
mod n
B
Jorge Rami Aguirre Madrid (Espaa) 2003
Seguridad Informtica y Criptografa. Tema 17: Protocolos y Esquemas Criptogrficos
Diapositiva 829
Ejemplo de algoritmo de firma ciega
Adelaida (A) desea que Benito (B) le firme el mensaje M = 65
Claves pblicas de B: n
B
= 299, e
B
= 7
Clave privada y datos de B: p
B
= 13; q
B
= 23; (n
B
) = 264, d
B
= 151
A elige k / mcd (k, n
B
), por ejemplo k = 60. Luego inv (k, n
B
) = 5
A enmascara el mensaje: t
A
= M k
eB
mod n
B
= 65 60
7
mod 299
A enva a B: t
A
= 65226 mod 299 = 39
B firma t
A
con clave privada: t
B
= t
A
eB
mod n
B
= 39
151
mod 299 = 104
A quita la mscara: s = t
B
inv (k, n
B
) = 104 5 mod 299 = 221
Este valor (221) es el mismo que se obtendra si B firmase su con
clave privada el mensaje M, es decir 65
151
mod 299 = 221
Jorge Rami Aguirre Madrid (Espaa) 2003
Seguridad Informtica y Criptografa. Tema 17: Protocolos y Esquemas Criptogrficos
Diapositiva 830
Existe el correo electrnico certificado?
Cmo podemos estar seguros que un mensaje enviado por
correo electrnico ha sido abierto y su contenido conocido
slo por su destinatario autorizado?
Los sistemas actuales de e-mail permiten emitir desde
el cliente de correo del receptor un acuse de recibo.
No obstante, esto slo significa que alguien en
extremo receptor desde el buzn de entrada pincha
sobre un mensaje nuevo y a la pregunta enviar acuse
recibo al emisor? pulsa Enter eligiendo la opcin S.
Ser para m
ese e-mail?
Para evitar estas situaciones podemos
usar el protocolo del correo certificado
Jorge Rami Aguirre Madrid (Espaa) 2003
Seguridad Informtica y Criptografa. Tema 17: Protocolos y Esquemas Criptogrficos
Diapositiva 831
El correo electrnico certificado
El usuario A desea enviar un mensaje electrnico
como correo certificado al usuario B.
El usuario A le descubre el mensaje (le enva la
clave) slo despus de que el usuario B le enve
el acuse de recibo correspondiente. De la misma
manera que actuamos ante un correo certificado:
nos entregan la multa si primero firmamos.
El algoritmo ser muy similar al anterior de
firma de contratos propuesto por Even.
Veamos una implementacin del algoritmo
Jorge Rami Aguirre Madrid (Espaa) 2003
Seguridad Informtica y Criptografa. Tema 17: Protocolos y Esquemas Criptogrficos
Diapositiva 832
Un algoritmo de correo certificado (1)
A elige de forma aleatoria n+1 claves (a
0
, a
1
, a
2
, ... a
n
) de un sistema
de cifra simtrico. Las claves a
i
no estn relacionadas.
Con la clave a
0
A cifrar el documento o carta, C
0
= E
a0
(M) y se lo
enva a B.
Las claves (a
1
, a
2
, ... a
n
) sern la parte izquierda de la clave KI
Ai
.
A calcula a
n+i
= a
0
a
i
para 1 i n, obteniendo as la parte derecha
de la clave (a
n+1
, a
n+2
, ... a
2n
) es decir KD
Ai
.
A y B se ponen de acuerdo en un mensaje estndar de validacin, por
ejemplo V = Mensaje de Validacin.
A cifra el mensaje de validacin V con las 2n claves secretas, es decir
n claves KI
Ai
y n claves KD
Ai
.
Cifrado de validacin de la parte izquierda: VI
Ai
= E
KIAi
(V).
Cifrado de validacin de la parte derecha: VD
Ai
= E
KDAi
(V).
A enva a B los pares ordenados (VI
Ai
, VD
Ai
) para i = 1, 2, ... n.
Jorge Rami Aguirre Madrid (Espaa) 2003
Seguridad Informtica y Criptografa. Tema 17: Protocolos y Esquemas Criptogrficos
Diapositiva 833
Un algoritmo de correo certificado (2)
B genera de forma similar n parejas de claves KI
Bi
y KD
Bi
, 2n claves.
B genera n parejas de mensajes Acuse de Recibo de la parte i
Izquierda (RI
i
) y Acuse de Recibo de la parte i Derecha (RD
i
).
B cifra las parejas (RI
i
, RD
i
) con un sistema simtrico usando las claves
KI
Bi
y KD
Bi
.
B enva a A las parejas ordenadas (IB
i
, DB
i
) = [E
KIBi
(RIi), E
KDBi
(RDi)].
Mediante una trasferencia trascordada A enva a B una de las dos claves
secretas (K
IA1
o K
DA1
) y lo mismo hace B que enva a A (K
IB1
o K
DB1
).
Este proceso se repite hasta que se envan los n valores de claves.
B usa las claves enviadas por A en el paso anterior para comprobar que
al descifrar D
KIAi
(V
Ai
) o D
KDAi
(V
Ai
) obtiene el Mensaje de Validacin.
A usa las claves enviadas por B en el paso anterior para comprobar que
al descifrar D
KIBi
(I
Bi
) o D
KDBi
(I
Bi
) obtiene siempre RI
i
o RD
i
.
Jorge Rami Aguirre Madrid (Espaa) 2003
Seguridad Informtica y Criptografa. Tema 17: Protocolos y Esquemas Criptogrficos
Diapositiva 834
Un algoritmo de correo certificado (3)
No pueden hacer trampa. A y B ya tienen informacin suficiente para
demostrar ante un ministro de fe que el otro no ha seguido el protocolo.
A y B se intercambian ahora bit a bit todos los bits de las claves de
forma alterna. El primer bit de KI
A1
, el primer bit de KI
B1
, el primer bit
de KI
A2
, el primer bit de KI
B2
, ... el primer bit de KD
A1
, etc.
Este paso se conoce como compromiso bit a bit entre A y B.
A obtiene todas las claves de B y comprueba todos los Acuse de Recibo
pareados, la parte i Izquierda y su correspondiente parte i Derecha.
B obtiene todas las claves de A y comprueba que todos los envos de A
contienen el Mensaje de Validacin. A deber mostrar todas sus claves
a B para que B compruebe que A ha usado la funcin a
n+1
= a
0
a
i
.
Como B tiene todas las claves de A calcula ahora a
0
= KI
Ai
KD
Ai
.
Para ello cualquiera de las parejas de Acuse de Recibo son vlidas.
B descifra el criptograma Da
0
(C
0
) = M y recupera el mensaje .
Jorge Rami Aguirre Madrid (Espaa) 2003
Seguridad Informtica y Criptografa. Tema 17: Protocolos y Esquemas Criptogrficos
Diapositiva 835
El pker mental con cifra simtrica (1)
Se trata de encontrar un protocolo que permita el juego
del pker a travs de una red de computadores. Se debe asegurar un
juego limpio y sin trampas. Aunque el nmero de jugadores puede
ser cualquiera,veremos un ejemplo slo para dos jugadores.
1. A y B usan un sistema de cifra simtrica que tenga propiedades
conmutativas, usando claves K
A
y K
B
respectivamente.
2. B cifra -accin mezcla- las 52 cartas (codificadas con un nmero
aleatorio c
i
) con su clave secreta K
B
: E
KB
(c
i
) y las enva a A.
3. A elige al azar 5 valores y enva a B: E
KB
(c
B1
, c
B2
, c
B3
, c
B4
, c
B5
).
4. B recibe estos valores y los descifra con su clave secreta K
B
. As
obtiene: D
KB
[E
KB
(c
B1
, c
B2
, c
B3
, c
B4
, c
B5
)] = c
B1
, c
B2
, c
B3
, c
B4
, c
B5
.
Estas cinco cartas c
Bi
corresponden a la mano de B.
Jorge Rami Aguirre Madrid (Espaa) 2003
Seguridad Informtica y Criptografa. Tema 17: Protocolos y Esquemas Criptogrficos
Diapositiva 836
El pker mental con cifra simtrica (2)
5. A elige otras cinco cartas de las 47 restantes, las cifra con su
clave secreta K
A
y enva a B: E
KA
[E
KB
(c
A1
, c
A2
, c
A3
, c
A4
, c
A5
)].
6. B descifra con su clave secreta K
B
la cifra anterior y enva a A el
resultado: E
KA
(c
A1
, c
A2
, c
A3
, c
A4
, c
A5
).
7. A descifra lo anterior con su clave secreta K
A
y obtiene su mano
c
Ai
: D
KA
[E
KA
(c
A1
, c
A2
, c
A3
, c
A4
, c
A5
)] = c
A1
, c
A2
, c
A3
, c
A4
, c
A5
.
8. Las restantes 42 cartas permanecern en poder de A que es quien
las reparte. Estas cartas siguen cifradas con la clave de B.
9. Si los jugadores desean cambiar algunas cartas, siguen el mismo
procedimiento anterior.
Como ejercicio, intente generalizar este esquema para cuatro jugadores.
Esquema con cifra asimtrica
Jorge Rami Aguirre Madrid (Espaa) 2003
Seguridad Informtica y Criptografa. Tema 17: Protocolos y Esquemas Criptogrficos
Diapositiva 837
Pker mental con cifra asimtrica RSA (1)
En este caso se usar un sistema RSA en el que el mdulo de trabajo
n ser compartido y el par de claves asimtricas de cada jugador, e y
d, sern ambas secretas. Veamos un ejemplo para 4 jugadores.
1. El jugador A que repartir las cartas, todas ellas codificadas con
un nmero aleatorio c
i
, las mezclar cifrndolas con su clave
pblica e
A
: E
eA
[c
1
, c
2
, c
3
, ... c
50
, c
51
, c
52
] y las enva a B.
2. B elige cinco cartas, las cifra con su clave pblica e
B
y devuelve
a A : E
eB
{E
eA
[c
B1
, c
B2
, c
B3
, c
B4
, c
B5
]}.
3. A descifra lo recibido con su clave privada d
A
y se lo enva a B:
E
eB
[c
B1
, c
B2
, c
B3
, c
B4
, c
B5
].
4. B descifra ahora con su clave privada d
B
lo recibido y se queda
con su mano c
Bi
= c
B1
, c
B2
, c
B3
, c
B4
, c
B5
.
Jorge Rami Aguirre Madrid (Espaa) 2003
Seguridad Informtica y Criptografa. Tema 17: Protocolos y Esquemas Criptogrficos
Diapositiva 838
Pker mental con cifra asimtrica RSA (2)
5. El jugador B pasa las restantes 47 cartas al jugador C y se
repiten los pasos 2 al 4 anteriores entre C y A, usando ahora las
claves e
C
, d
A
y d
C
.
6. Terminado el paso 5, el jugador C tendr entonces como mano
c
Ci
= c
C1
, c
C2
, c
C3
, c
C4
, c
C5
.
7. El jugador C pasa las restantes 42 cartas al jugador D y se
repiten los pasos 2 al 4 entre D y A, usando ahora las claves e
D
,
d
A
y d
D
.
8. Terminado el paso 7, el jugador D tendr entonces como mano
c
Di
= c
D1
, c
D2
, c
D3
, c
D4
, c
D5
.
9. El jugador D devuelve las 37 cartas que quedan y que estn
cifradas con su clave pblica: E
eD
{E
eA
[c
1
, c
2
, c
3
, ... c
36
, c
37
]} al
jugador A.
Jorge Rami Aguirre Madrid (Espaa) 2003
Seguridad Informtica y Criptografa. Tema 17: Protocolos y Esquemas Criptogrficos
Diapositiva 839
Pker mental con cifra asimtrica RSA (3)
10. El jugador A elige 5 cartas entre las 37 y devuelve al jugador
D: E
eD
{E
eA
[c
A1
, c
A2
, c
A3
, c
A4
, c
A5
]}.
11. El jugador D descifra con su clave privada d
D
lo recibido y
enva a A: E
eA
[c
A1
, c
A2
, c
A3
, c
A4
, c
A5
].
12. El jugador A descifra con su clave privada d
A
lo recibido y se
queda con su mano c
Ai
= c
A1
, c
A2
, c
A3
, c
A4
, c
A5
.
13. Todos tienen su mano de juego. Las restantes 32 cartas quedan
en poder de A cifradas por D y A: E
eD
{E
eA
[c
1
, c
2
, ... c
31
, c
32
]}.
14. Si un jugador X desea descarte, pide las cartas a A, elige las
que desea, las cifra con su clave pblica e
X
y se las devuelve a
A, quien las enva a D para que descifre con su clave privada
d
D
. D las devuelve a A para que descifre con su clave privada
d
A
y A enva a X: E
eX
[cartas elegidas en su descarte].
Jorge Rami Aguirre Madrid (Espaa) 2003
Seguridad Informtica y Criptografa. Tema 17: Protocolos y Esquemas Criptogrficos
Diapositiva 840
El canal subliminal
Como ejemplo de canal subliminal, en un supermercado podran incluir
en la msica ambiental una informacin no audible y que slo nuestro
subconsciente sea capaz de interpretar. No se extrae de ello, este tipo
de experimentos se han probado hace muchos aos atrs.
El concepto de canal subliminal fue propuesto por Gustavus Simmons
en 1983. Se conoce tambin como el problema de los prisioneros.
Dos prisioneros cmplices de un delito son encarcelados en celdas
separadas. Si entre ellos pueden intercambiarse mensajes a travs de un
carcelero que los puede leer, cmo hacen para que esos mensajes en
principio inocentes, lleven de forma subliminal un mensaje cifrado y
que el carcelero sea incapaz de dilucidar ese secreto?
La tcnica denominada esteganografa, hoy en da de moda, realiza una
operacin similar, normalmente ocultando un texto bajo una fotografa.
Jorge Rami Aguirre Madrid (Espaa) 2003
Seguridad Informtica y Criptografa. Tema 17: Protocolos y Esquemas Criptogrficos
Diapositiva 841
El problema de los prisioneros
El prisionero A genera un mensaje inocente M que desea enviar al
prisionero B a travs del guardia.
Utilizando una clave secreta K acordada con anterioridad, el prisionero
A firma el mensaje de forma que en esa firma se esconda el mensaje
subliminal.
El guardia recibe el mensaje firmado por A y como no observa nada
anormal se lo entrega al prisionero B.
El prisionero B comprueba la firma de su compaero A, autentica el
mensaje y lee la informacin subliminal en M.
Existen varios esquemas de uso del canal subliminal para proteger la
informacin, entre ellos el propio esquema de Simmons basado en la
factorizacin de un nmero grande n compuesto por tres primos p, q y r.
Habr por tanto 2
3
= 8 races de las cuales slo algunas se usarn como
valores vlidos y otras no. Hace uso del Teorema del Resto Chino.
Jorge Rami Aguirre Madrid (Espaa) 2003
Seguridad Informtica y Criptografa. Tema 17: Protocolos y Esquemas Criptogrficos
Diapositiva 842
Transferencia con conocimiento nulo TCN
La cueva de Al Bab
A
B
C D
Puerta de la cueva: palabra secreta
Algoritmo:
1. Mortadelo y Filemn se acercan a la
cueva en el punto A.
2. Mortadelo se adentra en la cueva hasta
llegar al punto C o D.
3. Filemn se acerca al punto B de la cueva
y le pide a Mortadelo que salga por la
ladera derecha o izquierda, segn desee.
4. Mortadelo satisface la peticin de
Filemn y sale por la ladera que ste le
ha solicitado, usando si es menester la
palabra secreta para abrir la puerta.
5. Se repite el proceso desde el comienzo
hasta que Filemn se convence que
Mortadelo conoce la palabra secreta.
Este modelo fue presentado
por J. Quisquater y L. Guillou
en Crypto 89 para explicar el
protocolo de transferencia con
conocimiento cero o nulo.
brete ssamo
Jorge Rami Aguirre Madrid (Espaa) 2003
Seguridad Informtica y Criptografa. Tema 17: Protocolos y Esquemas Criptogrficos
Diapositiva 843
Esquema de TCN de Koyama
A desea demostrar a B que conoce la clave secreta RSA de un tercer
usuario C, es decir d
C
. Como es lgico tambin conocer p
C
, q
C
y (n
C
).
Las claves pblicas de C son n
C
y e
C
que conocen tanto A como B.
A y B se ponen de acuerdo y eligen dos valores aleatorios k y m con la
condicin de que km = e
C
mod (n
C
).
Como A debe mantener en secreto el valor de (n
C
) le propone a B que
en cada ejecucin del algoritmo elija un nmero m primo por lo que A
calcula k = [{inv (m, (n
C
)}e
C
] mod (n
C
).
A propone a B un texto aleatorio M o bien A y B generan este texto
usando, por ejemplo, un algoritmo de transferencia trascordada.
Usando la clave privada d
C
de C, ahora A calcula C = M
dC
mod n
C
.
Luego calcula X = C
k
mod n
C
y enva el valor X a B.
B recibe X y comprueba si X
m
mod n
C
es igual al texto M. Si es as,
quiere decir que A ha usado d
C
, la clave privada de C.
Se repite el proceso las veces que haga falta hasta que B acepte que A
conoce clave privada de C.
Jorge Rami Aguirre Madrid (Espaa) 2003
Seguridad Informtica y Criptografa. Tema 17: Protocolos y Esquemas Criptogrficos
Diapositiva 844
Por qu funciona el esquema de Koyama?
Por simplicidad supondremos que los datos de C no tienen subndice:
1. A conoce n, e, d, p, q, (n) y el texto M; B conoce n, e y el texto M.
2. B elige un primo m y se lo enva enva a A.
3. A calcula k = [{inv (m, (n)}e] mod (n).
4. A calcula C = M
d
mod n y X = C
k
mod n = M
dk
mod n y enva este
valor X a B.
5. B recibe X y calcula X
m
mod n = M
(dk)m
mod n = M
kmd
mod n, pero
como km = e mod (n) entonces M
kmd
mod n = M
ed
mod n = M.
6. La nica posibilidad para que B recupere el texto M en el paso 5, es
que A haya usado en la cifra del paso 4 la clave privada d.
7. Si B no se convence en el primer intento, ambos repiten el algoritmo
con valores primos m distintos en cada iteracin, hasta que se cumpla
un umbral ante el que B acepte que A est en posesin de ese secreto.
Jorge Rami Aguirre Madrid (Espaa) 2003
Seguridad Informtica y Criptografa. Tema 17: Protocolos y Esquemas Criptogrficos
Diapositiva 845
Ejemplo del esquema de TCN de Koyama
Supongamos que A desea demostrar a B que conoce la clave privada de
C. Los valores pblicos de C son n = 77, e = 13.
El mensaje M acordado por A y B es la palabra PADRINO con la
codificacin que se muestra a continuacin:
A B C D E F G H I J K L M N O P Q R S T U V W X Y Z
02 03 04 05 06 07 08 09 10 11 12 13 14 15 16 17 18 19 20 21 22 23 24 25 26 27 28
Supongamos que B elige como valor aleatorio m = 29.
A calcula k segn el algoritmo de Koyama y para cada valor M
i
del
mensaje (P = 18, A = 2, D = 5, etc.) calcula primero C = M
i
d
mod n y
luego X = C
k
mod n = 30, 39, 31, 27, 54, 36, 68 que enva a B.
B calcula 30
29
mod 77, 39
29
mod 77, 31
29
mod 77, 27
29
mod 77, 54
29
mod 77, 36
29
mod 77, 68
29
mod 77 y obtiene la cadena de caracteres
PADRINO.
El protocolo puede repetirse para otros valores primos m que elija B y
siempre se obtendr como resultado el mismo mensaje M.
Jorge Rami Aguirre Madrid (Espaa) 2003
Seguridad Informtica y Criptografa. Tema 17: Protocolos y Esquemas Criptogrficos
Diapositiva 846
Solucin del ejemplo de TCN de Koyama
Como n = 77, es obvio que p = 7, q = 11, (n) = 60. Por lo tanto, puesto
que e = 13 entonces d = inv {e, (n)} = inv (13, 60) = 37.
M
1
= 18; M
2
= 2; M
3
= 5; M
4
= 20; M
5
= 10; M
6
= 15; M
7
= 17.
C
1
= 18
37
mod 77 = 39; C
2
= 2
37
mod 77 = 51; C
3
= 5
37
mod 77 = 47;
C
4
= 20
37
mod 77 = 48; C
5
= 10
37
mod 77 = 10; C
6
= 15
37
mod 77 = 71;
C
7
= 17
37
mod 77 = 52.
k = [{inv (m, (n)}e] mod (n) = inv (29, 60)13 mod 60 = 17.
X
1
= 39
17
mod 77 = 30; X
2
= 51
17
mod 77 = 39; X
3
= 47
17
mod 77 = 31;
X
4
= 48
17
mod 77 = 27; X
5
= 10
17
mod 77 = 54; X
6
= 71
17
mod 77 = 36;
X
7
= 52
17
mod 77 = 68. Luego X = 30, 39, 31, 27, 54, 36, 68.
30
29
mod 77 = 18 = P; 39
29
mod 77 = 2 = A; 31
29
mod 77 = 5 = D
27
29
mod 77 = 20 = R; 54
29
mod 77 = 10 = I; 36
29
mod 77 = 15 = N
68
29
mod 77 = 17 = O.
En este ejemplo el valor de n es muy pequeo y resulta muy fcil
romper la clave privada simplemente factorizando el mdulo .
Jorge Rami Aguirre Madrid (Espaa) 2003
Seguridad Informtica y Criptografa. Tema 17: Protocolos y Esquemas Criptogrficos
Diapositiva 847
El voto electrnico o por ordenador
Todos tenemos de una u otra forma una idea intuitiva,
aunque quizs no completa, sobre cmo se desarrolla un
proceso electoral.
La pregunta es si es posible realizar este tipo de eventos
desde Internet, lo que se conoce como esquema electoral.
La respuesta es s con la ayuda de tcnicas y protocolos
criptogrficos aunque no se trata slo de un problema de
implementacin tcnica; es menester tener en cuenta otros
factores importantes, a saber:
Socio-polticos, econmicos, jurdicos, legislativos...
Jorge Rami Aguirre Madrid (Espaa) 2003
Seguridad Informtica y Criptografa. Tema 17: Protocolos y Esquemas Criptogrficos
Diapositiva 848
Definicin de esquema electoral
Un esquema de votacin electrnica es una
aplicacin distribuida y constituida por un conjunto
de mecanismos criptogrficos y protocolos que, de
forma conjunta, permiten que se realicen elecciones
en una red de computadores, de forma segura,
incluso suponiendo que los electores legtimos
pueden tener un comportamiento malicioso.
Andreu Riera (Tesis Doctoral, UAB, 1999)
Jorge Rami Aguirre Madrid (Espaa) 2003
Seguridad Informtica y Criptografa. Tema 17: Protocolos y Esquemas Criptogrficos
Diapositiva 849
Requisitos de un esquema electoral (1)
Requisitos de un esquema electoral:
Slo pueden votar quienes estn censados.
El voto debe ser secreto.
El voto debe ser nico por cada votante.
Se contabilizarn todos los votos vlidos.
El recuento parcial no debe afectar a votos
que se emitan con posterioridad.
sigue
Jorge Rami Aguirre Madrid (Espaa) 2003
Seguridad Informtica y Criptografa. Tema 17: Protocolos y Esquemas Criptogrficos
Diapositiva 850
Requisitos de un esquema electoral (2)
Requisitos de un esquema electoral:
Cada votante podr comprobar que su voto
ha sido tenido en cuenta en el escrutinio.
Esto ltimo es muy importante
Y, adems:
Se debe proteger el proceso contra ataques en red.
El proceso debe ser factible, prctico y dentro de lo
posible de uso universal.
Jorge Rami Aguirre Madrid (Espaa) 2003
Seguridad Informtica y Criptografa. Tema 17: Protocolos y Esquemas Criptogrficos
Diapositiva 851
Primera aproximacin del voto electrnico
MCV = Mesa Central de Votacin
> El votante cifra su voto con la clave pblica de MCV.
> El votante enva su voto a la MCV.
> La MCV descifra el voto y lo contabiliza.
> La MCV hace pblico el resultado.
Qu problemas presenta este esquema? TODOS...
La MCV no sabe de dnde vienen los votos, si stos
son vlidos o no y si alguien vota ms de una vez.
Adems puede conocer la identidad del votante por lo
que se vulnera el secreto del voto. Lo nico que aqu
se protege es el secreto del voto ante terceros.
Jorge Rami Aguirre Madrid (Espaa) 2003
Seguridad Informtica y Criptografa. Tema 17: Protocolos y Esquemas Criptogrficos
Diapositiva 852
Segunda aproximacin del voto electrnico
MCV = Mesa Central de Votacin
> El votante firma su voto con su clave privada y lo
cifra luego con la clave pblica de MCV.
> El votante enva su voto a la MCV.
> La MCV descifra el voto, lo contabiliza y hace
pblico el resultado.
Qu problema tenemos ahora?
En este nuevo esquema se satisface que cada votante
autorizado vote una sola vez, no obstante seguimos
vulnerando el secreto del voto ante la MCV.
Jorge Rami Aguirre Madrid (Espaa) 2003
Seguridad Informtica y Criptografa. Tema 17: Protocolos y Esquemas Criptogrficos
Diapositiva 853
Tercera aproximacin del voto electrnico
El tercer esquema contempla dos mesas:
MCV = Mesa Central de Votacin
MCL = Mesa Central de Legitimacin
> Evita que la MCV conozca a quin ha votado el
votante, mediante un protocolo entre ambas, y
adems gestionan una lista de votantes censados.

MCV y MCL deben ser rganos independientes


Veamos cmo funciona este esquema
Jorge Rami Aguirre Madrid (Espaa) 2003
Seguridad Informtica y Criptografa. Tema 17: Protocolos y Esquemas Criptogrficos
Diapositiva 854
Un protocolo de voto electrnico (1)
1. El votante A enva a la MCL el mensaje:
Buenos das, soy A y vengo a votar.
2. La MCL verifica si A est censado. Si no es
un votante legtimo rechaza la solicitud. Si es
legtimo, le enva un nmero aleatorio de
identificacin nico i(A) y le borra de la lista
para impedir que vuelva a votar.
Caractersticas
de i(A)
Toda la informacin ir
cifrada y firmada
Jorge Rami Aguirre Madrid (Espaa) 2003
Seguridad Informtica y Criptografa. Tema 17: Protocolos y Esquemas Criptogrficos
Diapositiva 855
Un protocolo de voto electrnico (2)
Mucho mayor que el
nmero de votantes.
Por ejemplo, para un
milln de votantes,
unos 10
100
nmeros.
Cules deben ser
las caractersticas
de este nmero
aleatorio?
I(A)
Jorge Rami Aguirre Madrid (Espaa) 2003
Seguridad Informtica y Criptografa. Tema 17: Protocolos y Esquemas Criptogrficos
Diapositiva 856
Un protocolo de voto electrnico (3)
3. La MCL enva a la MCV la lista de nmeros de
validacin.
4. El votante A escoge una identificacin secreta
s(A) y enva a la MCV el mensaje formado por
el tro [i(A), v(A), s(A)] es decir:
su identificacin i(A)
su voto v(A)
su nmero secreto s(A)
Puede generarlo
internamente con su
sistema de cifra. Ser
tambin un valor de
muchos dgitos.
Jorge Rami Aguirre Madrid (Espaa) 2003
Seguridad Informtica y Criptografa. Tema 17: Protocolos y Esquemas Criptogrficos
Diapositiva 857
Un protocolo de voto electrnico (4)
5. La MCV verifica que el nmero i(A) de
identificacin se encuentra en el conjunto N de
los nmeros censados y cruza los datos para
evitar que se vote ms de una vez. Quita i(A) del
conjunto N y aade s(A) al conjunto de electores
que han optado por la opcin v(A).
6. La MCV contabiliza los votos y hace pblico el
resultado, junto con la lista de nmeros secretos
s(A) que han votado a la opcin v(A) ... luego
Jorge Rami Aguirre Madrid (Espaa) 2003
Seguridad Informtica y Criptografa. Tema 17: Protocolos y Esquemas Criptogrficos
Diapositiva 858
Un protocolo de voto electrnico (5)
Cada elector puede comprobar si su voto ha sido
contabilizado sin hacer pblica su opcin.
Qu pasa si MCV y MCL no son independientes?
Si las dos mesas, MCV y MCL, no tienen la idoneidad
y la integridad que se presume, la solucin est en el
uso de una diversidad de esquemas ms desarrollados
que evitan esta anomala mediante protocolos, entre
ellos ANDOS (All-or-Nothing Disclosure Of Secrets)
Distribucin Annima de Nmeros de Validacin,
pero esto ya se escapa del objetivo de este libro.
Jorge Rami Aguirre Madrid (Espaa) 2003
Seguridad Informtica y Criptografa. Tema 17: Protocolos y Esquemas Criptogrficos
Diapositiva 859
Otros esquemas de mesas electorales
Hay muchos otros esquemas con dos mesas, una nica
mesa e incluso ninguna, cada uno con sus caractersticas
propias. Entre ellos tenemos:
- Modelo de Cohen y Fisher (1985)
- Modelo de Fujioka y otros (1992)
- Modelo de Park y otros (1993)
- Modelo de Sako y Killian (1995)
- Modelo de Borrel y Rif (1996)
Observe que son modelos y esquemas muy recientes.
Jorge Rami Aguirre Madrid (Espaa) 2003
Seguridad Informtica y Criptografa. Tema 17: Protocolos y Esquemas Criptogrficos
Diapositiva 860
Estado del arte en voto electrnico
Existen diversos modelos y esquemas, algunos de ellos
probados con xito con un nmero reducido de electores.
No est todava bien solucionado el problema de la
proteccin fsica y lgica de la red ante ataques masivos,
denegacin de servicio, etc. Es el principal problema al
que se enfrentan estos esquemas, su difcil escalabilidad
en sistemas grandes y abiertos.
No obstante, el proceso de unas elecciones va Internet
realizable, prctico y seguro en cuanto a la privacidad y
autenticidad, es completamente factible.
Fin del Tema 17
Jorge Rami Aguirre Madrid (Espaa) 2003
Seguridad Informtica y Criptografa. Tema 17: Protocolos y Esquemas Criptogrficos
Diapositiva 861
Cuestiones y ejercicios (1 de 4)
1. Qu diferencia hay entre un protocolo de red como por ejemplo
TCP/IP con un protocolo criptogrfico?
2. En una transferencia inconsciente de Rabin, A y B se intercambian
lo siguiente. A enva a B el nmero compuesto n = 55, B elige el
valor x = 9 y enva x
2
mod n a A. Qu valores de los 4 que puede
devolver A a B permiten a este ltimo factorizar el cuerpo n?
3. Qu sucede si en el ejemplo anterior B elige x = 10?
4. En el ejemplo anterior, estn bien elegidos por A los valores de p y
q? Qu valores usara si p y q fuesen nmeros mayores que 10?
5. Presente una solucin al problema del lanzamiento de la moneda a
travs del esquema de transferencia inconsciente de Rabin.
6. Calcule todos los valores de x
2
mod 13. Sea a = 2, 3, 4, 5, 6. Cules
son restos cuadrticos de Blum en el cuerpo n = 13?, por qu?
Jorge Rami Aguirre Madrid (Espaa) 2003
Seguridad Informtica y Criptografa. Tema 17: Protocolos y Esquemas Criptogrficos
Diapositiva 862
Cuestiones y ejercicios (2 de 4)
7. Para los restos cuadrticos encontrados en el ejercicio anterior, se
cumple la paridad en el valor de x? Qu significa esto?
8. Cules de los siguientes siete nmeros compuestos son enteros de
Blum: 69, 143, 161, 189, 319, 713, 1.333? Justifquelo?
9. Encuentre todos los restos de y, z para el entero de Blum n = 33.
10. En un protocolo con enteros de Blum, A trabaja en n = 77 y elige el
valor x = 15. Calcula y = x
2
mod n y luego z = y
2
mod n. Enva el
valor z a B. Cul es el escenario del protocolo y cmo trabaja?
11. Qu sucede si en el esquema anterior de Blum el usuario B conoce
el valor de los primos p y q? Funciona as el protocolo?
12. En el algoritmo de firma de contratos con claves asimtricas y una
clave simtrica, cmo puede comprobar el usuario B que A est
usando en cada vuelta una clave privada distinta y no hace trampa?
Jorge Rami Aguirre Madrid (Espaa) 2003
Seguridad Informtica y Criptografa. Tema 17: Protocolos y Esquemas Criptogrficos
Diapositiva 863
Cuestiones y ejercicios (3 de 4)
13. Cmo se entiende el compromiso de firma de A y B en el esquema
de firma de contratos de Even?
14. En el esquema anterior de Even qu relacin tiene el compromiso
bit a bit con el trmino correcto del protocolo? Por qu estn A y
B obligados a terminar el protocolo hasta el ltimo bit?
15. Se desea que el usuario B le firme de forma ciega al usuario A el
mensaje M = 100. Si n
B
= 253, e
B
= 19 y el usuario A elige k = 25,
realice y compruebe el protocolo de firma ciega.
16. Para qu podra servir un protocolo como el de firma ciega?
17. Por qu decimos que el actual acuse de recibo de los clientes de
correo electrnico no corresponde a uno verdadero?
18. En el algoritmo de correo con acuse de recibo, compruebe que B
obtiene la clave de descifrado del mensaje haciendo KI
Ai
KD
Ai
.
Jorge Rami Aguirre Madrid (Espaa) 2003
Seguridad Informtica y Criptografa. Tema 17: Protocolos y Esquemas Criptogrficos
Diapositiva 864
Cuestiones y ejercicios (4 de 4)
19. Generalice el pker mental con cifra simtrica para 4 jugadores.
20. Qu diferencia hay en cuanto a la eleccin de cartas de una mano
entre el esquema de pker mental con cifra simtrica y el esquema
con cifra asimtrica? Es esto un inconveniente o no?
21. En el esquema de Quisquater y Guillou de conocimiento nulo, si
Mortadelo y Filemn repiten el protocolo 20 veces, cul es la
probabilidad de que el primero engae al segundo?
22. Usando el software Fortaleza de la asignatura (ver Web) , repita el
ejercicio de TCN de Koyama con n = 465.256.980.233 y e = 4.171.
B elige el valor m = 131, el mensaje M es el mismo y se recibe:
X
1
= 394.106.275.745; X
2
= 342.981.204.125; X
3
= 49.911.481.740;
X
4
= 366.983.136.296; X
5
= 56.903.681.682; X
6
= 246.374.030.904;
X
7
= 254.152.395.874. Qu valor tiene la clave privada d?
Tema 18
Bibliografa, Enlaces, SW y Tablas
Seguridad Informtica y Criptografa
Ultima actualizacin: 03/03/03
Archivo con 50 diapositivas
Jorge Rami Aguirre
Universidad Politcnica de Madrid
Material Docente de
Libre Distribucin
v 3.1
Este archivo forma parte de un curso sobre Seguridad Informtica y Criptografa. Se autoriza la
reproduccin en computador e impresin en papel slo con fines docentes o personales, respetando
en todo caso los crditos del autor. Queda prohibida su venta, excepto a travs del Departamento de
Publicaciones de la Escuela Universitaria de Informtica, Universidad Politcnica de Madrid, Espaa.
Curso de Seguridad Informtica y Criptografa JRA
Jorge Rami Aguirre Madrid (Espaa) 2003
Seguridad Informtica y Criptografa. Tema 18: Bibliografa, Enlaces, SW y Tablas
Diapositiva 866
Bibliografa recomendada en castellano (1)
Bibliografa en castellano comentada ordenada alfabticamente
Caballero, Pino
INTRODUCCIN A LA CRIPTOGRAFA. SEGUNDA EDICIN
Editorial Ra-Ma, Textos Universitarios, Madrid
Ao 2002 (133 pginas)
Libro de introduccin a las tcnicas criptogrficas que presenta estos
temas bajo una orientacin matemtica clara y precisa. Actualizacin de la
primera edicin de 1996 en la que trata los temas de criptografa terica,
criptografa de clave secreta y pblica, problemas de autenticacin y
accesos y algunas aplicaciones criptogrficas. Para un buen seguimiento
de la lectura, en algunos apartados es recomendable contar con una base
de conocimientos en matemticas a nivel universitario.
Jorge Rami Aguirre Madrid (Espaa) 2003
Seguridad Informtica y Criptografa. Tema 18: Bibliografa, Enlaces, SW y Tablas
Diapositiva 867
Bibliografa recomendada en castellano (2)
Fster, Amparo; De la Gua, Dolores; Hernndez, Luis; Montoya, Fausto;
Muoz, Jaime
TCNICAS CRIPTOGRFICAS DE PROTECCIN DE DATOS.
SEGUNDA EDICIN
Editorial Ra-Ma
Ao 2000 (372 pginas)
Detallado y actualizado resumen de las tcnicas de cifra modernas,
profundizando en los sistemas de clave secreta con cifra en flujo y en
bloque, de clave pblica y sus aplicaciones en redes. De especial inters
resulta el captulo dedicado a protocolos criptogrficos y sus apndices en
donde explica los mtodos matemticos usados en criptografa y nociones
sobre complejidad computacional. La segunda edicin incluye adems una
coleccin de problemas y sus soluciones en un disquete.
Jorge Rami Aguirre Madrid (Espaa) 2003
Seguridad Informtica y Criptografa. Tema 18: Bibliografa, Enlaces, SW y Tablas
Diapositiva 868
Bibliografa recomendada en castellano (3)
Morant Ramn, J.L.; Ribagorda Garnacho, A.; Sancho Rodrguez J.
SEGURIDAD Y PROTECCIN DE LA INFORMACIN
Coleccin de Informtica, Editorial Centro de Estudios Ramn Areces,
S.A., Madrid
Ao 1994 (388 pginas)
Libro que trata, adems de los temas genricos de la criptografa clsica y
moderna, aspectos de seguridad en sistemas operativos, en bases de datos
y en redes de computadores. Buen texto descriptivo que profundiza en
ciertos aspectos matemticos y hace un buen estudio de la gestin de
claves. Tiene adems como caracterstica ser el primer libro con formato
universitario sobre criptografa y seguridad informtica en Espaa, y
seguramente de lengua espaola.
Jorge Rami Aguirre Madrid (Espaa) 2003
Seguridad Informtica y Criptografa. Tema 18: Bibliografa, Enlaces, SW y Tablas
Diapositiva 869
Bibliografa recomendada en castellano (4)
Pastor, Jos; Sarasa, Miguel Angel
CRIPTOGRAFA DIGITAL. FUNDAMENTOS Y APLICACIONES
Prensas Universitarias de Zaragoza
Ao 1998 (597 pginas)
Extenso y completo texto sobre tcnicas criptogrficas modernas, con una
buena cantidad de ejemplos y profusin de tablas con datos de inters.
Destacan los captulos de cifra con clave secreta, en donde se estudian
ms de una docena de criptosistemas, los de clave pblica y su aplicacin
en firmas digitales y un captulo dedicado a protocolos criptogrficos. En
particular, estn muy bien tratados los apndices con temas matemticos
as como los algoritmos de factorizacin y del logaritmo discreto, algo no
muy comn y que se agradece. Para el buen seguimiento es necesario
contar con una buena base matemtica.
Jorge Rami Aguirre Madrid (Espaa) 2003
Seguridad Informtica y Criptografa. Tema 18: Bibliografa, Enlaces, SW y Tablas
Diapositiva 870
Bibliografa recomendada en castellano (5)
Singh, Simon (traduccin de Jos Ignacio Moraza)
LOS CDIGOS SECRETOS
Editorial Debate S.A.
Ao 2000 (382 pginas)
Interesante y completo libro de Simon Singh editado en 1999 en el que se
hace un repaso extenso de la criptografa denominada clsica, mquinas y
artilugios de cifra, mquina Enigma, etc., desde una perspectiva un tanto
novelesca que, sin desmerecer en absoluto la calidad tcnica del mismo, lo
convierte en un excelente libro de lectura. Encontrar en l una interesante
presentacin de los sistemas de cifra asimtrica, la historia inmersa en la
bsqueda de la criptografa de clave pblica y el intercambio de clave,
para terminar con PGP y un captulo dedicado a la criptografa cuntica.
Jorge Rami Aguirre Madrid (Espaa) 2003
Seguridad Informtica y Criptografa. Tema 18: Bibliografa, Enlaces, SW y Tablas
Diapositiva 871
Bibliografa recomendada en ingls (1)
Bibliografa en ingls comentada ordenada alfabticamente
Denning, Dorothy
CRYPTOGRAPHY AND DATA SECURITY
Addison-Wesley Publishing Company, London
Ao 1982 (400 pginas)
Libro clsico de criptografa, si bien antiguo en comparacin con la
bibliografa actual. Trata los algoritmos y tcnicas criptogrficas clsicas y
modernas, control de accesos y de flujo. Tiene un buen nmero de
ejemplos resueltos y ejercicios propuestos aunque sin sus soluciones. De
especial inters son algunos algoritmos desarrollados en PASCAL. Es, no
obstante, un referente bsico de acuerdo a su fecha de edicin.
Jorge Rami Aguirre Madrid (Espaa) 2003
Seguridad Informtica y Criptografa. Tema 18: Bibliografa, Enlaces, SW y Tablas
Diapositiva 872
Bibliografa recomendada en ingls (2)
Menezes, Alfred; Oorsschof, Paul; Vanstone, Scott
HANDBOOK OF APPLIED CRYPTOGRAPHY
CRC Press Inc.
Ao 1997 (780 pginas)
Interesante y completo libro dedicado al estudio de los algoritmos con una
visin matemtica de alto nivel. Sus captulos estn orientados a bases
matemticas para la criptografa, sistemas de claves secretas y pblicas,
cifradores de flujo y de bloque, hash, autenticacin, firma digital y gestin
de claves. Obra imprescindible para el estudiante universitario que desea
profundizar en el anlisis de los algoritmos, si bien el seguimiento del
mismo puede resultar algo complejo por el nivel matemtico comentado.
Le recomiendo que descargue el libro de forma gratuita desde la pgina
Web de su autor http://www.cacr.math.uwaterloo.ca/hac/.
Jorge Rami Aguirre Madrid (Espaa) 2003
Seguridad Informtica y Criptografa. Tema 18: Bibliografa, Enlaces, SW y Tablas
Diapositiva 873
Bibliografa recomendada en ingls (3)
Pflegger P., Charles
SECURITY IN COMPUTING
Prentice-Hall International Editions, London
Ao 1989 (538 pginas)
Texto de consulta general sobre seguridad informtica que trata los
criptosistemas y adems estudia la seguridad en los programas, en
informtica personal, en las comunicaciones, anlisis de riesgos, as como
los aspectos legales y ticos de esta especialidad. Incluye varios ejercicios
propuestos sin incluir sus soluciones. Debido a la fecha de edicin, no
profundiza en aspectos de cifra asimtrica y algoritmos actuales.
Jorge Rami Aguirre Madrid (Espaa) 2003
Seguridad Informtica y Criptografa. Tema 18: Bibliografa, Enlaces, SW y Tablas
Diapositiva 874
Bibliografa recomendada en ingls (4)
Salomaa, Arto
PUBLIC-KEY CRYPTOGRAPHY. SECOND EDITION
EATCS Monographics on Theoretical Computer Science
W. Brauer, G. Rozenberg, A. Salomaa (Eds.), Springer-Verlag, New York
Ao 1996 (268 pginas)
Interesante y ameno, profundiza en los criptosistemas de clave pblica y
protocolos criptogrficos en esta segunda edicin. En algunos captulos es
necesario contar con una base matemtica de nivel universitario para una
mejor comprensin. No obstante, su lectura es muy agradable y presenta
algunos ejemplos resueltos. Incluye un amplio estudio de los sistemas de
mochilas, su implementacin, debilidades, tipos de ataques, etc.
Jorge Rami Aguirre Madrid (Espaa) 2003
Seguridad Informtica y Criptografa. Tema 18: Bibliografa, Enlaces, SW y Tablas
Diapositiva 875
Bibliografa recomendada en ingls (5)
Schneier, Bruce
APPLIED CRYPTOGRAPHY: PROTOCOLS, ALGORITHMS, AND
SOURCE CODE IN C. SECOND EDITION
John Wiley & Sons, Inc., New York
Ao 1996 (758 pginas)
Segunda edicin del libro con mismo ttulo, con mayor nfasis en los
algoritmos de cifra, protocolos, firmas, etc., especialmente en los sistemas
con clave privada y pblica. El texto de Schneier resulta fundamental para
los estudiantes universitarios de asignaturas de criptografa. Incluye una
bibliografa es muy completa, hay profusin de tablas y estudia una
infinidad de criptosistemas, adjuntando el cdigo fuente en C de muchos
de los algoritmos estudiados. Se echa en falta, no obstante, que no tenga
algunos ejemplos resueltos en cada captulo.
Jorge Rami Aguirre Madrid (Espaa) 2003
Seguridad Informtica y Criptografa. Tema 18: Bibliografa, Enlaces, SW y Tablas
Diapositiva 876
Bibliografa recomendada en ingls (6)
Seberry, Jennifer; Pieprzyk, Josef
CRYPTOGRAPHY. AN INTRODUCTION TO COMPUTER
SECURITY
Prentice-Hall, New York
Ao 1989 (375 pginas)
Adems de los temas propios de criptografa clsica y moderna, incluye
un captulo de introduccin a la aritmtica modular bien estructurado.
Trata tambin la seguridad informtica en bases de datos, en sistemas
operativos y en redes. Como lector se agradece en especial la gran
cantidad de ejercicios propuestos y resueltos en cada captulo, incluyendo
el cdigo en PASCAL.
Jorge Rami Aguirre Madrid (Espaa) 2003
Seguridad Informtica y Criptografa. Tema 18: Bibliografa, Enlaces, SW y Tablas
Diapositiva 877
Bibliografa recomendada en ingls (7)
Stallings, William
CRYPTOGRAPHY AND NETWORK SECURITY. PRINCIPLES AND
PRACTICE. THIRD EDITION
Prentice-Hall Inc.
Ao 2002 (696 pginas)
Con 130 pginas ms que la segunda edicin de mismo ttulo (1999), el
texto est estructurado de una forma ptima que permite una agradable
lectura. Adems de cifra simtrica y asimtrica, algoritmos, firmas, hash,
autenticacin y seguridad en redes, esta edicin se centra en la seguridad
en Internet, profundizando en temas como correo seguro, protocolos de
redes, IP seguro, seguridad en Web, intrusiones, cortafuegos, etc. Incluye
algunos ejemplos y ejercicios. Es para muchos probablemente el mejor
libro de criptografa y seguridad informtica en estos momentos.
Jorge Rami Aguirre Madrid (Espaa) 2003
Seguridad Informtica y Criptografa. Tema 18: Bibliografa, Enlaces, SW y Tablas
Diapositiva 878
Bibliografa de consulta y de inters (1)
Bibliografa interesante ordenada alfabticamente por nombre del autor
Amoroso, Edward. Fundamental of Computer Security Technology.
Prentice Hall, 1994.
Anderson, Ross. Security Engineering. John Wiley & Sons, 2001.
Bauer, Friedrich. Decrypted Secrets. Methods and Maxims of
Cryptology. Springer-Verlag, 1997.
Brassars, Gilles. Cryptologie Contemporaine. Edit. Masson, Paris,
1993.
Braun, Christoph. Unix System Security Essentials. Addison-Wesley,
1994.
Buck, Edward. Introduction to Data Security & Controls. QED
Technical Publishing Group, 1991.
Contreras, J.M.; Gonzlez, M.; Chamorro, R. Correo Electrnico en
Internet. Paraninfo, 1997.
Cobb, Sthephen. Manual de Seguridad para PC y Redes Locales.
McGraw-Hill, 1994.
Jorge Rami Aguirre Madrid (Espaa) 2003
Seguridad Informtica y Criptografa. Tema 18: Bibliografa, Enlaces, SW y Tablas
Diapositiva 879
Bibliografa de consulta y de inters (2)
Chapman, Brent; Zwicky, Elizabeth. Construya Firewalls para Internet.
McGraw-Hill, 1997.
Cheswick, William; Bellovin, Steven. Firewalls and Internet Security.
Addison-Wesley, 1994.
De Marcelo, Jess. Virus de Sistemas Informticos e Internet. Ra-Ma,
2000.
Deavours, C.; Khan, D.; Kruh, L.; Mellen, G. Winkel, B. Cryptology:
Machines, History & Methods. Artech House, 1989.
Del Peso Navarro, Emilio; Ramos Gonzlez, Miguel Angel.
Confidencialidad y Seguridad de la Informacin: La LORTAD y sus
implicaciones socioeconmicas. Daz de Santos, 1994.
Diffie, Whitfield; Landau, Susan. Privacy on the Line. The Politics of
Wiretapping and Encryption. MIT Press, 1998.
Farley, Marc; Stearns, Tom; Hsu, Jeffrey. Seguridad e Integridad de los
Datos. McGraw-Hill, 1998.
Jorge Rami Aguirre Madrid (Espaa) 2003
Seguridad Informtica y Criptografa. Tema 18: Bibliografa, Enlaces, SW y Tablas
Diapositiva 880
Bibliografa de consulta y de inters (3)
Feghhi, Jalal; Feghhi, Jalil; Williams, Peter. Digital Certificates.
Applied Internet Security. Addison-Wesley, 1998.
Fisher, Royal. Seguridad en los Sistemas Informticos. Ediciones Daz
de Santos, 1988.
Ford, Warmick. Computer Communications Security. Principles,
Standards Protocols and Techniques. Prentice Hall, 1994.
Hunter, John. An Information Security Handbook. Springer, 2001.
Garfinkel, Simson; Spafford, Gene. Seguridad Prctica en Unix e
Internet. McGraw-Hill, 1999.
Giblin, Peter. Primes and Programming. An Introduction to Number
Theory with Computing. Cambridge University Press, 1994.
Golomb, Solomon. Shift Register Sequences. Aegean Park Press, 1982.
Gollman, Dieter. Computer Securiry. John Wiley & Sons, 1999.
Graff, Jon. Cryptography and E-Commerce. John Wiley & Sons, 2001.
Jaworski, Jamie; Perrone, Paul. Seguridad en Java. Prentice-Hall, 2001.
Jorge Rami Aguirre Madrid (Espaa) 2003
Seguridad Informtica y Criptografa. Tema 18: Bibliografa, Enlaces, SW y Tablas
Diapositiva 881
Bibliografa de consulta y de inters (4)
Klander, Lars. Hacker Proof. The Ultimate Guide to Network Security.
Gulf Publishing Company, 1997. A Prueba de Hackers. Anaya, 1998.
Knudsen, Jonathan. Java Cryptography. OReilly, 1998.
Konheim, Alan G. Cryptography: A Primer. John Wiley & Sons, 1981.
Loshin, Pete. Big Book of IPSec RFCs. Internet Security Architecture.
Morgan Kaufmann, 2000.
Madron, Thomas W. Network Security in the '90s. Issues and Solutions
for Managers. John-Wiley & Sons, Inc., 1992.
Menezes, Alfred. Elliptic Curve Public Key Cryptosystems. Kluwer
Academic Publishers, Fourth Printing, 1997.
Meyer, C.; Matyas, S. Cryptography: A New Dimension in Computer
Data Security. John Wiley & Sons, 1982.
New Riders (varios autores). Implementing Internet Security. New
Riders Publishing NRP, 1995.
Khan, David. The Codebreakers. The Story of Secret Writing.
Macmillan Publishing Company, New York, 1967.
Jorge Rami Aguirre Madrid (Espaa) 2003
Seguridad Informtica y Criptografa. Tema 18: Bibliografa, Enlaces, SW y Tablas
Diapositiva 882
Bibliografa de consulta y de inters (5)
Northcutt, Stephen; Novak, Judy. Deteccin de Intrusos. Segunda
Edicin. Gua Avanzada. Prentice Hall, 2001.
Oppliger, Rof. Sistemas de Autentificacin para Seguridad en Redes.
Ra-Ma, 1998.
Piattini, Mario, Del Peso, Emilio. Auditora Informtica. Un Enfoque
Prctico. Ra-ma, 1998.
Poe, Edgar Allan. El Escarabajo de Oro. Anaya Tus Libros, 1995.
Ribagorda, Arturo. Glosario de Trminos de Seguridad de las T.I.
Ediciones CODA, 1997.
Ribenboim, Paulo. The Little Book of Big Primes. Springer-Verlag,
1991.
Rodrguez Prieto, A. Proteccin de la Informacin. Paraninfo, 1986.
Rueppel, Rainer A. Analysis and Design of Stream Ciphers. Springer-
Verlag, 1986.
Russell, Deborah; Gangemi Sr., G.T. Computer Security Basics.
O'Reilly & Associates, Inc., 1991.
Jorge Rami Aguirre Madrid (Espaa) 2003
Seguridad Informtica y Criptografa. Tema 18: Bibliografa, Enlaces, SW y Tablas
Diapositiva 883
Bibliografa de consulta y de inters (6)
Schneier, Bruce. E-Mail Security. How to Keep your Electronic
Messages Private. John-Wileys & Sons, Inc., 1995.
Sgarro, A. Cdigos Secretos. Pirmide, 1989.
Smith, Richard E. Internet Cryptography. Addison-Wesley, 1997.
Stallings, William. Network Security Essential. Aplications and
Standards. Prentice Hall, 2000.
Stallings, William. Protect Your Privacy. A Guide for PGO Users.
Prentice Hall, 1995.
Stein, Lincoln D. Web Security. A Step-by-Step Reference Guide.
Addison-Wesley, 1998.
Tena, Juan. Codificacin de la Informacin . Universidad de
Valladolid, 1997.
Thomas, Stephen. SSL and TLS Essentials. John Wiley & Sons, 2000.
Tung, Brian. Kerberos. A Network Authentication System, Addison-
Wesley, 1999.
Jorge Rami Aguirre Madrid (Espaa) 2003
Seguridad Informtica y Criptografa. Tema 18: Bibliografa, Enlaces, SW y Tablas
Diapositiva 884
Bibliografa de consulta y de inters (7)
Van Tilborg, H.C.A. An Introduction to Cryptology. Kluwer Academic
Publishers, 1988.
Wadlow, Thomas. The Pricess of Network Security. Addison-Wesley,
2000.
White, Gregory, Fisch, Eric, Pooch, Udo. Computer System and
Network Security. CRC Press Inc., 1996.
Zimmermann, Philip R. The Official PGP User's Guide. MIT Press,
1995.
En los ltimos dos o tres aos, la bibliografa relacionada con la
criptografa y seguridad informtica ha aumentado de una forma
espectacular. La bibliografa presentada en este captulo como
de consulta est actualizada slo hasta finales del ao 2001. Hoy
cada mes aparecen tres o cuatro libros sobre esta temtica en
el mercado. No obstante, los libros que se han recomendado y
comentado s son actuales y permiten estar al da en este tema.
Jorge Rami Aguirre Madrid (Espaa) 2003
Seguridad Informtica y Criptografa. Tema 18: Bibliografa, Enlaces, SW y Tablas
Diapositiva 885
Enlaces de inters en Internet (1)
Los enlaces que puede encontrar en Internet sobre temas de seguridad
informtica son muchos. Por ejemplo, una consulta al buscador Google
por criptografa entrega 45.000 enlaces; sobre computer security
417.000; por cryptography 937.000; por hackers ms de 4 millones,
etc. En estas tres diapositivas pondremos slo unos cuantos enlaces que
se recomiendan al lector, en los que encontrar informacin interesante.
RedIRIS
http://www.rediris.es/
CSIC Dpto. de Tratamiento de la Informacin
http://www.iec.csic.es/
Criptonomicn
http://www.iec.csic.es/criptonomicon/
Hispasec
http://www.hispasec.com/
VirusProt
http://www.virusprot.com/
Kriptpolis
http://www.kriptopolis.com/
CriptoRed
http://www.criptored.upm.es/
Revista SIC
http://www.revistasic.com/
Organizacin ISACA
http://www.isaca.org/
Computer Security Resource Center del NIST
http://csrc.nist.gov/
CERT de Carnegie Mellon
http://www.cert.org/
National Security Agency
http://www.nsa.gov/
Jorge Rami Aguirre Madrid (Espaa) 2003
Seguridad Informtica y Criptografa. Tema 18: Bibliografa, Enlaces, SW y Tablas
Diapositiva 886
Enlaces de inters en Internet (2)
Pgina Web de Alfred Menezes
http://www.cacr.math.uwaterloo.ca/~ajmeneze/
Pgina Web de Bruce Schneier
http://www.counterpane.com/schneier.html
Pgina Web del libro de William Stallings
http://williamstallings.com/Crypto3e.html
Pgina Web de Solomon Golomb
http://csi.usc.edu/faculty/golomb.html
Pgina Web de Vincent Rijmen: Rijndael
http://www.esat.kuleuven.ac.be/~rijmen/rijndael/
Pgina Web de Philip Zimmermann
http://www.philzimmermann.com/
Crypto++ Library
http://www.eskimo.com/~weidai/cryptlib.html
Conceptos sobre voto electrnico
http://oasis.dit.upm.es/~jantonio/documentos/voto-
electronico/article.html
Esteganografa
http://www.stegoarchive.com/
Software libre de esteganografa
http://members.tripod.com/steganography/stego/so
ftware.html
Pgina de criptografa visual de Doug Stinson
http://www.cacr.math.uwaterloo.ca/~dstinson/vis
ual.html
Quantum Cryptography Tutorial
http://www.cs.dartmouth.edu/~jford/crypto.html
PGP internacional
http://www.pgpi.org/
Computer Security Information
http://www.alw.nih.gov/Security/security.html
Cryptographic Compendium de John Savard
http://home.ecn.ab.ca/~jsavard/crypto/entry.htm
PKCS standards: RSA Security Corporation
http://www.rsasecurity.com/rsalabs/pkcs/
The Prime Pages
http://www.utm.edu/research/primes/
Polinomios primitivos
http://mathworld.wolfran.com/primitivepolynomi
al.html
SW para generacin de polinomios primitivos
http://www.theory.csc.uvic.ca/~cos/gen/poly.html
Cryptome: documentos sobre criptografa
http://cryptome.org/
Jorge Rami Aguirre Madrid (Espaa) 2003
Seguridad Informtica y Criptografa. Tema 18: Bibliografa, Enlaces, SW y Tablas
Diapositiva 887
Enlaces de inters en Internet (3)
Delitos Informticos y Leyes
http://www.delitosinformaticos.com/
Electronic Frontier Foundation EFF
http://www.eff.org/
Leyes, LOPD, 17799, etc. CSI MAP - Espaa
http://www.map.es/csi/fr600001.htm#5
SSH Cryptography A-Z
http://www.ssh.com/support/cryptography/
Linux Security
http://www.linuxsecurity.com/
Ley Orgnica de Proteccin de Datos LOPD
http://www.igsap.map.es/cia/dispo/lo15-99.htm
The Internet Engineering Task Force
http://www.ietf.org/
FIPS Federal Inf. Process. Standards Publications
http://www.itl.nist.gov/fipspubs/
RFC Editor Homepage
http://www.rfc-editor.org/
Open SSL
http://www.openssl.org/
ANSI American National Standards Institute
http://www.ansi.org/
ETSI. Poltica y normas de seguridad Europa
http://www.etsi.org/technicalfocus/home.htm
Cryptography and Inform. Security Group MIT
http://theory.lcs.mit.edu/~cis/
Computer Forensics Laboratory
http://www.dcfl.gov/index.shtml
VeriSign
http://www.verisign.com/
The Hacker Quarterly
http://www.2600.com/
Web spoofing. Universidad de Princeton
http://www.cs.princeton.edu/sip/pub/spoofing.h
tml
Apache Software Foundation
http://www.apache.org/
GNU Software de seguridad
http://www.gnu.org/directory/security/
Netscape Security
http://wp.netscape.com/security/
Windows Security
http://www.microsoft.com/security/
Jorge Rami Aguirre Madrid (Espaa) 2003
Seguridad Informtica y Criptografa. Tema 18: Bibliografa, Enlaces, SW y Tablas
Diapositiva 888
Software libro electrnico de cifra clsica
Autor: Ana Mara Camacho Hernndez (1998).
Enlace: http://www.criptored.upm.es/paginas/software.htm#propio.
Libro electrnico realizado con ToolBook que hace un repaso a los temas
principales de la criptografa clsica, incluyendo fotografas de mquinas de
cifrar as como de los algoritmos ms caractersticos para la realizacin de
prcticas sencillas en un entorno Windows. Cuenta con cinco secciones:
Seccin 0: Historia de la criptografa. Principios de las tcnicas criptogrficas.
Presentacin de los algoritmos esctala, de Polybios y del Csar.
Seccin 1: Mquinas de cifrar. Rueda de Jefferson, discos de Alberti y
Wheatstone, cifrador de Vernam, mquinas Enigma, M-325 y Hagelin.
Seccin 2: Cifrados por sustitucin. Monoalfabtica, polialfabtica, cifra de
Vigenre, de Beaufort, por homofonas, de Beale, de Playfair y de Hill.
Seccin 3: Cifrados por transposicin. Cifra por grupos, series, columnas y filas.
Seccin 4: Algoritmos. Estn implementados en la misma aplicacin todos los
algoritmos de cifrado y descifrado por sustitucin y transposicin para poder
ejercitarse con ejemplos de textos introducidos por teclado.
Jorge Rami Aguirre Madrid (Espaa) 2003
Seguridad Informtica y Criptografa. Tema 18: Bibliografa, Enlaces, SW y Tablas
Diapositiva 889
Software CriptoClsicos
Autor: Luis Miguel Motrel Berjano (1999).
Enlace: http://www.criptored.upm.es/paginas/software.htm#propio.
Aplicacin para prcticas de sistemas de cifra clsicos que incluye algoritmos
de cifra monoalfabtica por sustitucin y por transposicin, cifra polialfabtica
con los cifradores de Vigenre, de Beaufort y de clave continua.
Permite, adems de cifrar y descifrar, realizar ataques por criptoanlisis a los
sistemas anteriores mediante el uso de tcnicas de estadsticas del lenguaje.
Incluye adems el cifrador de Vernam, el cifrador de Playfair, el cifrador de Hill
digrmico y cifrado por transposiciones.
Todas las operaciones de cifra pueden realizarse con los alfabetos castellano
mdulo 27 (letras maysculas), castellano mdulo 37 (letras y dgitos) e ingls
mdulo 26 y mdulo 36.
Incluye un apartado con herramientas caractersticas de trabajo dentro de un
cuerpo finito y estadsticas bsicas del lenguaje. Las operaciones estn
limitadas a cuerpos menores que 65.536.
Ayuda en formato Windows estndar y contextual mediante la tecla F1.
Jorge Rami Aguirre Madrid (Espaa) 2003
Seguridad Informtica y Criptografa. Tema 18: Bibliografa, Enlaces, SW y Tablas
Diapositiva 890
Software cifrador de Hill
Autor: M Carmen Cogolludo Alcarazo (2001).
Enlace: http://www.criptored.upm.es/paginas/software.htm#propio.
Aplicacin para prcticas de laboratorio con el cifrador poligrmico de Hill.
Permite cifrar y descifrar archivos txt con una matriz clave de tamao 2x2 hasta
10x10 dentro de los siguientes cuerpos: alfabeto castellano con letras en
maysculas (mod 27), alfabeto incluyendo adems los dgitos (mod 37) y por
ltimo un subconjunto de caracteres ASCII imprimibles (mod 191). En este
ltimo caso, la salida puede guardarse como un archivo en formato base 64.
Las matrices clave pueden guardarse como un archivo.
Permite adems realizar ataques por criptoanlisis segn el mtodo de Gauss-
Jordan. Una vez criptoanalizada la matriz clave, entrega un seguimiento de las
ecuaciones que han permitido romper el sistema.
El programa incluye una herramienta para el clculo del determinante de una
matriz, la matriz inversa y el nmero de matrices vlidas dentro de un cuerpo.
Ayuda en formato Windows estndar y contextual mediante la tecla F1.
Jorge Rami Aguirre Madrid (Espaa) 2003
Seguridad Informtica y Criptografa. Tema 18: Bibliografa, Enlaces, SW y Tablas
Diapositiva 891
Software cifrador de mochilas M-H
Autor: Juan Carlos Rodrguez Castellano (1997).
Enlace: http://www.criptored.upm.es/paginas/software.htm#propio.
Software para prcticas de cifrado y descifrado del sistema de cifra con mochila
de Merkle - Hellman realizado en Delphi.
Se ha incluido una librera para trabajar con nmeros grandes: decenas o
centenas de dgitos, y herramientas bsicas de trabajo dentro de un cuerpo.
Permite el diseo de mochilas del tamao y datos que desee el usuario, con
tamao recomendable M-H o bien mochilas con un tamao proporcional al
modelo recomendado por Merkle y Hellman.
Una vez creada una mochila, el programa incluye la opcin de criptoanlisis de
la msima segn el mtodo de Shamir y Zippel, indicando luego de romper la
mochila clave los valores analizados hasta lograr su objetivo
Ayuda en formato Windows estndar y contextual mediante la tecla F1.
Jorge Rami Aguirre Madrid (Espaa) 2003
Seguridad Informtica y Criptografa. Tema 18: Bibliografa, Enlaces, SW y Tablas
Diapositiva 892
Software: CripMod sistemas modernos
Autores: Jos Alberto Charfol Sancho y D. Abel Gregorio Palomino (1997).
Enlace: http://www.criptored.upm.es/paginas/software.htm#propio.
Software para prcticas de sistemas de cifra modernos de clave privada y
pblica realizado en Delphi y con entorno de ayuda en castellano e ingls.
Incluye cifrado y descifrado con DES, Data Encryption Standard, en todos sus
modos de cifra, adems de Triple DES.
La generacin de claves puede ser por parte del usuario o bien la genera el
programa. Detecta claves dbiles y semidbiles.
Incluye una aplicacin de cifra y firma RSA y ElGamal, limitados a cuerpos de
tamao menor que 65.536.
Incluye un sistema bsico de cifra con curvas elpticas.
Cuenta con un apartado dedicado a herramientas tpicas de trabajo dentro de
un cuerpo: primalidad, mximo comn divisor, inversos, indicador de Euler.
Ayuda en formato Windows estndar y contextual mediante la tecla F1.
Presentacin de entradas y resultados en ASCII y hexadecimal.
Jorge Rami Aguirre Madrid (Espaa) 2003
Seguridad Informtica y Criptografa. Tema 18: Bibliografa, Enlaces, SW y Tablas
Diapositiva 893
Software: Fortaleza de cifrados
Autor: Cristina Chrcoles Larriba (1999).
Enlace: http://www.criptored.upm.es/paginas/software.htm#propio.
Software de prcticas realizado en Visual Basic que permite realizar y simular
las operaciones dentro de un cuerpo ms caractersticas en sistemas de cifra
exponencial como son RSA y ElGamal: operaciones bsicas de suma, resta,
multiplicacin y divisin, con o sin mdulo, raz, mximo comn divisor, clculo
de inversos, potencia y primalidad.
Usa una librera de nmeros grandes: decenas hasta centenas de dgitos.
Incluye un mdulo de factorizacin de nmeros compuestos por dos primos
mediante los mtodos de Pollard Rho, Dixon y Fraccciones Continuas. Adems
se muestra una lista de primos para poder trabajar con ellos, un conjunto de
ejemplos y una tabla de primos titnicos.
Incluye un mdulo de clculo del logaritmo discreto mediante los mtodos de
Bsqueda Exhaustiva, Paso Gigante - Paso Enano y Pohlig - Hellman. Adems
presenta un conjunto de ejemplos.
Ayuda en formato Windows estndar y contextual mediante la tecla F1.
Jorge Rami Aguirre Madrid (Espaa) 2003
Seguridad Informtica y Criptografa. Tema 18: Bibliografa, Enlaces, SW y Tablas
Diapositiva 894
Software: CriptoRES funciones hash
Autor: Jos Azaa Alonso (2001).
Enlace: http://www.criptored.upm.es/paginas/software.htm#propio.
Software para el estudio y seguimiento de las funciones hash ms conocidas
usadas en la compresin del documento para la firma digital y tambin en otras
aplicaciones de autenticacin como los certificados digitales.
Tanto para MD5 como para SHA-1, la aplicacin obtiene los resmenes de
documentos de archivos o texto introducido por teclado.
Permite hacer un seguimiento del algoritmo de resumen, a nivel de bloques
mostrando todas las operaciones en hexadecimal o bien siguiendo los pasos de
las operaciones en bajo nivel. Como en este ltimo caso la informacin
mostrada est en bits, por su gran extensin muestra slo el primer bloque de
resumen que es precisamente donde se incluyen los rellenos para congruencia
con el tamao del bloque de 512 bits.
Incluye una representacin grfica de la ecuacin matemtica del problema del
ataque por la paradoja del cumpleaos.
Ayuda en formato Windows estndar y contextual mediante la tecla F1.
Jorge Rami Aguirre Madrid (Espaa) 2003
Seguridad Informtica y Criptografa. Tema 18: Bibliografa, Enlaces, SW y Tablas
Diapositiva 895
Software: CryptoIDEA
Autor: Esther Snchez Mellado (1999).
Enlace: http://www.criptored.upm.es/paginas/software.htm#propio.
Software de prcticas realizado en Delphi que permite cifrar y descifrar con el
algoritmo IDEA as como realizar un seguimiento del proceso y la generacin
de claves directas e inversas de cifra.
Muestra adems los ficheros en nmeros enteros y en binario.
Incluye un sistema de gestin de bases de datos tipo Paradox 5.0 para el
mantenimiento (creacin, modificacin, borrado, etc.) de claves.
La generacin de claves se hace a partir de un texto o clave ASCII.
Incluye un apartado de herramientas para clculos tpicos dentro de un cuerpo,
en especial mod 65.536 y mod 65.537, valores usados en IDEA. Adems el
clculo del mximo comn divisor, inversos y conversiones de carcter a ASCII,
de entero a binario y de binario a entero.
Ayuda en formato Windows estndar y contextual mediante la tecla F1.
Jorge Rami Aguirre Madrid (Espaa) 2003
Seguridad Informtica y Criptografa. Tema 18: Bibliografa, Enlaces, SW y Tablas
Diapositiva 896
Otro software que se usa en la asignatura
El en proyecto de desarrollo de software de prcticas para la asignatura de
Seguridad Informtica, hay otros programas que se estn llevando a cabo y que
en los siguientes meses se pondrn en el servidor Web ya comentado, entre
ellos:
Laboratorio de cifra RSA que permite comprobar las debilidades asociadas a
este tipo de cifra: claves parejas, mensajes no cifrables, ataque cclico, etc.
Laboratorio de ataque al DES. Adems de cifrado y descifrado con claves en
ASCII o hexadecimal, muestra la debilidad de las claves del DES ante un
ataque monousuario, distribuido simulado o distribuido a travs de la red.
Simulacin de las operaciones de autenticacin de Kerberos en una aplicacin
de tipo Windows para prcticas.
Laboratorio de seguimiento del algoritmo Rijndael.
Etc.
Otros programas ya desarrollados y que se usan de forma interna, no se han
hecho pblicos al no cumplir alguna condicin bsica para su libre distribucin
como puede ser el tamao de la aplicacin, su funcionalidad, etc.
Jorge Rami Aguirre Madrid (Espaa) 2003
Seguridad Informtica y Criptografa. Tema 18: Bibliografa, Enlaces, SW y Tablas
Diapositiva 897
Tabla de frecuencia de monogramas
A 7,49 A 10,60 A 9,83 -,-- 0,10 0,07
B 1,29 B 1,16 B 0,86 O 7,37 O 8,23 O 7,75
C 3,54 C 4,85 C 4,15 P 2,43 P 2,71 P 2,41
D 3,62 D 5,87 D 4,04 Q 0,26 Q 0,74 Q 0,73
E 14,00 E 13,11 E 11,41 R 6,14 R 6,95 R 5,26
F 2,18 F 1,13 F 0,81 S 6,95 S 8,47 S 7,13
G 1,74 G 1,40 G 0,85 T 9,85 T 5,40 T 3,62
H 4,22 H 0,60 H 0,57 U 3,00 U 4,34 U 3,24
I 6,65 I 7,16 I 6,04 V 1,16 V 0,82 V 0,69
J 0,27 J 0,25 J 0,17 W 1,69 W 0,12 W 0,00
K 0,47 K 0,11 K 0,00 X 0,28 X 0,15 X 0,18
L 3,57 L 4,42 L 4,34 Y 1,64 Y 0,79 Y 0,63
M 3,39 M 3,11 M 2,42 Z 0,04 Z 0,26 Z 0,29
N 6,74 N 7,14 N 6,03 Blanco 6,33
Valores de frecuencia en tanto por ciento en archivos de 50.000 caracteres.
Columnas: 1 Ingls (mod 26); 2 Castellano (mod 27); 3 Castellano (mod 28)
Jorge Rami Aguirre Madrid (Espaa) 2003
Seguridad Informtica y Criptografa. Tema 18: Bibliografa, Enlaces, SW y Tablas
Diapositiva 898
Monogramas ms frecuentes mod 27
E 13,11 C 4,85 Y 0,79
A 10,60 L 4,42 Q 0,74
S 8,47 U 4,34 H 0,60
O 8,23 M 3,11 Z 0,26
I 7,16 P 2,71 J 0,25
N 7,14 G 1,40 X 0,15
R 6,95 B 1,16 W 0,12
D 5.87 F 1,13 K 0,11
T 5,40 V 0,82 0,10
Frecuencia alta Frecuencia media Frecuencia baja
Con los 9 caracteres ms frecuentes podemos formar la palabra ESTIRANDO
Jorge Rami Aguirre Madrid (Espaa) 2003
Seguridad Informtica y Criptografa. Tema 18: Bibliografa, Enlaces, SW y Tablas
Diapositiva 899
Alfabeto castellano y sus inversos mod 27
A B C D E F G H I J K L M N O P Q R S T U V W X Y Z
00 01 02 03 04 05 06 07 08 09 10 11 12 13 14 15 16 17 18 19 20 21 22 23 24 25 26
Alfabeto y tabla mdulo 27
26 26 13 25 20 23
16 22 23 20 10 19
8 17 22 16 2 14
25 13 5 11 19 10
17 8 4 7 11 5
7 4 14 2 1 1
inv (a,27) a inv (a,27) a inv (a,27) a
Jorge Rami Aguirre Madrid (Espaa) 2003
Seguridad Informtica y Criptografa. Tema 18: Bibliografa, Enlaces, SW y Tablas
Diapositiva 900
Alfabeto castellano y sus inversos mod 37
A B C D E F G H I J K L M N O P Q R S T U V W X Y Z
00 01 02 03 04 05 06 07 08 09 10 11 12 13 14 15 16 17 18 19 20 21 22 23 24 25 26
Alfabeto y tabla mdulo 37
0 1 2 3 4 5 6 7 8 9
27 28 29 30 31 32 33 34 35 36
36
30
24
18
12
6
a
18
23
29
24
27
15
inv (a,37)
35
29
23
17
11
5
a
12
4
32
7
26
28
inv (a,37)
34
28
22
16
10
4
a
33
27
21
15
9
3
a
32
26
20
14
8
2
a
6
3
2
20
16
1
inv (a,37)
22
10
13
8
14
19
inv (a,37)
9
11
30
5
33
25
inv (a,37)
31
25
19
13
7
1
a
36
21
17
35
34
31
inv (a,37)
Jorge Rami Aguirre Madrid (Espaa) 2003
Seguridad Informtica y Criptografa. Tema 18: Bibliografa, Enlaces, SW y Tablas
Diapositiva 901
Tabla de Vigenre
A B C D E F G H I J K L M N O P Q R S T U V W X Y Z
A A B C D E F G H I J K L M N O P Q R S T U V W X Y Z
B B C D E F G H I J K L M N O P Q R S T U V W X Y Z A
C C D E F G H I J K L M N O P Q R S T U V W X Y Z A B
D D E F G H I J K L M N O P Q R S T U V W X Y Z A B C
E E F G H I J K L M N O P Q R S T U V W X Y Z A B C D
F F G H I J K L M N O P Q R S T U V W X Y Z A B C D E
G G H I J K L M N O P Q R S T U V W X Y Z A B C D E F
H H I J K L M N O P Q R S T U V W X Y Z A B C D E F G
I I J K L M N O P Q R S T U V W X Y Z A B C D E F G H
J J K L M N O P Q R S T U V W X Y Z A B C D E F G H I
K K L M N O P Q R S T U V W X Y Z A B C D E F G H I J
L L M N O P Q R S T U V W X Y Z A B C D E F G H I J K
M M N O P Q R S T U V W X Y Z A B C D E F G H I J K L
N N O P Q R S T U V W X Y Z A B C D E F G H I J K L M
O P Q R S T U V W X Y Z A B C D E F G H I J K L M N
O O P Q R S T U V W X Y Z A B C D E F G H I J K L M N
P P Q R S T U V W X Y Z A B C D E F G H I J K L M N O
Q Q R S T U V W X Y Z A B C D E F G H I J K L M N O P
R R S T U V W X Y Z A B C D E F G H I J K L M N O P Q
S S T U V W X Y Z A B C D E F G H I J K L M N O P Q R
T T U V W X Y Z A B C D E F G H I J K L M N O P Q R S
U U V W X Y Z A B C D E F G H I J K L M N O P Q R S T
V V W X Y Z A B C D E F G H I J K L M N O P Q R S T U
W W X Y Z A B C D E F G H I J K L M N O P Q R S T U V
X X Y Z A B C D E F G H I J K L M N O P Q R S T U V W
Y Y Z A B C D E F G H I J K L M N O P Q R S T U V W X
Z Z A B C D E F G H I J K L M N O P Q R S T U V W X Y
Jorge Rami Aguirre Madrid (Espaa) 2003
Seguridad Informtica y Criptografa. Tema 18: Bibliografa, Enlaces, SW y Tablas
Diapositiva 902
Cdigo Baudot (cifrador de Vernam)
Cdigo Binario Carcter Cdigo Binario Carcter
00000 Blanco 10000 T
00001 E 10001 Z
00010 = 10010 L
00011 A 10011 W
00100 Espacio 10100 H
00101 S 10101 Y
00110 I 10110 P
00111 U 10111 Q
01000 < 11000 O
01001 D 11001 B
01010 R 11010 G
01011 J 11011
01100 N 11100 M
01101 F 11101 X
01110 C 11110 V
01111 K 11111
Jorge Rami Aguirre Madrid (Espaa) 2003
Seguridad Informtica y Criptografa. Tema 18: Bibliografa, Enlaces, SW y Tablas
Diapositiva 903
Cdigo ASCII/ANSI de nivel bajo (1)
Byte carcter
0010 0000 Espacio
0010 0001 !
0010 0010
0010 0011 #
0010 0100 $
0010 0101 %
0010 0110 &
0010 0111
0010 1000 (
0010 1001 )
0010 1010 *
0010 1011 +
0010 1100 ,
0010 1101 -
0010 1110 .
0010 1111 /
Byte carcter
0100 0000 @
0100 0001 A
0100 0010 B
0100 0011 C
0100 0100 D
0100 0101 E
0100 0110 F
0100 0111 G
0100 1000 H
0100 1001 I
0100 1010 J
0100 1011 K
0100 1100 L
0100 1101 M
0100 1110 N
0100 1111 O
Byte carcter
0011 0000 0
0011 0001 1
0011 0010 2
0011 0011 3
0011 0100 4
0011 0101 5
0011 0110 6
0011 0111 7
0011 1000 8
0011 1001 9
0011 1010 :
0011 1011 ;
0011 1100 <
0011 1101 =
0011 1110 >
0011 1111 ?
Jorge Rami Aguirre Madrid (Espaa) 2003
Seguridad Informtica y Criptografa. Tema 18: Bibliografa, Enlaces, SW y Tablas
Diapositiva 904
Cdigo ASCII/ANSI de nivel bajo (2)
Byte carcter
0101 0000 P
0101 0001 Q
0101 0010 R
0101 0011 S
0101 0100 T
0101 0101 U
0101 0110 V
0101 0111 W
0101 1000 X
0101 1001 Y
0101 1010 Z
0101 1011 [
0101 1100 \
0101 1101 ]
0101 1110 ^
0101 1111 _
Byte carcter
0111 0000 p
0111 0001 q
0111 0010 r
0111 0011 s
0111 0100 t
0111 0101 u
0111 0110 v
0111 0111 w
0111 1000 x
0111 1001 y
0111 1010 z
0111 1011 {
0111 1100 |
0111 1101 }
0111 1110 ~
0111 1111
Byte carcter
0110 0000 `
0110 0001 a
0110 0010 b
0110 0011 c
0110 0100 d
0110 0101 e
0110 0110 f
0110 0111 g
0110 1000 h
0110 1001 i
0110 1010 j
0110 1011 k
0110 1100 l
0110 1101 m
0110 1110 n
0110 1111 o
Jorge Rami Aguirre Madrid (Espaa) 2003
Seguridad Informtica y Criptografa. Tema 18: Bibliografa, Enlaces, SW y Tablas
Diapositiva 905
Tabla de cdigo ASCII extendido
00-0F: Valor decimal: 000015
10-1F: Valor decimal: 016031
202F: Valor decimal: 032047
303F: Valor decimal: 048063
404F: Valor decimal: 064079
505F: Valor decimal: 080095
606F: Valor decimal: 096111
707F: Valor decimal: 112127
808F: Valor decimal: 128143
909F: Valor decimal: 144159
A0AF: Valor decimal: 160175
B0BF: Valor decimal: 176191
C0CF: Valor decimal: 192207
D0DF: Valor decimal: 208223
E0EF: Valor decimal: 224239
F0FF: Valor decimal: 240255
Jorge Rami Aguirre Madrid (Espaa) 2003
Seguridad Informtica y Criptografa. Tema 18: Bibliografa, Enlaces, SW y Tablas
Diapositiva 906
Tabla de cdigo ANSI extendido
00-0F: Valor decimal: 000015
10-1F: Valor decimal: 016031
202F: Valor decimal: 032047
303F: Valor decimal: 048063
404F: Valor decimal: 064079
505F: Valor decimal: 080095
606F: Valor decimal: 096111
707F: Valor decimal: 112127
808F: Valor decimal: 128143
909F: Valor decimal: 144159
A0AF: Valor decimal: 160175
B0BF: Valor decimal: 176191
C0CF: Valor decimal: 192207
D0DF: Valor decimal: 208223
E0EF: Valor decimal: 224239
F0FF: Valor decimal: 240255
Jorge Rami Aguirre Madrid (Espaa) 2003
Seguridad Informtica y Criptografa. Tema 18: Bibliografa, Enlaces, SW y Tablas
Diapositiva 907
La codificacin en Base 64
Valor Carcter
6 bits codificado
Valor Carcter
6 bits codificado
Valor Carcter
6 bits codificado
Valor Carcter
6 bits codificado
0 000000 A 16 010000 Q 32 100000 g 48 110000 w
1 000001 B 17 010001 R 33 100001 h 49 110001 x
2 000010 C 18 010010 S 34 100010 i 50 110010 y
3 000011 D 19 010011 T 35 100011 j 51 110011 z
4 000100 E 20 010100 U 36 100100 k 52 110100 0
5 000101 F 21 010101 V 37 100101 l 53 110101 1
6 000110 G 22 010110 W 38 100110 m 54 110110 2
7 000111 H 23 010111 X 39 100111 n 55 110111 3
8 001000 I 24 011000 Y 40 101000 o 56 111000 4
9 001001 J 25 011001 Z 41 101001 p 57 111001 5
10 001010 K 26 011010 a 42 101010 q 58 111010 6
11 001011 L 27 011011 b 43 101011 r 59 111011 7
12 001100 M 28 011100 c 44 101100 s 60 111100 8
13 001101 N 29 011101 d 45 101101 t 61 111101 9
14 001110 O 30 011110 e 46 101110 u 62 111110 +
15 001111 P 31 011111 f 47 101111 v 63 111111 /
(Relleno) =
Tabla de codificacin en Base 64
Cada 3 bytes ANSI (24 bits) se convierten en 4 elementos Base 64 de 6 bits cada
uno. El fichero aumenta un 33% pero ello se compensar al usar la compresin zip.
Jorge Rami Aguirre Madrid (Espaa) 2003
Seguridad Informtica y Criptografa. Tema 18: Bibliografa, Enlaces, SW y Tablas
Diapositiva 908
Ejemplo de codificacin Base 64
Valor Carcter
6 bits codificado
Valor Carcter
6 bits codificado
Valor Carcter
6 bits codificado
Valor Carcter
6 bits codificado
0 000000 A 16 010000 Q 32 100000 g 48 110000 w
1 000001 B 17 010001 R 33 100001 h 49 110001 x
2 000010 C 18 010010 S 34 100010 i 50 110010 y
3 000011 D 19 010011 T 35 100011 j 51 110011 z
4 000100 E 20 010100 U 36 100100 k 52 110100 0
5 000101 F 21 010101 V 37 100101 l 53 110101 1
6 000110 G 22 010110 W 38 100110 m 54 110110 2
7 000111 H 23 010111 X 39 100111 n 55 110111 3
8 001000 I 24 011000 Y 40 101000 o 56 111000 4
9 001001 J 25 011001 Z 41 101001 p 57 111001 5
10 001010 K 26 011010 a 42 101010 q 58 111010 6
11 001011 L 27 011011 b 43 101011 r 59 111011 7
12 001100 M 28 011100 c 44 101100 s 60 111100 8
13 001101 N 29 011101 d 45 101101 t 61 111101 9
14 001110 O 30 011110 e 46 101110 u 62 111110 +
15 001111 P 31 011111 f 47 101111 v 63 111111 /
(Relleno) =
Tabla de codificacin en Base 64
Hola
ANSI
= 01001000 01101111 01101100 01100001
Hola
B64
= 010010 000110 111101 101100 011000 01 (00 00) = SG9sYQ==
Jorge Rami Aguirre Madrid (Espaa) 2003
Seguridad Informtica y Criptografa. Tema 18: Bibliografa, Enlaces, SW y Tablas
Diapositiva 909
Tabla de primos del 1 al 1000
997 991 983 977 971 967 953 947 941 937 929 919 911 907
887 883 881 877 863 859 857 853 839 829 827 823 821 811 809
797 787 773 769 761 757 751 743 739 733 727 711 709 701
691 683 677 673 661 659 653 647 643 641 631 619 617 613 607 601
599 593 587 577 571 569 563 557 547 541 523 521 509 503
499 491 487 479 467 463 461 457 449 443 439 433 431 421 419 409 401
397 389 383 379 373 367 359 353 349 347 337 331 317 313 311 307
293 283 281 277 271 269 263 257 251 241 239 233 229 227 223 211
199 197 193 191 181 179 173 167 163 157 151 149 139 137 131 127 113 109 107 103 101
97 89 83 79 73 71 67 61 59 53 47 43 41 37 31 29 23 19 17 13 11 7 5 3 2
Jorge Rami Aguirre Madrid (Espaa) 2003
Seguridad Informtica y Criptografa. Tema 18: Bibliografa, Enlaces, SW y Tablas
Diapositiva 910
Tabla de primos del 1001 al 2000
1999 1997 1993 1987 1979 1973 1951 1949 1933 1931 1913 1907 1901
1889 1879 1877 1873 1871 1867 1861 1847 1831 1823 1811 1801
1789 1787 1783 1777 1759 1753 1747 1741 1733 1723 1721 1709
1699 1667 1693 1669 1667 1663 1657 1637 1627 1621 1619 1613 1609 1607 1601
1597 1583 1579 1571 1567 1559 1553 1549 1543 1531 1523 1511
1499 1493 1489 1487 1483 1481 1471 1459 1453 1451 1447 1439 1433 1429 1427 1423 1409
1399 1381 1373 1367 1361 1327 1321 1319 1307 1303 1301
1297 1291 1289 1283 1279 1277 1259 1249 1237 1231 1229 1223 1217 1213 1201
1193 1187 1181 1171 1163 1153 1151 1129 1123 1117 1109 1103
1097 1093 1091 1087 1069 1063 1061 1051 1049 1039 1033 1031 1021 1019 1013 1009
Jorge Rami Aguirre Madrid (Espaa) 2003
Seguridad Informtica y Criptografa. Tema 18: Bibliografa, Enlaces, SW y Tablas
Diapositiva 911
Polinomios primitivos
Polinomios para n = 3: x
3
+ x + 1; x
3
+ x
2
+ 1
Polinomios para n = 4: x
4
+ x + 1; x
4
+ x
3
+ 1
Polinomios para n = 5: x
5
+ x
2
+ 1; x
5
+ x
4
+ x
3
+ x
2
+ 1; x
5
+ x
4
+ x
2
+ 1; x
5
+ x
3
+ x
2
+ x + 1;
x
5
+ x
4
+ x
3
+ x + 1; x
5
+ x
3
+ x + 1
Polinomios para n = 6: x
6
+ x + 1; x
6
+ x
5
+ x
2
+ x + 1; x
6
+ x
5
+ x
3
+ x
2
+ 1; x
6
+ x
4
+ x
3
+ x + 1;
x
6
+ x
5
+ x
4
+ x + 1; x
6
+ x
5
+ 1
(1, 0) (11, 2, 0) (21, 2, 0) (31, 3, 0) (41, 3, 0)
(2, 1, 0) (12, 6, 4, 1, 0) (22, 1, 0) (32, 7, 6, 2, 0) (42, 5, 4, 3, 2, 1, 0)
(3, 1, 0) (13, 4, 3, 1, 0) (23, 5, 0) (33, 13, 0) (43, 6, 4, 3, 0)
(4, 1, 0) (14, 5, 3, 1, 0) (24, 4, 3, 1, 0) (34, 8, 4, 3, 0) (44, 6, 5, 2, 0)
(5, 2, 0) (15, 1, 0) (25, 3, 0) (35, 2, 0) (45, 4, 3, 1, 0)
(6, 1, 0) (16, 5, 3, 2, 0) (26, 6, 2, 1, 0) (36, 11, 0) (46, 8, 5, 3, 2, 1, 0)
(7, 1, 0) (17, 3, 0) (27, 5, 2, 1, 0) (37, 6, 4, 1, 0) (47, 5, 0)
(8, 4, 3, 2, 0) (18, 7, 0) (28, 3, 0) (38, 6, 5, 1, 0) (48, 7, 5, 4, 2, 1, 0)
(9, 4, 0) (19, 5, 2, 1, 0) (29, 2, 0) (39, 4, 0) (49, 9, 0)
(10, 3, 0) (20, 3, 0) (30, 6, 4, 1, 0) (40, 5, 4, 3, 0) (50, 4, 3, 2, 0) (*)
Algunos polinomios de x
n
para n = 1 hasta n = 50
(*) Explicacin: (50, 4, 3, 2, 0) p(x) = x
50
+ x
4
+ x
3
+ x
2
+ 1
Jorge Rami Aguirre Madrid (Espaa) 2003
Seguridad Informtica y Criptografa. Tema 18: Bibliografa, Enlaces, SW y Tablas
Diapositiva 912
Magnitudes de tiempo y criptoanlisis
22.020.985.858.787.784.059 (2
64
) aos 128 bits
336.013.578.167.538 (2
48
) aos 112 bits
5.127.160.311 (2
32
) aos 96 bits
78.250 (2
16
) aos 80 bits
305 aos 72 bits
14 meses 64 bits
40 horas 56 bits
9 minutos 48 bits
2 segundos 40 bits
Tiempo necesario para romper la clave Longitud de la clave
La tabla muestra el tiempo
medio de criptoanlisis
necesario para romper una
clave del DES por fuerza
bruta, usando la potencia
de clculo alcanzada en el
DES Challenge III, unos
250.000 millones de claves
por segundo con mquina
DES Cracker y 100.000
PCs conectados en red.
10.000.000.000 (10
10
= 2
34
) aos Edad del planeta
100.000.000.000 (10
11
= 2
37
) aos Edad del universo
Valores de tiempo con nmeros grandes
Fin del Tema 18
Jorge Rami Aguirre Madrid (Espaa) 2003
Seguridad Informtica y Criptografa. Tema 18: Bibliografa, Enlaces, SW y Tablas
Diapositiva 913
Cuestiones y ejercicios (1 de 2)
1. En un texto cifrado por sustitucin afn, se tiene como criptograma
C = PCERC QBCSK AQBGR LGFJQ KCXK LCECN RKZHL
KKXGF LCAFB. Qu es lo primero que hacemos? Ataque el
criptograma con el software de la asignatura. Qu puede concluir?
2. Con el cifrador de Vernam y cdigo Baudot ciframos M = SOL con
la clave K = MAR. Qu se obtiene como criptograma?
3. Observando el cdigo ASCII/ANSI de nivel bajo en binario, Por
qu es desaconsejable cifrar con mochilas de tamao 4 u 8?
4. La clave DES escrita en cdigo ASCII es K = HOLAhola. Cul
sera en este caso la clave de 56 bits? Qu puede decir de esto?
5. Observando la tabla de primos del 1 al 1.000 y de 1.001 al 2.000,
podramos concluir que en una ventana igual (2.001 al 3.000; 3.001
al 4.000, etc.) cada vez hay ms nmeros primos? Por qu?
Jorge Rami Aguirre Madrid (Espaa) 2003
Seguridad Informtica y Criptografa. Tema 18: Bibliografa, Enlaces, SW y Tablas
Diapositiva 914
Cuestiones y ejercicios (2 de 2)
6. Por qu en mdulo 37 existen ms inversos que en mdulo 27?
7. Codifique en base 64 el texto de 10 caracteres M = Qu tal!
8. Qu mensaje hay en M = v01lb nNham Ugb2N 1bHRv Pw==
que est codificado en base 64? Por qu el relleno es dos?
9. En cunto aumenta el tamao cuando convertimos ASCII/ANSI a
cdigo base 64? Es eso significativo en PGP? Por qu s o no?
10. En qu zona podemos decir que el cdigo ASCII y el ANSI son
iguales?
11. Se codificar igual en ASCII que en ANSI el mensaje M
1
= Voy
a entrar? Y si ahora el mensaje es M
2
= Pasa, est abierto Qu
consecuencias puede tener esto en un programa?
12. Un mensaje codificado en ANSI hexadecimal es 43 72 69 70 74 6F
67 72 61 66 ED 61, cul es el texto en castellano?

You might also like